Você está na página 1de 82

Answer Sheets

1 A
O B
O C
O D
O E
O 6 A
O B
O C
O D
O E
O 11 A
O B
O C
O D
O E
O 16 A
O B
O C
O D
O E
O
SECTION 2 A
O B
O C
O D
O E
O 7 A
O B
O C
O D
O E
O 12 A
O B
O C
O D
O E
O 17 A
O B
O C
O D
O E
O

1 3
4
5
A
O
A
O
A
O
B
O

B
O
C
O

B
O

C
O
D
O

C
O
D
O
E
O

D
O
E
O
E
O
8
9
10
A
O
A
O
A
O
B
O

B
O
C
O

B
O

C
O
D
O

C
O
D
O
E
O

D
O
E
O
E
O
13
14
15
A
O
A
O
A
O
B
O

B
O
C
O

B
O

C
O
D
O

C
O
D
O
E
O

D
O
E
O
E
O
18
19
20
A
O
A
O
A
O
B
O

B
O
B
O
C
O

C
O

C
O
D
O

D
O
D
O
E
O

E
O
E
O

1 A
O B
O C
O D
O E
O 7 A
O B
O C
O D
O E
O 12 A
O B
O C
O D
O E
O 17 A
O B
O C
O D
O E
O

SECTION 2 A
O B
O C
O D
O E
O 8 A
O B
O C
O D
O E
O 13 A
O B
O C
O D
O E
O 18 A
O B
O C
O D
O E
O

3 A
O B
O C
O D
O E
O 9 A
O B
O C
O D
O E
O 14 A
O B
O C
O D
O E
O 19 A
O B
O C
O D
O E
O

2 4
5
6
A
O
A
O
A
O
B
O

B
O
C
O
B
O

C
O
D
O

D
O
E
O

C
O
E
O
D
O E
O
10
11
A
O
A
O
B
O C
O
B
O
D
O E
O

C
O D
O E
O
15
16
A
O
A
O
B
O C
O
B
O
D
O E
O

C
O D
O E
O
20
21
A
O
A
O
B
O
B
O
C
O

C
O
D
O
D
O
E
O
E
O

1 A
O B
O C
O D
O E
O 9 A
O B
O C
O D
O E
O 17 A
O B
O C
O D
O E
O 24 A
O B
O C
O D
O E
O

2 A
O B
O C
O D
O E
O 10 A
O B
O C
O D
O E
O 18 A
O B
O C
O D
O E
O 25 A
O B
O C
O D
O E
O

SECTION 3 A
O B
O C
O D
O E
O 11 A
O B
O C
O D
O E
O 19 A
O B
O C
O D
O E
O 26 A
O B
O C
O D
O E
O

4 A
O B
O C
O D
O E
O 12 A
O B
O C
O D
O E
O 20 A
O B
O C
O D
O E
O 27 A
O B
O C
O D
O E
O

3 5
6
7
A
O
A
O
A
O
B
O

B
O
B
O

C
O

C
O
D
O

D
O
C
O
E
O

E
O
D
O E
O 13
14
15
A
O
A
O
A
O
B
O

B
O
B
O

C
O

C
O
D
O

D
O
C
O
E
O

E
O
D
O E
O 21
22
23
A
O
A
O
A
O
B
O

B
O
B
O

C
O

C
O
D
O

D
O
C
O
E
O

E
O
D
O E
O 28
29
30
A
O
A
O
A
O
B
O
B
O

B
O
C
O

C
O

C
O
D
O
D
O

D
O
E
O
E
O

E
O

8 A
O B
O C
O D
O E
O 16 A
O B
O C
O D
O E
O

1 A
O B
O C
O D
O E
O 8 A
O B
O C
O D
O E
O 15 A
O B
O C
O D
O E
O 22 A
O B
O C
O D
O E
O

2 A
O B
O C
O D
O E
O 9 A
O B
O C
O D
O E
O 16 A
O B
O C
O D
O E
O 23 A
O B
O C
O D
O E
O
SECTION 3 A
O B
O C
O D
O E
O 10 A
O B
O C
O D
O E
O 17 A
O B
O C
O D
O E
O 24 A
O B
O C
O D
O E
O

4 4
5
6
A
O
A
O
A
O
B
O
B
O
C
O

C
O
B
O
D
O
D
O

C
O
E
O
E
O
D
O E
O
11
12
13
A
O
A
O
A
O
B
O
B
O
C
O

C
O
B
O
D
O
D
O

C
O
E
O
E
O
D
O E
O
18
19
20
A
O
A
O
A
O
B
O
B
O
C
O

C
O
B
O
D
O
D
O

C
O
E
O
E
O
D
O E
O
25
26
27
A
O
A
O
A
O
B
O
B
O
B
O
C
O

C
O

C
O
D
O
D
O
D
O
E
O
E
O
E
O

7 A
O B
O C
O D
O E
O 14 A
O B
O C
O D
O E
O 21 A
O B
O C
O D
O E
O

1 A
O B
O C
O D
O E
O 7 A
O B
O C
O D
O E
O 12 A
O B
O C
O D
O E
O 17 A
O B
O C
O D
O E
O

SECTION 2 A
O B
O C
O D
O E
O 8 A
O B
O C
O D
O E
O 13 A
O B
O C
O D
O E
O 18 A
O B
O C
O D
O E
O

3 A
O B
O C
O D
O E
O 9 A
O B
O C
O D
O E
O 14 A
O B
O C
O D
O E
O 19 A
O B
O C
O D
O E
O

5 4
5
6
A
O
A
O
A
O
B
O

B
O
C
O
B
O

C
O
D
O

D
O
E
O

C
O

E
O
D
O E
O
10
11
A
O
A
O
B
O C
O
B
O
D
O E
O

C
O D
O E
O
15
16
A
O
A
O
B
O C
O
B
O
D
O E
O

C
O D
O E
O
20
21
A
O
A
O
B
O
B
O
C
O

C
O
D
O
D
O
E
O
E
O

Copyright © 2005 Thomson Peterson’s, a part of The Thomson Corporaton 1


SAT is a registered trademark of the College Entrance Examination Board, which
was not involved in the production of and does not endorse this product.
Answer Sheets

SECTION 1 A
O B
O C
O D
O E
O 5 A
O B
O C
O D
O E
O 9 A
O B
O C
O D
O E
O 13 A
O B
O C
O D
O E
O

2 A
O B
O C
O D
O E
O 6 A
O B
O C
O D
O E
O 10 A
O B
O C
O D
O E
O 14 A
O B
O C
O D
O E
O

6 3
4
A
O
A
O
B
O C
O

B
O
D
O

C
O
E
O

D
O E
O
7
8
A
O
A
O
B
O C
O

B
O
D
O

C
O
E
O

D
O E
O
11
12
A
O
A
O
B
O C
O

B
O
D
O

C
O
E
O

D
O E
O
15
16
A
O
A
O
B
O

B
O
C
O

C
O
D
O

D
O
E
O

E
O

SECTION
For Questions 1–13:

1
7 Only answers entered in the ovals in each grid area will be scored.
You will not receive credit for anything written in the boxes above the ovals.
2 3 4 5

6 7 8 9 10

11 12 13

2 Copyright © 2005 Thomson Peterson’s, a part of The Thomson Corporaton


SAT is a registered trademark of the College Entrance Examination Board, which
was not involved in the production of and does not endorse this product.
Practice Test

2
Section 1
20 Questions j Time—25 Minutes
12345678901234567890123456789012123456789012345678901234567890121234567
12234567890123456789012345678901212345678901234567890123456789012123456 7
1234567890123456789012345678901212345678901234567890123456789012123456 7
1234567890123456789012345678901212345678901234567890123456789012123456 7
1234567890123456789012345678901212345678901234567890123456789012123456 7
1234567890123456789012345678901212345678901234567890123456789012123456
Directions: Read each of the passages carefully, then answer the questions that come after them. 7
34567890123456789012345678901212345678901234567890123456789012123456
1234567890123456789012345678901212345678901234567890123456789012123456 7
1 The answer to each question may be stated overtly or only implied. You will not have to use 7
12234567890123456789012345678901212345678901234567890123456789012123456 7
1234567890123456789012345678901212345678901234567890123456789012123456
outside knowledge to answer the questions—all the material you will need will be in the passage 7
1234567890123456789012345678901212345678901234567890123456789012123456 7
1234567890123456789012345678901212345678901234567890123456789012123456 77
34567890123456789012345678901212345678901234567890123456789012123456
itself. In some cases, you will be asked to read two related passages and answer questions about
1234567890123456789012345678901212345678901234567890123456789012123456
their relationship to one another. Mark the letter of your choice on your answer sheet.
1234567890123456789012345678901212345678901234567890123456789012123456 7
1234567890123456789012345678901212345678901234567890123456789012123456 77
1234567890123456789012345678901212345678901234567890123456789012123456 7
1234567890123456789012345678901212345678901234567890123456789012123456 7
1234567890123456789012345678901212345678901234567890123456789012123456 7
1234567890123456789012345678901212345678901234567890123456789012123456
A bill is the form used for most legislation in 1. From the passage, it can be inferred that a 7
1234567890123456789012345678901212345678901234567890123456789012123456 7
1 the United State Congress. Only constitutional bill that is designated as H.R. 1 is the
12234567890123456789012345678901212345678901234567890123456789012123456 7
1234567890123456789012345678901212345678901234567890123456789012123456
amendments and procedural issues affecting the first bill 7
1 34567890123456789012345678901212345678901234567890123456789012123456 7
1234567890123456789012345678901212345678901234567890123456789012123456
House and Senate are adopted by a resolution, 7
12234567890123456789012345678901212345678901234567890123456789012123456
34567890123456789012345678901212345678901234567890123456789012123456
(A) voted upon by the House of Repre- 7
7
1234567890123456789012345678901212345678901234567890123456789012123456
rather than a bill. Bills can be written to be
1234567890123456789012345678901212345678901234567890123456789012123456 sentatives in a particular session of 7
1 permanent or temporary, general or special. A 7
12234567890123456789012345678901212345678901234567890123456789012123456 Congress. 7
1234567890123456789012345678901212345678901234567890123456789012123456
bill originating in the House of Representatives 7
34567890123456789012345678901212345678901234567890123456789012123456
1234567890123456789012345678901212345678901234567890123456789012123456 (B) submitted to the House of Represen- 7
1234567890123456789012345678901212345678901234567890123456789012123456
is designated by the letters “H.R.,” signifying 7
1 “House of Representatives,” followed by a tatives in a particular session of 7
1234567890123456789012345678901212345678901234567890123456789012123456 7
1234567890123456789012345678901212345678901234567890123456789012123456 Congress. 7
2 34567890123456789012345678901212345678901234567890123456789012123456
1234567890123456789012345678901212345678901234567890123456789012123456
number that it retains throughout all its
(C) sent to the Senate from the House of 7
1234567890123456789012345678901212345678901234567890123456789012123456 7
1234567890123456789012345678901212345678901234567890123456789012123456
parliamentary stages. The number on the bill is 7
1234567890123456789012345678901212345678901234567890123456789012123456 Representatives in a particular 7
1 determined by the order in which it was 7
2 34567890123456789012345678901212345678901234567890123456789012123456
1234567890123456789012345678901212345678901234567890123456789012123456 session of Congress. 7
1234567890123456789012345678901212345678901234567890123456789012123456
submitted during a particular session. Bills are 7
1234567890123456789012345678901212345678901234567890123456789012123456 (D) originating in the House of Repre- 7
1234567890123456789012345678901212345678901234567890123456789012123456
presented to the President for action when 7
1 approved in identical form by both the House sentatives signed by the President in 7
2 34567890123456789012345678901212345678901234567890123456789012123456
1234567890123456789012345678901212345678901234567890123456789012123456 a particular session of Congress. 7
1234567890123456789012345678901212345678901234567890123456789012123456 7
1234567890123456789012345678901212345678901234567890123456789012123456 (E) debated on the floor of the House of 7
of Representatives and the Senate.
1234567890123456789012345678901212345678901234567890123456789012123456 77
1 Representatives in a particular
12234567890123456789012345678901212345678901234567890123456789012123456 7
1 34567890123456789012345678901212345678901234567890123456789012123456 session of Congress. 7
12234567890123456789012345678901212345678901234567890123456789012123456 7
1 34567890123456789012345678901212345678901234567890123456789012123456 7
12234567890123456789012345678901212345678901234567890123456789012123456 7
1 34567890123456789012345678901212345678901234567890123456789012123456 7
1234567890123456789012345678901212345678901234567890123456789012123456 7
1234567890123456789012345678901212345678901234567890123456789012123456 7
1234567890123456789012345678901212345678901234567890123456789012123456 7
12345678901234567890123456789012123456789012345678901234567890121234567
Copyright © 2005 Thomson Peterson’s, a part of The Thomson Corporaton 3
SAT is a registered trademark of the College Entrance Examination Board, which
was not involved in the production of and does not endorse this product.
12345678901234567890123456789012123456789012345678901234567890121234567
1234567890123456789012345678901212345678901234567890123456789012123456 7
12 34567890123456789012345678901212345678901234567890123456789012123456 7
1234567890123456789012345678901212345678901234567890123456789012123456
2. It is implied in the passage that once a bill 4. Given what the passage states about 7
12 34567890123456789012345678901212345678901234567890123456789012123456 7
12 34567890123456789012345678901212345678901234567890123456789012123456
is passed in the House of Representatives Native American views of nature, which 7
1234567890123456789012345678901212345678901234567890123456789012123456 7
12
2
34567890123456789012345678901212345678901234567890123456789012123456 7
that it might be sent to which of the of the
34567890123456789012345678901212345678901234567890123456789012123456
following scenarios most accords
7
1
12 34567890123456789012345678901212345678901234567890123456789012123456
following two places? with a Native American view? 7
12 34567890123456789012345678901212345678901234567890123456789012123456 7
12 34567890123456789012345678901212345678901234567890123456789012123456 7
12
2
34567890123456789012345678901212345678901234567890123456789012123456
(A) Senate, conference committee (A) Studying a microorganism removed
34567890123456789012345678901212345678901234567890123456789012123456
7
7
1
12 34567890123456789012345678901212345678901234567890123456789012123456
(B) Senate, House committee from its habitat. 7
12 34567890123456789012345678901212345678901234567890123456789012123456 7
12 34567890123456789012345678901212345678901234567890123456789012123456 7
(C) Senate, President (B) Studying Earth through satellite
12 34567890123456789012345678901212345678901234567890123456789012123456 7
1234567890123456789012345678901212345678901234567890123456789012123456
(D) President, Supreme Court images. 7
12 34567890123456789012345678901212345678901234567890123456789012123456 7
12 34567890123456789012345678901212345678901234567890123456789012123456 7
(E) President, Congress (C) Studying only animals and sub-
12 34567890123456789012345678901212345678901234567890123456789012123456 7
12
2
34567890123456789012345678901212345678901234567890123456789012123456
34567890123456789012345678901212345678901234567890123456789012123456
stances with spiritual symbolism. 7
7
1 Native American views of nature have impor- (D) Studying a specific organism’s
12 34567890123456789012345678901212345678901234567890123456789012123456 7
12 34567890123456789012345678901212345678901234567890123456789012123456
tant parallels in contemporary ecology. interrelationships with its habitat. 7
12 34567890123456789012345678901212345678901234567890123456789012123456 7
12 34567890123456789012345678901212345678901234567890123456789012123456
Through traditional customs and symbols like (E) Studying a habitat as a whole. 7
2 34567890123456789012345678901212345678901234567890123456789012123456
1234567890123456789012345678901212345678901234567890123456789012123456 7
1234567890123456789012345678901212345678901234567890123456789012123456
the medicine wheel, a circular arrangement of 7
1234567890123456789012345678901212345678901234567890123456789012123456 7
1234567890123456789012345678901212345678901234567890123456789012123456
stones often interpreted as representing the 7
1234567890123456789012345678901212345678901234567890123456789012123456
Questions 5–12 are based on the following 7
1234567890123456789012345678901212345678901234567890123456789012123456
relationship between Earth, air, water, and fire, 7
1 Native Americans have long recognized and passage. 7
12 34567890123456789012345678901212345678901234567890123456789012123456 7
12 34567890123456789012345678901212345678901234567890123456789012123456
This passage is about Aaron Copland, one of the 7
12 34567890123456789012345678901212345678901234567890123456789012123456
celebrated the connectedness among all natural 7
2 34567890123456789012345678901212345678901234567890123456789012123456
1234567890123456789012345678901212345678901234567890123456789012123456
most celebrated American composers. 7
1234567890123456789012345678901212345678901234567890123456789012123456
things. Indeed, the Native American view of the 7
1234567890123456789012345678901212345678901234567890123456789012123456 7
1234567890123456789012345678901212345678901234567890123456789012123456
world has always been consistent with that of Line Copland’s music of the late 1920s 7
1234567890123456789012345678901212345678901234567890123456789012123456 7
1234567890123456789012345678901212345678901234567890123456789012123456 77
Earth ecology—that Earth is a single system of culminates in two key works, both
1234567890123456789012345678901212345678901234567890123456789012123456
interconnected parts. uncompromising in their modernism: the 7
1234567890123456789012345678901212345678901234567890123456789012123456
1234567890123456789012345678901212345678901234567890123456789012123456 7
1 Symphonic Ode of 1929 and the Piano 7
2 34567890123456789012345678901212345678901234567890123456789012123456
1234567890123456789012345678901212345678901234567890123456789012123456 7
1234567890123456789012345678901212345678901234567890123456789012123456
3. The symbol of the medicine wheel is given (5) Variations of 1930. The fate of these 7
1 7
1234567890123456789012345678901212345678901234567890123456789012123456 7
as a(n) compositions contrasts sharply. While the
12 34567890123456789012345678901212345678901234567890123456789012123456 7
12 34567890123456789012345678901212345678901234567890123456789012123456
Piano Variations is not often performed 7
12 34567890123456789012345678901212345678901234567890123456789012123456
(A) illustration of how Native Americans 7
1234567890123456789012345678901212345678901234567890123456789012123456
in concert, it is well known to pianists 7
12 34567890123456789012345678901212345678901234567890123456789012123456
view the Earth as an interconnected 7
12 34567890123456789012345678901212345678901234567890123456789012123456
because, although it does contain 7
2 34567890123456789012345678901212345678901234567890123456789012123456
1234567890123456789012345678901212345678901234567890123456789012123456
system. 7
1234567890123456789012345678901212345678901234567890123456789012123456
(B) example of the Native American
(10) virtuoso passages, even those of very 7
1 7
1234567890123456789012345678901212345678901234567890123456789012123456
understanding of the four elements.
modest ability can “play at” the work in 7
1234567890123456789012345678901212345678901234567890123456789012123456 7
2 34567890123456789012345678901212345678901234567890123456789012123456
1234567890123456789012345678901212345678901234567890123456789012123456
(C) example of the interrelatedness of the
private. It represents the twentieth- 7
1234567890123456789012345678901212345678901234567890123456789012123456
century continuation of the great 7
1234567890123456789012345678901212345678901234567890123456789012123456
four basic elements. 7
1234567890123456789012345678901212345678901234567890123456789012123456
tradition of keyboard variations—the 7
1 (D) critique of contemporary ecological 7
2 34567890123456789012345678901212345678901234567890123456789012123456
1234567890123456789012345678901212345678901234567890123456789012123456
(15) tradition that produced such works as 7
1234567890123456789012345678901212345678901234567890123456789012123456
understandings of the Earth. 7
1234567890123456789012345678901212345678901234567890123456789012123456
the Bach Goldberg Variations, and 7
1234567890123456789012345678901212345678901234567890123456789012123456
(E) contrast to contemporary ecological 7
1 Beethoven’s Diabelli Variations. Cop- 7
2 34567890123456789012345678901212345678901234567890123456789012123456
1234567890123456789012345678901212345678901234567890123456789012123456
understandings of the Earth. 7
1234567890123456789012345678901212345678901234567890123456789012123456
land’s Symphonic Ode, on the other 7
1234567890123456789012345678901212345678901234567890123456789012123456 7
1234567890123456789012345678901212345678901234567890123456789012123456
hand, remains almost unknown: An 7
1 7
1234567890123456789012345678901212345678901234567890123456789012123456 77
2 34567890123456789012345678901212345678901234567890123456789012123456
(20) intense symphonic movement, it was
1
12 34567890123456789012345678901212345678901234567890123456789012123456
considered unperformable by the 7
1234567890123456789012345678901212345678901234567890123456789012123456 7
12
2
34567890123456789012345678901212345678901234567890123456789012123456 7
conductor
34567890123456789012345678901212345678901234567890123456789012123456
Serge Koussevitzky, otherwise
7
1
1234567890123456789012345678901212345678901234567890123456789012123456
the most potent American champion of 7
1234567890123456789012345678901212345678901234567890123456789012123456 77
1234567890123456789012345678901212345678901234567890123456789012123456
12345678901234567890123456789012123456789012345678901234567890121234567
4 Copyright © 2005 Thomson Peterson’s, a part of The Thomson Corporaton
SAT is a registered trademark of the College Entrance Examination Board, which
was not involved in the production of and does not endorse this product.
12345678901234567890123456789012123456789012345678901234567890121234567
1234567890123456789012345678901212345678901234567890123456789012123456 7
12234567890123456789012345678901212345678901234567890123456789012123456 7
1 34567890123456789012345678901212345678901234567890123456789012123456
Copland’s work during the first half of (Some of the folk music he heard in Rio 7
12234567890123456789012345678901212345678901234567890123456789012123456 7
1234567890123456789012345678901212345678901234567890123456789012123456
(25) the century. Koussevitzky did perform a de Janeiro on this trip appears in his later 7
1 34567890123456789012345678901212345678901234567890123456789012123456 7
12234567890123456789012345678901212345678901234567890123456789012123456
revised version in 1932; but even with a works.) Copland in fact envisioned
34567890123456789012345678901212345678901234567890123456789012123456
7
7
1
12234567890123456789012345678901212345678901234567890123456789012123456
second, more extensive revision in 1955, (70) “American music” as being music of the 7
1234567890123456789012345678901212345678901234567890123456789012123456 7
1234567890123456789012345678901212345678901234567890123456789012123456
the Ode is seldom played. It is Copland’s Americas as a whole. His own use of 7
1234567890123456789012345678901212345678901234567890123456789012123456
34567890123456789012345678901212345678901234567890123456789012123456
7
7
1 single longest orchestral movement. Mexican material in the mid-1930s
12234567890123456789012345678901212345678901234567890123456789012123456 7
1234567890123456789012345678901212345678901234567890123456789012123456
(30) Perhaps as a reaction to the perfor- helped make his style more accessible to 7
1234567890123456789012345678901212345678901234567890123456789012123456 7
1234567890123456789012345678901212345678901234567890123456789012123456
mance problems of the Symphonic Ode, listeners
34567890123456789012345678901212345678901234567890123456789012123456
not willing to accept the 7
1 Copland’s next two orchestral works (75) challenges of modern symphonic music. 7
12234567890123456789012345678901212345678901234567890123456789012123456 7
1234567890123456789012345678901212345678901234567890123456789012123456 7
1234567890123456789012345678901212345678901234567890123456789012123456
deal in shorter units of time: the Short 7
1234567890123456789012345678901212345678901234567890123456789012123456
34567890123456789012345678901212345678901234567890123456789012123456
7
7
1 Symphony of 1933 requires fifteen 5. What is the author’s tone toward Cop-
12234567890123456789012345678901212345678901234567890123456789012123456 7
1234567890123456789012345678901212345678901234567890123456789012123456
(35) minutes for three movements and the six land’s music? 7
1234567890123456789012345678901212345678901234567890123456789012123456
Statements for orchestra of 1935 last 7
1234567890123456789012345678901212345678901234567890123456789012123456 7
34567890123456789012345678901212345678901234567890123456789012123456
1234567890123456789012345678901212345678901234567890123456789012123456
only nineteen minutes. Yet, in fact, these (A) Strident skepticism 7
1234567890123456789012345678901212345678901234567890123456789012123456 7
1234567890123456789012345678901212345678901234567890123456789012123456
works were more complex than the Ode; (B) Clinical objectivity 7
1234567890123456789012345678901212345678901234567890123456789012123456 7
1234567890123456789012345678901212345678901234567890123456789012123456
in particular, the wiry, agile rhythms of (C) Respectful description 7
1234567890123456789012345678901212345678901234567890123456789012123456 7
1 (40) the opening movement of the Short (D) Qualified enthusiasm 7
12234567890123456789012345678901212345678901234567890123456789012123456 7
1234567890123456789012345678901212345678901234567890123456789012123456
Symphony proved too much for both the (E) Unqualified praise 7
1234567890123456789012345678901212345678901234567890123456789012123456 7
1234567890123456789012345678901212345678901234567890123456789012123456 77
34567890123456789012345678901212345678901234567890123456789012123456
conductors Serge Koussevitzky and
1234567890123456789012345678901212345678901234567890123456789012123456
6. The word “virtuoso” in line 10 could best
1234567890123456789012345678901212345678901234567890123456789012123456
Leopold Stokowski. In the end it was 7
1234567890123456789012345678901212345678901234567890123456789012123456
be replaced with 7
1234567890123456789012345678901212345678901234567890123456789012123456 77
Carlos Chávez and the Orquesta Sin-
1234567890123456789012345678901212345678901234567890123456789012123456
1234567890123456789012345678901212345678901234567890123456789012123456
(45) fónica de México who gave the Short (A) ostentatious. 7
1234567890123456789012345678901212345678901234567890123456789012123456 7
1234567890123456789012345678901212345678901234567890123456789012123456 77
Symphony its premiere. (B) intricate
1
12234567890123456789012345678901212345678901234567890123456789012123456
It may have been partly Copland’s (C) raucous. 7
1234567890123456789012345678901212345678901234567890123456789012123456 7
1 34567890123456789012345678901212345678901234567890123456789012123456 7
friendship with Carlos Chávez that drew (D) abstruse.
1234567890123456789012345678901212345678901234567890123456789012123456 7
12234567890123456789012345678901212345678901234567890123456789012123456
him to Mexico. Copland first visited (E) publicized.
34567890123456789012345678901212345678901234567890123456789012123456
7
7
1234567890123456789012345678901212345678901234567890123456789012123456
1234567890123456789012345678901212345678901234567890123456789012123456
(50) Mexico in 1932 and returned frequently 7
1 7
12234567890123456789012345678901212345678901234567890123456789012123456
in later years. His initial delight in the 7
1234567890123456789012345678901212345678901234567890123456789012123456 7
1234567890123456789012345678901212345678901234567890123456789012123456 77
34567890123456789012345678901212345678901234567890123456789012123456
country is related in his letter of January
1234567890123456789012345678901212345678901234567890123456789012123456
13, 1933, to Mary Lescaze, in which he 7
1
1234567890123456789012345678901212345678901234567890123456789012123456
glowingly describes the Mexican people 7
1234567890123456789012345678901212345678901234567890123456789012123456 7
2 34567890123456789012345678901212345678901234567890123456789012123456
1234567890123456789012345678901212345678901234567890123456789012123456
(55) and the Mexican landscape. His interest 7
1234567890123456789012345678901212345678901234567890123456789012123456 7
1234567890123456789012345678901212345678901234567890123456789012123456
in Mexico is also reflected in his music, 7
1234567890123456789012345678901212345678901234567890123456789012123456 77
1 including El Salón México (1936) and
12234567890123456789012345678901212345678901234567890123456789012123456 7
1234567890123456789012345678901212345678901234567890123456789012123456
the Three Latin American Sketches 7
1234567890123456789012345678901212345678901234567890123456789012123456 7
1234567890123456789012345678901212345678901234567890123456789012123456
(1972). 7
1 34567890123456789012345678901212345678901234567890123456789012123456 7
1234567890123456789012345678901212345678901234567890123456789012123456 77
2 34567890123456789012345678901212345678901234567890123456789012123456
(60) Mexico was not Copland’s only Latin
1234567890123456789012345678901212345678901234567890123456789012123456 7
1234567890123456789012345678901212345678901234567890123456789012123456
American interest. A 1941 trip to
7
1234567890123456789012345678901212345678901234567890123456789012123456 7
1 Havana suggested his Danzón Cubano.
2 34567890123456789012345678901212345678901234567890123456789012123456
1234567890123456789012345678901212345678901234567890123456789012123456 7
1 By the early 1940s he was friends with 7
12234567890123456789012345678901212345678901234567890123456789012123456 7
1 34567890123456789012345678901212345678901234567890123456789012123456 7
South American composers such as
12234567890123456789012345678901212345678901234567890123456789012123456 7
1 34567890123456789012345678901212345678901234567890123456789012123456
(65) Jacobo Ficher, and in 1947 he toured 7
1234567890123456789012345678901212345678901234567890123456789012123456 7
1234567890123456789012345678901212345678901234567890123456789012123456 77
South America for the State Department.
1234567890123456789012345678901212345678901234567890123456789012123456
12345678901234567890123456789012123456789012345678901234567890121234567
Copyright © 2005 Thomson Peterson’s, a part of The Thomson Corporaton 5
SAT is a registered trademark of the College Entrance Examination Board, which
was not involved in the production of and does not endorse this product.
12345678901234567890123456789012123456789012345678901234567890121234567
1234567890123456789012345678901212345678901234567890123456789012123456 7
12 34567890123456789012345678901212345678901234567890123456789012123456 7
1234567890123456789012345678901212345678901234567890123456789012123456
7. In the first paragraph the author states 9. The author of the passage believes that 7
12 34567890123456789012345678901212345678901234567890123456789012123456 7
12 34567890123456789012345678901212345678901234567890123456789012123456
that Symphonic Ode and Piano Variations Copland’s works immediately subsequent 7
1234567890123456789012345678901212345678901234567890123456789012123456 7
12
2
34567890123456789012345678901212345678901234567890123456789012123456
had different fates in that to the Symphonic Ode were possibly
34567890123456789012345678901212345678901234567890123456789012123456
7
7
1
12 34567890123456789012345678901212345678901234567890123456789012123456
written 7
12 34567890123456789012345678901212345678901234567890123456789012123456 7
(A) one was largely ignored while the
12 34567890123456789012345678901212345678901234567890123456789012123456 7
12
2
34567890123456789012345678901212345678901234567890123456789012123456
other was almost universally praised. (A) for Copland’s new relationship with
34567890123456789012345678901212345678901234567890123456789012123456
7
7
1
12 34567890123456789012345678901212345678901234567890123456789012123456
(B) one, a simpler piece, won popular Carlos Chávez and the Orquesta 7
12 34567890123456789012345678901212345678901234567890123456789012123456 7
12 34567890123456789012345678901212345678901234567890123456789012123456 7
acclaim, while the other, a more Sinfónica de México.
12 34567890123456789012345678901212345678901234567890123456789012123456 7
1234567890123456789012345678901212345678901234567890123456789012123456
complex piece, won critical acclaim. (B) to be simpler than the Symphonic 7
12 34567890123456789012345678901212345678901234567890123456789012123456 7
12 34567890123456789012345678901212345678901234567890123456789012123456 7
(C) one, a simpler piece, became widely Ode, on account of its difficulty in
12 34567890123456789012345678901212345678901234567890123456789012123456 7
12
2
34567890123456789012345678901212345678901234567890123456789012123456
known by pianists, but the other, a being performed.
34567890123456789012345678901212345678901234567890123456789012123456
7
7
1 more complex piece, remained (C) to be shorter than the Symphonic
12 34567890123456789012345678901212345678901234567890123456789012123456 7
12 34567890123456789012345678901212345678901234567890123456789012123456
largely unknown. Ode, because the Ode was not being 7
12 34567890123456789012345678901212345678901234567890123456789012123456 7
12 34567890123456789012345678901212345678901234567890123456789012123456
(D) one, featuring Mexican influences, performed. 7
1234567890123456789012345678901212345678901234567890123456789012123456
2 7
1234567890123456789012345678901212345678901234567890123456789012123456
was popular in Latin America, and (D) to demand even more of conductors 7
1234567890123456789012345678901212345678901234567890123456789012123456 7
1234567890123456789012345678901212345678901234567890123456789012123456
the other, a modernist piece, was and musicians attempting to play 7
1234567890123456789012345678901212345678901234567890123456789012123456 7
1234567890123456789012345678901212345678901234567890123456789012123456 77
34567890123456789012345678901212345678901234567890123456789012123456
popular in the United States. Copland’s music.
1
12 34567890123456789012345678901212345678901234567890123456789012123456
(E) both were initially acclaimed but (E) to reflect Copland’s new interest in 7
12 34567890123456789012345678901212345678901234567890123456789012123456 7
12 34567890123456789012345678901212345678901234567890123456789012123456 7
only one became part of Copland’s Latin America.
2 34567890123456789012345678901212345678901234567890123456789012123456
1234567890123456789012345678901212345678901234567890123456789012123456 7
1234567890123456789012345678901212345678901234567890123456789012123456
corpus of beloved works. 7
1234567890123456789012345678901212345678901234567890123456789012123456 7
1234567890123456789012345678901212345678901234567890123456789012123456 77
10. In the sentence beginning “Yet, in fact,
1234567890123456789012345678901212345678901234567890123456789012123456
1234567890123456789012345678901212345678901234567890123456789012123456
8. Koussevitzky is mentioned as an example these works. . .” in lines 37–43 [second 7
1234567890123456789012345678901212345678901234567890123456789012123456 7
1234567890123456789012345678901212345678901234567890123456789012123456 77
of a(n) paragraph], the author suggests that
1234567890123456789012345678901212345678901234567890123456789012123456 7
1 (A) American conductor who admired (A) parts of the Short Symphony simply
12 34567890123456789012345678901212345678901234567890123456789012123456 7
12 34567890123456789012345678901212345678901234567890123456789012123456
Copland’s work, but nonetheless weren’t melodic enough to engage 7
1234567890123456789012345678901212345678901234567890123456789012123456 7
1234567890123456789012345678901212345678901234567890123456789012123456
found some pieces too difficult to audiences. 7
12 34567890123456789012345678901212345678901234567890123456789012123456 7
1234567890123456789012345678901212345678901234567890123456789012123456 77
2 34567890123456789012345678901212345678901234567890123456789012123456
perform. (B) the Statements were too brief to
1234567890123456789012345678901212345678901234567890123456789012123456 7
1 (B) friend of Copland’s who agreed to warrant a formal performance.
12 34567890123456789012345678901212345678901234567890123456789012123456 7
12 34567890123456789012345678901212345678901234567890123456789012123456 7
perform his less popular works. (C) even those who admired Copland’s
2 34567890123456789012345678901212345678901234567890123456789012123456
1234567890123456789012345678901212345678901234567890123456789012123456 7
1234567890123456789012345678901212345678901234567890123456789012123456
(C) European composer who took issue work lost patience with the Short 7
1 7
1234567890123456789012345678901212345678901234567890123456789012123456 7
with the difficulty of Copland’s Symphony and Statements.
1234567890123456789012345678901212345678901234567890123456789012123456 7
12 34567890123456789012345678901212345678901234567890123456789012123456
early work. (D) the Statements and Short Symphony 7
12 34567890123456789012345678901212345678901234567890123456789012123456 7
12 34567890123456789012345678901212345678901234567890123456789012123456 7
(D) musician who appreciated Copland’s determined which performers were
12 34567890123456789012345678901212345678901234567890123456789012123456 7
1234567890123456789012345678901212345678901234567890123456789012123456
work but was unable to play it. truly excellent and which were 7
2 34567890123456789012345678901212345678901234567890123456789012123456
1234567890123456789012345678901212345678901234567890123456789012123456 7
1234567890123456789012345678901212345678901234567890123456789012123456
(E) European conductor who performed mediocre. 7
1234567890123456789012345678901212345678901234567890123456789012123456 7
1234567890123456789012345678901212345678901234567890123456789012123456 77
Copland’s work. (E) the Short Symphony had melodies
1
12 34567890123456789012345678901212345678901234567890123456789012123456
that were too quick to be played 7
12 34567890123456789012345678901212345678901234567890123456789012123456
even by famous musicians. 7
12 34567890123456789012345678901212345678901234567890123456789012123456 7
12 34567890123456789012345678901212345678901234567890123456789012123456 7
1234567890123456789012345678901212345678901234567890123456789012123456 7
1234567890123456789012345678901212345678901234567890123456789012123456
2 7
1 34567890123456789012345678901212345678901234567890123456789012123456 7
12 34567890123456789012345678901212345678901234567890123456789012123456 7
1234567890123456789012345678901212345678901234567890123456789012123456 7
12 34567890123456789012345678901212345678901234567890123456789012123456 7
1234567890123456789012345678901212345678901234567890123456789012123456 7
1234567890123456789012345678901212345678901234567890123456789012123456 7
1234567890123456789012345678901212345678901234567890123456789012123456 7
1234567890123456789012345678901212345678901234567890123456789012123456 7
12345678901234567890123456789012123456789012345678901234567890121234567
6 Copyright © 2005 Thomson Peterson’s, a part of The Thomson Corporaton
SAT is a registered trademark of the College Entrance Examination Board, which
was not involved in the production of and does not endorse this product.
12345678901234567890123456789012123456789012345678901234567890121234567
1234567890123456789012345678901212345678901234567890123456789012123456 7
12234567890123456789012345678901212345678901234567890123456789012123456 7
1 34567890123456789012345678901212345678901234567890123456789012123456
11. The author suggests that Copland believed round, we are still just barely skimming 7
12234567890123456789012345678901212345678901234567890123456789012123456 7
1234567890123456789012345678901212345678901234567890123456789012123456
Latin American music (10) the surface when you consider that the 7
1 34567890123456789012345678901212345678901234567890123456789012123456 7
12234567890123456789012345678901212345678901234567890123456789012123456
diameter
34567890123456789012345678901212345678901234567890123456789012123456
of the Earth is over 8,000 miles. 7
7
1 (A) was unfamiliar enough to a North
12234567890123456789012345678901212345678901234567890123456789012123456 So how much of the Earth can we see 7
1234567890123456789012345678901212345678901234567890123456789012123456
American audience that he needed to 7
1234567890123456789012345678901212345678901234567890123456789012123456
at one time? When you are standing on 7
1234567890123456789012345678901212345678901234567890123456789012123456
introduce them to it.
34567890123456789012345678901212345678901234567890123456789012123456
7
7
1 the ground, the horizon is a few miles
12234567890123456789012345678901212345678901234567890123456789012123456
(B) was different enough from North 7
1234567890123456789012345678901212345678901234567890123456789012123456
(15) away. When in a tall building, the 7
1234567890123456789012345678901212345678901234567890123456789012123456
American music that incorporating 7
1234567890123456789012345678901212345678901234567890123456789012123456
horizon
34567890123456789012345678901212345678901234567890123456789012123456
can be as far as about 40 miles. 7
1 From the International Space Station, the 7
aspects of it would make his music
12234567890123456789012345678901212345678901234567890123456789012123456 7
1234567890123456789012345678901212345678901234567890123456789012123456
unique and exciting. 7
1234567890123456789012345678901212345678901234567890123456789012123456
distance to the horizon is over 1,000 7
1234567890123456789012345678901212345678901234567890123456789012123456
(C) influenced and was influenced by
34567890123456789012345678901212345678901234567890123456789012123456
7
7
1 North American music.
miles. So from horizon to horizon, the
12234567890123456789012345678901212345678901234567890123456789012123456 7
1234567890123456789012345678901212345678901234567890123456789012123456
(D) primarily originated in Mexico
(20) section of the Earth you can see at any 7
1234567890123456789012345678901212345678901234567890123456789012123456
one time is a patch about 2,000 miles 7
1234567890123456789012345678901212345678901234567890123456789012123456
and Cuba. 7
34567890123456789012345678901212345678901234567890123456789012123456
1234567890123456789012345678901212345678901234567890123456789012123456
across, almost enough to see the entire 7
1234567890123456789012345678901212345678901234567890123456789012123456
(E) embodied the polar opposite of 7
1234567890123456789012345678901212345678901234567890123456789012123456
United States at once. It isn’t exactly 7
1234567890123456789012345678901212345678901234567890123456789012123456
modernist aesthetics. 7
1234567890123456789012345678901212345678901234567890123456789012123456
seeing the Earth like a big blue marble, 7
1234567890123456789012345678901212345678901234567890123456789012123456 77
1 12. The sentence beginning “His own use of (25) it’s more like having your face up against
12234567890123456789012345678901212345678901234567890123456789012123456 7
1234567890123456789012345678901212345678901234567890123456789012123456
a big blue beach ball. When I look out a 7
1234567890123456789012345678901212345678901234567890123456789012123456
Mexican material. . . “ in lines 71–75 sug- 7
1234567890123456789012345678901212345678901234567890123456789012123456
window that faces straight down, it is 7
1234567890123456789012345678901212345678901234567890123456789012123456
gests that the modernist music which also 7
1234567890123456789012345678901212345678901234567890123456789012123456
actually pretty hard to see the horizon— 7
1234567890123456789012345678901212345678901234567890123456789012123456
influenced Copland’s compositions was 7
1234567890123456789012345678901212345678901234567890123456789012123456
you need to get your face very close to 7
34567890123456789012345678901212345678901234567890123456789012123456
1234567890123456789012345678901212345678901234567890123456789012123456
(A) superior in quality to his Latin 7
1234567890123456789012345678901212345678901234567890123456789012123456
(30) the window. So what you see out a 7
1234567890123456789012345678901212345678901234567890123456789012123456
American influences. 7
1234567890123456789012345678901212345678901234567890123456789012123456 77
window like that is a moving patch of
1 (B) dry and passionless.
12234567890123456789012345678901212345678901234567890123456789012123456
ground (or water). 7
1234567890123456789012345678901212345678901234567890123456789012123456
(C) technically more challenging to 7
1 34567890123456789012345678901212345678901234567890123456789012123456 From the time a place on the ground 7
1234567890123456789012345678901212345678901234567890123456789012123456
perform. 7
12234567890123456789012345678901212345678901234567890123456789012123456
comes
34567890123456789012345678901212345678901234567890123456789012123456
into view until it disappears over 7
7
1234567890123456789012345678901212345678901234567890123456789012123456
(D) inaccessible but rewarding.
1234567890123456789012345678901212345678901234567890123456789012123456
(35) the horizon is only a few minutes, since 7
1 (E) outmoded by the 1930s. 7
12234567890123456789012345678901212345678901234567890123456789012123456
we are traveling 300 miles per minute. 7
1234567890123456789012345678901212345678901234567890123456789012123456 7
1234567890123456789012345678901212345678901234567890123456789012123456 77
34567890123456789012345678901212345678901234567890123456789012123456
When looking out a sideward facing
1234567890123456789012345678901212345678901234567890123456789012123456
1 Questions 13–20 are based on the following window, you can see the horizon of the 7
1234567890123456789012345678901212345678901234567890123456789012123456
passage. Earth against the black background of 7
1234567890123456789012345678901212345678901234567890123456789012123456 7
2 34567890123456789012345678901212345678901234567890123456789012123456
1234567890123456789012345678901212345678901234567890123456789012123456
(40) space. The horizon is distinctly curved. 7
1234567890123456789012345678901212345678901234567890123456789012123456
The following passage was written by Ed Lu, an 7
1234567890123456789012345678901212345678901234567890123456789012123456
The edge of the Earth isn’t distinct but 7
1234567890123456789012345678901212345678901234567890123456789012123456
astronaut, while a crew member of the Interna- 7
1 tional Space Station. rather is smeared out due to the atmo- 7
2 34567890123456789012345678901212345678901234567890123456789012123456
1234567890123456789012345678901212345678901234567890123456789012123456 7
1234567890123456789012345678901212345678901234567890123456789012123456
sphere. Here you can get a feel for how 7
1234567890123456789012345678901212345678901234567890123456789012123456 7
1234567890123456789012345678901212345678901234567890123456789012123456
Line Whenever I get a chance, I spend time relatively thin the atmosphere is com- 7
1 7
1234567890123456789012345678901212345678901234567890123456789012123456 77
2 34567890123456789012345678901212345678901234567890123456789012123456
just observing the planet below. It turns (45) pared to the Earth as a whole. I can see
1234567890123456789012345678901212345678901234567890123456789012123456
that the width of the atmosphere on the 7
1234567890123456789012345678901212345678901234567890123456789012123456
out you can see a lot more from up here
1 than you might expect. First off, we horizon is about 1 degree in angular size, 7
1234567890123456789012345678901212345678901234567890123456789012123456 7
2 34567890123456789012345678901212345678901234567890123456789012123456
1234567890123456789012345678901212345678901234567890123456789012123456 7
1 (5) aren’t as far away as some people which is about the width of your index 7
12234567890123456789012345678901212345678901234567890123456789012123456 7
1 34567890123456789012345678901212345678901234567890123456789012123456 7
think—our orbit is only about 240 miles finger held out at arms length. There
12234567890123456789012345678901212345678901234567890123456789012123456
above the surface of the Earth. While this 7
1 34567890123456789012345678901212345678901234567890123456789012123456
(50) really isn’t a sharp boundary to the 7
1234567890123456789012345678901212345678901234567890123456789012123456
is high enough to see that the Earth is 7
1234567890123456789012345678901212345678901234567890123456789012123456 77
atmosphere, but it gets rapidly thinner
1234567890123456789012345678901212345678901234567890123456789012123456
12345678901234567890123456789012123456789012345678901234567890121234567
Copyright © 2005 Thomson Peterson’s, a part of The Thomson Corporaton 7
SAT is a registered trademark of the College Entrance Examination Board, which
was not involved in the production of and does not endorse this product.
12345678901234567890123456789012123456789012345678901234567890121234567
1234567890123456789012345678901212345678901234567890123456789012123456 7
12 34567890123456789012345678901212345678901234567890123456789012123456 7
1234567890123456789012345678901212345678901234567890123456789012123456
the higher you go. Not many airplanes 14. The second half of the second paragraph 7
12 34567890123456789012345678901212345678901234567890123456789012123456 7
12 34567890123456789012345678901212345678901234567890123456789012123456
can fly higher than about 10 miles, and is primarily concerned with 7
1234567890123456789012345678901212345678901234567890123456789012123456 7
12
2
34567890123456789012345678901212345678901234567890123456789012123456
the highest mountains are only about 6
34567890123456789012345678901212345678901234567890123456789012123456
7
7
1 (55) miles high. Above about 30 miles there is (A) how one’s location affects one’s
12 34567890123456789012345678901212345678901234567890123456789012123456 7
12 34567890123456789012345678901212345678901234567890123456789012123456 visual horizon. 7
12 34567890123456789012345678901212345678901234567890123456789012123456
very little air to speak of, but at night 7
12
2
34567890123456789012345678901212345678901234567890123456789012123456
34567890123456789012345678901212345678901234567890123456789012123456
(B) the thickness and density of the 7
7
1 you can see a faint glow from what little
12 34567890123456789012345678901212345678901234567890123456789012123456 atmosphere. 7
12 34567890123456789012345678901212345678901234567890123456789012123456
air there is at that height. 7
12 34567890123456789012345678901212345678901234567890123456789012123456 (C) the speed of the International Space 7
12
2
34567890123456789012345678901212345678901234567890123456789012123456
Since we orbit at an altitude about 40
34567890123456789012345678901212345678901234567890123456789012123456
7
7
1 (60) times higher than the tallest mountain, Station.
12 34567890123456789012345678901212345678901234567890123456789012123456 7
12 34567890123456789012345678901212345678901234567890123456789012123456 (D) the visual horizon from atop a 7
12 34567890123456789012345678901212345678901234567890123456789012123456
the surface of the Earth is pretty smooth 7
12
2
34567890123456789012345678901212345678901234567890123456789012123456
34567890123456789012345678901212345678901234567890123456789012123456
tall building. 7
1 (E) being able to see all the Earth at once. 7
from our perspective. A good way to
12 34567890123456789012345678901212345678901234567890123456789012123456 7
12 34567890123456789012345678901212345678901234567890123456789012123456
imagine our view is to stand up and look 7
12 34567890123456789012345678901212345678901234567890123456789012123456
down at your feet. Imagine that your eyes 7
12 34567890123456789012345678901212345678901234567890123456789012123456 7
15. The author compares the view of the
2 34567890123456789012345678901212345678901234567890123456789012123456
1234567890123456789012345678901212345678901234567890123456789012123456
(65) are where the International Space Station Earth from a downward-facing window in 7
7
1234567890123456789012345678901212345678901234567890123456789012123456
1234567890123456789012345678901212345678901234567890123456789012123456
is orbiting, and the floor is the surface of 7
1234567890123456789012345678901212345678901234567890123456789012123456 the International Space Station to 7
1234567890123456789012345678901212345678901234567890123456789012123456
the Earth. The atmosphere would be 7
1234567890123456789012345678901212345678901234567890123456789012123456 7
1 about 6 inches high, and the height of the (A) holding a blue marble at arm’s 7
12 34567890123456789012345678901212345678901234567890123456789012123456 7
12 34567890123456789012345678901212345678901234567890123456789012123456
tallest mountain is less than 2 inches, or length. 7
12 34567890123456789012345678901212345678901234567890123456789012123456 7
1234567890123456789012345678901212345678901234567890123456789012123456 77
2 34567890123456789012345678901212345678901234567890123456789012123456
(70) about the height of the tops of your feet. (B) having your face up-close to a big
1234567890123456789012345678901212345678901234567890123456789012123456
1234567890123456789012345678901212345678901234567890123456789012123456
Almost all of the people below you blue beach ball. 7
1234567890123456789012345678901212345678901234567890123456789012123456 (C) looking at the tips of your shoes 7
1234567890123456789012345678901212345678901234567890123456789012123456 77
would live in the first one quarter of an
1234567890123456789012345678901212345678901234567890123456789012123456 when standing up.
1234567890123456789012345678901212345678901234567890123456789012123456
inch from the floor. The horizon of the 7
1234567890123456789012345678901212345678901234567890123456789012123456 (D) looking at an object that is on the 7
1234567890123456789012345678901212345678901234567890123456789012123456 77
Earth is a little over 20 feet away from
1 (75) where you are standing. If you are ground fifteen feet away when you
12 34567890123456789012345678901212345678901234567890123456789012123456 7
12 34567890123456789012345678901212345678901234567890123456789012123456 are standing up. 7
1234567890123456789012345678901212345678901234567890123456789012123456
standing on top of Denver, then about 15 7
1234567890123456789012345678901212345678901234567890123456789012123456 (E) the view from a high-flying plane. 7
12
2
34567890123456789012345678901212345678901234567890123456789012123456 7
feet to one side you can see San Fran-
34567890123456789012345678901212345678901234567890123456789012123456 7
1234567890123456789012345678901212345678901234567890123456789012123456
1234567890123456789012345678901212345678901234567890123456789012123456
cisco, and about 15 feet to the other side 7
1 16. In the passage, the author contrasts the 7
12 34567890123456789012345678901212345678901234567890123456789012123456
you can see Chicago. 7
12 34567890123456789012345678901212345678901234567890123456789012123456 7view from a window looking “straight
2 34567890123456789012345678901212345678901234567890123456789012123456
1234567890123456789012345678901212345678901234567890123456789012123456 7
1234567890123456789012345678901212345678901234567890123456789012123456 down” with the view from 7
1 13. The primary purpose of this passage is to 7
1234567890123456789012345678901212345678901234567890123456789012123456 (A) the observational deck. 7
1234567890123456789012345678901212345678901234567890123456789012123456 7
2 34567890123456789012345678901212345678901234567890123456789012123456
1234567890123456789012345678901212345678901234567890123456789012123456
(A) provide a layperson’s account of the (B) a sideward-facing window. 7
1234567890123456789012345678901212345678901234567890123456789012123456 7
1234567890123456789012345678901212345678901234567890123456789012123456
Space Station’s motion over the Earth. (C) a passenger airliner. 7
1234567890123456789012345678901212345678901234567890123456789012123456 7
1 (B) explain the relationship between the (D) a window looking “straight up.” 7
2 34567890123456789012345678901212345678901234567890123456789012123456
1234567890123456789012345678901212345678901234567890123456789012123456 7
1234567890123456789012345678901212345678901234567890123456789012123456
diameter of the Earth and the (E) the circular windows on the 7
1234567890123456789012345678901212345678901234567890123456789012123456 7
1234567890123456789012345678901212345678901234567890123456789012123456
thickness of the Earth’s atmosphere. space station. 7
1 7
1234567890123456789012345678901212345678901234567890123456789012123456 77
2 34567890123456789012345678901212345678901234567890123456789012123456
(C) answer the imagined question, “What
1234567890123456789012345678901212345678901234567890123456789012123456
do astronauts see from space?”
7
1234567890123456789012345678901212345678901234567890123456789012123456 7
1234567890123456789012345678901212345678901234567890123456789012123456
(D) give a glimpse of some of the daily
7
1
2 34567890123456789012345678901212345678901234567890123456789012123456
1234567890123456789012345678901212345678901234567890123456789012123456
activities of astronauts in space. 7
1 7
12 34567890123456789012345678901212345678901234567890123456789012123456
(E) discuss the thickness and composi- 7
1234567890123456789012345678901212345678901234567890123456789012123456 7
12
2
34567890123456789012345678901212345678901234567890123456789012123456
tion of the atmosphere.
34567890123456789012345678901212345678901234567890123456789012123456
7
7
1
1234567890123456789012345678901212345678901234567890123456789012123456 7
1234567890123456789012345678901212345678901234567890123456789012123456 7
1234567890123456789012345678901212345678901234567890123456789012123456 7
12345678901234567890123456789012123456789012345678901234567890121234567
8 Copyright © 2005 Thomson Peterson’s, a part of The Thomson Corporaton
SAT is a registered trademark of the College Entrance Examination Board, which
was not involved in the production of and does not endorse this product.
12345678901234567890123456789012123456789012345678901234567890121234567
1234567890123456789012345678901212345678901234567890123456789012123456 7
12234567890123456789012345678901212345678901234567890123456789012123456 7
1 34567890123456789012345678901212345678901234567890123456789012123456
17. The “faint glow” at night that the author 19. The tone of the passage is best described as 7
12234567890123456789012345678901212345678901234567890123456789012123456 7
1234567890123456789012345678901212345678901234567890123456789012123456
speaks of in the passage comes from 7
1 34567890123456789012345678901212345678901234567890123456789012123456 7
(A) fairly technical.
12234567890123456789012345678901212345678901234567890123456789012123456 7
1 34567890123456789012345678901212345678901234567890123456789012123456
(A) low-lying atmosphere. B. highly professional. 7
12234567890123456789012345678901212345678901234567890123456789012123456 7
1234567890123456789012345678901212345678901234567890123456789012123456
(B) the outer edges of the atmosphere. (C) refreshingly irreverent. 7
1234567890123456789012345678901212345678901234567890123456789012123456 7
1234567890123456789012345678901212345678901234567890123456789012123456
(C) the eastern horizon of the Earth just (D) engagingly
34567890123456789012345678901212345678901234567890123456789012123456
conversational. 7
7
1
12234567890123456789012345678901212345678901234567890123456789012123456
before sunrise. (E) lyrically impassioned. 7
1234567890123456789012345678901212345678901234567890123456789012123456 7
1234567890123456789012345678901212345678901234567890123456789012123456
(D) haze from foreign particulates in the 7
1234567890123456789012345678901212345678901234567890123456789012123456
20. From the passage as a whole, it can be
34567890123456789012345678901212345678901234567890123456789012123456
7
7
1 atmosphere.
12234567890123456789012345678901212345678901234567890123456789012123456
inferred that the astronauts’ training 7
1234567890123456789012345678901212345678901234567890123456789012123456
(E) the sun reflecting off aircraft in the 7
1234567890123456789012345678901212345678901234567890123456789012123456 7
1234567890123456789012345678901212345678901234567890123456789012123456
high atmosphere. (A) did
34567890123456789012345678901212345678901234567890123456789012123456
not prepare them for their free 7
7
1
12234567890123456789012345678901212345678901234567890123456789012123456
time in space. 7
1234567890123456789012345678901212345678901234567890123456789012123456
18. In the last paragraph the author provides 7
1234567890123456789012345678901212345678901234567890123456789012123456
(B) included a great deal of zero-gravity 7
1234567890123456789012345678901212345678901234567890123456789012123456
the thought exercise with the reader’s 7
34567890123456789012345678901212345678901234567890123456789012123456
1234567890123456789012345678901212345678901234567890123456789012123456exercises. 7
1234567890123456789012345678901212345678901234567890123456789012123456
height primarily to 7
1234567890123456789012345678901212345678901234567890123456789012123456 77
(C) was more physical than technical.
1234567890123456789012345678901212345678901234567890123456789012123456
1234567890123456789012345678901212345678901234567890123456789012123456
(A) demonstrate the distance from (D) involved a strong background 7
1234567890123456789012345678901212345678901234567890123456789012123456 7
1 Denver to San Francisco. in math. 7
12234567890123456789012345678901212345678901234567890123456789012123456 7
1234567890123456789012345678901212345678901234567890123456789012123456
(B) give the reader a concrete sense of (E) focused on the astronauts’ communi- 7
1234567890123456789012345678901212345678901234567890123456789012123456 7
1234567890123456789012345678901212345678901234567890123456789012123456 77
34567890123456789012345678901212345678901234567890123456789012123456
the proportions involved in looking cation procedures and abilities.
1234567890123456789012345678901212345678901234567890123456789012123456
1234567890123456789012345678901212345678901234567890123456789012123456
down from the space station. 7
1234567890123456789012345678901212345678901234567890123456789012123456
(C) point out that most humans live at a 7
1234567890123456789012345678901212345678901234567890123456789012123456 77
1234567890123456789012345678901212345678901234567890123456789012123456
low altitude relative to the height of
1234567890123456789012345678901212345678901234567890123456789012123456 7
1234567890123456789012345678901212345678901234567890123456789012123456
the atmosphere. 7
1234567890123456789012345678901212345678901234567890123456789012123456 77
1 (D) illustrate the expansion of one’s
12234567890123456789012345678901212345678901234567890123456789012123456 7
1234567890123456789012345678901212345678901234567890123456789012123456
horizon at high altitudes. 7
1 34567890123456789012345678901212345678901234567890123456789012123456 7
1234567890123456789012345678901212345678901234567890123456789012123456
(E) provide visual details of his activities 7
12234567890123456789012345678901212345678901234567890123456789012123456 7
1234567890123456789012345678901212345678901234567890123456789012123456 77
34567890123456789012345678901212345678901234567890123456789012123456
in space.
1234567890123456789012345678901212345678901234567890123456789012123456 7
1
12234567890123456789012345678901212345678901234567890123456789012123456 7
1234567890123456789012345678901212345678901234567890123456789012123456 7
1234567890123456789012345678901212345678901234567890123456789012123456 77
34567890123456789012345678901212345678901234567890123456789012123456
1234567890123456789012345678901212345678901234567890123456789012123456 7
1
1234567890123456789012345678901212345678901234567890123456789012123456 7
1234567890123456789012345678901212345678901234567890123456789012123456 7
12234567890123456789012345678901212345678901234567890123456789012123456 7
1234567890123456789012345678901212345678901234567890123456789012123456 7
1234567890123456789012345678901212345678901234567890123456789012123456 7
1234567890123456789012345678901212345678901234567890123456789012123456 7
1 34567890123456789012345678901212345678901234567890123456789012123456 7
1234567890123456789012345678901212345678901234567890123456789012123456 77
2 34567890123456789012345678901212345678901234567890123456789012123456
1234567890123456789012345678901212345678901234567890123456789012123456 7
1234567890123456789012345678901212345678901234567890123456789012123456 7
1234567890123456789012345678901212345678901234567890123456789012123456 7
1
12234567890123456789012345678901212345678901234567890123456789012123456 7
1234567890123456789012345678901212345678901234567890123456789012123456 7
1234567890123456789012345678901212345678901234567890123456789012123456 7
1234567890123456789012345678901212345678901234567890123456789012123456 7
1 34567890123456789012345678901212345678901234567890123456789012123456 7
1234567890123456789012345678901212345678901234567890123456789012123456 77
2 34567890123456789012345678901212345678901234567890123456789012123456
1
12234567890123456789012345678901212345678901234567890123456789012123456 7
1 34567890123456789012345678901212345678901234567890123456789012123456 7
12234567890123456789012345678901212345678901234567890123456789012123456 7
STOP
1 34567890123456789012345678901212345678901234567890123456789012123456
Do not proceed to the next section until time is up.
1234567890123456789012345678901212345678901234567890123456789012123456 7
7
1234567890123456789012345678901212345678901234567890123456789012123456 7
1234567890123456789012345678901212345678901234567890123456789012123456 7
12345678901234567890123456789012123456789012345678901234567890121234567
Copyright © 2005 Thomson Peterson’s, a part of The Thomson Corporaton 9
SAT is a registered trademark of the College Entrance Examination Board, which
was not involved in the production of and does not endorse this product.
Section 2
21 Questions j Time—25 Minutes

12345678901234567890123456789012123456789012345678901234567890121234567
12 34567890123456789012345678901212345678901234567890123456789012123456 7
12 34567890123456789012345678901212345678901234567890123456789012123456 7
1 34567890123456789012345678901212345678901234567890123456789012123456 7
2
12 34567890123456789012345678901212345678901234567890123456789012123456 7
12 34567890123456789012345678901212345678901234567890123456789012123456
Directions: Solve the following problems using any available space on the page for scratchwork. 7
12 34567890123456789012345678901212345678901234567890123456789012123456
Mark the letter of your choice on the answer sheet that best corresponds to the correct answer. 7
12 34567890123456789012345678901212345678901234567890123456789012123456 7
1234567890123456789012345678901212345678901234567890123456789012123456 77
2 34567890123456789012345678901212345678901234567890123456789012123456
1234567890123456789012345678901212345678901234567890123456789012123456
Notes:
1234567890123456789012345678901212345678901234567890123456789012123456 7
1234567890123456789012345678901212345678901234567890123456789012123456
1. You may use a calculator. All of the numbers used are real numbers. 7
1234567890123456789012345678901212345678901234567890123456789012123456 77
1234567890123456789012345678901212345678901234567890123456789012123456 7
1 2. You may use the figures that accompany the problems to help you find the solution. Unless
12 34567890123456789012345678901212345678901234567890123456789012123456 7
12 34567890123456789012345678901212345678901234567890123456789012123456 7
the instructions say that a figure is not drawn to scale, assume that it has been drawn
12 34567890123456789012345678901212345678901234567890123456789012123456 7
12 34567890123456789012345678901212345678901234567890123456789012123456
accurately. Each figure lies in a plane unless the instructions say otherwise. 7
12 34567890123456789012345678901212345678901234567890123456789012123456 7
12 34567890123456789012345678901212345678901234567890123456789012123456 7
12 34567890123456789012345678901212345678901234567890123456789012123456 7
12 34567890123456789012345678901212345678901234567890123456789012123456 7
1234567890123456789012345678901212345678901234567890123456789012123456
2 7
Reference Information

2s
1234567890123456789012345678901212345678901234567890123456789012123456
r w 2x
45
7
1234567890123456789012345678901212345678901234567890123456789012123456
h h
r
c x s 60
7
1234567890123456789012345678901212345678901234567890123456789012123456 h b 7
1 34567890123456789012345678901212345678901234567890123456789012123456 7
30
45
w
2 34567890123456789012345678901212345678901234567890123456789012123456
1234567890123456789012345678901212345678901234567890123456789012123456 7
1234567890123456789012345678901212345678901234567890123456789012123456
A 5 pr2 b a 3x s 7
1 1 7
1234567890123456789012345678901212345678901234567890123456789012123456 7
2 2 2 2
C 5 2pr A 5 ,w A 5 bh V 5 ,wh V 5 pr h c 5 a 1 b Special Right Triangles
12
2
34567890123456789012345678901212345678901234567890123456789012123456 7
2
34567890123456789012345678901212345678901234567890123456789012123456 7
1234567890123456789012345678901212345678901234567890123456789012123456
The number of degrees of arc in a circle is 360.
1234567890123456789012345678901212345678901234567890123456789012123456 7
1 The measure in degrees of a straight angle is 180. 7
12 34567890123456789012345678901212345678901234567890123456789012123456 7
12 34567890123456789012345678901212345678901234567890123456789012123456 7
The sum of the measures in degrees of the angles of a triangle is 180.
1234567890123456789012345678901212345678901234567890123456789012123456 77
2 34567890123456789012345678901212345678901234567890123456789012123456
1234567890123456789012345678901212345678901234567890123456789012123456 7
1
1234567890123456789012345678901212345678901234567890123456789012123456 7
1234567890123456789012345678901212345678901234567890123456789012123456
1. If x 1 4y 5 3 and x 5 2y, then y 5 2. It takes Michael three hours to mow 2d 7
2 34567890123456789012345678901212345678901234567890123456789012123456
acres. If Michael mows d acres at the same 7
1234567890123456789012345678901212345678901234567890123456789012123456
1234567890123456789012345678901212345678901234567890123456789012123456 7
1234567890123456789012345678901212345678901234567890123456789012123456
(A) 21 7
1234567890123456789012345678901212345678901234567890123456789012123456
(B) 0
rate, how many minutes would it take? 7
1 7
2 34567890123456789012345678901212345678901234567890123456789012123456
1234567890123456789012345678901212345678901234567890123456789012123456
(C) 1 (A) 75 7
1234567890123456789012345678901212345678901234567890123456789012123456 7
1234567890123456789012345678901212345678901234567890123456789012123456
(D) 2 (B) 90 7
1234567890123456789012345678901212345678901234567890123456789012123456 77
1 (E) 3 (C) 100
12 34567890123456789012345678901212345678901234567890123456789012123456 7
12 34567890123456789012345678901212345678901234567890123456789012123456
(D) 120 7
12 34567890123456789012345678901212345678901234567890123456789012123456 7
12 34567890123456789012345678901212345678901234567890123456789012123456
(E) 150 7
1234567890123456789012345678901212345678901234567890123456789012123456 7
1234567890123456789012345678901212345678901234567890123456789012123456 77
2 34567890123456789012345678901212345678901234567890123456789012123456
1
12 34567890123456789012345678901212345678901234567890123456789012123456 7
1234567890123456789012345678901212345678901234567890123456789012123456 7
12 34567890123456789012345678901212345678901234567890123456789012123456 7
1234567890123456789012345678901212345678901234567890123456789012123456 7
1234567890123456789012345678901212345678901234567890123456789012123456 7
1234567890123456789012345678901212345678901234567890123456789012123456 7
1234567890123456789012345678901212345678901234567890123456789012123456 7
12345678901234567890123456789012123456789012345678901234567890121234567
10 Copyright © 2005 Thomson Peterson’s, a part of The Thomson Corporaton
SAT is a registered trademark of the College Entrance Examination Board, which
was not involved in the production of and does not endorse this product.
12345678901234567890123456789012123456789012345678901234567890121234567
1234567890123456789012345678901212345678901234567890123456789012123456 7
12234567890123456789012345678901212345678901234567890123456789012123456 7
1 34567890123456789012345678901212345678901234567890123456789012123456
3. If 3f 1 15 5 27, then 3f 2 6 5 7. If =x2 5 2x 1 3, then x 5 7
12234567890123456789012345678901212345678901234567890123456789012123456 7
1234567890123456789012345678901212345678901234567890123456789012123456 7
1 34567890123456789012345678901212345678901234567890123456789012123456 7
(A) 6 (A) 2x
12234567890123456789012345678901212345678901234567890123456789012123456 7
1 34567890123456789012345678901212345678901234567890123456789012123456
(B) 10 (B) 23 7
12234567890123456789012345678901212345678901234567890123456789012123456 7
1234567890123456789012345678901212345678901234567890123456789012123456
(C) 12 (C) x 7
1234567890123456789012345678901212345678901234567890123456789012123456 7
1234567890123456789012345678901212345678901234567890123456789012123456
(D) 17 (D) 21
34567890123456789012345678901212345678901234567890123456789012123456
7
7
1
12234567890123456789012345678901212345678901234567890123456789012123456
(E) 20 (E) 4 7
1234567890123456789012345678901212345678901234567890123456789012123456 7
1234567890123456789012345678901212345678901234567890123456789012123456 7
1234567890123456789012345678901212345678901234567890123456789012123456
4. 8. If |x 1 1| . |y| then which of the following 7
34567890123456789012345678901212345678901234567890123456789012123456 7
1
12234567890123456789012345678901212345678901234567890123456789012123456
expresses the relationship between 7
1234567890123456789012345678901212345678901234567890123456789012123456 7
1234567890123456789012345678901212345678901234567890123456789012123456
x and y? 7
1234567890123456789012345678901212345678901234567890123456789012123456
34567890123456789012345678901212345678901234567890123456789012123456
7
7
1
12234567890123456789012345678901212345678901234567890123456789012123456
(A) x 1 1 . y 7
1234567890123456789012345678901212345678901234567890123456789012123456 7
1234567890123456789012345678901212345678901234567890123456789012123456
(B) x 1 1 . y 7
1234567890123456789012345678901212345678901234567890123456789012123456 7
1234567890123456789012345678901212345678901234567890123456789012123456 77
34567890123456789012345678901212345678901234567890123456789012123456
(C) x , y
1234567890123456789012345678901212345678901234567890123456789012123456
1234567890123456789012345678901212345678901234567890123456789012123456
What is the value of x 1 y? (D) x . y 7
1234567890123456789012345678901212345678901234567890123456789012123456 7
1234567890123456789012345678901212345678901234567890123456789012123456
(A) 7 (E) It cannot be determined. 7
1234567890123456789012345678901212345678901234567890123456789012123456 77
1 (B) 7=2
12234567890123456789012345678901212345678901234567890123456789012123456
9. 75% of 104 is the same value as 60% of 7
1234567890123456789012345678901212345678901234567890123456789012123456
= 7
1234567890123456789012345678901212345678901234567890123456789012123456
(C) 7 3 what number? 7
1234567890123456789012345678901212345678901234567890123456789012123456 77
34567890123456789012345678901212345678901234567890123456789012123456
1234567890123456789012345678901212345678901234567890123456789012123456
(D) 14
1234567890123456789012345678901212345678901234567890123456789012123456
(A) 130 7
1234567890123456789012345678901212345678901234567890123456789012123456 7
1234567890123456789012345678901212345678901234567890123456789012123456 77
(E) It cannot be determined. (B) 133
1234567890123456789012345678901212345678901234567890123456789012123456
1234567890123456789012345678901212345678901234567890123456789012123456
(C) 136 7
1234567890123456789012345678901212345678901234567890123456789012123456
5. Steve bought a snack and a drink for 7
1234567890123456789012345678901212345678901234567890123456789012123456 77
(D) 140
1 $1.30. If the snack costs twenty cents less
12234567890123456789012345678901212345678901234567890123456789012123456
(E) 144 7
1234567890123456789012345678901212345678901234567890123456789012123456
than the drink, how much does the 7
1 34567890123456789012345678901212345678901234567890123456789012123456 7
1234567890123456789012345678901212345678901234567890123456789012123456
drink cost? 10. If 3y 2 x 5 12 is the equation of a line, 7
12234567890123456789012345678901212345678901234567890123456789012123456
34567890123456789012345678901212345678901234567890123456789012123456
7
7
1234567890123456789012345678901212345678901234567890123456789012123456
what is twice the value of this line’s
1234567890123456789012345678901212345678901234567890123456789012123456
(A) $0.50 7
1 y-intercept? 7
12234567890123456789012345678901212345678901234567890123456789012123456
(B) $0.55 7
1234567890123456789012345678901212345678901234567890123456789012123456 7
1234567890123456789012345678901212345678901234567890123456789012123456 77
34567890123456789012345678901212345678901234567890123456789012123456
(C) $0.65 (A) 22
1234567890123456789012345678901212345678901234567890123456789012123456 7
1 (D) $0.75 (B) 2
1234567890123456789012345678901212345678901234567890123456789012123456 7
1234567890123456789012345678901212345678901234567890123456789012123456 7
(E) $0.80 (C) 4
2 34567890123456789012345678901212345678901234567890123456789012123456
1234567890123456789012345678901212345678901234567890123456789012123456 7
1234567890123456789012345678901212345678901234567890123456789012123456
(D) 8 7
1234567890123456789012345678901212345678901234567890123456789012123456
x
6. If f(x) 5 x 1 x , when x 5 2, f(2x) 5 7
1234567890123456789012345678901212345678901234567890123456789012123456 77
(E) 24
1
12234567890123456789012345678901212345678901234567890123456789012123456
(A) 6 7
1234567890123456789012345678901212345678901234567890123456789012123456 7
1234567890123456789012345678901212345678901234567890123456789012123456
(B) 20 7
1234567890123456789012345678901212345678901234567890123456789012123456 7
1 34567890123456789012345678901212345678901234567890123456789012123456 7
(C) 200
2 34567890123456789012345678901212345678901234567890123456789012123456
1234567890123456789012345678901212345678901234567890123456789012123456 7
1234567890123456789012345678901212345678901234567890123456789012123456
(D) 260 7
1234567890123456789012345678901212345678901234567890123456789012123456 7
1234567890123456789012345678901212345678901234567890123456789012123456 77
(E) 4096
1
12234567890123456789012345678901212345678901234567890123456789012123456 7
1 34567890123456789012345678901212345678901234567890123456789012123456 7
12234567890123456789012345678901212345678901234567890123456789012123456 7
1 34567890123456789012345678901212345678901234567890123456789012123456 7
12234567890123456789012345678901212345678901234567890123456789012123456 7
1 34567890123456789012345678901212345678901234567890123456789012123456 7
1234567890123456789012345678901212345678901234567890123456789012123456 7
1234567890123456789012345678901212345678901234567890123456789012123456 7
1234567890123456789012345678901212345678901234567890123456789012123456 7
12345678901234567890123456789012123456789012345678901234567890121234567
Copyright © 2005 Thomson Peterson’s, a part of The Thomson Corporaton 11
SAT is a registered trademark of the College Entrance Examination Board, which
was not involved in the production of and does not endorse this product.
12345678901234567890123456789012123456789012345678901234567890121234567
1234567890123456789012345678901212345678901234567890123456789012123456 7
12 34567890123456789012345678901212345678901234567890123456789012123456 7
1234567890123456789012345678901212345678901234567890123456789012123456
11. 14. If A . B . C . D . E, and each is a digit 7
12 34567890123456789012345678901212345678901234567890123456789012123456
p q
7
12 34567890123456789012345678901212345678901234567890123456789012123456
ᐉ 1 through 9, then ABCD 2 { (ABCD) is 7
1234567890123456789012345678901212345678901234567890123456789012123456 7
12 34567890123456789012345678901212345678901234567890123456789012123456 7
1234567890123456789012345678901212345678901234567890123456789012123456
(A) less than zero. 7
12 34567890123456789012345678901212345678901234567890123456789012123456 7
12 34567890123456789012345678901212345678901234567890123456789012123456 7
(B) between zero and 100.
12 34567890123456789012345678901212345678901234567890123456789012123456 7
12
2
34567890123456789012345678901212345678901234567890123456789012123456
(C) between 100 and 500.
34567890123456789012345678901212345678901234567890123456789012123456
7
7
1
12 34567890123456789012345678901212345678901234567890123456789012123456
(D) between 500 and 1000. 7
12 34567890123456789012345678901212345678901234567890123456789012123456 7
12 34567890123456789012345678901212345678901234567890123456789012123456 7
(E) more than 1000.
12 34567890123456789012345678901212345678901234567890123456789012123456 7
1234567890123456789012345678901212345678901234567890123456789012123456 7
12 34567890123456789012345678901212345678901234567890123456789012123456
m 15. 7
12 34567890123456789012345678901212345678901234567890123456789012123456 7
12 34567890123456789012345678901212345678901234567890123456789012123456 7
12 34567890123456789012345678901212345678901234567890123456789012123456 7
1 34567890123456789012345678901212345678901234567890123456789012123456
2 7
12 34567890123456789012345678901212345678901234567890123456789012123456
Given the figure above, which of the 7
12 34567890123456789012345678901212345678901234567890123456789012123456 7
12 34567890123456789012345678901212345678901234567890123456789012123456 7
following must be a true statement?
12 34567890123456789012345678901212345678901234567890123456789012123456 7
12 34567890123456789012345678901212345678901234567890123456789012123456
(A) x . y 7
12 34567890123456789012345678901212345678901234567890123456789012123456 7
12 34567890123456789012345678901212345678901234567890123456789012123456
(B) x , y 7
12 34567890123456789012345678901212345678901234567890123456789012123456 7
12 34567890123456789012345678901212345678901234567890123456789012123456
(C) x 5 y 7
12 34567890123456789012345678901212345678901234567890123456789012123456 7
1234567890123456789012345678901212345678901234567890123456789012123456
(D) x 1 y 5 90 7
12 34567890123456789012345678901212345678901234567890123456789012123456 7
12 34567890123456789012345678901212345678901234567890123456789012123456
(E) x 2 y 5 90 7
12 34567890123456789012345678901212345678901234567890123456789012123456 7
1234567890123456789012345678901212345678901234567890123456789012123456 77
2 34567890123456789012345678901212345678901234567890123456789012123456
1234567890123456789012345678901212345678901234567890123456789012123456 7
1234567890123456789012345678901212345678901234567890123456789012123456
1234567890123456789012345678901212345678901234567890123456789012123456
Questions 12–14 refer to the following 7
1234567890123456789012345678901212345678901234567890123456789012123456 7
1234567890123456789012345678901212345678901234567890123456789012123456 77
definition.
1234567890123456789012345678901212345678901234567890123456789012123456
1234567890123456789012345678901212345678901234567890123456789012123456
Let { m be defined for any positive integer m as 7
1234567890123456789012345678901212345678901234567890123456789012123456 7
1 the number obtained when the first and last digit 7
2 34567890123456789012345678901212345678901234567890123456789012123456
1234567890123456789012345678901212345678901234567890123456789012123456 7
1234567890123456789012345678901212345678901234567890123456789012123456
of m switch places. If RT 5 18, what is the area of the 7
1 above circle? 7
1234567890123456789012345678901212345678901234567890123456789012123456
For example { 4 5 4, { 35 5 53, and 7
12
2
34567890123456789012345678901212345678901234567890123456789012123456 7
34567890123456789012345678901212345678901234567890123456789012123456 7
1234567890123456789012345678901212345678901234567890123456789012123456
{ 2003 5 3002. (A) 6p
1234567890123456789012345678901212345678901234567890123456789012123456 7
1 (B) 12p 7
12 34567890123456789012345678901212345678901234567890123456789012123456 7
12 34567890123456789012345678901212345678901234567890123456789012123456
12. { 2323 2 { 2321 5 (C) 36p 7
2 34567890123456789012345678901212345678901234567890123456789012123456
1234567890123456789012345678901212345678901234567890123456789012123456 7
1234567890123456789012345678901212345678901234567890123456789012123456
(D) 81p 7
1 (A) 2 7
1234567890123456789012345678901212345678901234567890123456789012123456
(E) 324p 7
1234567890123456789012345678901212345678901234567890123456789012123456 7
(B) 20
2 34567890123456789012345678901212345678901234567890123456789012123456
1234567890123456789012345678901212345678901234567890123456789012123456 7
1234567890123456789012345678901212345678901234567890123456789012123456
(C) 200 7
1234567890123456789012345678901212345678901234567890123456789012123456 7
1234567890123456789012345678901212345678901234567890123456789012123456 77
(D) 2000
1
12 34567890123456789012345678901212345678901234567890123456789012123456
(E) 20,000 7
12 34567890123456789012345678901212345678901234567890123456789012123456 7
12 34567890123456789012345678901212345678901234567890123456789012123456 7
12 34567890123456789012345678901212345678901234567890123456789012123456
13. If A is a two-digit number between 10 and 7
1234567890123456789012345678901212345678901234567890123456789012123456 7
1234567890123456789012345678901212345678901234567890123456789012123456 77
2 34567890123456789012345678901212345678901234567890123456789012123456
20 and ({ A)2
5 {(A 2
), then A 5
1234567890123456789012345678901212345678901234567890123456789012123456 7
1234567890123456789012345678901212345678901234567890123456789012123456 7
1234567890123456789012345678901212345678901234567890123456789012123456
(A) 11
7
1
1234567890123456789012345678901212345678901234567890123456789012123456 77
2 34567890123456789012345678901212345678901234567890123456789012123456
(B) 13
1
12 34567890123456789012345678901212345678901234567890123456789012123456
(C) 14 7
1234567890123456789012345678901212345678901234567890123456789012123456 7
12
2
34567890123456789012345678901212345678901234567890123456789012123456 7
(D) 16
34567890123456789012345678901212345678901234567890123456789012123456 7
1
1234567890123456789012345678901212345678901234567890123456789012123456
(E) 18 7
1234567890123456789012345678901212345678901234567890123456789012123456 77
1234567890123456789012345678901212345678901234567890123456789012123456
12345678901234567890123456789012123456789012345678901234567890121234567
12 Copyright © 2005 Thomson Peterson’s, a part of The Thomson Corporaton
SAT is a registered trademark of the College Entrance Examination Board, which
was not involved in the production of and does not endorse this product.
12345678901234567890123456789012123456789012345678901234567890121234567
1234567890123456789012345678901212345678901234567890123456789012123456 7
12234567890123456789012345678901212345678901234567890123456789012123456 7
1 34567890123456789012345678901212345678901234567890123456789012123456
16. 18. Which of the following is the product of 7
12234567890123456789012345678901212345678901234567890123456789012123456 7
1234567890123456789012345678901212345678901234567890123456789012123456
two prime numbers whose difference is 7
1 34567890123456789012345678901212345678901234567890123456789012123456 7
12234567890123456789012345678901212345678901234567890123456789012123456
twenty one?
34567890123456789012345678901212345678901234567890123456789012123456
7
7
1
12234567890123456789012345678901212345678901234567890123456789012123456
(A) 46 7
1234567890123456789012345678901212345678901234567890123456789012123456 7
1234567890123456789012345678901212345678901234567890123456789012123456
(B) 51 7
1234567890123456789012345678901212345678901234567890123456789012123456
34567890123456789012345678901212345678901234567890123456789012123456
7
7
1 (C) 55
12234567890123456789012345678901212345678901234567890123456789012123456 7
1234567890123456789012345678901212345678901234567890123456789012123456
(D) 57 7
1234567890123456789012345678901212345678901234567890123456789012123456 7
1234567890123456789012345678901212345678901234567890123456789012123456
(E) 58
34567890123456789012345678901212345678901234567890123456789012123456
7
7
1
12234567890123456789012345678901212345678901234567890123456789012123456 7
1234567890123456789012345678901212345678901234567890123456789012123456
19. What is the distance from the midpoint of 7
1234567890123456789012345678901212345678901234567890123456789012123456 7
1234567890123456789012345678901212345678901234567890123456789012123456
DE to the origin, if D(0,12) and E(5,0)?
34567890123456789012345678901212345678901234567890123456789012123456
7
7
1
12234567890123456789012345678901212345678901234567890123456789012123456 7
1234567890123456789012345678901212345678901234567890123456789012123456
(A) 5 7
1234567890123456789012345678901212345678901234567890123456789012123456
According to the above table, which of the (B) 6.5 7
1234567890123456789012345678901212345678901234567890123456789012123456 7
1234567890123456789012345678901212345678901234567890123456789012123456 77
34567890123456789012345678901212345678901234567890123456789012123456
following statements is true? (C) 8.2
1234567890123456789012345678901212345678901234567890123456789012123456
1234567890123456789012345678901212345678901234567890123456789012123456
(A) September total sales are 3.5% (D) 12 7
1234567890123456789012345678901212345678901234567890123456789012123456 7
1234567890123456789012345678901212345678901234567890123456789012123456
greater than August total sales. (E) 13 7
1234567890123456789012345678901212345678901234567890123456789012123456 77
1 (B) September total sales are 2.5% less
12234567890123456789012345678901212345678901234567890123456789012123456
20. If 2b2 5 (b 2 7)(b 1 3) 2 (2b 1 2)(b 1 5), 7
1234567890123456789012345678901212345678901234567890123456789012123456
than August total sales. 7
1234567890123456789012345678901212345678901234567890123456789012123456
then b equals 7
1234567890123456789012345678901212345678901234567890123456789012123456 77
34567890123456789012345678901212345678901234567890123456789012123456
(C) September total sales are 2.5%
1234567890123456789012345678901212345678901234567890123456789012123456 15
1234567890123456789012345678901212345678901234567890123456789012123456
greater than August total sales. (A) 21 7
1234567890123456789012345678901212345678901234567890123456789012123456 16 7
1234567890123456789012345678901212345678901234567890123456789012123456 77
(D) September total sales are 5.4% less
12345678901234567890123456789012123456789012345678901234567890121234563
1234567890123456789012345678901212345678901234567890123456789012123456
than August total sales. (B) 1 7
1234567890123456789012345678901212345678901234567890123456789012123456
8 7
1234567890123456789012345678901212345678901234567890123456789012123456 77
(E) September total sales are 5.4%
1 (C) 2
12234567890123456789012345678901212345678901234567890123456789012123456
greater than August total sales. 7
1234567890123456789012345678901212345678901234567890123456789012123456
10 7
1 34567890123456789012345678901212345678901234567890123456789012123456
(D) 2 7
1234567890123456789012345678901212345678901234567890123456789012123456 7
12234567890123456789012345678901212345678901234567890123456789012123456
17. 11
34567890123456789012345678901212345678901234567890123456789012123456
7
7
1234567890123456789012345678901212345678901234567890123456789012123456
1
1234567890123456789012345678901212345678901234567890123456789012123456
(E) 5 7
1 7
12234567890123456789012345678901212345678901234567890123456789012123456
3 7
1234567890123456789012345678901212345678901234567890123456789012123456 7
34567890123456789012345678901212345678901234567890123456789012123456
21. Between 1950 and 2000, the population of 7
1234567890123456789012345678901212345678901234567890123456789012123456
1234567890123456789012345678901212345678901234567890123456789012123456 7
1 7
1234567890123456789012345678901212345678901234567890123456789012123456 7
Cree County tripled every ten years, except
1234567890123456789012345678901212345678901234567890123456789012123456
for the decade of the 1980s when the popu- 7
12234567890123456789012345678901212345678901234567890123456789012123456 7
1234567890123456789012345678901212345678901234567890123456789012123456
lation of the country fell by one-half. If the 7
1234567890123456789012345678901212345678901234567890123456789012123456 7
1234567890123456789012345678901212345678901234567890123456789012123456
population of Cree country was 1000 people 7
1 34567890123456789012345678901212345678901234567890123456789012123456 7
12234567890123456789012345678901212345678901234567890123456789012123456
What is the perimeter of the above figure? in 1950, what is the population now? 7
1234567890123456789012345678901212345678901234567890123456789012123456 7
1234567890123456789012345678901212345678901234567890123456789012123456 7
1234567890123456789012345678901212345678901234567890123456789012123456
(A) 24 (A) 40,500 7
1 34567890123456789012345678901212345678901234567890123456789012123456 7
1234567890123456789012345678901212345678901234567890123456789012123456 77
2 34567890123456789012345678901212345678901234567890123456789012123456
(B) 25 (B) 72,900
1234567890123456789012345678901212345678901234567890123456789012123456
(C) 26 (C) 121,500
7
1234567890123456789012345678901212345678901234567890123456789012123456 7
1234567890123456789012345678901212345678901234567890123456789012123456
(D) 27 (D) 218,700
7
1
2 34567890123456789012345678901212345678901234567890123456789012123456
1234567890123456789012345678901212345678901234567890123456789012123456
(E) 28 (E) 243,000 7
1 7
12234567890123456789012345678901212345678901234567890123456789012123456 7
1 34567890123456789012345678901212345678901234567890123456789012123456 7
12234567890123456789012345678901212345678901234567890123456789012123456 7
STOP
1 34567890123456789012345678901212345678901234567890123456789012123456
Do not proceed to the next section until time is up.
1234567890123456789012345678901212345678901234567890123456789012123456 7
7
1234567890123456789012345678901212345678901234567890123456789012123456 7
1234567890123456789012345678901212345678901234567890123456789012123456 7
12345678901234567890123456789012123456789012345678901234567890121234567
Copyright © 2005 Thomson Peterson’s, a part of The Thomson Corporaton 13
SAT is a registered trademark of the College Entrance Examination Board, which
was not involved in the production of and does not endorse this product.
Section 3
30 Questions j Time—25 Minutes

12345678901234567890123456789012123456789012345678901234567890121234567
12 34567890123456789012345678901212345678901234567890123456789012123456 7
12 34567890123456789012345678901212345678901234567890123456789012123456 7
1 34567890123456789012345678901212345678901234567890123456789012123456
2 Identifying Sentence Errors 7
12 34567890123456789012345678901212345678901234567890123456789012123456 7
12 34567890123456789012345678901212345678901234567890123456789012123456 7
12 34567890123456789012345678901212345678901234567890123456789012123456 7
12 34567890123456789012345678901212345678901234567890123456789012123456
Directions: Mark the letter of your choice on the answer sheet that best corresponds to the 7
1234567890123456789012345678901212345678901234567890123456789012123456
2 7
1234567890123456789012345678901212345678901234567890123456789012123456
correct answer. 7
1234567890123456789012345678901212345678901234567890123456789012123456 7
1234567890123456789012345678901212345678901234567890123456789012123456 7
1234567890123456789012345678901212345678901234567890123456789012123456
Notes: 7
1234567890123456789012345678901212345678901234567890123456789012123456 77
34567890123456789012345678901212345678901234567890123456789012123456
1
12 34567890123456789012345678901212345678901234567890123456789012123456
1. The following questions test your knowledge of the rules of English grammar, as well as 7
12 34567890123456789012345678901212345678901234567890123456789012123456 7
12 34567890123456789012345678901212345678901234567890123456789012123456 7
word usage, word choice, and idioms.
1234567890123456789012345678901212345678901234567890123456789012123456 77
2 34567890123456789012345678901212345678901234567890123456789012123456
1234567890123456789012345678901212345678901234567890123456789012123456
2. Some sentences are correct, but others contain a single error. No sentence contains more
1234567890123456789012345678901212345678901234567890123456789012123456 7
1234567890123456789012345678901212345678901234567890123456789012123456
than one error. 7
1234567890123456789012345678901212345678901234567890123456789012123456 77
1234567890123456789012345678901212345678901234567890123456789012123456
1234567890123456789012345678901212345678901234567890123456789012123456
3. Any errors that occur will be found in the underlined portion of the sentence. Choose the 7
1234567890123456789012345678901212345678901234567890123456789012123456 7
1234567890123456789012345678901212345678901234567890123456789012123456
letter underneath the error to indicate the part of the sentence that must be changed. 7
1 7
2 34567890123456789012345678901212345678901234567890123456789012123456
1234567890123456789012345678901212345678901234567890123456789012123456 7
1234567890123456789012345678901212345678901234567890123456789012123456
4. If there is no error, pick answer choice (E). 7
1 7
1234567890123456789012345678901212345678901234567890123456789012123456 7
12 34567890123456789012345678901212345678901234567890123456789012123456 7
5. There will be no change in any parts of the sentence that are not underlined.
1234567890123456789012345678901212345678901234567890123456789012123456 77
2 34567890123456789012345678901212345678901234567890123456789012123456
1234567890123456789012345678901212345678901234567890123456789012123456 7
1
12 34567890123456789012345678901212345678901234567890123456789012123456 7
12 34567890123456789012345678901212345678901234567890123456789012123456 7
1234567890123456789012345678901212345678901234567890123456789012123456
2 1. The scientists found that there were less 3. The team, which is composed of four 7
1234567890123456789012345678901212345678901234567890123456789012123456 7
1 34567890123456789012345678901212345678901234567890123456789012123456 7
A A
1234567890123456789012345678901212345678901234567890123456789012123456
strands of the mold they were studying cyclists, compete against other cycling 7
1234567890123456789012345678901212345678901234567890123456789012123456 7
2 34567890123456789012345678901212345678901234567890123456789012123456
1234567890123456789012345678901212345678901234567890123456789012123456
B C B 7
1234567890123456789012345678901212345678901234567890123456789012123456 7
1234567890123456789012345678901212345678901234567890123456789012123456 77
than they needed to conduct the experi- teams from around the nation and around
1234567890123456789012345678901212345678901234567890123456789012123456 7
1 D C D
12 34567890123456789012345678901212345678901234567890123456789012123456
ment. No error the world. No error 7
12 34567890123456789012345678901212345678901234567890123456789012123456 7
12 34567890123456789012345678901212345678901234567890123456789012123456
E E 7
12 34567890123456789012345678901212345678901234567890123456789012123456 7
1234567890123456789012345678901212345678901234567890123456789012123456 7
2 34567890123456789012345678901212345678901234567890123456789012123456
1234567890123456789012345678901212345678901234567890123456789012123456 7
1234567890123456789012345678901212345678901234567890123456789012123456
2. The painter gave a fantastic demonstration 4. Depending on which historian you read, 7
1234567890123456789012345678901212345678901234567890123456789012123456 7
1234567890123456789012345678901212345678901234567890123456789012123456 77
A A B
1 for Ada and I and we vowed to utilize the the Pax Romana was either as long as five
12 34567890123456789012345678901212345678901234567890123456789012123456 7
1234567890123456789012345678901212345678901234567890123456789012123456
B C C 7
12 34567890123456789012345678901212345678901234567890123456789012123456 7
1234567890123456789012345678901212345678901234567890123456789012123456 7
new technique in our own work . No error centuries or as short as three. No error
12 34567890123456789012345678901212345678901234567890123456789012123456 7
1234567890123456789012345678901212345678901234567890123456789012123456
D E D E 7
1234567890123456789012345678901212345678901234567890123456789012123456 7
1234567890123456789012345678901212345678901234567890123456789012123456 7
1234567890123456789012345678901212345678901234567890123456789012123456 7
12345678901234567890123456789012123456789012345678901234567890121234567
14 Copyright © 2005 Thomson Peterson’s, a part of The Thomson Corporaton
SAT is a registered trademark of the College Entrance Examination Board, which
was not involved in the production of and does not endorse this product.
12345678901234567890123456789012123456789012345678901234567890121234567
1234567890123456789012345678901212345678901234567890123456789012123456 7
12234567890123456789012345678901212345678901234567890123456789012123456 7
1 34567890123456789012345678901212345678901234567890123456789012123456
5. Because they have extensively surveyed the 8. The census statistics was a complex source 7
12234567890123456789012345678901212345678901234567890123456789012123456 7
1234567890123456789012345678901212345678901234567890123456789012123456
A A 7
1 34567890123456789012345678901212345678901234567890123456789012123456 7
12234567890123456789012345678901212345678901234567890123456789012123456
Lower Ohio Valley , Louis and Clark of information,
34567890123456789012345678901212345678901234567890123456789012123456
in that the demographers 7
7
1
12234567890123456789012345678901212345678901234567890123456789012123456
B B 7
1234567890123456789012345678901212345678901234567890123456789012123456
were well prepared to navigate uncharted knew that certain groups of people were 7
1234567890123456789012345678901212345678901234567890123456789012123456 7
1234567890123456789012345678901212345678901234567890123456789012123456
C
34567890123456789012345678901212345678901234567890123456789012123456 C 7
7
1
12234567890123456789012345678901212345678901234567890123456789012123456
territory along the Missouri River. No error more likely to respond than others. 7
1234567890123456789012345678901212345678901234567890123456789012123456 7
1234567890123456789012345678901212345678901234567890123456789012123456
D E D 7
1234567890123456789012345678901212345678901234567890123456789012123456
34567890123456789012345678901212345678901234567890123456789012123456
7
7
1 No error
12234567890123456789012345678901212345678901234567890123456789012123456
6. The administrator suggested that the staff 7
1234567890123456789012345678901212345678901234567890123456789012123456
E 7
1234567890123456789012345678901212345678901234567890123456789012123456 7
1234567890123456789012345678901212345678901234567890123456789012123456
34567890123456789012345678901212345678901234567890123456789012123456
7
7
1 first focus on contacting 9. Because of the large traffic jam, scarcely
12234567890123456789012345678901212345678901234567890123456789012123456 7
1234567890123456789012345678901212345678901234567890123456789012123456
A A 7
1234567890123456789012345678901212345678901234567890123456789012123456
prospective participants for the upcoming no one from the group made it to the 7
1234567890123456789012345678901212345678901234567890123456789012123456 7
34567890123456789012345678901212345678901234567890123456789012123456
1234567890123456789012345678901212345678901234567890123456789012123456
B B C 7
1234567890123456789012345678901212345678901234567890123456789012123456 7
1234567890123456789012345678901212345678901234567890123456789012123456
seminar and then on follow-up with airport in time to catch the flight to the 7
1234567890123456789012345678901212345678901234567890123456789012123456 7
1234567890123456789012345678901212345678901234567890123456789012123456 77
C D D
1234567890123456789012345678901212345678901234567890123456789012123456 7
1 previous speakers. No error conference in San Antonio. No error
12234567890123456789012345678901212345678901234567890123456789012123456 7
1234567890123456789012345678901212345678901234567890123456789012123456
E E 7
1234567890123456789012345678901212345678901234567890123456789012123456 7
34567890123456789012345678901212345678901234567890123456789012123456
1234567890123456789012345678901212345678901234567890123456789012123456 7
1234567890123456789012345678901212345678901234567890123456789012123456
7. The Ottoman Empire initiated better trade 10. Even though he had the title of 7
1234567890123456789012345678901212345678901234567890123456789012123456 7
1234567890123456789012345678901212345678901234567890123456789012123456 77
A B A
1234567890123456789012345678901212345678901234567890123456789012123456
1234567890123456789012345678901212345678901234567890123456789012123456
relations with the Austrian Empire when Vice-President of Operations, his duties 7
1234567890123456789012345678901212345678901234567890123456789012123456 7
1234567890123456789012345678901212345678901234567890123456789012123456
B
and responsibilities were not much greater 7
7
1234567890123456789012345678901212345678901234567890123456789012123456
its border disputes with Russia
7
1
12234567890123456789012345678901212345678901234567890123456789012123456
C C 7
1234567890123456789012345678901212345678901234567890123456789012123456 7
1 34567890123456789012345678901212345678901234567890123456789012123456 7
were resolved (D). No error than a midlevel manager . No error
1234567890123456789012345678901212345678901234567890123456789012123456 7
12234567890123456789012345678901212345678901234567890123456789012123456
D E D E 7
1234567890123456789012345678901212345678901234567890123456789012123456 77
34567890123456789012345678901212345678901234567890123456789012123456
1234567890123456789012345678901212345678901234567890123456789012123456 7
1
12234567890123456789012345678901212345678901234567890123456789012123456 7
1234567890123456789012345678901212345678901234567890123456789012123456 7
1234567890123456789012345678901212345678901234567890123456789012123456 77
34567890123456789012345678901212345678901234567890123456789012123456
1234567890123456789012345678901212345678901234567890123456789012123456 7
1
1234567890123456789012345678901212345678901234567890123456789012123456 7
1234567890123456789012345678901212345678901234567890123456789012123456 7
12234567890123456789012345678901212345678901234567890123456789012123456 7
1234567890123456789012345678901212345678901234567890123456789012123456 7
1234567890123456789012345678901212345678901234567890123456789012123456 7
1234567890123456789012345678901212345678901234567890123456789012123456 7
1 34567890123456789012345678901212345678901234567890123456789012123456 7
1234567890123456789012345678901212345678901234567890123456789012123456 77
2 34567890123456789012345678901212345678901234567890123456789012123456
1234567890123456789012345678901212345678901234567890123456789012123456 7
1234567890123456789012345678901212345678901234567890123456789012123456 7
1234567890123456789012345678901212345678901234567890123456789012123456 7
1
12234567890123456789012345678901212345678901234567890123456789012123456 7
1234567890123456789012345678901212345678901234567890123456789012123456 7
1234567890123456789012345678901212345678901234567890123456789012123456 7
1234567890123456789012345678901212345678901234567890123456789012123456 7
1 34567890123456789012345678901212345678901234567890123456789012123456 7
1234567890123456789012345678901212345678901234567890123456789012123456 77
2 34567890123456789012345678901212345678901234567890123456789012123456
1
12234567890123456789012345678901212345678901234567890123456789012123456 7
1 34567890123456789012345678901212345678901234567890123456789012123456 7
12234567890123456789012345678901212345678901234567890123456789012123456 7
1 34567890123456789012345678901212345678901234567890123456789012123456 7
1234567890123456789012345678901212345678901234567890123456789012123456 7
1234567890123456789012345678901212345678901234567890123456789012123456 7
1234567890123456789012345678901212345678901234567890123456789012123456 7
12345678901234567890123456789012123456789012345678901234567890121234567
Copyright © 2005 Thomson Peterson’s, a part of The Thomson Corporaton 15
SAT is a registered trademark of the College Entrance Examination Board, which
was not involved in the production of and does not endorse this product.
12345678901234567890123456789012123456789012345678901234567890121234567
1234567890123456789012345678901212345678901234567890123456789012123456 7
12 34567890123456789012345678901212345678901234567890123456789012123456 7
1234567890123456789012345678901212345678901234567890123456789012123456
Improving Sentences 7
12 34567890123456789012345678901212345678901234567890123456789012123456 7
12 34567890123456789012345678901212345678901234567890123456789012123456 7
1234567890123456789012345678901212345678901234567890123456789012123456 7
12 34567890123456789012345678901212345678901234567890123456789012123456
Directions: 7
1234567890123456789012345678901212345678901234567890123456789012123456 7
12 34567890123456789012345678901212345678901234567890123456789012123456 7
12 34567890123456789012345678901212345678901234567890123456789012123456 7
1. The following questions test your knowledge of English grammar, word usage, word
12 34567890123456789012345678901212345678901234567890123456789012123456 7
12
2
34567890123456789012345678901212345678901234567890123456789012123456
choice, sentence construction, and punctuation.
34567890123456789012345678901212345678901234567890123456789012123456
7
7
1
12 34567890123456789012345678901212345678901234567890123456789012123456
2. Every sentence contains a portion that is underlined. 7
12 34567890123456789012345678901212345678901234567890123456789012123456 7
12 34567890123456789012345678901212345678901234567890123456789012123456 7
12
2
34567890123456789012345678901212345678901234567890123456789012123456
3. Any errors that occur will be found in the underlined portion of the sentence. If you
34567890123456789012345678901212345678901234567890123456789012123456
7
7
1
12 34567890123456789012345678901212345678901234567890123456789012123456
believe there is an error, choose the answer choice that corrects the original mistake. 7
12 34567890123456789012345678901212345678901234567890123456789012123456 7
12 34567890123456789012345678901212345678901234567890123456789012123456 7
Answer choices (B), (C), (D), and (E) contain alternative phrasings of the underlined
12 34567890123456789012345678901212345678901234567890123456789012123456 7
1234567890123456789012345678901212345678901234567890123456789012123456
portion. If the sentence contains an error, one of these alternate phrasings will correct it. 7
12 34567890123456789012345678901212345678901234567890123456789012123456 7
12 34567890123456789012345678901212345678901234567890123456789012123456
4. Choice (A) repeats the original underlined portion. If you believe the underlined portion 7
12 34567890123456789012345678901212345678901234567890123456789012123456 7
12 34567890123456789012345678901212345678901234567890123456789012123456 7
does not contain any errors, select answer choice (A).
1234567890123456789012345678901212345678901234567890123456789012123456 77
2 34567890123456789012345678901212345678901234567890123456789012123456
1234567890123456789012345678901212345678901234567890123456789012123456
5. There will be no change in any parts of the sentence that are not underlined.
1234567890123456789012345678901212345678901234567890123456789012123456 7
1234567890123456789012345678901212345678901234567890123456789012123456 77
1234567890123456789012345678901212345678901234567890123456789012123456 7
1234567890123456789012345678901212345678901234567890123456789012123456 7
1
12 34567890123456789012345678901212345678901234567890123456789012123456
11. In an effort to make the Constitution both 12. Widely considered one of the most 7
12 34567890123456789012345678901212345678901234567890123456789012123456 7
12 34567890123456789012345678901212345678901234567890123456789012123456
more accessible and understandable to the original poets of all time, Gerard Manley 7
1234567890123456789012345678901212345678901234567890123456789012123456 77
2 34567890123456789012345678901212345678901234567890123456789012123456
1234567890123456789012345678901212345678901234567890123456789012123456
public, the House of Representatives has Hopkins’s poems display utterly noncon-
1234567890123456789012345678901212345678901234567890123456789012123456 7
1234567890123456789012345678901212345678901234567890123456789012123456
authorized the publication of a series of ventional systems of rhyme. 7
1234567890123456789012345678901212345678901234567890123456789012123456 77
1234567890123456789012345678901212345678901234567890123456789012123456
pamphlets about the Constitution.
1234567890123456789012345678901212345678901234567890123456789012123456
(A) Gerard Manley Hopkins’s poems 7
1234567890123456789012345678901212345678901234567890123456789012123456 7
1234567890123456789012345678901212345678901234567890123456789012123456
(A) both more accessible and under- display utterly nonconventional 7
1 7
1234567890123456789012345678901212345678901234567890123456789012123456 77
2 34567890123456789012345678901212345678901234567890123456789012123456
standable to the public systems of rhyme.
1234567890123456789012345678901212345678901234567890123456789012123456 7
1 (B) more both accessible and under- (B) Gerard Manley Hopkins’s poems
1234567890123456789012345678901212345678901234567890123456789012123456 7
12
2
34567890123456789012345678901212345678901234567890123456789012123456 7
standable to the public
34567890123456789012345678901212345678901234567890123456789012123456
displayed utterly nonconventional
7
1234567890123456789012345678901212345678901234567890123456789012123456
1234567890123456789012345678901212345678901234567890123456789012123456
(C) more accessible to the public and systems of rhyme. 7
1 7
12 34567890123456789012345678901212345678901234567890123456789012123456 7
more understandable for it (C) Gerard Manley Hopkins’s poems
12 34567890123456789012345678901212345678901234567890123456789012123456 7
12 34567890123456789012345678901212345678901234567890123456789012123456
(D) both more accessible and more have systems of rhyme that are 7
12 34567890123456789012345678901212345678901234567890123456789012123456 7
1234567890123456789012345678901212345678901234567890123456789012123456 7
understandable to the public utterly nonconventional.
1234567890123456789012345678901212345678901234567890123456789012123456 7
1234567890123456789012345678901212345678901234567890123456789012123456
(E) accessible to the public and under- (D) Gerard Manley Hopkins wrote 7
2 34567890123456789012345678901212345678901234567890123456789012123456
1234567890123456789012345678901212345678901234567890123456789012123456
standable poems using utterly nonconventional 7
1234567890123456789012345678901212345678901234567890123456789012123456 7
1234567890123456789012345678901212345678901234567890123456789012123456 systems of rhyme. 7
1234567890123456789012345678901212345678901234567890123456789012123456 77
1 (E) Gerard Manley Hopkins had written
12 34567890123456789012345678901212345678901234567890123456789012123456 7
12 34567890123456789012345678901212345678901234567890123456789012123456poems that were displaying utterly 7
12 34567890123456789012345678901212345678901234567890123456789012123456 7
12 34567890123456789012345678901212345678901234567890123456789012123456nonconventional systems of rhyme. 7
1234567890123456789012345678901212345678901234567890123456789012123456 7
1234567890123456789012345678901212345678901234567890123456789012123456 77
2 34567890123456789012345678901212345678901234567890123456789012123456
1234567890123456789012345678901212345678901234567890123456789012123456 7
1234567890123456789012345678901212345678901234567890123456789012123456 7
1234567890123456789012345678901212345678901234567890123456789012123456 7
1
12 34567890123456789012345678901212345678901234567890123456789012123456 7
1234567890123456789012345678901212345678901234567890123456789012123456 7
12 34567890123456789012345678901212345678901234567890123456789012123456 7
1234567890123456789012345678901212345678901234567890123456789012123456 7
12 34567890123456789012345678901212345678901234567890123456789012123456 7
1234567890123456789012345678901212345678901234567890123456789012123456 7
1234567890123456789012345678901212345678901234567890123456789012123456 7
1234567890123456789012345678901212345678901234567890123456789012123456 7
1234567890123456789012345678901212345678901234567890123456789012123456 7
12345678901234567890123456789012123456789012345678901234567890121234567
16 Copyright © 2005 Thomson Peterson’s, a part of The Thomson Corporaton
SAT is a registered trademark of the College Entrance Examination Board, which
was not involved in the production of and does not endorse this product.
12345678901234567890123456789012123456789012345678901234567890121234567
1234567890123456789012345678901212345678901234567890123456789012123456 7
12234567890123456789012345678901212345678901234567890123456789012123456 7
1 34567890123456789012345678901212345678901234567890123456789012123456
13. Eleanor Roosevelt, the wife of President 14. Sputnik was the first artificial satellite 7
12234567890123456789012345678901212345678901234567890123456789012123456 7
1234567890123456789012345678901212345678901234567890123456789012123456
Franklin D. Roosevelt, made an active successfully propelled into orbit and began 7
1 34567890123456789012345678901212345678901234567890123456789012123456 7
12234567890123456789012345678901212345678901234567890123456789012123456
contribution to many political organiza- the space race between the United States
34567890123456789012345678901212345678901234567890123456789012123456
7
7
1
12234567890123456789012345678901212345678901234567890123456789012123456
tions, this included the Human Rights and the Soviet Union, in 1957 it was 7
1234567890123456789012345678901212345678901234567890123456789012123456 7
1234567890123456789012345678901212345678901234567890123456789012123456
Commission. launched by the Soviets. 7
1234567890123456789012345678901212345678901234567890123456789012123456
34567890123456789012345678901212345678901234567890123456789012123456
7
7
1
12234567890123456789012345678901212345678901234567890123456789012123456
(A) organizations, this included the (A) Sputnik was the first artificial 7
1234567890123456789012345678901212345678901234567890123456789012123456 7
1234567890123456789012345678901212345678901234567890123456789012123456
Human Rights Commission. satellite successfully propelled into 7
1234567890123456789012345678901212345678901234567890123456789012123456
34567890123456789012345678901212345678901234567890123456789012123456
7
7
1 (B) organizations, being included the orbit and began the space race
12234567890123456789012345678901212345678901234567890123456789012123456 7
1234567890123456789012345678901212345678901234567890123456789012123456
Human Rights Commission. between the United States and the 7
1234567890123456789012345678901212345678901234567890123456789012123456 7
(C) organizations, whose participation Soviet Union,
1234567890123456789012345678901212345678901234567890123456789012123456
34567890123456789012345678901212345678901234567890123456789012123456
in 1957 it was 7
7
1 included the Human Rights launched by the Soviets.
12234567890123456789012345678901212345678901234567890123456789012123456 7
1234567890123456789012345678901212345678901234567890123456789012123456
Commission. (B) In 1957, the first satellite and space 7
1234567890123456789012345678901212345678901234567890123456789012123456 7
1234567890123456789012345678901212345678901234567890123456789012123456
(D) organizations; this including the race were beginning when Sputnik 7
1234567890123456789012345678901212345678901234567890123456789012123456 77
34567890123456789012345678901212345678901234567890123456789012123456
1234567890123456789012345678901212345678901234567890123456789012123456
Human Rights Commission. was launched.
1234567890123456789012345678901212345678901234567890123456789012123456 7
1234567890123456789012345678901212345678901234567890123456789012123456
(E) organizations, including the Human (C) Launched by the Soviets in 1957, 7
1234567890123456789012345678901212345678901234567890123456789012123456 7
1234567890123456789012345678901212345678901234567890123456789012123456 77
Rights Commission. Sputnik was the first artificial
1
12234567890123456789012345678901212345678901234567890123456789012123456
satellite successfully propelled into 7
1234567890123456789012345678901212345678901234567890123456789012123456 7
1234567890123456789012345678901212345678901234567890123456789012123456
orbit, beginning the space race 7
34567890123456789012345678901212345678901234567890123456789012123456
1234567890123456789012345678901212345678901234567890123456789012123456 7
1234567890123456789012345678901212345678901234567890123456789012123456
between the United States and the 7
1234567890123456789012345678901212345678901234567890123456789012123456 7
1234567890123456789012345678901212345678901234567890123456789012123456 77
Soviet Union.
1234567890123456789012345678901212345678901234567890123456789012123456
1234567890123456789012345678901212345678901234567890123456789012123456
(D) The launching of Sputnik was in 7
1234567890123456789012345678901212345678901234567890123456789012123456 7
1234567890123456789012345678901212345678901234567890123456789012123456 77
1957, the first artificial satellite was
1234567890123456789012345678901212345678901234567890123456789012123456
successfully propelled and the space 7
1
12234567890123456789012345678901212345678901234567890123456789012123456 7
1234567890123456789012345678901212345678901234567890123456789012123456
race between the United States and 7
1 34567890123456789012345678901212345678901234567890123456789012123456 7
1234567890123456789012345678901212345678901234567890123456789012123456 7
the Soviet Union was begun.
12234567890123456789012345678901212345678901234567890123456789012123456 7
1234567890123456789012345678901212345678901234567890123456789012123456
(E) The first artificial satellite success- 7
1234567890123456789012345678901212345678901234567890123456789012123456
fully propelled into orbit was when 7
1 34567890123456789012345678901212345678901234567890123456789012123456 7
12234567890123456789012345678901212345678901234567890123456789012123456
Sputnik was launched in 1957, and 7
1234567890123456789012345678901212345678901234567890123456789012123456 7
34567890123456789012345678901212345678901234567890123456789012123456
1234567890123456789012345678901212345678901234567890123456789012123456
the space race between the United 7
1234567890123456789012345678901212345678901234567890123456789012123456 7
1 States and the Soviet Union was 7
1234567890123456789012345678901212345678901234567890123456789012123456 7
1234567890123456789012345678901212345678901234567890123456789012123456
begun as well. 7
12234567890123456789012345678901212345678901234567890123456789012123456 7
1234567890123456789012345678901212345678901234567890123456789012123456 7
1234567890123456789012345678901212345678901234567890123456789012123456 7
1234567890123456789012345678901212345678901234567890123456789012123456 7
1 34567890123456789012345678901212345678901234567890123456789012123456 7
1234567890123456789012345678901212345678901234567890123456789012123456 77
2 34567890123456789012345678901212345678901234567890123456789012123456
1234567890123456789012345678901212345678901234567890123456789012123456 7
1234567890123456789012345678901212345678901234567890123456789012123456 7
1234567890123456789012345678901212345678901234567890123456789012123456 7
1
12234567890123456789012345678901212345678901234567890123456789012123456 7
1234567890123456789012345678901212345678901234567890123456789012123456 7
1234567890123456789012345678901212345678901234567890123456789012123456 7
1234567890123456789012345678901212345678901234567890123456789012123456 7
1 34567890123456789012345678901212345678901234567890123456789012123456 7
1234567890123456789012345678901212345678901234567890123456789012123456 77
2 34567890123456789012345678901212345678901234567890123456789012123456
1
12234567890123456789012345678901212345678901234567890123456789012123456 7
1 34567890123456789012345678901212345678901234567890123456789012123456 7
12234567890123456789012345678901212345678901234567890123456789012123456 7
1 34567890123456789012345678901212345678901234567890123456789012123456 7
1234567890123456789012345678901212345678901234567890123456789012123456 7
1234567890123456789012345678901212345678901234567890123456789012123456 7
1234567890123456789012345678901212345678901234567890123456789012123456 7
12345678901234567890123456789012123456789012345678901234567890121234567
Copyright © 2005 Thomson Peterson’s, a part of The Thomson Corporaton 17
SAT is a registered trademark of the College Entrance Examination Board, which
was not involved in the production of and does not endorse this product.
12345678901234567890123456789012123456789012345678901234567890121234567
1234567890123456789012345678901212345678901234567890123456789012123456 7
12 34567890123456789012345678901212345678901234567890123456789012123456 7
1234567890123456789012345678901212345678901234567890123456789012123456
15. The newly hired CEO stated clearly in her 17. Domesticated over 5,000 years ago, the 7
12 34567890123456789012345678901212345678901234567890123456789012123456 7
12 34567890123456789012345678901212345678901234567890123456789012123456
opening address to the company that her camel is a useful pack animal because of 7
1234567890123456789012345678901212345678901234567890123456789012123456 7
12
2
34567890123456789012345678901212345678901234567890123456789012123456 7
plans for reinvigorating the company were its tolerance
34567890123456789012345678901212345678901234567890123456789012123456
for hot sand, extreme
7
1
12 34567890123456789012345678901212345678901234567890123456789012123456
to cut back on discretionary spending, temperatures, and it needs little drink- 7
12 34567890123456789012345678901212345678901234567890123456789012123456 7
12 34567890123456789012345678901212345678901234567890123456789012123456 7
refinance the company’s largest loans, and ing water.
12 34567890123456789012345678901212345678901234567890123456789012123456 7
1234567890123456789012345678901212345678901234567890123456789012123456
her plans of keeping the company’s
(A) of its tolerance for hot sand, extreme 7
12 34567890123456789012345678901212345678901234567890123456789012123456 7
12 34567890123456789012345678901212345678901234567890123456789012123456
holdings in the stock market. 7
12 34567890123456789012345678901212345678901234567890123456789012123456 7
temperatures, and it needs little
12 34567890123456789012345678901212345678901234567890123456789012123456 7
1234567890123456789012345678901212345678901234567890123456789012123456
(A) her plans of keeping the company’s drinking water. 7
12 34567890123456789012345678901212345678901234567890123456789012123456 7
12 34567890123456789012345678901212345678901234567890123456789012123456 7
holdings in the stock market. (B) of its tolerance for hot sand and
12 34567890123456789012345678901212345678901234567890123456789012123456
(B) keep the company’s holdings in the 7
12
2
extreme temperatures and its need
34567890123456789012345678901212345678901234567890123456789012123456
34567890123456789012345678901212345678901234567890123456789012123456
7
7
1 stock market. for water is very small.
12 34567890123456789012345678901212345678901234567890123456789012123456 7
12 34567890123456789012345678901212345678901234567890123456789012123456
(C) to get the company to keep its (C) it can tolerate hot sand, extreme 7
12 34567890123456789012345678901212345678901234567890123456789012123456 7
12 34567890123456789012345678901212345678901234567890123456789012123456
holdings in the stock market. temperatures, and a lack of drink- 7
12 34567890123456789012345678901212345678901234567890123456789012123456 7
12 34567890123456789012345678901212345678901234567890123456789012123456
(D) her plans to keep the company’s ing water. 7
12 34567890123456789012345678901212345678901234567890123456789012123456 7
12 34567890123456789012345678901212345678901234567890123456789012123456
holdings in the stock market. (D) it can tolerate hot stand, withstand- 7
12 34567890123456789012345678901212345678901234567890123456789012123456 7
1234567890123456789012345678901212345678901234567890123456789012123456 77
2 34567890123456789012345678901212345678901234567890123456789012123456
(E) keeping the company’s holdings in ing extreme temperatures, and needs
1
12 34567890123456789012345678901212345678901234567890123456789012123456
the stock market. little water. 7
12 34567890123456789012345678901212345678901234567890123456789012123456 7
12 34567890123456789012345678901212345678901234567890123456789012123456 7
(E) it can tolerate hot sand and extreme
2 34567890123456789012345678901212345678901234567890123456789012123456
1234567890123456789012345678901212345678901234567890123456789012123456 7
1234567890123456789012345678901212345678901234567890123456789012123456
16. Typically, a restaurant’s kitchen is divided temperatures, and needs little water. 7
1234567890123456789012345678901212345678901234567890123456789012123456 7
1234567890123456789012345678901212345678901234567890123456789012123456 77
into a number of sections, each with a
1234567890123456789012345678901212345678901234567890123456789012123456
1234567890123456789012345678901212345678901234567890123456789012123456
particular aspect of food preparation 18. Legally, an agreement among two people 7
1234567890123456789012345678901212345678901234567890123456789012123456 7
1234567890123456789012345678901212345678901234567890123456789012123456 77
performed there. to commit a crime or concealing it
1234567890123456789012345678901212345678901234567890123456789012123456
constitutes a criminal conspiracy.
7
1 (A) each with a particular aspect of food
1234567890123456789012345678901212345678901234567890123456789012123456 77
2 34567890123456789012345678901212345678901234567890123456789012123456
1234567890123456789012345678901212345678901234567890123456789012123456
preparation performed there. (A) among two people to commit a crime
7
1
1234567890123456789012345678901212345678901234567890123456789012123456
(B) each corresponding to a particular or concealing it 7
12 34567890123456789012345678901212345678901234567890123456789012123456 7
1234567890123456789012345678901212345678901234567890123456789012123456 77
2 34567890123456789012345678901212345678901234567890123456789012123456
aspect of food preparation. (B) among two people to commit a crime
1234567890123456789012345678901212345678901234567890123456789012123456 7
1 (C) where they each have their particu- or agreeing to conceal it
12 34567890123456789012345678901212345678901234567890123456789012123456 7
12 34567890123456789012345678901212345678901234567890123456789012123456 7
lar aspect of food preparation to (C) among two people committing a
2 34567890123456789012345678901212345678901234567890123456789012123456
1234567890123456789012345678901212345678901234567890123456789012123456 7
1234567890123456789012345678901212345678901234567890123456789012123456
perform. crime or concealing it 7
1 7
1234567890123456789012345678901212345678901234567890123456789012123456 7
(D) which has a particular aspect of food (D) between two people to commit a
1234567890123456789012345678901212345678901234567890123456789012123456 7
12 34567890123456789012345678901212345678901234567890123456789012123456
preparation performed there. crime or conceal it 7
12 34567890123456789012345678901212345678901234567890123456789012123456 7
12 34567890123456789012345678901212345678901234567890123456789012123456 7
(E) they each correspond to a particular (E) between two people in the commit-
12 34567890123456789012345678901212345678901234567890123456789012123456 7
1234567890123456789012345678901212345678901234567890123456789012123456
aspect of food preparation. ting of a crime or the concealment 7
2 34567890123456789012345678901212345678901234567890123456789012123456
1234567890123456789012345678901212345678901234567890123456789012123456 7
1234567890123456789012345678901212345678901234567890123456789012123456
of it 7
1234567890123456789012345678901212345678901234567890123456789012123456 77
1234567890123456789012345678901212345678901234567890123456789012123456 7
1
12 34567890123456789012345678901212345678901234567890123456789012123456 7
12 34567890123456789012345678901212345678901234567890123456789012123456 7
12 34567890123456789012345678901212345678901234567890123456789012123456 7
12 34567890123456789012345678901212345678901234567890123456789012123456 7
1234567890123456789012345678901212345678901234567890123456789012123456 7
1234567890123456789012345678901212345678901234567890123456789012123456
2 7
1 34567890123456789012345678901212345678901234567890123456789012123456 7
12 34567890123456789012345678901212345678901234567890123456789012123456 7
1234567890123456789012345678901212345678901234567890123456789012123456 7
12 34567890123456789012345678901212345678901234567890123456789012123456 7
1234567890123456789012345678901212345678901234567890123456789012123456 7
1234567890123456789012345678901212345678901234567890123456789012123456 7
1234567890123456789012345678901212345678901234567890123456789012123456 7
1234567890123456789012345678901212345678901234567890123456789012123456 7
12345678901234567890123456789012123456789012345678901234567890121234567
18 Copyright © 2005 Thomson Peterson’s, a part of The Thomson Corporaton
SAT is a registered trademark of the College Entrance Examination Board, which
was not involved in the production of and does not endorse this product.
12345678901234567890123456789012123456789012345678901234567890121234567
1234567890123456789012345678901212345678901234567890123456789012123456 7
12234567890123456789012345678901212345678901234567890123456789012123456 7
1 34567890123456789012345678901212345678901234567890123456789012123456
19. Not only was Sir Isaac Newton famous 20. If the high levels of stock market invest- 7
12234567890123456789012345678901212345678901234567890123456789012123456 7
1234567890123456789012345678901212345678901234567890123456789012123456
for his pioneering work in Physics, he was ment are to continue it will depend upon 7
1 34567890123456789012345678901212345678901234567890123456789012123456 7
12234567890123456789012345678901212345678901234567890123456789012123456
also a talented and well-respected eco- both how
34567890123456789012345678901212345678901234567890123456789012123456
long the stock market remains 7
7
1
12234567890123456789012345678901212345678901234567890123456789012123456
nomic advisor to the king. stable and its long-term durability. 7
1234567890123456789012345678901212345678901234567890123456789012123456 7
1234567890123456789012345678901212345678901234567890123456789012123456 7
1234567890123456789012345678901212345678901234567890123456789012123456
(A) he was also a talented and well- (A) If the
34567890123456789012345678901212345678901234567890123456789012123456
high levels of stock market 7
7
1
12234567890123456789012345678901212345678901234567890123456789012123456
respected economic advisor to the king. investment are to continue it 7
1234567890123456789012345678901212345678901234567890123456789012123456 7
1234567890123456789012345678901212345678901234567890123456789012123456
(B) he had also been a talented and will depend 7
1234567890123456789012345678901212345678901234567890123456789012123456
34567890123456789012345678901212345678901234567890123456789012123456
7
7
1 well-respected economic advisor to (B) If the high levels of stock market
12234567890123456789012345678901212345678901234567890123456789012123456 7
1234567890123456789012345678901212345678901234567890123456789012123456
the king. investment are to continue, depend- 7
1234567890123456789012345678901212345678901234567890123456789012123456 7
(C) but he was also a talented and ing
1234567890123456789012345678901212345678901234567890123456789012123456
34567890123456789012345678901212345678901234567890123456789012123456
upon 7
7
1 well-respected economic advisor to (C) If the high levels of stock market
12234567890123456789012345678901212345678901234567890123456789012123456 7
1234567890123456789012345678901212345678901234567890123456789012123456
the king. investment continue, it will depend 7
1234567890123456789012345678901212345678901234567890123456789012123456 7
1234567890123456789012345678901212345678901234567890123456789012123456
(D) as he also was a talented and (D) Whether the high levels of stock 7
1234567890123456789012345678901212345678901234567890123456789012123456 77
34567890123456789012345678901212345678901234567890123456789012123456
1234567890123456789012345678901212345678901234567890123456789012123456
well-respected economic advisor to market investment continue will
1234567890123456789012345678901212345678901234567890123456789012123456 7
1234567890123456789012345678901212345678901234567890123456789012123456
the king. depend 7
1234567890123456789012345678901212345678901234567890123456789012123456 7
1234567890123456789012345678901212345678901234567890123456789012123456 77
(E) but he had also been a talented and (E) Whether the high levels of stock
1
12234567890123456789012345678901212345678901234567890123456789012123456
well-respected economic advisor to market investment continue it 7
1234567890123456789012345678901212345678901234567890123456789012123456 7
1234567890123456789012345678901212345678901234567890123456789012123456
the king. will depend 7
1234567890123456789012345678901212345678901234567890123456789012123456 77
34567890123456789012345678901212345678901234567890123456789012123456
1234567890123456789012345678901212345678901234567890123456789012123456 7
1234567890123456789012345678901212345678901234567890123456789012123456 7
1234567890123456789012345678901212345678901234567890123456789012123456 7
1234567890123456789012345678901212345678901234567890123456789012123456 7
1234567890123456789012345678901212345678901234567890123456789012123456 7
1234567890123456789012345678901212345678901234567890123456789012123456 7
1234567890123456789012345678901212345678901234567890123456789012123456 7
1234567890123456789012345678901212345678901234567890123456789012123456 7
1
12234567890123456789012345678901212345678901234567890123456789012123456 7
1234567890123456789012345678901212345678901234567890123456789012123456 7
1 34567890123456789012345678901212345678901234567890123456789012123456 7
1234567890123456789012345678901212345678901234567890123456789012123456 7
12234567890123456789012345678901212345678901234567890123456789012123456 7
1234567890123456789012345678901212345678901234567890123456789012123456 7
1234567890123456789012345678901212345678901234567890123456789012123456 7
1 34567890123456789012345678901212345678901234567890123456789012123456 7
12234567890123456789012345678901212345678901234567890123456789012123456 7
1234567890123456789012345678901212345678901234567890123456789012123456 7
1234567890123456789012345678901212345678901234567890123456789012123456 77
34567890123456789012345678901212345678901234567890123456789012123456
1234567890123456789012345678901212345678901234567890123456789012123456 7
1
1234567890123456789012345678901212345678901234567890123456789012123456 7
1234567890123456789012345678901212345678901234567890123456789012123456 7
12234567890123456789012345678901212345678901234567890123456789012123456 7
1234567890123456789012345678901212345678901234567890123456789012123456 7
1234567890123456789012345678901212345678901234567890123456789012123456 7
1234567890123456789012345678901212345678901234567890123456789012123456 7
1 34567890123456789012345678901212345678901234567890123456789012123456 7
1234567890123456789012345678901212345678901234567890123456789012123456 77
2 34567890123456789012345678901212345678901234567890123456789012123456
1234567890123456789012345678901212345678901234567890123456789012123456 7
1234567890123456789012345678901212345678901234567890123456789012123456 7
1234567890123456789012345678901212345678901234567890123456789012123456 7
1
12234567890123456789012345678901212345678901234567890123456789012123456 7
1234567890123456789012345678901212345678901234567890123456789012123456 7
1234567890123456789012345678901212345678901234567890123456789012123456 7
1234567890123456789012345678901212345678901234567890123456789012123456 7
1 34567890123456789012345678901212345678901234567890123456789012123456 7
1234567890123456789012345678901212345678901234567890123456789012123456 77
2 34567890123456789012345678901212345678901234567890123456789012123456
1
12234567890123456789012345678901212345678901234567890123456789012123456 7
1 34567890123456789012345678901212345678901234567890123456789012123456 7
12234567890123456789012345678901212345678901234567890123456789012123456 7
1 34567890123456789012345678901212345678901234567890123456789012123456 7
1234567890123456789012345678901212345678901234567890123456789012123456 7
1234567890123456789012345678901212345678901234567890123456789012123456 7
1234567890123456789012345678901212345678901234567890123456789012123456 7
12345678901234567890123456789012123456789012345678901234567890121234567
Copyright © 2005 Thomson Peterson’s, a part of The Thomson Corporaton 19
SAT is a registered trademark of the College Entrance Examination Board, which
was not involved in the production of and does not endorse this product.
12345678901234567890123456789012123456789012345678901234567890121234567
1234567890123456789012345678901212345678901234567890123456789012123456 7
12 34567890123456789012345678901212345678901234567890123456789012123456 7
1234567890123456789012345678901212345678901234567890123456789012123456
Improving Paragraphs 7
12 34567890123456789012345678901212345678901234567890123456789012123456 7
12 34567890123456789012345678901212345678901234567890123456789012123456 7
1234567890123456789012345678901212345678901234567890123456789012123456 7
12 34567890123456789012345678901212345678901234567890123456789012123456
Directions: 7
1234567890123456789012345678901212345678901234567890123456789012123456 7
12 34567890123456789012345678901212345678901234567890123456789012123456 7
12 34567890123456789012345678901212345678901234567890123456789012123456
1. The following questions test your knowledge of paragraph and sentence construction. 7
12 34567890123456789012345678901212345678901234567890123456789012123456 7
12 34567890123456789012345678901212345678901234567890123456789012123456 7
1234567890123456789012345678901212345678901234567890123456789012123456
2. The following passage is a rough draft of an essay. This rough draft contains various 7
12 34567890123456789012345678901212345678901234567890123456789012123456 7
12 34567890123456789012345678901212345678901234567890123456789012123456 7
errors.
12 34567890123456789012345678901212345678901234567890123456789012123456 7
12 34567890123456789012345678901212345678901234567890123456789012123456 7
1 34567890123456789012345678901212345678901234567890123456789012123456
2 3. Read the rough draft and then answer the questions that follow. Some questions will focus 7
12 34567890123456789012345678901212345678901234567890123456789012123456 7
12 34567890123456789012345678901212345678901234567890123456789012123456 7
on specific sentences and ask if there are any problems with that sentence’s word choice,
12 34567890123456789012345678901212345678901234567890123456789012123456
word usage, or overall structure. Other questions will ask about the paragraph itself. 7
12
2
34567890123456789012345678901212345678901234567890123456789012123456
34567890123456789012345678901212345678901234567890123456789012123456
7
7
1 These questions will focus on paragraph organization and development.
12 34567890123456789012345678901212345678901234567890123456789012123456 7
12 34567890123456789012345678901212345678901234567890123456789012123456 7
12 34567890123456789012345678901212345678901234567890123456789012123456
4. Select the answer that best reflects the rules of English grammar and proper essay and 7
12 34567890123456789012345678901212345678901234567890123456789012123456 7
1234567890123456789012345678901212345678901234567890123456789012123456 77
2 34567890123456789012345678901212345678901234567890123456789012123456
paragraph writing.
1234567890123456789012345678901212345678901234567890123456789012123456 7
1234567890123456789012345678901212345678901234567890123456789012123456 7
1234567890123456789012345678901212345678901234567890123456789012123456 7
1234567890123456789012345678901212345678901234567890123456789012123456 7
1234567890123456789012345678901212345678901234567890123456789012123456 7
1 Questions 21–25 are based on the following century after when the philosophers
12 34567890123456789012345678901212345678901234567890123456789012123456 7
12 34567890123456789012345678901212345678901234567890123456789012123456
passage. reckon the beginning of modern times. 7
12 34567890123456789012345678901212345678901234567890123456789012123456 7
2 34567890123456789012345678901212345678901234567890123456789012123456
1234567890123456789012345678901212345678901234567890123456789012123456
(25) (11) The philosophers and the historians 7
1234567890123456789012345678901212345678901234567890123456789012123456
The following passage is part of an essay about 7
1234567890123456789012345678901212345678901234567890123456789012123456 have a bit of a discrepancy here. 7
1234567890123456789012345678901212345678901234567890123456789012123456
the different meanings of the word “modern.” 7
1234567890123456789012345678901212345678901234567890123456789012123456 77 (12) Of course, you might think both
1234567890123456789012345678901212345678901234567890123456789012123456 the philosophers and the historians are a 7
1234567890123456789012345678901212345678901234567890123456789012123456
Line (1) The word modern is a curious word
1234567890123456789012345678901212345678901234567890123456789012123456
(2) It is curious for all its different 7
1234567890123456789012345678901212345678901234567890123456789012123456 bit off on this whole modernity thing. 7
1 meanings. (3) If you take a freshman 7
1234567890123456789012345678901212345678901234567890123456789012123456 77
2 34567890123456789012345678901212345678901234567890123456789012123456
(30) (13) Who thinks of the mud and cobble-
1234567890123456789012345678901212345678901234567890123456789012123456
class in philosophy, for instance, your stone streets of Paris in the early decades 7
1
1234567890123456789012345678901212345678901234567890123456789012123456
(5) professor might tell you about Rene 7
12
2
34567890123456789012345678901212345678901234567890123456789012123456
34567890123456789012345678901212345678901234567890123456789012123456
of the seventeenth century as modern? 7
7
1234567890123456789012345678901212345678901234567890123456789012123456
Descartes. (4) Descartes was a French
1234567890123456789012345678901212345678901234567890123456789012123456 (14) Then again, London a century later 7
1 philosopher from the seventeenth with the power of steam harnessed does 7
12 34567890123456789012345678901212345678901234567890123456789012123456 7
12 34567890123456789012345678901212345678901234567890123456789012123456
century. (5) Many consider him as the (35) not strike most as the picture of modern 7
2 34567890123456789012345678901212345678901234567890123456789012123456
1234567890123456789012345678901212345678901234567890123456789012123456 7
1234567890123456789012345678901212345678901234567890123456789012123456
first figure in modern philosophy. (6) So times. (15) Most of us, when we think of 7
1 (10) if you are talking to a philosophy 7
1234567890123456789012345678901212345678901234567890123456789012123456 what modern means, we are thinking 7
1234567890123456789012345678901212345678901234567890123456789012123456 7
2 34567890123456789012345678901212345678901234567890123456789012123456
1234567890123456789012345678901212345678901234567890123456789012123456
professor the word modern denotes about computers and cell phones and 7
1234567890123456789012345678901212345678901234567890123456789012123456 7
1234567890123456789012345678901212345678901234567890123456789012123456
anytime between about 1615 and now. wireless networks. (16) We are not 7
1234567890123456789012345678901212345678901234567890123456789012123456 7
1 (7) (Of course, some philosophers think (40) thinking about some philosophical 7
2 34567890123456789012345678901212345678901234567890123456789012123456
1234567890123456789012345678901212345678901234567890123456789012123456 7
1234567890123456789012345678901212345678901234567890123456789012123456
that modern times ended a few decades discourse that a Frenchman wrote four 7
1234567890123456789012345678901212345678901234567890123456789012123456 7
1234567890123456789012345678901212345678901234567890123456789012123456 77
(15) back and that we are already in to centuries back.
1
12 34567890123456789012345678901212345678901234567890123456789012123456
postmodern times.) (8) Other academics 7
12 34567890123456789012345678901212345678901234567890123456789012123456 7
12 34567890123456789012345678901212345678901234567890123456789012123456 7
have a different timeline for the birth of
12 34567890123456789012345678901212345678901234567890123456789012123456 7
1234567890123456789012345678901212345678901234567890123456789012123456
modern times, or modernity. (9) And 7
2 34567890123456789012345678901212345678901234567890123456789012123456
1234567890123456789012345678901212345678901234567890123456789012123456
some historians point to the Industrial 7
1 7
12 34567890123456789012345678901212345678901234567890123456789012123456
(20) Revolution in England as the beginning 7
1234567890123456789012345678901212345678901234567890123456789012123456 7
12 34567890123456789012345678901212345678901234567890123456789012123456
of modernity. (10) This dating technique 7
1234567890123456789012345678901212345678901234567890123456789012123456 7
1234567890123456789012345678901212345678901234567890123456789012123456
puts the beginning of modernity at least a 7
1234567890123456789012345678901212345678901234567890123456789012123456 77
1234567890123456789012345678901212345678901234567890123456789012123456
12345678901234567890123456789012123456789012345678901234567890121234567
20 Copyright © 2005 Thomson Peterson’s, a part of The Thomson Corporaton
SAT is a registered trademark of the College Entrance Examination Board, which
was not involved in the production of and does not endorse this product.
12345678901234567890123456789012123456789012345678901234567890121234567
1234567890123456789012345678901212345678901234567890123456789012123456 7
12234567890123456789012345678901212345678901234567890123456789012123456 7
1 34567890123456789012345678901212345678901234567890123456789012123456
21. Which of the following is the best 23. Which of the following would be the best 7
12234567890123456789012345678901212345678901234567890123456789012123456 7
1234567890123456789012345678901212345678901234567890123456789012123456
combination of sentences 1 and 2 replacement for “And” at the beginning of 7
1 34567890123456789012345678901212345678901234567890123456789012123456 7
12234567890123456789012345678901212345678901234567890123456789012123456
(reproduced below)?
34567890123456789012345678901212345678901234567890123456789012123456
sentence 9 (reproduced below)? 7
7
1
12234567890123456789012345678901212345678901234567890123456789012123456 7
1234567890123456789012345678901212345678901234567890123456789012123456
The word modern is a curious word. It is And some historians point to the Indus- 7
1234567890123456789012345678901212345678901234567890123456789012123456 7
1234567890123456789012345678901212345678901234567890123456789012123456
curious for all its different meanings.
34567890123456789012345678901212345678901234567890123456789012123456
trial Revolution in England as the 7
7
1
12234567890123456789012345678901212345678901234567890123456789012123456 beginning of modernity. 7
1234567890123456789012345678901212345678901234567890123456789012123456
(A) The word modern is a curious word. It 7
1234567890123456789012345678901212345678901234567890123456789012123456 7
1234567890123456789012345678901212345678901234567890123456789012123456
is curious for all its different meanings.
34567890123456789012345678901212345678901234567890123456789012123456
(A) However, 7
7
1 (B) Modern is a curious word because it (B) Moreover,
12234567890123456789012345678901212345678901234567890123456789012123456 7
1234567890123456789012345678901212345678901234567890123456789012123456
has all its different meanings. (C) Even so, 7
1234567890123456789012345678901212345678901234567890123456789012123456 7
1234567890123456789012345678901212345678901234567890123456789012123456
(C) It is curious that the word modern
34567890123456789012345678901212345678901234567890123456789012123456(D) Considering this, 7
7
1
12234567890123456789012345678901212345678901234567890123456789012123456
has various different meanings. (E) For example, 7
1234567890123456789012345678901212345678901234567890123456789012123456 7
1234567890123456789012345678901212345678901234567890123456789012123456
(D) Curiously, the word modern has 7
1234567890123456789012345678901212345678901234567890123456789012123456 7
1234567890123456789012345678901212345678901234567890123456789012123456 77
34567890123456789012345678901212345678901234567890123456789012123456
many different meanings. 24. Which of the following is the best revision
1234567890123456789012345678901212345678901234567890123456789012123456 of the underlined portions of sentence 10 7
1234567890123456789012345678901212345678901234567890123456789012123456
(E) Modernity is a curious word for all
1234567890123456789012345678901212345678901234567890123456789012123456 7
1234567890123456789012345678901212345678901234567890123456789012123456
its various different meanings. (reproduced below)? 7
1234567890123456789012345678901212345678901234567890123456789012123456 77
1
12234567890123456789012345678901212345678901234567890123456789012123456 This dating technique puts the beginning 7
1234567890123456789012345678901212345678901234567890123456789012123456
22. Which of the following is the best way to
of modernity at least a century after when 7
1234567890123456789012345678901212345678901234567890123456789012123456 7
34567890123456789012345678901212345678901234567890123456789012123456
1234567890123456789012345678901212345678901234567890123456789012123456
revise sentences 4 and 5 (reproduced
the philosophers reckon the beginning of 7
1234567890123456789012345678901212345678901234567890123456789012123456 7
1234567890123456789012345678901212345678901234567890123456789012123456
below) so that they are condensed into
modern times. 7
1234567890123456789012345678901212345678901234567890123456789012123456
one sentence? 7
1234567890123456789012345678901212345678901234567890123456789012123456 7
1234567890123456789012345678901212345678901234567890123456789012123456 77
(A) As it is now.
1234567890123456789012345678901212345678901234567890123456789012123456
Descartes was a French philosopher from
1234567890123456789012345678901212345678901234567890123456789012123456 (B) at least a century after when the 7
1234567890123456789012345678901212345678901234567890123456789012123456
the seventeenth century. Many consider him 7
1 philosophers reckon it. 7
2 34567890123456789012345678901212345678901234567890123456789012123456
1234567890123456789012345678901212345678901234567890123456789012123456
as the first figure in modern philosophy. 7
1234567890123456789012345678901212345678901234567890123456789012123456 (C) at least a century after when the 7
1 7
1234567890123456789012345678901212345678901234567890123456789012123456 7
(A) Descartes was a French philosopher philosopher’s date modernity’s
12234567890123456789012345678901212345678901234567890123456789012123456 7
1234567890123456789012345678901212345678901234567890123456789012123456
from the seventeenth century, inception. 7
1234567890123456789012345678901212345678901234567890123456789012123456 7
1 34567890123456789012345678901212345678901234567890123456789012123456 7
considering him the first figure in (D) at least one century after when the
12234567890123456789012345678901212345678901234567890123456789012123456 7
1234567890123456789012345678901212345678901234567890123456789012123456
modern philosophy. philosophers had reckoned the 7
34567890123456789012345678901212345678901234567890123456789012123456
1234567890123456789012345678901212345678901234567890123456789012123456 7
(B) Descartes was a French philosopher
1234567890123456789012345678901212345678901234567890123456789012123456 inception of modernity. 7
1 from the seventeenth century, whom 7
1234567890123456789012345678901212345678901234567890123456789012123456 7(E) at least a century after its having
1234567890123456789012345678901212345678901234567890123456789012123456
many consider the first figure in 7
12234567890123456789012345678901212345678901234567890123456789012123456 been reckoned by the philosophers. 7
1234567890123456789012345678901212345678901234567890123456789012123456
modern philosophy. 7
1234567890123456789012345678901212345678901234567890123456789012123456 7
1234567890123456789012345678901212345678901234567890123456789012123456
(C) Descartes was a French philosopher 7
1 34567890123456789012345678901212345678901234567890123456789012123456 7
1234567890123456789012345678901212345678901234567890123456789012123456 77
2 34567890123456789012345678901212345678901234567890123456789012123456
from the seventeenth century, and
1234567890123456789012345678901212345678901234567890123456789012123456 7
1234567890123456789012345678901212345678901234567890123456789012123456
many consider him the first figure in
7
1234567890123456789012345678901212345678901234567890123456789012123456 7
1 modern philosophy.
2 34567890123456789012345678901212345678901234567890123456789012123456
1234567890123456789012345678901212345678901234567890123456789012123456 7
1234567890123456789012345678901212345678901234567890123456789012123456
(D) Descartes was a French philosopher 7
1234567890123456789012345678901212345678901234567890123456789012123456 7
1234567890123456789012345678901212345678901234567890123456789012123456 77
from the seventeenth century, so he was
1
12234567890123456789012345678901212345678901234567890123456789012123456
the first figure in modern philosophy. 7
1 34567890123456789012345678901212345678901234567890123456789012123456
(E) Descartes was a French philosopher 7
12234567890123456789012345678901212345678901234567890123456789012123456 7
1 34567890123456789012345678901212345678901234567890123456789012123456
from the seventeenth century and 7
12234567890123456789012345678901212345678901234567890123456789012123456
34567890123456789012345678901212345678901234567890123456789012123456
7
7
1 figuring him the first figure in
1234567890123456789012345678901212345678901234567890123456789012123456 7
1234567890123456789012345678901212345678901234567890123456789012123456
modern philosophy. 7
1234567890123456789012345678901212345678901234567890123456789012123456 7
12345678901234567890123456789012123456789012345678901234567890121234567
Copyright © 2005 Thomson Peterson’s, a part of The Thomson Corporaton 21
SAT is a registered trademark of the College Entrance Examination Board, which
was not involved in the production of and does not endorse this product.
12345678901234567890123456789012123456789012345678901234567890121234567
1234567890123456789012345678901212345678901234567890123456789012123456 7
12 34567890123456789012345678901212345678901234567890123456789012123456 7
1234567890123456789012345678901212345678901234567890123456789012123456
25. Which of the following is the best revision larger group of people. (6) Those people 7
12 34567890123456789012345678901212345678901234567890123456789012123456 7
12 34567890123456789012345678901212345678901234567890123456789012123456
of sentence 15 (reproduced below)? (15) could be potential consumers. 7
1234567890123456789012345678901212345678901234567890123456789012123456 7
12
2
34567890123456789012345678901212345678901234567890123456789012123456
34567890123456789012345678901212345678901234567890123456789012123456
(7) Many of our most famous 7
7
1 Most of us, when we think of what
12 34567890123456789012345678901212345678901234567890123456789012123456
department store chains first expanded 7
12 34567890123456789012345678901212345678901234567890123456789012123456
modern means, we are thinking about 7
12 34567890123456789012345678901212345678901234567890123456789012123456
during this time. (8) These include Sears, 7
12
2
34567890123456789012345678901212345678901234567890123456789012123456
computers and cell phones and wireless
34567890123456789012345678901212345678901234567890123456789012123456
7
7
1 Roebuck; Woolworth’s; the Great
12 34567890123456789012345678901212345678901234567890123456789012123456
networks. 7
12 34567890123456789012345678901212345678901234567890123456789012123456 7
(20) Atlantic and Pacific Tea Company (the
12 34567890123456789012345678901212345678901234567890123456789012123456 7
12
2
34567890123456789012345678901212345678901234567890123456789012123456
(A) As it is now. A&P); and Walgreen Drug. (9) Among
34567890123456789012345678901212345678901234567890123456789012123456
7
7
1 (B) Most of us, when we think of what
12 34567890123456789012345678901212345678901234567890123456789012123456
the most successful department stores 7
12 34567890123456789012345678901212345678901234567890123456789012123456
modern means, computers and cell 7
12 34567890123456789012345678901212345678901234567890123456789012123456 7
was Filene’s in Boston and Macy’s in
12 34567890123456789012345678901212345678901234567890123456789012123456
phones and wireless networks 7
1234567890123456789012345678901212345678901234567890123456789012123456
New York. (10) Initially, department 7
12 34567890123456789012345678901212345678901234567890123456789012123456
coming to mind. 7
12 34567890123456789012345678901212345678901234567890123456789012123456 7
(25) stores were more like the malls of today.
12 34567890123456789012345678901212345678901234567890123456789012123456
(C) When we think of what modern (11) Each department was leased to an 7
12 34567890123456789012345678901212345678901234567890123456789012123456 7
2 34567890123456789012345678901212345678901234567890123456789012123456
1234567890123456789012345678901212345678901234567890123456789012123456
means, think about computers and individual owner. (12) Nowadays, 7
1234567890123456789012345678901212345678901234567890123456789012123456 7
1234567890123456789012345678901212345678901234567890123456789012123456
cell phones and wireless networks. virtually all departments are run by the 7
1234567890123456789012345678901212345678901234567890123456789012123456 7
1234567890123456789012345678901212345678901234567890123456789012123456
(D) Most of us, when we think of what larger company, including restaurants. 7
1234567890123456789012345678901212345678901234567890123456789012123456 7
1 modern means, we are thinking (30) (13) Also, with their radio cam- 7
12 34567890123456789012345678901212345678901234567890123456789012123456 7
12 34567890123456789012345678901212345678901234567890123456789012123456
about computers, cell phones, paigns, the new department stores of the 7
12 34567890123456789012345678901212345678901234567890123456789012123456 7
12 34567890123456789012345678901212345678901234567890123456789012123456
wireless networks. 1920s put on extravagant advertising 7
12 34567890123456789012345678901212345678901234567890123456789012123456 7
12 34567890123456789012345678901212345678901234567890123456789012123456
(E) Most of us, when we think of what spectacles. (14) Sometimes, they even 7
12 34567890123456789012345678901212345678901234567890123456789012123456 7
12 34567890123456789012345678901212345678901234567890123456789012123456 7
modern means, think about comput- hosted entertainment events to attract
2 34567890123456789012345678901212345678901234567890123456789012123456
1234567890123456789012345678901212345678901234567890123456789012123456 7
1234567890123456789012345678901212345678901234567890123456789012123456
ers and cell phones and wireless (35) consumers. (15) The Macy’s Thanksgiv- 7
1234567890123456789012345678901212345678901234567890123456789012123456 7
1234567890123456789012345678901212345678901234567890123456789012123456 77
networks. ing Day Parade, an attempt to capture
1
12 34567890123456789012345678901212345678901234567890123456789012123456
the children’s toy market, is one example 7
12 34567890123456789012345678901212345678901234567890123456789012123456 7
1234567890123456789012345678901212345678901234567890123456789012123456
Questions 26–30 are based on the following of popular merchandising. (16) Bloom- 7
1234567890123456789012345678901212345678901234567890123456789012123456 7
12
2
34567890123456789012345678901212345678901234567890123456789012123456 7
passage. ingdale’s
34567890123456789012345678901212345678901234567890123456789012123456
posted ads on all New York
7
1234567890123456789012345678901212345678901234567890123456789012123456
1234567890123456789012345678901212345678901234567890123456789012123456
(40) public transit, pronouncing, “All Cars 7
1 The following is a first draft of an essay about 7
12 34567890123456789012345678901212345678901234567890123456789012123456
Transfer to Bloomingdale’s.” 7
12 34567890123456789012345678901212345678901234567890123456789012123456 7
the growth of the department store industry in
2 34567890123456789012345678901212345678901234567890123456789012123456
1234567890123456789012345678901212345678901234567890123456789012123456 7
1234567890123456789012345678901212345678901234567890123456789012123456
the 1920s. 7
1 7
1234567890123456789012345678901212345678901234567890123456789012123456
Line (1) Automobiles and radios became far 7
1234567890123456789012345678901212345678901234567890123456789012123456 7
2 34567890123456789012345678901212345678901234567890123456789012123456
1234567890123456789012345678901212345678901234567890123456789012123456
more affordable in the 1920s. (2) By 7
1234567890123456789012345678901212345678901234567890123456789012123456 7
1234567890123456789012345678901212345678901234567890123456789012123456
1925 there was one automobile for every 7
1234567890123456789012345678901212345678901234567890123456789012123456 77
1 six people in the United States, by 1930
12 34567890123456789012345678901212345678901234567890123456789012123456 7
12 34567890123456789012345678901212345678901234567890123456789012123456
(5) this had increased to one for every 4.6 7
12 34567890123456789012345678901212345678901234567890123456789012123456 7
12 34567890123456789012345678901212345678901234567890123456789012123456
people. (3) Also by 1930, about 4 in 10 7
1234567890123456789012345678901212345678901234567890123456789012123456 7
1234567890123456789012345678901212345678901234567890123456789012123456 77
2 34567890123456789012345678901212345678901234567890123456789012123456
American families owned radios. (4) The
1234567890123456789012345678901212345678901234567890123456789012123456 7
1234567890123456789012345678901212345678901234567890123456789012123456
popularity of automobiles and radios led
7
1234567890123456789012345678901212345678901234567890123456789012123456 7
1 to the spread of chain stores of all kinds.
2 34567890123456789012345678901212345678901234567890123456789012123456
1234567890123456789012345678901212345678901234567890123456789012123456 7
1 (10) (5) Automobiles allowed consumers to 7
12 34567890123456789012345678901212345678901234567890123456789012123456 7
1234567890123456789012345678901212345678901234567890123456789012123456 7
travel further in search of the right item
12 34567890123456789012345678901212345678901234567890123456789012123456 7
1234567890123456789012345678901212345678901234567890123456789012123456
for the right price, while radios allowed 7
1234567890123456789012345678901212345678901234567890123456789012123456 7
1234567890123456789012345678901212345678901234567890123456789012123456 77
businesses to advertise their products to a
1234567890123456789012345678901212345678901234567890123456789012123456
12345678901234567890123456789012123456789012345678901234567890121234567
22 Copyright © 2005 Thomson Peterson’s, a part of The Thomson Corporaton
SAT is a registered trademark of the College Entrance Examination Board, which
was not involved in the production of and does not endorse this product.
12345678901234567890123456789012123456789012345678901234567890121234567
1234567890123456789012345678901212345678901234567890123456789012123456 7
12234567890123456789012345678901212345678901234567890123456789012123456 7
1 34567890123456789012345678901212345678901234567890123456789012123456
26. Which of the following is the best revision 28. If you were to combine sentences 10 and 7
12234567890123456789012345678901212345678901234567890123456789012123456 7
1234567890123456789012345678901212345678901234567890123456789012123456
of the underlined portions of sentences 5 11 (reproduced below), which would be 7
1 34567890123456789012345678901212345678901234567890123456789012123456 7
12234567890123456789012345678901212345678901234567890123456789012123456
and 6 (reproduced below)? the most
34567890123456789012345678901212345678901234567890123456789012123456
appropriate and precise punctua- 7
7
1
12234567890123456789012345678901212345678901234567890123456789012123456
tion mark to use? 7
1234567890123456789012345678901212345678901234567890123456789012123456
Automobiles allowed consumers to travel 7
1234567890123456789012345678901212345678901234567890123456789012123456 7
1234567890123456789012345678901212345678901234567890123456789012123456
further in search of the right item for the Initially,
34567890123456789012345678901212345678901234567890123456789012123456
department stores were more like 7
7
1
12234567890123456789012345678901212345678901234567890123456789012123456
right price, while radios allowed busi- the malls of today. Each department was 7
1234567890123456789012345678901212345678901234567890123456789012123456 7
1234567890123456789012345678901212345678901234567890123456789012123456
nesses to advertise their products to a leased to an individual owner. 7
1234567890123456789012345678901212345678901234567890123456789012123456
34567890123456789012345678901212345678901234567890123456789012123456
7
7
1 larger group of people. Those people
(A) today ‘each. . . owner.’
12234567890123456789012345678901212345678901234567890123456789012123456 7
1234567890123456789012345678901212345678901234567890123456789012123456
could be potential consumers.
(B) today; each 7
1234567890123456789012345678901212345678901234567890123456789012123456 7
1234567890123456789012345678901212345678901234567890123456789012123456
(A) As it is now. (C) today: each
34567890123456789012345678901212345678901234567890123456789012123456
7
7
1
12234567890123456789012345678901212345678901234567890123456789012123456
(B) a larger group of people, who could (D) today (each. . . owner). 7
1234567890123456789012345678901212345678901234567890123456789012123456 7
1234567890123456789012345678901212345678901234567890123456789012123456
be potential consumers. (E) today, each 7
1234567890123456789012345678901212345678901234567890123456789012123456 7
1234567890123456789012345678901212345678901234567890123456789012123456 77
34567890123456789012345678901212345678901234567890123456789012123456
(C) a larger group of people, being
1234567890123456789012345678901212345678901234567890123456789012123456
29. Which of the following is the best revision 7
1234567890123456789012345678901212345678901234567890123456789012123456
potential consumers.
1234567890123456789012345678901212345678901234567890123456789012123456 7
1234567890123456789012345678901212345678901234567890123456789012123456
(D) a larger group of people, whom they of the underlined portion of sentence 13 7
1234567890123456789012345678901212345678901234567890123456789012123456 7
1 made into potential consumers. (reproduced below)? 7
12234567890123456789012345678901212345678901234567890123456789012123456 7
1234567890123456789012345678901212345678901234567890123456789012123456
(E) a larger group of people, having the Also, with their radio campaigns, the new 7
1234567890123456789012345678901212345678901234567890123456789012123456 7
34567890123456789012345678901212345678901234567890123456789012123456
1234567890123456789012345678901212345678901234567890123456789012123456
potential to become consumers. department stores of the 1920s put on 7
1234567890123456789012345678901212345678901234567890123456789012123456 7
1234567890123456789012345678901212345678901234567890123456789012123456
extravagant advertising spectacles. 7
1234567890123456789012345678901212345678901234567890123456789012123456
27. “This time” in sentence 7 (reproduced 7
1234567890123456789012345678901212345678901234567890123456789012123456 7
1234567890123456789012345678901212345678901234567890123456789012123456
below) is best made more specific. Which (A) Moreover 7
1234567890123456789012345678901212345678901234567890123456789012123456 7
1234567890123456789012345678901212345678901234567890123456789012123456
of the following phrases is the best (B) Too 7
1234567890123456789012345678901212345678901234567890123456789012123456 7
1 revision? (C) What’s more to their radio campaigns 7
2 34567890123456789012345678901212345678901234567890123456789012123456
(D) In addition to their radio campaigns 7
1234567890123456789012345678901212345678901234567890123456789012123456
1234567890123456789012345678901212345678901234567890123456789012123456 7
1 Many of our most famous department 7
1234567890123456789012345678901212345678901234567890123456789012123456
(E) The radio campaigns being included 7
12234567890123456789012345678901212345678901234567890123456789012123456
store chains first expanded during
34567890123456789012345678901212345678901234567890123456789012123456
7
7
1234567890123456789012345678901212345678901234567890123456789012123456
1234567890123456789012345678901212345678901234567890123456789012123456
this time. Which of the following sentences, if added 7
1 30. 7
12234567890123456789012345678901212345678901234567890123456789012123456
at the end of paragraph 3, is the best 7
1234567890123456789012345678901212345678901234567890123456789012123456
(A) these years 7
34567890123456789012345678901212345678901234567890123456789012123456
1234567890123456789012345678901212345678901234567890123456789012123456
concluding sentence for the passage? 7
1234567890123456789012345678901212345678901234567890123456789012123456
(B) the twentieth century 7
1 7
1234567890123456789012345678901212345678901234567890123456789012123456
(C) the same when automobile sales and (A) In the 20s, shopping and advertising 7
1234567890123456789012345678901212345678901234567890123456789012123456 7
12234567890123456789012345678901212345678901234567890123456789012123456
radio sales were also on the rise started to look a lot like they do now. 7
1234567890123456789012345678901212345678901234567890123456789012123456
(D) the years described in the previous 7
1234567890123456789012345678901212345678901234567890123456789012123456
(B) Other stores also had extravaganzas. 7
1234567890123456789012345678901212345678901234567890123456789012123456
paragraph (C) So now get in your car and drive to a 7
1 34567890123456789012345678901212345678901234567890123456789012123456 7
2 34567890123456789012345678901212345678901234567890123456789012123456
1234567890123456789012345678901212345678901234567890123456789012123456
(E) the 1920s 7
1234567890123456789012345678901212345678901234567890123456789012123456 department store! 7
1234567890123456789012345678901212345678901234567890123456789012123456 7
1234567890123456789012345678901212345678901234567890123456789012123456 77
(D) Department Stores having become
1
12234567890123456789012345678901212345678901234567890123456789012123456huge successes. 7
1234567890123456789012345678901212345678901234567890123456789012123456 7
1234567890123456789012345678901212345678901234567890123456789012123456
(E) Macy’s Thanksgiving Day Parade 7
1234567890123456789012345678901212345678901234567890123456789012123456 7
1 34567890123456789012345678901212345678901234567890123456789012123456 7
was always a big hit.
1234567890123456789012345678901212345678901234567890123456789012123456 77
2 34567890123456789012345678901212345678901234567890123456789012123456
1
12234567890123456789012345678901212345678901234567890123456789012123456 7
1 34567890123456789012345678901212345678901234567890123456789012123456 7
12234567890123456789012345678901212345678901234567890123456789012123456 7
STOP
1 34567890123456789012345678901212345678901234567890123456789012123456
Do not proceed to the next section until time is up.
1234567890123456789012345678901212345678901234567890123456789012123456 7
7
1234567890123456789012345678901212345678901234567890123456789012123456 7
1234567890123456789012345678901212345678901234567890123456789012123456 7
12345678901234567890123456789012123456789012345678901234567890121234567
Copyright © 2005 Thomson Peterson’s, a part of The Thomson Corporaton 23
SAT is a registered trademark of the College Entrance Examination Board, which
was not involved in the production of and does not endorse this product.
Section 4
27 Questions j Time—25 Minutes

12345678901234567890123456789012123456789012345678901234567890121234567
12 34567890123456789012345678901212345678901234567890123456789012123456 7
12 34567890123456789012345678901212345678901234567890123456789012123456 7
1 34567890123456789012345678901212345678901234567890123456789012123456 7
2
12 34567890123456789012345678901212345678901234567890123456789012123456 7
12 34567890123456789012345678901212345678901234567890123456789012123456
Directions: Each sentence below has either one or two blanks in it and is followed by five 7
12 34567890123456789012345678901212345678901234567890123456789012123456
choices, labeled (A) through (E). These choices represent words or phrases that have been left 7
12 34567890123456789012345678901212345678901234567890123456789012123456 7
2 34567890123456789012345678901212345678901234567890123456789012123456
1234567890123456789012345678901212345678901234567890123456789012123456
out. Choose the word or phrase that, if inserted into the sentence, would best fit the meaning of 7
1234567890123456789012345678901212345678901234567890123456789012123456 7
1234567890123456789012345678901212345678901234567890123456789012123456
the sentence as a whole. 7
1234567890123456789012345678901212345678901234567890123456789012123456 77
1234567890123456789012345678901212345678901234567890123456789012123456
1234567890123456789012345678901212345678901234567890123456789012123456
Example: 7
1 7
12 34567890123456789012345678901212345678901234567890123456789012123456 7
12 34567890123456789012345678901212345678901234567890123456789012123456 7
Canine massage is a veterinary technique
12 34567890123456789012345678901212345678901234567890123456789012123456 7
12 34567890123456789012345678901212345678901234567890123456789012123456
for calming dogs that are extremely __________. 7
12 34567890123456789012345678901212345678901234567890123456789012123456 7
12 34567890123456789012345678901212345678901234567890123456789012123456 7
12 34567890123456789012345678901212345678901234567890123456789012123456
(A) inept 7
12 34567890123456789012345678901212345678901234567890123456789012123456 7
1234567890123456789012345678901212345678901234567890123456789012123456 77
2 34567890123456789012345678901212345678901234567890123456789012123456
(B) disciplined
1234567890123456789012345678901212345678901234567890123456789012123456
(C) controlled
1234567890123456789012345678901212345678901234567890123456789012123456 7
1234567890123456789012345678901212345678901234567890123456789012123456
(D) stressed 7
1 7
2 34567890123456789012345678901212345678901234567890123456789012123456
1234567890123456789012345678901212345678901234567890123456789012123456
(E) restrained 7
1234567890123456789012345678901212345678901234567890123456789012123456 7
1 7
1234567890123456789012345678901212345678901234567890123456789012123456
A
O Þ
Þ B
O Þ C
O Þ O ÞE 7
12 34567890123456789012345678901212345678901234567890123456789012123456 7
1234567890123456789012345678901212345678901234567890123456789012123456 77
2 34567890123456789012345678901212345678901234567890123456789012123456
1234567890123456789012345678901212345678901234567890123456789012123456 7
1
12 34567890123456789012345678901212345678901234567890123456789012123456 7
12 34567890123456789012345678901212345678901234567890123456789012123456
1. Jerome is a true __________ ; he rarely 2. The property tax hike in Colson County 7
2 34567890123456789012345678901212345678901234567890123456789012123456
1234567890123456789012345678901212345678901234567890123456789012123456 7
1234567890123456789012345678901212345678901234567890123456789012123456
buys anything other than food, and even was not just __________ by the county 7
1 7
1234567890123456789012345678901212345678901234567890123456789012123456 7
his food is plain and minimal. residents; a series of protests were even
1234567890123456789012345678901212345678901234567890123456789012123456 7
12 34567890123456789012345678901212345678901234567890123456789012123456
__________. 7
12 34567890123456789012345678901212345678901234567890123456789012123456
(A) hermit 7
12 34567890123456789012345678901212345678901234567890123456789012123456 7
12 34567890123456789012345678901212345678901234567890123456789012123456 7
(B) teetotaler (A) disliked. .organized
1234567890123456789012345678901212345678901234567890123456789012123456 7
12 34567890123456789012345678901212345678901234567890123456789012123456
(C) gourmand (B) espoused. .planned 7
12 34567890123456789012345678901212345678901234567890123456789012123456 7
12 34567890123456789012345678901212345678901234567890123456789012123456 7
(D) ascetic (C) discouraged. .theorized
12 34567890123456789012345678901212345678901234567890123456789012123456 7
1234567890123456789012345678901212345678901234567890123456789012123456
(E) fanatic (D) bolstered. .analyzed 7
2 34567890123456789012345678901212345678901234567890123456789012123456
1234567890123456789012345678901212345678901234567890123456789012123456
(E) detested. .negotiated 7
1234567890123456789012345678901212345678901234567890123456789012123456 7
1234567890123456789012345678901212345678901234567890123456789012123456 77
1234567890123456789012345678901212345678901234567890123456789012123456 7
1
12 34567890123456789012345678901212345678901234567890123456789012123456 7
1234567890123456789012345678901212345678901234567890123456789012123456 7
12 34567890123456789012345678901212345678901234567890123456789012123456 7
1234567890123456789012345678901212345678901234567890123456789012123456 7
12 34567890123456789012345678901212345678901234567890123456789012123456 7
1234567890123456789012345678901212345678901234567890123456789012123456 7
1234567890123456789012345678901212345678901234567890123456789012123456 7
1234567890123456789012345678901212345678901234567890123456789012123456 7
1234567890123456789012345678901212345678901234567890123456789012123456 7
12345678901234567890123456789012123456789012345678901234567890121234567
24 Copyright © 2005 Thomson Peterson’s, a part of The Thomson Corporaton
SAT is a registered trademark of the College Entrance Examination Board, which
was not involved in the production of and does not endorse this product.
12345678901234567890123456789012123456789012345678901234567890121234567
1234567890123456789012345678901212345678901234567890123456789012123456 7
12234567890123456789012345678901212345678901234567890123456789012123456 7
1 34567890123456789012345678901212345678901234567890123456789012123456
3. There was criticism that the councilman 7. Despite the longstanding __________ 7
12234567890123456789012345678901212345678901234567890123456789012123456 7
1234567890123456789012345678901212345678901234567890123456789012123456
was __________ when he seized the between the clans, both clans __________ 7
1 34567890123456789012345678901212345678901234567890123456789012123456 7
12234567890123456789012345678901212345678901234567890123456789012123456
ceremony __________ by the girl’s tragic each other in the aftermath of the disaster. 7
34567890123456789012345678901212345678901234567890123456789012123456 7
1
12234567890123456789012345678901212345678901234567890123456789012123456
death to speak out against his opponent. 7
1234567890123456789012345678901212345678901234567890123456789012123456
(A) feud. .assisted 7
1234567890123456789012345678901212345678901234567890123456789012123456 7
1234567890123456789012345678901212345678901234567890123456789012123456
(A) militaristic. . negated (B) grudge.
34567890123456789012345678901212345678901234567890123456789012123456
.maligned 7
7
1
12234567890123456789012345678901212345678901234567890123456789012123456
(B) opportunistic. .afforded (C) detente. .withstood 7
1234567890123456789012345678901212345678901234567890123456789012123456 7
1234567890123456789012345678901212345678901234567890123456789012123456
(C) unreceptive. .preempted (D) skirmish. . ameliorated 7
1234567890123456789012345678901212345678901234567890123456789012123456
34567890123456789012345678901212345678901234567890123456789012123456
7
7
1 (D) passive. . created (E) alliance. . discounted
12234567890123456789012345678901212345678901234567890123456789012123456 7
1234567890123456789012345678901212345678901234567890123456789012123456
(E) defeatist. .overshadowed 7
1234567890123456789012345678901212345678901234567890123456789012123456
8. The oral tradition of the Bambara people 7
1234567890123456789012345678901212345678901234567890123456789012123456 7
1 34567890123456789012345678901212345678901234567890123456789012123456
4. The computer expert underscored that the of West Africa is rich with humor and 7
12234567890123456789012345678901212345678901234567890123456789012123456 7
1234567890123456789012345678901212345678901234567890123456789012123456
new software would _____ the prior __________, characteristics which are 7
1234567890123456789012345678901212345678901234567890123456789012123456 7
1234567890123456789012345678901212345678901234567890123456789012123456
version; users could simply __________ evident in the merriment of their every- 7
34567890123456789012345678901212345678901234567890123456789012123456
1234567890123456789012345678901212345678901234567890123456789012123456 7
1234567890123456789012345678901212345678901234567890123456789012123456
the old one. day life. 7
1234567890123456789012345678901212345678901234567890123456789012123456 77
1234567890123456789012345678901212345678901234567890123456789012123456
1234567890123456789012345678901212345678901234567890123456789012123456
(A) preclude. .destroy (A) irony 7
1234567890123456789012345678901212345678901234567890123456789012123456 7
1 (B) outdo. .implement (B) mirth 7
12234567890123456789012345678901212345678901234567890123456789012123456 7
1234567890123456789012345678901212345678901234567890123456789012123456
(C) infect. .disregard (C) cynicism 7
1234567890123456789012345678901212345678901234567890123456789012123456 7
1234567890123456789012345678901212345678901234567890123456789012123456 77
34567890123456789012345678901212345678901234567890123456789012123456
(D) undermine. .detach (D) history
1234567890123456789012345678901212345678901234567890123456789012123456
1234567890123456789012345678901212345678901234567890123456789012123456
(E) supercede. .discard (E) mystery 7
1234567890123456789012345678901212345678901234567890123456789012123456 77
1234567890123456789012345678901212345678901234567890123456789012123456
9. Although Astropithicus has more subspe- 7
1234567890123456789012345678901212345678901234567890123456789012123456
5. Romania has a long and __________
1234567890123456789012345678901212345678901234567890123456789012123456
cies than Pthicalitius, the latter populates 7
7
1234567890123456789012345678901212345678901234567890123456789012123456
tradition of activist-poets, who through
1234567890123456789012345678901212345678901234567890123456789012123456 7
1 poetry have __________ the dignity and the Earth with __________ numbers and 7
2 34567890123456789012345678901212345678901234567890123456789012123456
1234567890123456789012345678901212345678901234567890123456789012123456 7
1234567890123456789012345678901212345678901234567890123456789012123456
equality of humanity. in more __________ geographic regions. 7
1 7
1234567890123456789012345678901212345678901234567890123456789012123456 7
12234567890123456789012345678901212345678901234567890123456789012123456
(A) reputable. .initiated (A) greater.
34567890123456789012345678901212345678901234567890123456789012123456
.diverse 7
7
1234567890123456789012345678901212345678901234567890123456789012123456
1234567890123456789012345678901212345678901234567890123456789012123456
(B) storied. .articulated (B) lesser. .secluded 7
1 7
12234567890123456789012345678901212345678901234567890123456789012123456
(C) tenuous. .supported (C) milder. .remote 7
1234567890123456789012345678901212345678901234567890123456789012123456 7
1234567890123456789012345678901212345678901234567890123456789012123456 77
34567890123456789012345678901212345678901234567890123456789012123456
(D) tactile. .decried (D) scanter. .familiar
1234567890123456789012345678901212345678901234567890123456789012123456
(E) exemplary. .countered (E) larger. .ominous 7
1
1234567890123456789012345678901212345678901234567890123456789012123456 7
1234567890123456789012345678901212345678901234567890123456789012123456 7
12234567890123456789012345678901212345678901234567890123456789012123456
6. Dr. Patel expected the surgery to be 10. It is generally __________ by medical 7
1234567890123456789012345678901212345678901234567890123456789012123456 7
1234567890123456789012345678901212345678901234567890123456789012123456
__________ and laborious, but it turned practitioners that the last few weeks of a 7
1234567890123456789012345678901212345678901234567890123456789012123456 7
1 34567890123456789012345678901212345678901234567890123456789012123456 7
out to be speedy and __________. pregnancy are crucial in the __________ of
2 34567890123456789012345678901212345678901234567890123456789012123456
1234567890123456789012345678901212345678901234567890123456789012123456 7
1234567890123456789012345678901212345678901234567890123456789012123456
the fetus. 7
1234567890123456789012345678901212345678901234567890123456789012123456 77
(A) fragile. .simple
1234567890123456789012345678901212345678901234567890123456789012123456 7
1 (B) compelling. .forthcoming (A) acknowledged. .progression
12234567890123456789012345678901212345678901234567890123456789012123456 7
1234567890123456789012345678901212345678901234567890123456789012123456
(C) intricate. .straightforward (B) hedged. .health 7
1234567890123456789012345678901212345678901234567890123456789012123456 7
1234567890123456789012345678901212345678901234567890123456789012123456
(D) complicated. .locatable (C) endorsed. .birthing 7
1 34567890123456789012345678901212345678901234567890123456789012123456
(E) hard. .mechanical (D) accepted. .development 7
1234567890123456789012345678901212345678901234567890123456789012123456 77
2 34567890123456789012345678901212345678901234567890123456789012123456
1 (E) negated. .vitality
12234567890123456789012345678901212345678901234567890123456789012123456 7
1 34567890123456789012345678901212345678901234567890123456789012123456 7
12234567890123456789012345678901212345678901234567890123456789012123456 7
1 34567890123456789012345678901212345678901234567890123456789012123456 7
1234567890123456789012345678901212345678901234567890123456789012123456 7
1234567890123456789012345678901212345678901234567890123456789012123456 7
1234567890123456789012345678901212345678901234567890123456789012123456 7
12345678901234567890123456789012123456789012345678901234567890121234567
Copyright © 2005 Thomson Peterson’s, a part of The Thomson Corporaton 25
SAT is a registered trademark of the College Entrance Examination Board, which
was not involved in the production of and does not endorse this product.
12345678901234567890123456789012123456789012345678901234567890121234567
1234567890123456789012345678901212345678901234567890123456789012123456 7
12 34567890123456789012345678901212345678901234567890123456789012123456 7
1234567890123456789012345678901212345678901234567890123456789012123456
11. Though the organization espoused 15. Technology, instead of alleviating the 7
12 34567890123456789012345678901212345678901234567890123456789012123456 7
12 34567890123456789012345678901212345678901234567890123456789012123456
outward-focused ideals, in practice, it was demands upon our time, has made the 7
1234567890123456789012345678901212345678901234567890123456789012123456 7
12
2
34567890123456789012345678901212345678901234567890123456789012123456 7
quite __________. pace of modern-day
34567890123456789012345678901212345678901234567890123456789012123456
life increase to a near
7
1
12 34567890123456789012345678901212345678901234567890123456789012123456
__________ pace. 7
12 34567890123456789012345678901212345678901234567890123456789012123456 7
(A) gregarious
12 34567890123456789012345678901212345678901234567890123456789012123456 7
12
2
34567890123456789012345678901212345678901234567890123456789012123456
(B) vigilant (A) elicit
34567890123456789012345678901212345678901234567890123456789012123456
7
7
1
12 34567890123456789012345678901212345678901234567890123456789012123456
(C) insular (B) frenetic 7
12 34567890123456789012345678901212345678901234567890123456789012123456 7
12 34567890123456789012345678901212345678901234567890123456789012123456 7
(D) hermetic (C) lethargic
12 34567890123456789012345678901212345678901234567890123456789012123456 7
1234567890123456789012345678901212345678901234567890123456789012123456
(E) idiosyncratic (D) dilatory 7
12 34567890123456789012345678901212345678901234567890123456789012123456 7
12 34567890123456789012345678901212345678901234567890123456789012123456 7
(E) cavalier
12 34567890123456789012345678901212345678901234567890123456789012123456
12. The prosecuting attorney described the 7
12
2
34567890123456789012345678901212345678901234567890123456789012123456
34567890123456789012345678901212345678901234567890123456789012123456
7
7
1 defendant’s character as __________ and
12 34567890123456789012345678901212345678901234567890123456789012123456
Questions 16–27 are based on the following 7
12 34567890123456789012345678901212345678901234567890123456789012123456
base, but the defense attorney rejoined 7
12 34567890123456789012345678901212345678901234567890123456789012123456
passage. 7
12 34567890123456789012345678901212345678901234567890123456789012123456
that the prosecution was __________ the 7
2 34567890123456789012345678901212345678901234567890123456789012123456
1234567890123456789012345678901212345678901234567890123456789012123456
The following passage tells the story of Juan 7
1234567890123456789012345678901212345678901234567890123456789012123456
testimonials about the defendant.
Bautista Rael, an Hispanic-American scholar. It 7
7
1234567890123456789012345678901212345678901234567890123456789012123456
1234567890123456789012345678901212345678901234567890123456789012123456
is adapted from a biography written by Enrique 7
7
1234567890123456789012345678901212345678901234567890123456789012123456
(A) dark. .misunderstanding
1234567890123456789012345678901212345678901234567890123456789012123456 7
1 (B) nefarious. .misconstruing R. Lamadrid. 7
12 34567890123456789012345678901212345678901234567890123456789012123456 7
12 34567890123456789012345678901212345678901234567890123456789012123456
(C) lackluster. .misinterpreting 7
12 34567890123456789012345678901212345678901234567890123456789012123456
Line Linguist and folklorist Juan Bautista Rael 7
2 34567890123456789012345678901212345678901234567890123456789012123456
1234567890123456789012345678901212345678901234567890123456789012123456
(D) blustery. .misapplying 7
1234567890123456789012345678901212345678901234567890123456789012123456
was a highly regarded academic pioneer. 7
1234567890123456789012345678901212345678901234567890123456789012123456
(E) motley. .misrepresenting 7
1234567890123456789012345678901212345678901234567890123456789012123456 77
His major contribution to linguistics was
1234567890123456789012345678901212345678901234567890123456789012123456
1234567890123456789012345678901212345678901234567890123456789012123456
13. Many believe that the new drug regime collecting and documenting the Hispano 7
1234567890123456789012345678901212345678901234567890123456789012123456 7
folk stories,
1234567890123456789012345678901212345678901234567890123456789012123456
will be a __________ for helping cure (5) plays, and religious tradi- 7
1234567890123456789012345678901212345678901234567890123456789012123456
tions of northern New Mexico and 7
1 diverse ailments related to spinal dysfunc- 7
2 34567890123456789012345678901212345678901234567890123456789012123456
1234567890123456789012345678901212345678901234567890123456789012123456
southern Colorado. 7
1234567890123456789012345678901212345678901234567890123456789012123456
tions. 7
1 Rael was born on August 14, 1900, 7
1234567890123456789012345678901212345678901234567890123456789012123456 7
12
2
34567890123456789012345678901212345678901234567890123456789012123456
(A) bane in Arroyo Hondo, New Mexico. Famous 7
34567890123456789012345678901212345678901234567890123456789012123456 7
1234567890123456789012345678901212345678901234567890123456789012123456
1234567890123456789012345678901212345678901234567890123456789012123456
(B) panacea (10) for its spectacular setting north of Taos, 7
1 7
12 34567890123456789012345678901212345678901234567890123456789012123456
(C) module the village lies in a deep, narrow valley 7
12 34567890123456789012345678901212345678901234567890123456789012123456 7
1234567890123456789012345678901212345678901234567890123456789012123456 77
2 34567890123456789012345678901212345678901234567890123456789012123456
(D) fledgling between Taos mountain and the gorge of
1234567890123456789012345678901212345678901234567890123456789012123456
(E) pariah 7
1 the Rio Grande to the west. His family
1234567890123456789012345678901212345678901234567890123456789012123456 7
1234567890123456789012345678901212345678901234567890123456789012123456
prospered in sheep and cattle ranching 7
2 34567890123456789012345678901212345678901234567890123456789012123456
1234567890123456789012345678901212345678901234567890123456789012123456
14. The executive charged that the whistle- 7
1234567890123456789012345678901212345678901234567890123456789012123456
(15) and owned a mercantile business that 7
1234567890123456789012345678901212345678901234567890123456789012123456
blower’s actions were so self-centered that 7
1234567890123456789012345678901212345678901234567890123456789012123456
served surrounding Hispano communities 7
1 they were not just __________ but even 7
2 34567890123456789012345678901212345678901234567890123456789012123456
1234567890123456789012345678901212345678901234567890123456789012123456
as well as nearby Taos Pueblo. 7
1234567890123456789012345678901212345678901234567890123456789012123456
__________. 7
1234567890123456789012345678901212345678901234567890123456789012123456 77
Juan’s parents, José Ignacio Rael and
1234567890123456789012345678901212345678901234567890123456789012123456 7
1 (A) self-involved. .altruistic Soledad Santistevan, raised a family of
12 34567890123456789012345678901212345678901234567890123456789012123456 7
12 34567890123456789012345678901212345678901234567890123456789012123456
(B) ill-tempered. .narcissistic (20) four sons and a daughter. José Ignacio 7
12 34567890123456789012345678901212345678901234567890123456789012123456 7
12 34567890123456789012345678901212345678901234567890123456789012123456 7
(C) egocentric. .solipsistic had the foresight to recognize the
1234567890123456789012345678901212345678901234567890123456789012123456
(D) unduly. .respectable changes that were coming with the 7
12 34567890123456789012345678901212345678901234567890123456789012123456 7
1234567890123456789012345678901212345678901234567890123456789012123456
(E) erratic. .destructive increasing Americanization of New 7
12 34567890123456789012345678901212345678901234567890123456789012123456 7
1234567890123456789012345678901212345678901234567890123456789012123456
Mexico and realized that a fluent 7
12
2
34567890123456789012345678901212345678901234567890123456789012123456
34567890123456789012345678901212345678901234567890123456789012123456
7
7
1 (25) knowledge of English and a good
1234567890123456789012345678901212345678901234567890123456789012123456 7
1234567890123456789012345678901212345678901234567890123456789012123456
education would be necessary for his 7
1234567890123456789012345678901212345678901234567890123456789012123456 7
12345678901234567890123456789012123456789012345678901234567890121234567
26 Copyright © 2005 Thomson Peterson’s, a part of The Thomson Corporaton
SAT is a registered trademark of the College Entrance Examination Board, which
was not involved in the production of and does not endorse this product.
12345678901234567890123456789012123456789012345678901234567890121234567
1234567890123456789012345678901212345678901234567890123456789012123456 7
12234567890123456789012345678901212345678901234567890123456789012123456 7
1 34567890123456789012345678901212345678901234567890123456789012123456
family to excel. Since local schools were (70) relinquished his family inheritance in 7
12234567890123456789012345678901212345678901234567890123456789012123456 7
1234567890123456789012345678901212345678901234567890123456789012123456
rudimentary at best, the family relied land, cattle, and sheep to his three 7
1 34567890123456789012345678901212345678901234567890123456789012123456 7
12234567890123456789012345678901212345678901234567890123456789012123456
upon its own resources to get the best brothers
34567890123456789012345678901212345678901234567890123456789012123456
and his sister. 7
7
1 (30) possible education for the children. Juan
12234567890123456789012345678901212345678901234567890123456789012123456 Rael realized that the wealth in 7
1234567890123456789012345678901212345678901234567890123456789012123456 7
1234567890123456789012345678901212345678901234567890123456789012123456
was a dedicated student from his earliest northern New Mexico that interested 7
1234567890123456789012345678901212345678901234567890123456789012123456
34567890123456789012345678901212345678901234567890123456789012123456
7
7
1 years, and his father’s ambition was for (75) him was the vast repertory of folk
12234567890123456789012345678901212345678901234567890123456789012123456 7
1234567890123456789012345678901212345678901234567890123456789012123456
him to become a lawyer and tend to the narrative, song, and custom that had 7
1234567890123456789012345678901212345678901234567890123456789012123456 7
1234567890123456789012345678901212345678901234567890123456789012123456
family lands and business. Juan’s scarcely
34567890123456789012345678901212345678901234567890123456789012123456
been documented. While 7
1 (35) elementary schooling was at Saint
12234567890123456789012345678901212345678901234567890123456789012123456teaching at the University of Oregon, he 7 7
1234567890123456789012345678901212345678901234567890123456789012123456 7
1234567890123456789012345678901212345678901234567890123456789012123456
Michael’s College in Santa Fe, and his returned to Arroyo Hondo in the summer 7
1234567890123456789012345678901212345678901234567890123456789012123456
34567890123456789012345678901212345678901234567890123456789012123456
7
7
1 high school studies were at the Christian (80) of 1930 to begin compiling his famous
12234567890123456789012345678901212345678901234567890123456789012123456 7
1234567890123456789012345678901212345678901234567890123456789012123456
Brothers’ College in St. Louis, Missouri. collection of over five hundred Nuevo 7
1234567890123456789012345678901212345678901234567890123456789012123456
The boy’s semester-long absences from 7
1234567890123456789012345678901212345678901234567890123456789012123456 Mexicano folk tales. 7
34567890123456789012345678901212345678901234567890123456789012123456
1234567890123456789012345678901212345678901234567890123456789012123456
his family led him to treasure the simple By then his work had attracted the 7
(40)
1234567890123456789012345678901212345678901234567890123456789012123456 7
1234567890123456789012345678901212345678901234567890123456789012123456
pleasures of village life. Summers are attention of pioneer Hispano folklorist 7
1234567890123456789012345678901212345678901234567890123456789012123456 77
1234567890123456789012345678901212345678901234567890123456789012123456
especially beautiful in Arroyo Hondo, (85) Aurelio Espinosa who invited Rael to
1234567890123456789012345678901212345678901234567890123456789012123456 7
1 and Christmas and Easter vacations were Stanford in 1933. Rael completed his 7
12234567890123456789012345678901212345678901234567890123456789012123456 7
1234567890123456789012345678901212345678901234567890123456789012123456
filled with colorful festivities and solemn doctoral studies in 1937 there with a 7
1234567890123456789012345678901212345678901234567890123456789012123456 7
1234567890123456789012345678901212345678901234567890123456789012123456 77
34567890123456789012345678901212345678901234567890123456789012123456
(45) ceremony. Rael later reminisced about dissertation on the phonology and
1234567890123456789012345678901212345678901234567890123456789012123456 morphology of New Mexico Spanish that 7
1234567890123456789012345678901212345678901234567890123456789012123456
how much the Pastores, or Shepherds’
1234567890123456789012345678901212345678901234567890123456789012123456 amplified the work of Espinosa with the 7
7
1234567890123456789012345678901212345678901234567890123456789012123456
plays, impressed him as a child. Un- (90)
1234567890123456789012345678901212345678901234567890123456789012123456
1234567890123456789012345678901212345678901234567890123456789012123456
doubtedly, the instincts and sympathies huge corpus of folk tales, later published 7 7
1234567890123456789012345678901212345678901234567890123456789012123456 7
1234567890123456789012345678901212345678901234567890123456789012123456 77
of Rael the folklorist can be traced to as, Cuentos Españoles de Colorado y
1 (50) these beginnings—watching rehearsals
12234567890123456789012345678901212345678901234567890123456789012123456Nuevo Mexico: Spanish Folk Tales of 7
1234567890123456789012345678901212345678901234567890123456789012123456 7
1 34567890123456789012345678901212345678901234567890123456789012123456 7
and performances depicting shepherds, Colorado and New Mexico.
1234567890123456789012345678901212345678901234567890123456789012123456 Well versed in the historic-geographic 7
12234567890123456789012345678901212345678901234567890123456789012123456
hermits, and the rich ensemble of (95)
34567890123456789012345678901212345678901234567890123456789012123456
7
7
1234567890123456789012345678901212345678901234567890123456789012123456
1234567890123456789012345678901212345678901234567890123456789012123456
pastoral characters. theory of transmission and diffusion of 7
1 7
12234567890123456789012345678901212345678901234567890123456789012123456
What became clear in his post- motifs, tale types, and genres, Dr. Rael 7
1234567890123456789012345678901212345678901234567890123456789012123456 7
1234567890123456789012345678901212345678901234567890123456789012123456 77
34567890123456789012345678901212345678901234567890123456789012123456
(55) secondary studies is that he was much set out on the formidable, almost
1234567890123456789012345678901212345678901234567890123456789012123456
more attracted to literature, philology, quixotic task of gathering all the possible 7
1
1234567890123456789012345678901212345678901234567890123456789012123456
and the emerging disciplines of linguistics (100) versions and texts of the tales, hymns, 7
1234567890123456789012345678901212345678901234567890123456789012123456 7
2 34567890123456789012345678901212345678901234567890123456789012123456
1234567890123456789012345678901212345678901234567890123456789012123456
and folklore. His Bachelor’s degree, from and plays he was studying. The vast 7
1234567890123456789012345678901212345678901234567890123456789012123456 7
1234567890123456789012345678901212345678901234567890123456789012123456
St. Mary’s College in Oakland in 1923, majority of tales are of European 7
1234567890123456789012345678901212345678901234567890123456789012123456 77
1 (60) led to a Master’s degree from the provenance, with only minimal local
12234567890123456789012345678901212345678901234567890123456789012123456 7
1234567890123456789012345678901212345678901234567890123456789012123456
University of California at Berkeley in references. He meticulously traced the 7
1234567890123456789012345678901212345678901234567890123456789012123456 7
1234567890123456789012345678901212345678901234567890123456789012123456
1927. In the meantime, in 1923, he (105) shepherds’ plays to several root sources 7
1 34567890123456789012345678901212345678901234567890123456789012123456 7
1234567890123456789012345678901212345678901234567890123456789012123456 77
2 34567890123456789012345678901212345678901234567890123456789012123456
married Quirina Espinoza of Antonito, in Mexico, and his study, The Sources
1234567890123456789012345678901212345678901234567890123456789012123456 and Diffusion of the Mexican Shepherds’ 7
1234567890123456789012345678901212345678901234567890123456789012123456
Colorado. Rael’s first inclination was to
7
1234567890123456789012345678901212345678901234567890123456789012123456 7
1 (65) become an English teacher, but his bride Plays, is a standard reference on the
2 34567890123456789012345678901212345678901234567890123456789012123456
1234567890123456789012345678901212345678901234567890123456789012123456 subject. His ground-breaking study of the 7
1 helped convince him that his opportuni- 7
12234567890123456789012345678901212345678901234567890123456789012123456 7
1 34567890123456789012345678901212345678901234567890123456789012123456 7
ties and strengths would be as an (110) alabado hymn, The New Mexican
12234567890123456789012345678901212345678901234567890123456789012123456 7
1 34567890123456789012345678901212345678901234567890123456789012123456
Hispanist. After deciding on a university Alabado, is also a prime resource. But 7
1234567890123456789012345678901212345678901234567890123456789012123456 7
1234567890123456789012345678901212345678901234567890123456789012123456 77
career of teaching and research, Rael inevitably the historic-geographic
1234567890123456789012345678901212345678901234567890123456789012123456
12345678901234567890123456789012123456789012345678901234567890121234567
Copyright © 2005 Thomson Peterson’s, a part of The Thomson Corporaton 27
SAT is a registered trademark of the College Entrance Examination Board, which
was not involved in the production of and does not endorse this product.
12345678901234567890123456789012123456789012345678901234567890121234567
1234567890123456789012345678901212345678901234567890123456789012123456 7
12 34567890123456789012345678901212345678901234567890123456789012123456 7
1234567890123456789012345678901212345678901234567890123456789012123456
approach led more to collection building 19. The author probably uses the word 7
12 34567890123456789012345678901212345678901234567890123456789012123456 7
12 34567890123456789012345678901212345678901234567890123456789012123456
than to analysis. Later generations of “relinquished” in line 70 to emphasize 7
1234567890123456789012345678901212345678901234567890123456789012123456 7
12
2
34567890123456789012345678901212345678901234567890123456789012123456 7
(115) scholars would develop interests in that
34567890123456789012345678901212345678901234567890123456789012123456 7
1
12 34567890123456789012345678901212345678901234567890123456789012123456
performance-centered studies, but the 7
12 34567890123456789012345678901212345678901234567890123456789012123456
(A) Rael had had friction with his 7
12 34567890123456789012345678901212345678901234567890123456789012123456
collections of Rael continue to be an 7
12
2
34567890123456789012345678901212345678901234567890123456789012123456
siblings.
34567890123456789012345678901212345678901234567890123456789012123456
7
7
1 indispensable landmark in the field.
12 34567890123456789012345678901212345678901234567890123456789012123456
(B) Rael’s family was very wealthy. 7
12 34567890123456789012345678901212345678901234567890123456789012123456 7
12 34567890123456789012345678901212345678901234567890123456789012123456 7
(C) Rael had tried to be a rancher for
12 34567890123456789012345678901212345678901234567890123456789012123456
16. The author’s attitude towards Dr. Rael’s 7
1234567890123456789012345678901212345678901234567890123456789012123456
some time. 7
12 34567890123456789012345678901212345678901234567890123456789012123456
work can best be described as 7
12 34567890123456789012345678901212345678901234567890123456789012123456 7
(D) Rael was relieved to be free of his
12 34567890123456789012345678901212345678901234567890123456789012123456 7
12
2
34567890123456789012345678901212345678901234567890123456789012123456
(A) laudatory. family duties.
34567890123456789012345678901212345678901234567890123456789012123456
7
7
1 (E) Rael’s career path was difficult and
12 34567890123456789012345678901212345678901234567890123456789012123456
(B) engaged. 7
12 34567890123456789012345678901212345678901234567890123456789012123456
not lucrative. 7
12 34567890123456789012345678901212345678901234567890123456789012123456 7
(C) ambivalent.
12 34567890123456789012345678901212345678901234567890123456789012123456 7
12 34567890123456789012345678901212345678901234567890123456789012123456
(D) disinterested. 7
12 34567890123456789012345678901212345678901234567890123456789012123456
20. It can be inferred from the passage that 7
12 34567890123456789012345678901212345678901234567890123456789012123456
(E) condescending. 7
12 34567890123456789012345678901212345678901234567890123456789012123456 7
northern New Mexico in the early decades
12 34567890123456789012345678901212345678901234567890123456789012123456 7
12 34567890123456789012345678901212345678901234567890123456789012123456
17. The passage primarily of the twentieth century 7
1234567890123456789012345678901212345678901234567890123456789012123456 7
12 34567890123456789012345678901212345678901234567890123456789012123456 7
12 34567890123456789012345678901212345678901234567890123456789012123456
(A) analyzes the academic contributions (A) had a ranch-driven economy. 7
12 34567890123456789012345678901212345678901234567890123456789012123456 7
1234567890123456789012345678901212345678901234567890123456789012123456 77
2 34567890123456789012345678901212345678901234567890123456789012123456
of Dr. Rael. (B) was suffering severe economic
1234567890123456789012345678901212345678901234567890123456789012123456
1234567890123456789012345678901212345678901234567890123456789012123456
(B) contrasts Dr. Rael’s work with the depression. 7
1234567890123456789012345678901212345678901234567890123456789012123456
(C) was economically booming because 7
1234567890123456789012345678901212345678901234567890123456789012123456 77
work of Dr. Espinosa.
1234567890123456789012345678901212345678901234567890123456789012123456
of a newly opened southern railroad
1234567890123456789012345678901212345678901234567890123456789012123456
(C) tells the story of Dr. Rael’s life and 7
1234567890123456789012345678901212345678901234567890123456789012123456
route to California. 7
1234567890123456789012345678901212345678901234567890123456789012123456 77
work.
1 (D) had a strong public education system.
12 34567890123456789012345678901212345678901234567890123456789012123456
(D) discusses the basic assumptions of 7
12 34567890123456789012345678901212345678901234567890123456789012123456
(E) was not a primarily English-speaking 7
1234567890123456789012345678901212345678901234567890123456789012123456
Hispano scholarship. 7
1234567890123456789012345678901212345678901234567890123456789012123456
region. 7
12 34567890123456789012345678901212345678901234567890123456789012123456 7
(E) relates the story of the Rael family.
1234567890123456789012345678901212345678901234567890123456789012123456 77
2 34567890123456789012345678901212345678901234567890123456789012123456
1234567890123456789012345678901212345678901234567890123456789012123456 7
1 18. The passage implies that Rael decided that 21. Which of the following best describes Dr.
12 34567890123456789012345678901212345678901234567890123456789012123456 7
12 34567890123456789012345678901212345678901234567890123456789012123456
he would have more opportunity as a Espinosa’s relationship to Dr. Rael? 7
2 34567890123456789012345678901212345678901234567890123456789012123456
1234567890123456789012345678901212345678901234567890123456789012123456 7
1234567890123456789012345678901212345678901234567890123456789012123456
Hispanist than as an English teacher (A) Boss 7
1 7
1234567890123456789012345678901212345678901234567890123456789012123456
because (B) Critic 7
1234567890123456789012345678901212345678901234567890123456789012123456 7
2 34567890123456789012345678901212345678901234567890123456789012123456
1234567890123456789012345678901212345678901234567890123456789012123456
(C) Mentor 7
1234567890123456789012345678901212345678901234567890123456789012123456
(A) Hispanics weren’t often hired to 7
1234567890123456789012345678901212345678901234567890123456789012123456
(D) Pastor 7
1234567890123456789012345678901212345678901234567890123456789012123456 77
teach English.
1 (E) Father
12 34567890123456789012345678901212345678901234567890123456789012123456
(B) Hispanic folklore would soon vanish. 7
12 34567890123456789012345678901212345678901234567890123456789012123456 7
12 34567890123456789012345678901212345678901234567890123456789012123456
(C) he would have to live far from his 7
12 34567890123456789012345678901212345678901234567890123456789012123456
22. The word “corpus” in line 91 most closely 7
1234567890123456789012345678901212345678901234567890123456789012123456
family to teach English. 7
2 34567890123456789012345678901212345678901234567890123456789012123456
1234567890123456789012345678901212345678901234567890123456789012123456
means 7
1234567890123456789012345678901212345678901234567890123456789012123456
(D) his English skills were mediocre. 7
1234567890123456789012345678901212345678901234567890123456789012123456 7
1234567890123456789012345678901212345678901234567890123456789012123456 77
(E) becoming an Hispanist was a nearer (A) religious text.
1 match to his educational back-
12 34567890123456789012345678901212345678901234567890123456789012123456
(B) oeuvre. 7
1234567890123456789012345678901212345678901234567890123456789012123456
ground. (C) published collection. 7
12 34567890123456789012345678901212345678901234567890123456789012123456 7
1234567890123456789012345678901212345678901234567890123456789012123456
(D) dissertation. 7
12
2
34567890123456789012345678901212345678901234567890123456789012123456
34567890123456789012345678901212345678901234567890123456789012123456
7
7
1 (E) dialect.
1234567890123456789012345678901212345678901234567890123456789012123456 7
1234567890123456789012345678901212345678901234567890123456789012123456 77
1234567890123456789012345678901212345678901234567890123456789012123456
12345678901234567890123456789012123456789012345678901234567890121234567
28 Copyright © 2005 Thomson Peterson’s, a part of The Thomson Corporaton
SAT is a registered trademark of the College Entrance Examination Board, which
was not involved in the production of and does not endorse this product.
12345678901234567890123456789012123456789012345678901234567890121234567
1234567890123456789012345678901212345678901234567890123456789012123456 7
12234567890123456789012345678901212345678901234567890123456789012123456 7
1 34567890123456789012345678901212345678901234567890123456789012123456
23. According to the passage, which of the 26. The main criticism the author offers of Dr. 7
12234567890123456789012345678901212345678901234567890123456789012123456 7
1234567890123456789012345678901212345678901234567890123456789012123456
following of Dr. Rael’s activities as a Rael’s work is that it 7
1 34567890123456789012345678901212345678901234567890123456789012123456 7
12234567890123456789012345678901212345678901234567890123456789012123456
young man was most important for the
34567890123456789012345678901212345678901234567890123456789012123456
7
7
1 (A) focuses too much on the Pastores.
12234567890123456789012345678901212345678901234567890123456789012123456
development of his later academic 7
1234567890123456789012345678901212345678901234567890123456789012123456
(B) privileges Spanish-language stories 7
1234567890123456789012345678901212345678901234567890123456789012123456
interests? 7
1234567890123456789012345678901212345678901234567890123456789012123456 over
34567890123456789012345678901212345678901234567890123456789012123456
English-language stories. 7
7
1
12234567890123456789012345678901212345678901234567890123456789012123456
(A) Working on his family’s ranch (C) focuses too much on European 7
1234567890123456789012345678901212345678901234567890123456789012123456 7
1234567890123456789012345678901212345678901234567890123456789012123456
(B) Watching Pastores as a young man stories and not enough on Mexican 7
1234567890123456789012345678901212345678901234567890123456789012123456
34567890123456789012345678901212345678901234567890123456789012123456
7
7
1 (C) Studying in a religious school stories.
12234567890123456789012345678901212345678901234567890123456789012123456 7
1234567890123456789012345678901212345678901234567890123456789012123456
(D) Reading books on the shepherds of (D) doesn’t offer enough analysis of the 7
1234567890123456789012345678901212345678901234567890123456789012123456 7
1234567890123456789012345678901212345678901234567890123456789012123456
northern New Mexico folklore.
34567890123456789012345678901212345678901234567890123456789012123456
7
1 (E) Struggling to retain his Spanish when (E) includes too many materials, without 7
12234567890123456789012345678901212345678901234567890123456789012123456 7
1234567890123456789012345678901212345678901234567890123456789012123456
his schooling was in English differentiating between good and 7
1234567890123456789012345678901212345678901234567890123456789012123456 7
1234567890123456789012345678901212345678901234567890123456789012123456 bad. 7
34567890123456789012345678901212345678901234567890123456789012123456
1234567890123456789012345678901212345678901234567890123456789012123456 7
1234567890123456789012345678901212345678901234567890123456789012123456
24. The phrase “diffusion of motifs” in lines 7
1234567890123456789012345678901212345678901234567890123456789012123456 7
1234567890123456789012345678901212345678901234567890123456789012123456 77
96–97 refers to 27. The passage suggests, in lines 97–101 that
1234567890123456789012345678901212345678901234567890123456789012123456
1234567890123456789012345678901212345678901234567890123456789012123456
Dr. Rael 7
1 (A) variations in the same stories that 7
12234567890123456789012345678901212345678901234567890123456789012123456 7
1234567890123456789012345678901212345678901234567890123456789012123456
occur over time or by region. (A) was mistaken about how much 7
1234567890123456789012345678901212345678901234567890123456789012123456 7
1234567890123456789012345678901212345678901234567890123456789012123456 77
34567890123456789012345678901212345678901234567890123456789012123456
(B) adherence to standard literary folklore was circulating.
1234567890123456789012345678901212345678901234567890123456789012123456
1234567890123456789012345678901212345678901234567890123456789012123456
structures. (B) traveled extensively as he gathered 7
1234567890123456789012345678901212345678901234567890123456789012123456
(C) variations in language use by region. folklore. 7
1234567890123456789012345678901212345678901234567890123456789012123456 77
1234567890123456789012345678901212345678901234567890123456789012123456
(D) the surprising similarity of stories in (C) was disorganized but intelligent in
1234567890123456789012345678901212345678901234567890123456789012123456 7
1234567890123456789012345678901212345678901234567890123456789012123456
different cultures. his methods. 7
1234567890123456789012345678901212345678901234567890123456789012123456 77
1 (E) the loss of folklore in more industri- (D) had an historic insight about the
12234567890123456789012345678901212345678901234567890123456789012123456 7
1234567890123456789012345678901212345678901234567890123456789012123456
alized societies. source of New Mexican folklore. 7
1 34567890123456789012345678901212345678901234567890123456789012123456 7
1234567890123456789012345678901212345678901234567890123456789012123456
(E) developed an excessively technical 7
12234567890123456789012345678901212345678901234567890123456789012123456 7
1234567890123456789012345678901212345678901234567890123456789012123456 77
34567890123456789012345678901212345678901234567890123456789012123456
25. The word “provenance,” line 103, could model for the development of
1234567890123456789012345678901212345678901234567890123456789012123456 7
1 best be replaced with which of the folklore.
12234567890123456789012345678901212345678901234567890123456789012123456 7
1234567890123456789012345678901212345678901234567890123456789012123456
following words? 7
1234567890123456789012345678901212345678901234567890123456789012123456 77
34567890123456789012345678901212345678901234567890123456789012123456
1234567890123456789012345678901212345678901234567890123456789012123456
(A) Authority 7
1
1234567890123456789012345678901212345678901234567890123456789012123456
(B) Origin 7
1234567890123456789012345678901212345678901234567890123456789012123456 7
2 34567890123456789012345678901212345678901234567890123456789012123456
1234567890123456789012345678901212345678901234567890123456789012123456
(C) Location 7
1234567890123456789012345678901212345678901234567890123456789012123456 7
1234567890123456789012345678901212345678901234567890123456789012123456
(D) Destiny 7
1234567890123456789012345678901212345678901234567890123456789012123456 77
1 (E) Ethos
12234567890123456789012345678901212345678901234567890123456789012123456 7
1234567890123456789012345678901212345678901234567890123456789012123456 7
1234567890123456789012345678901212345678901234567890123456789012123456 7
1234567890123456789012345678901212345678901234567890123456789012123456 7
1 34567890123456789012345678901212345678901234567890123456789012123456 7
1234567890123456789012345678901212345678901234567890123456789012123456 77
2 34567890123456789012345678901212345678901234567890123456789012123456
1234567890123456789012345678901212345678901234567890123456789012123456 7
1234567890123456789012345678901212345678901234567890123456789012123456 7
1234567890123456789012345678901212345678901234567890123456789012123456 7
1
12234567890123456789012345678901212345678901234567890123456789012123456 7
1 34567890123456789012345678901212345678901234567890123456789012123456 7
12234567890123456789012345678901212345678901234567890123456789012123456 7
1 34567890123456789012345678901212345678901234567890123456789012123456 7
12234567890123456789012345678901212345678901234567890123456789012123456 7
STOP
1 34567890123456789012345678901212345678901234567890123456789012123456
Do not proceed to the next section until time is up.
1234567890123456789012345678901212345678901234567890123456789012123456 7
7
1234567890123456789012345678901212345678901234567890123456789012123456 7
1234567890123456789012345678901212345678901234567890123456789012123456 7
12345678901234567890123456789012123456789012345678901234567890121234567
Copyright © 2005 Thomson Peterson’s, a part of The Thomson Corporaton 29
SAT is a registered trademark of the College Entrance Examination Board, which
was not involved in the production of and does not endorse this product.
Section 5
21 Questions j Time—25 Minutes

12345678901234567890123456789012123456789012345678901234567890121234567
12 34567890123456789012345678901212345678901234567890123456789012123456 7
12 34567890123456789012345678901212345678901234567890123456789012123456 7
1 34567890123456789012345678901212345678901234567890123456789012123456 7
2
12 34567890123456789012345678901212345678901234567890123456789012123456 7
12 34567890123456789012345678901212345678901234567890123456789012123456
Directions: Solve the following problems using any available space on the page for scratchwork. 7
12 34567890123456789012345678901212345678901234567890123456789012123456
Mark the letter of your choice on the answer sheet that best corresponds to the correct answer. 7
12 34567890123456789012345678901212345678901234567890123456789012123456 7
1234567890123456789012345678901212345678901234567890123456789012123456 77
2 34567890123456789012345678901212345678901234567890123456789012123456
1234567890123456789012345678901212345678901234567890123456789012123456
Notes:
1234567890123456789012345678901212345678901234567890123456789012123456 7
1234567890123456789012345678901212345678901234567890123456789012123456
1. You may use a calculator. All of the numbers used are real numbers. 7
1234567890123456789012345678901212345678901234567890123456789012123456 77
1234567890123456789012345678901212345678901234567890123456789012123456 7
1 2. You may use the figures that accompany the problems to help you find the solution. Unless
12 34567890123456789012345678901212345678901234567890123456789012123456 7
12 34567890123456789012345678901212345678901234567890123456789012123456 7
the instructions say that a figure is not drawn to scale, assume that it has been drawn
12 34567890123456789012345678901212345678901234567890123456789012123456 7
12 34567890123456789012345678901212345678901234567890123456789012123456
accurately. Each figure lies in a plane unless the instructions say otherwise. 7
12 34567890123456789012345678901212345678901234567890123456789012123456 7
12 34567890123456789012345678901212345678901234567890123456789012123456 7
12 34567890123456789012345678901212345678901234567890123456789012123456 7
12 34567890123456789012345678901212345678901234567890123456789012123456 7
1234567890123456789012345678901212345678901234567890123456789012123456
2 7
Reference Information

2s
1234567890123456789012345678901212345678901234567890123456789012123456
r w 2x
45
7
1234567890123456789012345678901212345678901234567890123456789012123456
h h
r
c x s
60
7
1234567890123456789012345678901212345678901234567890123456789012123456 h b 7
1 34567890123456789012345678901212345678901234567890123456789012123456 7
30
45
w
2 34567890123456789012345678901212345678901234567890123456789012123456
1234567890123456789012345678901212345678901234567890123456789012123456 7
1234567890123456789012345678901212345678901234567890123456789012123456
A 5 pr2 b a 3x s 7
1 1 7
1234567890123456789012345678901212345678901234567890123456789012123456 7
2 2 2 2
C 5 2pr A 5 ,w A 5 bh V 5 ,wh V 5 pr h c 5 a 1 b Special Right Triangles
12
2
34567890123456789012345678901212345678901234567890123456789012123456 7
2
34567890123456789012345678901212345678901234567890123456789012123456 7
1234567890123456789012345678901212345678901234567890123456789012123456
The number of degrees of arc in a circle is 360.
1234567890123456789012345678901212345678901234567890123456789012123456 7
1 The measure in degrees of a straight angle is 180. 7
12 34567890123456789012345678901212345678901234567890123456789012123456 7
12 34567890123456789012345678901212345678901234567890123456789012123456 7
The sum of the measures in degrees of the angles of a triangle is 180.
1234567890123456789012345678901212345678901234567890123456789012123456 77
2 34567890123456789012345678901212345678901234567890123456789012123456
1234567890123456789012345678901212345678901234567890123456789012123456 7
1
1234567890123456789012345678901212345678901234567890123456789012123456 2x 7
1234567890123456789012345678901212345678901234567890123456789012123456
1. At full capacity, Thompson Paper Factory 2. If Z 5 and Z 5 3, then x 5 7
2 34567890123456789012345678901212345678901234567890123456789012123456
1234567890123456789012345678901212345678901234567890123456789012123456 5 7
1234567890123456789012345678901212345678901234567890123456789012123456
produces 200 sheets of paper per second. 7
1234567890123456789012345678901212345678901234567890123456789012123456 5 7
1234567890123456789012345678901212345678901234567890123456789012123456
If the factory is operating at a quarter of (A) 7
1 3 7
1234567890123456789012345678901212345678901234567890123456789012123456 77
2 34567890123456789012345678901212345678901234567890123456789012123456
its full capacity, how many sheets of paper
1234567890123456789012345678901212345678901234567890123456789012123456(B) 2
7
1234567890123456789012345678901212345678901234567890123456789012123456
will the factory produce in twelve sec-
7
1234567890123456789012345678901212345678901234567890123456789012123456
onds? 7
1 (C) 5
1234567890123456789012345678901212345678901234567890123456789012123456 77
2 34567890123456789012345678901212345678901234567890123456789012123456
1234567890123456789012345678901212345678901234567890123456789012123456
(A) 600
7
1234567890123456789012345678901212345678901234567890123456789012123456 19
7
1234567890123456789012345678901212345678901234567890123456789012123456
(B) 900 (D)
7
1 3
1234567890123456789012345678901212345678901234567890123456789012123456 77
2 34567890123456789012345678901212345678901234567890123456789012123456
(C) 1200
1 15
12 34567890123456789012345678901212345678901234567890123456789012123456
(D) 2000 (E) 7
1234567890123456789012345678901212345678901234567890123456789012123456 2 7
12
2
34567890123456789012345678901212345678901234567890123456789012123456 7
(E) 2400
34567890123456789012345678901212345678901234567890123456789012123456 7
1
1234567890123456789012345678901212345678901234567890123456789012123456 7
1234567890123456789012345678901212345678901234567890123456789012123456 7
1234567890123456789012345678901212345678901234567890123456789012123456 7
12345678901234567890123456789012123456789012345678901234567890121234567
30 Copyright © 2005 Thomson Peterson’s, a part of The Thomson Corporaton
SAT is a registered trademark of the College Entrance Examination Board, which
was not involved in the production of and does not endorse this product.
12345678901234567890123456789012123456789012345678901234567890121234567
1234567890123456789012345678901212345678901234567890123456789012123456 7
12234567890123456789012345678901212345678901234567890123456789012123456 7
1 34567890123456789012345678901212345678901234567890123456789012123456
3. Which of the following is the greatest Questions 5–6 refer to the following table. 7
12234567890123456789012345678901212345678901234567890123456789012123456 7
1234567890123456789012345678901212345678901234567890123456789012123456
common factor of 32 and 42? 7
1 34567890123456789012345678901212345678901234567890123456789012123456 7
12234567890123456789012345678901212345678901234567890123456789012123456
5.
34567890123456789012345678901212345678901234567890123456789012123456
7
7
1 (A) 2
12234567890123456789012345678901212345678901234567890123456789012123456 7
1234567890123456789012345678901212345678901234567890123456789012123456
(B) 3 7
1234567890123456789012345678901212345678901234567890123456789012123456 7
1234567890123456789012345678901212345678901234567890123456789012123456
(C) 6
34567890123456789012345678901212345678901234567890123456789012123456
7
7
1
12234567890123456789012345678901212345678901234567890123456789012123456
(D) 8 7
1234567890123456789012345678901212345678901234567890123456789012123456 7
1234567890123456789012345678901212345678901234567890123456789012123456
(E) 12 7
1234567890123456789012345678901212345678901234567890123456789012123456
34567890123456789012345678901212345678901234567890123456789012123456
7
7
1
12234567890123456789012345678901212345678901234567890123456789012123456
4. 7
1234567890123456789012345678901212345678901234567890123456789012123456 7
1234567890123456789012345678901212345678901234567890123456789012123456 7
1234567890123456789012345678901212345678901234567890123456789012123456
34567890123456789012345678901212345678901234567890123456789012123456
7
7
1
12234567890123456789012345678901212345678901234567890123456789012123456
According to the table, how many 7
1234567890123456789012345678901212345678901234567890123456789012123456 rd th 7
1234567890123456789012345678901212345678901234567890123456789012123456
students are there in 3 and 4 grade at 7
1234567890123456789012345678901212345678901234567890123456789012123456
Hyde Park Elementary? 7
1234567890123456789012345678901212345678901234567890123456789012123456 77
34567890123456789012345678901212345678901234567890123456789012123456
1234567890123456789012345678901212345678901234567890123456789012123456 7
1234567890123456789012345678901212345678901234567890123456789012123456
(A) 30
1234567890123456789012345678901212345678901234567890123456789012123456 7
1234567890123456789012345678901212345678901234567890123456789012123456
(B) 32 7
1234567890123456789012345678901212345678901234567890123456789012123456 7
1 (C) 50 7
12234567890123456789012345678901212345678901234567890123456789012123456 7
1234567890123456789012345678901212345678901234567890123456789012123456
(D) 60 7
1234567890123456789012345678901212345678901234567890123456789012123456 7
1234567890123456789012345678901212345678901234567890123456789012123456 77
34567890123456789012345678901212345678901234567890123456789012123456
(E) 62
1234567890123456789012345678901212345678901234567890123456789012123456 7
1234567890123456789012345678901212345678901234567890123456789012123456
1234567890123456789012345678901212345678901234567890123456789012123456
6. If there are 18 girls in 4th grade, how 7
1234567890123456789012345678901212345678901234567890123456789012123456 7
1234567890123456789012345678901212345678901234567890123456789012123456 rd th 7
1234567890123456789012345678901212345678901234567890123456789012123456
many boys are there in 3 and 4 grade 7
1234567890123456789012345678901212345678901234567890123456789012123456
at Hyde Park Elementary? 7
1234567890123456789012345678901212345678901234567890123456789012123456 77
1
12234567890123456789012345678901212345678901234567890123456789012123456
(A) 18 7
1234567890123456789012345678901212345678901234567890123456789012123456 7
1 34567890123456789012345678901212345678901234567890123456789012123456
(B) 22 7
1234567890123456789012345678901212345678901234567890123456789012123456
What is the slope of the above line? 7
12234567890123456789012345678901212345678901234567890123456789012123456
(C) 26
34567890123456789012345678901212345678901234567890123456789012123456
7
7
1234567890123456789012345678901212345678901234567890123456789012123456
3
1234567890123456789012345678901212345678901234567890123456789012123456
(D) 30 7
1 A. 2 7
12234567890123456789012345678901212345678901234567890123456789012123456
2 (E) 32 7
1234567890123456789012345678901212345678901234567890123456789012123456 7
34567890123456789012345678901212345678901234567890123456789012123456
1234567890123456789012345678901212345678901234567890123456789012123456
2 7
1234567890123456789012345678901212345678901234567890123456789012123456
B. 2 7
1 3 7
1234567890123456789012345678901212345678901234567890123456789012123456 7
1234567890123456789012345678901212345678901234567890123456789012123456
2 7
12234567890123456789012345678901212345678901234567890123456789012123456
C. 7
1234567890123456789012345678901212345678901234567890123456789012123456
3 7
1234567890123456789012345678901212345678901234567890123456789012123456 7
1234567890123456789012345678901212345678901234567890123456789012123456 7
1 34567890123456789012345678901212345678901234567890123456789012123456 7
D. 1
1234567890123456789012345678901212345678901234567890123456789012123456 77
2 34567890123456789012345678901212345678901234567890123456789012123456
1234567890123456789012345678901212345678901234567890123456789012123456
3 7
1234567890123456789012345678901212345678901234567890123456789012123456 7
1234567890123456789012345678901212345678901234567890123456789012123456
E.
7
1 2
1234567890123456789012345678901212345678901234567890123456789012123456 77
2 34567890123456789012345678901212345678901234567890123456789012123456
1234567890123456789012345678901212345678901234567890123456789012123456 7
1234567890123456789012345678901212345678901234567890123456789012123456 7
1234567890123456789012345678901212345678901234567890123456789012123456 7
1
12234567890123456789012345678901212345678901234567890123456789012123456 7
1 34567890123456789012345678901212345678901234567890123456789012123456 7
12234567890123456789012345678901212345678901234567890123456789012123456 7
1 34567890123456789012345678901212345678901234567890123456789012123456 7
12234567890123456789012345678901212345678901234567890123456789012123456 7
1 34567890123456789012345678901212345678901234567890123456789012123456 7
1234567890123456789012345678901212345678901234567890123456789012123456 7
1234567890123456789012345678901212345678901234567890123456789012123456 7
1234567890123456789012345678901212345678901234567890123456789012123456 7
12345678901234567890123456789012123456789012345678901234567890121234567
Copyright © 2005 Thomson Peterson’s, a part of The Thomson Corporaton 31
SAT is a registered trademark of the College Entrance Examination Board, which
was not involved in the production of and does not endorse this product.
12345678901234567890123456789012123456789012345678901234567890121234567
1234567890123456789012345678901212345678901234567890123456789012123456 7
12
2
34567890123456789012345678901212345678901234567890123456789012123456
34567890123456789012345678901212345678901234567890123456789012123456
3x2 7
7
1 7. Which of the following is the value of the 10. If f ~ ! x2 1 3x 2 18
x 5 , for what values
12 34567890123456789012345678901212345678901234567890123456789012123456 7
12 34567890123456789012345678901212345678901234567890123456789012123456
~ 3 22
! 7
1234567890123456789012345678901212345678901234567890123456789012123456
m4 of x is the function undefined? 7
12
2
34567890123456789012345678901212345678901234567890123456789012123456
exponent when the expression is
34567890123456789012345678901212345678901234567890123456789012123456
7
7
1 5
(A) 0
12 34567890123456789012345678901212345678901234567890123456789012123456
m4 7
12 34567890123456789012345678901212345678901234567890123456789012123456
simplified? (B) 23,6 7
12 34567890123456789012345678901212345678901234567890123456789012123456 7
12 34567890123456789012345678901212345678901234567890123456789012123456
(C) 23,3 7
1234567890123456789012345678901212345678901234567890123456789012123456
11 7
12 34567890123456789012345678901212345678901234567890123456789012123456
(A) 2 (D) 2,5 7
12 34567890123456789012345678901212345678901234567890123456789012123456
4 7
12 34567890123456789012345678901212345678901234567890123456789012123456
(E) 3,26 7
12
2
34567890123456789012345678901212345678901234567890123456789012123456 7
29
34567890123456789012345678901212345678901234567890123456789012123456 7
1 (B) 2
12 34567890123456789012345678901212345678901234567890123456789012123456
16 7
12 34567890123456789012345678901212345678901234567890123456789012123456 7
11.
12 34567890123456789012345678901212345678901234567890123456789012123456 7
12
2
34567890123456789012345678901212345678901234567890123456789012123456
3
34567890123456789012345678901212345678901234567890123456789012123456
7
7
1 (C) 2
12 34567890123456789012345678901212345678901234567890123456789012123456
4 7
12 34567890123456789012345678901212345678901234567890123456789012123456 7
12 34567890123456789012345678901212345678901234567890123456789012123456
11 7
12 34567890123456789012345678901212345678901234567890123456789012123456
(D) 7
1234567890123456789012345678901212345678901234567890123456789012123456 77
2 34567890123456789012345678901212345678901234567890123456789012123456
4
1234567890123456789012345678901212345678901234567890123456789012123456 7
1234567890123456789012345678901212345678901234567890123456789012123456
29
1234567890123456789012345678901212345678901234567890123456789012123456
(E) 7
1234567890123456789012345678901212345678901234567890123456789012123456
6 7
1234567890123456789012345678901212345678901234567890123456789012123456 77
1
12 34567890123456789012345678901212345678901234567890123456789012123456 7
12 34567890123456789012345678901212345678901234567890123456789012123456
8. 7
12 34567890123456789012345678901212345678901234567890123456789012123456 7
1234567890123456789012345678901212345678901234567890123456789012123456 77
2 34567890123456789012345678901212345678901234567890123456789012123456
1234567890123456789012345678901212345678901234567890123456789012123456 7
1234567890123456789012345678901212345678901234567890123456789012123456
1234567890123456789012345678901212345678901234567890123456789012123456
If f(x) is graphed above, then f(x) 5 7
1234567890123456789012345678901212345678901234567890123456789012123456 77
1234567890123456789012345678901212345678901234567890123456789012123456
1234567890123456789012345678901212345678901234567890123456789012123456
(A) x2 1 2 7
1234567890123456789012345678901212345678901234567890123456789012123456 7
1234567890123456789012345678901212345678901234567890123456789012123456
In the right triangle above, what is the (B) x 2
2 2 7
1 7
2 34567890123456789012345678901212345678901234567890123456789012123456
1234567890123456789012345678901212345678901234567890123456789012123456 7
2
value of x? (C) 2x 1 2
1234567890123456789012345678901212345678901234567890123456789012123456 7
1 7
2
(D) 2(x 1 2)
1234567890123456789012345678901212345678901234567890123456789012123456
(A) 15 2 7
12 34567890123456789012345678901212345678901234567890123456789012123456
(E) 2(x 12) 7
1234567890123456789012345678901212345678901234567890123456789012123456 77
2 34567890123456789012345678901212345678901234567890123456789012123456
(B) 16
1234567890123456789012345678901212345678901234567890123456789012123456 7
1 (C) 18 12.
12 34567890123456789012345678901212345678901234567890123456789012123456 7
12 34567890123456789012345678901212345678901234567890123456789012123456 7
(D) 24
2 34567890123456789012345678901212345678901234567890123456789012123456
1234567890123456789012345678901212345678901234567890123456789012123456 7
1234567890123456789012345678901212345678901234567890123456789012123456
(E) 30 7
1 7
1234567890123456789012345678901212345678901234567890123456789012123456 7
1234567890123456789012345678901212345678901234567890123456789012123456
9. Taking the highway from Easton to 7
2 34567890123456789012345678901212345678901234567890123456789012123456
1234567890123456789012345678901212345678901234567890123456789012123456 7
1234567890123456789012345678901212345678901234567890123456789012123456
Bethsaida is 7 miles longer than taking 7
1234567890123456789012345678901212345678901234567890123456789012123456 7
1234567890123456789012345678901212345678901234567890123456789012123456
surface streets from Easton to Bethsaida. 7
1 If the above triangles are congruent, what 7
2 34567890123456789012345678901212345678901234567890123456789012123456
1234567890123456789012345678901212345678901234567890123456789012123456
It is 31 miles total if you travel from 7
1234567890123456789012345678901212345678901234567890123456789012123456
is the value of y? 7
1234567890123456789012345678901212345678901234567890123456789012123456
Easton to Bethsaida via highway and 7
1234567890123456789012345678901212345678901234567890123456789012123456 7
1 return via surface streets. How many miles (A) 5 7
2 34567890123456789012345678901212345678901234567890123456789012123456
1234567890123456789012345678901212345678901234567890123456789012123456 7
(B) =50
1234567890123456789012345678901212345678901234567890123456789012123456
then, is the highway route? 7
1234567890123456789012345678901212345678901234567890123456789012123456 77
1234567890123456789012345678901212345678901234567890123456789012123456
(C) 8
7
1 (A) 12
2 34567890123456789012345678901212345678901234567890123456789012123456
1234567890123456789012345678901212345678901234567890123456789012123456
(D) =75 7
1 (B) 13 7
12 34567890123456789012345678901212345678901234567890123456789012123456
(C) 15 7
1234567890123456789012345678901212345678901234567890123456789012123456
(E) 9 7
12
2
34567890123456789012345678901212345678901234567890123456789012123456
(D) 17
34567890123456789012345678901212345678901234567890123456789012123456
7
7
1
1234567890123456789012345678901212345678901234567890123456789012123456
(E) 19 7
1234567890123456789012345678901212345678901234567890123456789012123456 77
1234567890123456789012345678901212345678901234567890123456789012123456
12345678901234567890123456789012123456789012345678901234567890121234567
32 Copyright © 2005 Thomson Peterson’s, a part of The Thomson Corporaton
SAT is a registered trademark of the College Entrance Examination Board, which
was not involved in the production of and does not endorse this product.
12345678901234567890123456789012123456789012345678901234567890121234567
1234567890123456789012345678901212345678901234567890123456789012123456 7
12234567890123456789012345678901212345678901234567890123456789012123456 7
1 34567890123456789012345678901212345678901234567890123456789012123456
13. (y 1 2)2 5 (y 2 4)2 is true when y equals (B) 7
12234567890123456789012345678901212345678901234567890123456789012123456 7
1234567890123456789012345678901212345678901234567890123456789012123456 7
1 34567890123456789012345678901212345678901234567890123456789012123456 7
(A) 1 only
12234567890123456789012345678901212345678901234567890123456789012123456 7
1 34567890123456789012345678901212345678901234567890123456789012123456
(B) 1 and 21 7
12234567890123456789012345678901212345678901234567890123456789012123456 7
1234567890123456789012345678901212345678901234567890123456789012123456
(C) 2 and 22 7
1234567890123456789012345678901212345678901234567890123456789012123456 7
1234567890123456789012345678901212345678901234567890123456789012123456
(D) 1 and 2
34567890123456789012345678901212345678901234567890123456789012123456
7
7
1
12234567890123456789012345678901212345678901234567890123456789012123456
(E) 2 and 4 7
1234567890123456789012345678901212345678901234567890123456789012123456 7
1234567890123456789012345678901212345678901234567890123456789012123456 7
1234567890123456789012345678901212345678901234567890123456789012123456
14. Z is the set of numbers 1 through 50
34567890123456789012345678901212345678901234567890123456789012123456
7
7
1
12234567890123456789012345678901212345678901234567890123456789012123456
inclusive. How many members of Z are 7
1234567890123456789012345678901212345678901234567890123456789012123456 7
1234567890123456789012345678901212345678901234567890123456789012123456
evenly divisible by 2 and 3? 7
1234567890123456789012345678901212345678901234567890123456789012123456
34567890123456789012345678901212345678901234567890123456789012123456
7
7
1 (C)
12234567890123456789012345678901212345678901234567890123456789012123456
(A) 6 7
1234567890123456789012345678901212345678901234567890123456789012123456 7
1234567890123456789012345678901212345678901234567890123456789012123456
(B) 8 7
1234567890123456789012345678901212345678901234567890123456789012123456 7
1234567890123456789012345678901212345678901234567890123456789012123456 77
34567890123456789012345678901212345678901234567890123456789012123456
(C) 14
1234567890123456789012345678901212345678901234567890123456789012123456
1234567890123456789012345678901212345678901234567890123456789012123456
(D) 16 7
1234567890123456789012345678901212345678901234567890123456789012123456 7
1234567890123456789012345678901212345678901234567890123456789012123456 77
(E) 25
1234567890123456789012345678901212345678901234567890123456789012123456 7
1
12234567890123456789012345678901212345678901234567890123456789012123456
15. 7
1234567890123456789012345678901212345678901234567890123456789012123456 7
1234567890123456789012345678901212345678901234567890123456789012123456 7
1234567890123456789012345678901212345678901234567890123456789012123456 77
34567890123456789012345678901212345678901234567890123456789012123456
1234567890123456789012345678901212345678901234567890123456789012123456 7
1234567890123456789012345678901212345678901234567890123456789012123456 7
1234567890123456789012345678901212345678901234567890123456789012123456 7
1234567890123456789012345678901212345678901234567890123456789012123456 7
1234567890123456789012345678901212345678901234567890123456789012123456
(D)
1234567890123456789012345678901212345678901234567890123456789012123456 7
1234567890123456789012345678901212345678901234567890123456789012123456 77
1234567890123456789012345678901212345678901234567890123456789012123456 7
1
12234567890123456789012345678901212345678901234567890123456789012123456 7
1234567890123456789012345678901212345678901234567890123456789012123456 7
1 34567890123456789012345678901212345678901234567890123456789012123456 7
1234567890123456789012345678901212345678901234567890123456789012123456 7
12234567890123456789012345678901212345678901234567890123456789012123456 7
1234567890123456789012345678901212345678901234567890123456789012123456 7
1234567890123456789012345678901212345678901234567890123456789012123456 7
1 34567890123456789012345678901212345678901234567890123456789012123456 7
12234567890123456789012345678901212345678901234567890123456789012123456 7
1234567890123456789012345678901212345678901234567890123456789012123456 7
1234567890123456789012345678901212345678901234567890123456789012123456 77
34567890123456789012345678901212345678901234567890123456789012123456
Which of the following figures is similar
1234567890123456789012345678901212345678901234567890123456789012123456
to the shape shown above? 7
1
1234567890123456789012345678901212345678901234567890123456789012123456 7
1234567890123456789012345678901212345678901234567890123456789012123456
(A) 7
12234567890123456789012345678901212345678901234567890123456789012123456 7
1234567890123456789012345678901212345678901234567890123456789012123456
(E) 7
1234567890123456789012345678901212345678901234567890123456789012123456 7
1234567890123456789012345678901212345678901234567890123456789012123456 7
1 34567890123456789012345678901212345678901234567890123456789012123456 7
1234567890123456789012345678901212345678901234567890123456789012123456 77
2 34567890123456789012345678901212345678901234567890123456789012123456
1234567890123456789012345678901212345678901234567890123456789012123456 7
1234567890123456789012345678901212345678901234567890123456789012123456 7
1234567890123456789012345678901212345678901234567890123456789012123456 7
1
12234567890123456789012345678901212345678901234567890123456789012123456 7
1234567890123456789012345678901212345678901234567890123456789012123456 7
1234567890123456789012345678901212345678901234567890123456789012123456 7
1234567890123456789012345678901212345678901234567890123456789012123456 7
1 34567890123456789012345678901212345678901234567890123456789012123456 7
1234567890123456789012345678901212345678901234567890123456789012123456 77
2 34567890123456789012345678901212345678901234567890123456789012123456
1
12234567890123456789012345678901212345678901234567890123456789012123456 7
1 34567890123456789012345678901212345678901234567890123456789012123456 7
12234567890123456789012345678901212345678901234567890123456789012123456 7
1 34567890123456789012345678901212345678901234567890123456789012123456 7
1234567890123456789012345678901212345678901234567890123456789012123456 7
1234567890123456789012345678901212345678901234567890123456789012123456 7
1234567890123456789012345678901212345678901234567890123456789012123456 7
12345678901234567890123456789012123456789012345678901234567890121234567
Copyright © 2005 Thomson Peterson’s, a part of The Thomson Corporaton 33
SAT is a registered trademark of the College Entrance Examination Board, which
was not involved in the production of and does not endorse this product.
12345678901234567890123456789012123456789012345678901234567890121234567
1234567890123456789012345678901212345678901234567890123456789012123456 7
12 34567890123456789012345678901212345678901234567890123456789012123456 7
1234567890123456789012345678901212345678901234567890123456789012123456
16. 18. 7
12 34567890123456789012345678901212345678901234567890123456789012123456 7
12 34567890123456789012345678901212345678901234567890123456789012123456 7
1234567890123456789012345678901212345678901234567890123456789012123456 7
12 34567890123456789012345678901212345678901234567890123456789012123456 7
1234567890123456789012345678901212345678901234567890123456789012123456 7
12 34567890123456789012345678901212345678901234567890123456789012123456 7
12 34567890123456789012345678901212345678901234567890123456789012123456 7
12 34567890123456789012345678901212345678901234567890123456789012123456 7
12 34567890123456789012345678901212345678901234567890123456789012123456 7
1 34567890123456789012345678901212345678901234567890123456789012123456 7
2
12 34567890123456789012345678901212345678901234567890123456789012123456 7
12 34567890123456789012345678901212345678901234567890123456789012123456
ABC is an equilateral triangle and DEFG 7
12 34567890123456789012345678901212345678901234567890123456789012123456 7
12
2
34567890123456789012345678901212345678901234567890123456789012123456
34567890123456789012345678901212345678901234567890123456789012123456
7
7
1 is a square. If AB 5 DE, how many
12 34567890123456789012345678901212345678901234567890123456789012123456
different ways can ABC be placed in 7
12 34567890123456789012345678901212345678901234567890123456789012123456 7
12 34567890123456789012345678901212345678901234567890123456789012123456
DEFG such that two vertices of the 7
12
2
34567890123456789012345678901212345678901234567890123456789012123456
34567890123456789012345678901212345678901234567890123456789012123456
7
7
1 What is the area of the above figure? triangle coincide with two corners of
12 34567890123456789012345678901212345678901234567890123456789012123456 7
12 34567890123456789012345678901212345678901234567890123456789012123456
the square? 7
12 34567890123456789012345678901212345678901234567890123456789012123456 7
(A) 1
12 34567890123456789012345678901212345678901234567890123456789012123456 7
12 34567890123456789012345678901212345678901234567890123456789012123456
(B) =2 (A) 4 7
12 34567890123456789012345678901212345678901234567890123456789012123456 7
12 34567890123456789012345678901212345678901234567890123456789012123456
(B) 6 7
12 34567890123456789012345678901212345678901234567890123456789012123456
(C) 2 7
12 34567890123456789012345678901212345678901234567890123456789012123456
= (C) 8 7
2 34567890123456789012345678901212345678901234567890123456789012123456
1234567890123456789012345678901212345678901234567890123456789012123456
(D) 1 2 7
1 (D) 10 7
12 34567890123456789012345678901212345678901234567890123456789012123456
(E) 4 7
12 34567890123456789012345678901212345678901234567890123456789012123456 7
(E) 12
12 34567890123456789012345678901212345678901234567890123456789012123456 7
12 34567890123456789012345678901212345678901234567890123456789012123456 7
12 34567890123456789012345678901212345678901234567890123456789012123456
17. The sum of eight positive even integers is 19. G, S, and T are three points that lie on a 7
12 34567890123456789012345678901212345678901234567890123456789012123456 7
12 34567890123456789012345678901212345678901234567890123456789012123456
50. If no integer can appear more than plane. If the distance between G and S is 7
12 34567890123456789012345678901212345678901234567890123456789012123456 7
2 34567890123456789012345678901212345678901234567890123456789012123456
1234567890123456789012345678901212345678901234567890123456789012123456
twice in the set, what is the greatest 9, and the distance between S and T is 5, 7
1234567890123456789012345678901212345678901234567890123456789012123456 7
1234567890123456789012345678901212345678901234567890123456789012123456
possible value of one of the integers. which of the following are possible 7
1234567890123456789012345678901212345678901234567890123456789012123456 7
1 distances between G and T? 7
1234567890123456789012345678901212345678901234567890123456789012123456 77
2 34567890123456789012345678901212345678901234567890123456789012123456
(A) 8
1234567890123456789012345678901212345678901234567890123456789012123456
(B) 18 7
1 I. 3
1234567890123456789012345678901212345678901234567890123456789012123456
(C) 22 7
12
2
34567890123456789012345678901212345678901234567890123456789012123456 7
II. 5
34567890123456789012345678901212345678901234567890123456789012123456 7
1234567890123456789012345678901212345678901234567890123456789012123456
(D) 24
1234567890123456789012345678901212345678901234567890123456789012123456
III. 14 7
1 (E) 32 7
12 34567890123456789012345678901212345678901234567890123456789012123456 7
12 34567890123456789012345678901212345678901234567890123456789012123456 7
(A) I only
2 34567890123456789012345678901212345678901234567890123456789012123456
1234567890123456789012345678901212345678901234567890123456789012123456 7
1234567890123456789012345678901212345678901234567890123456789012123456
(B) II only 7
1 (C) I and II only 7
1234567890123456789012345678901212345678901234567890123456789012123456 7
1234567890123456789012345678901212345678901234567890123456789012123456
(D) II and III only 7
12 34567890123456789012345678901212345678901234567890123456789012123456 7
12 34567890123456789012345678901212345678901234567890123456789012123456
(E) I, II, and III 7
12 34567890123456789012345678901212345678901234567890123456789012123456 7
12 34567890123456789012345678901212345678901234567890123456789012123456 7
1234567890123456789012345678901212345678901234567890123456789012123456 7
1234567890123456789012345678901212345678901234567890123456789012123456
2 7
1234567890123456789012345678901212345678901234567890123456789012123456 7
1234567890123456789012345678901212345678901234567890123456789012123456 7
1234567890123456789012345678901212345678901234567890123456789012123456 7
1 34567890123456789012345678901212345678901234567890123456789012123456 7
1234567890123456789012345678901212345678901234567890123456789012123456 77
2 34567890123456789012345678901212345678901234567890123456789012123456
1234567890123456789012345678901212345678901234567890123456789012123456 7
1234567890123456789012345678901212345678901234567890123456789012123456 7
1234567890123456789012345678901212345678901234567890123456789012123456 7
1
12 34567890123456789012345678901212345678901234567890123456789012123456 7
1234567890123456789012345678901212345678901234567890123456789012123456 7
12 34567890123456789012345678901212345678901234567890123456789012123456 7
1234567890123456789012345678901212345678901234567890123456789012123456 7
12 34567890123456789012345678901212345678901234567890123456789012123456 7
1234567890123456789012345678901212345678901234567890123456789012123456 7
1234567890123456789012345678901212345678901234567890123456789012123456 7
1234567890123456789012345678901212345678901234567890123456789012123456 7
1234567890123456789012345678901212345678901234567890123456789012123456 7
12345678901234567890123456789012123456789012345678901234567890121234567
34 Copyright © 2005 Thomson Peterson’s, a part of The Thomson Corporaton
SAT is a registered trademark of the College Entrance Examination Board, which
was not involved in the production of and does not endorse this product.
12345678901234567890123456789012123456789012345678901234567890121234567
1234567890123456789012345678901212345678901234567890123456789012123456 7
12234567890123456789012345678901212345678901234567890123456789012123456 7
1 34567890123456789012345678901212345678901234567890123456789012123456
20. 21. 7
12234567890123456789012345678901212345678901234567890123456789012123456 7
1234567890123456789012345678901212345678901234567890123456789012123456 7
1 34567890123456789012345678901212345678901234567890123456789012123456 7
12234567890123456789012345678901212345678901234567890123456789012123456 7
1 34567890123456789012345678901212345678901234567890123456789012123456 7
12234567890123456789012345678901212345678901234567890123456789012123456 7
1234567890123456789012345678901212345678901234567890123456789012123456 7
1234567890123456789012345678901212345678901234567890123456789012123456 7
1234567890123456789012345678901212345678901234567890123456789012123456
34567890123456789012345678901212345678901234567890123456789012123456
7
7
1
12234567890123456789012345678901212345678901234567890123456789012123456 7
1234567890123456789012345678901212345678901234567890123456789012123456 7
1234567890123456789012345678901212345678901234567890123456789012123456 7
1234567890123456789012345678901212345678901234567890123456789012123456
34567890123456789012345678901212345678901234567890123456789012123456
7
7
1
12234567890123456789012345678901212345678901234567890123456789012123456 7
1234567890123456789012345678901212345678901234567890123456789012123456 7
1234567890123456789012345678901212345678901234567890123456789012123456
If AB 5 BC 5 AC 5 6, and D is the 7
1234567890123456789012345678901212345678901234567890123456789012123456
34567890123456789012345678901212345678901234567890123456789012123456
7
7
1 halfway between A and C, then BD 5
12234567890123456789012345678901212345678901234567890123456789012123456 7
1234567890123456789012345678901212345678901234567890123456789012123456
= 7
1234567890123456789012345678901212345678901234567890123456789012123456
(A) 2 3 7
1234567890123456789012345678901212345678901234567890123456789012123456 7
1234567890123456789012345678901212345678901234567890123456789012123456 77
34567890123456789012345678901212345678901234567890123456789012123456
(B) 3
1234567890123456789012345678901212345678901234567890123456789012123456 7
1234567890123456789012345678901212345678901234567890123456789012123456
(C) 3=3
1234567890123456789012345678901212345678901234567890123456789012123456 7
1234567890123456789012345678901212345678901234567890123456789012123456
= 7
1234567890123456789012345678901212345678901234567890123456789012123456 77
(D) 4 2 The figure above is the diagram of an
1
12234567890123456789012345678901212345678901234567890123456789012123456
(E) 4=3 industrial fan blade. If the fan’s maximum 7
1234567890123456789012345678901212345678901234567890123456789012123456 7
1234567890123456789012345678901212345678901234567890123456789012123456
blade speed is 100 revolutions per 10 7
34567890123456789012345678901212345678901234567890123456789012123456
1234567890123456789012345678901212345678901234567890123456789012123456 7
1234567890123456789012345678901212345678901234567890123456789012123456
seconds, what is the greatest distance (in 7
1234567890123456789012345678901212345678901234567890123456789012123456 7
1234567890123456789012345678901212345678901234567890123456789012123456 77
feet) that any point on the blade could
1234567890123456789012345678901212345678901234567890123456789012123456
1234567890123456789012345678901212345678901234567890123456789012123456
travel in 30 seconds? 7
1234567890123456789012345678901212345678901234567890123456789012123456 77
1234567890123456789012345678901212345678901234567890123456789012123456
(A) 100=2p
1234567890123456789012345678901212345678901234567890123456789012123456 7
1 7
1234567890123456789012345678901212345678901234567890123456789012123456 77
2 34567890123456789012345678901212345678901234567890123456789012123456
(B) 200 = 3p
1234567890123456789012345678901212345678901234567890123456789012123456 7
1 (C) 600=2p
1234567890123456789012345678901212345678901234567890123456789012123456 7
12234567890123456789012345678901212345678901234567890123456789012123456 = 7
1234567890123456789012345678901212345678901234567890123456789012123456 77
34567890123456789012345678901212345678901234567890123456789012123456
(D) 600 3p
1234567890123456789012345678901212345678901234567890123456789012123456 7
1 (E) 1200=2p
12234567890123456789012345678901212345678901234567890123456789012123456 7
1234567890123456789012345678901212345678901234567890123456789012123456 7
1234567890123456789012345678901212345678901234567890123456789012123456 77
34567890123456789012345678901212345678901234567890123456789012123456
1234567890123456789012345678901212345678901234567890123456789012123456 7
1
1234567890123456789012345678901212345678901234567890123456789012123456 7
1234567890123456789012345678901212345678901234567890123456789012123456 7
12234567890123456789012345678901212345678901234567890123456789012123456 7
1234567890123456789012345678901212345678901234567890123456789012123456 7
1234567890123456789012345678901212345678901234567890123456789012123456 7
1234567890123456789012345678901212345678901234567890123456789012123456 7
1 34567890123456789012345678901212345678901234567890123456789012123456 7
1234567890123456789012345678901212345678901234567890123456789012123456 77
2 34567890123456789012345678901212345678901234567890123456789012123456
1234567890123456789012345678901212345678901234567890123456789012123456 7
1234567890123456789012345678901212345678901234567890123456789012123456 7
1234567890123456789012345678901212345678901234567890123456789012123456 7
1
12234567890123456789012345678901212345678901234567890123456789012123456 7
1234567890123456789012345678901212345678901234567890123456789012123456 7
1234567890123456789012345678901212345678901234567890123456789012123456 7
1234567890123456789012345678901212345678901234567890123456789012123456 7
1 34567890123456789012345678901212345678901234567890123456789012123456 7
1234567890123456789012345678901212345678901234567890123456789012123456 77
2 34567890123456789012345678901212345678901234567890123456789012123456
1
12234567890123456789012345678901212345678901234567890123456789012123456 7
1 34567890123456789012345678901212345678901234567890123456789012123456 7
12234567890123456789012345678901212345678901234567890123456789012123456 7
STOP
1 34567890123456789012345678901212345678901234567890123456789012123456
Do not proceed to the next section until time is up.
1234567890123456789012345678901212345678901234567890123456789012123456 7
7
1234567890123456789012345678901212345678901234567890123456789012123456 7
1234567890123456789012345678901212345678901234567890123456789012123456 7
12345678901234567890123456789012123456789012345678901234567890121234567
Copyright © 2005 Thomson Peterson’s, a part of The Thomson Corporaton 35
SAT is a registered trademark of the College Entrance Examination Board, which
was not involved in the production of and does not endorse this product.
Section 6
16 Questions j Time—20 Minutes

12345678901234567890123456789012123456789012345678901234567890121234567
12 34567890123456789012345678901212345678901234567890123456789012123456 7
12 34567890123456789012345678901212345678901234567890123456789012123456 7
1 34567890123456789012345678901212345678901234567890123456789012123456 7
2
12 34567890123456789012345678901212345678901234567890123456789012123456 7
12 34567890123456789012345678901212345678901234567890123456789012123456
Directions: Each passage below is followed by a set of questions. Read each passage, then 7
12 34567890123456789012345678901212345678901234567890123456789012123456
answer the accompanying questions, basing your answers on what is stated or implied in the 7
12 34567890123456789012345678901212345678901234567890123456789012123456 7
2 34567890123456789012345678901212345678901234567890123456789012123456
1234567890123456789012345678901212345678901234567890123456789012123456
passage and any introductory material provided. Mark the letter of your choice on the answer 7
1234567890123456789012345678901212345678901234567890123456789012123456 7
1234567890123456789012345678901212345678901234567890123456789012123456
sheet that best corresponds to the correct answer. 7
1234567890123456789012345678901212345678901234567890123456789012123456 77
1234567890123456789012345678901212345678901234567890123456789012123456 7
1234567890123456789012345678901212345678901234567890123456789012123456 7
1
12 34567890123456789012345678901212345678901234567890123456789012123456 7
12 34567890123456789012345678901212345678901234567890123456789012123456
Line Frederic Remington (1861–1909) has Line The question of what counts as literature 7
12 34567890123456789012345678901212345678901234567890123456789012123456 7
2 34567890123456789012345678901212345678901234567890123456789012123456
1234567890123456789012345678901212345678901234567890123456789012123456
long been celebrated as one of the most has been strongly debated over the last 7
1234567890123456789012345678901212345678901234567890123456789012123456 7
1234567890123456789012345678901212345678901234567890123456789012123456
gifted interpreters of the American West. few decades both in and out of academia. 7
1234567890123456789012345678901212345678901234567890123456789012123456 7
1234567890123456789012345678901212345678901234567890123456789012123456
Initially, his western images appeared as Some argue that only the test of time 7
1234567890123456789012345678901212345678901234567890123456789012123456 7
1234567890123456789012345678901212345678901234567890123456789012123456
(5) illustrations in popular journals. As he (5) ultimately vindicates a fictional work’s 7
1234567890123456789012345678901212345678901234567890123456789012123456 7
1234567890123456789012345678901212345678901234567890123456789012123456
matured, however, Remington turned his claim to the status of literature. Their 7
1 7
1234567890123456789012345678901212345678901234567890123456789012123456 77
2 34567890123456789012345678901212345678901234567890123456789012123456
attention away from illustration, argument runs like this: if people still
1234567890123456789012345678901212345678901234567890123456789012123456 7
1 concentrating instead on painting and read, still reference, still care about a
1234567890123456789012345678901212345678901234567890123456789012123456 7
12
2
34567890123456789012345678901212345678901234567890123456789012123456 7
sculpture. About 1900 he began a series
34567890123456789012345678901212345678901234567890123456789012123456
work of fiction decades or even centuries
7
1234567890123456789012345678901212345678901234567890123456789012123456
1234567890123456789012345678901212345678901234567890123456789012123456
(10) of paintings that took as their subject the (10) after its original publication, then that 7
1 7
12 34567890123456789012345678901212345678901234567890123456789012123456 7
color of night. Before his premature work clearly rises to the auspicious status
12 34567890123456789012345678901212345678901234567890123456789012123456of literature. Critics of this view, though, 7
12 34567890123456789012345678901212345678901234567890123456789012123456
death in 1909 at age 48, Remington 7
12 34567890123456789012345678901212345678901234567890123456789012123456 7
1234567890123456789012345678901212345678901234567890123456789012123456 7
completed more than seventy paintings in point out that this method of determining
1234567890123456789012345678901212345678901234567890123456789012123456 what is and is not literature by definition 7
1234567890123456789012345678901212345678901234567890123456789012123456
which he explored the technical and 7
2 34567890123456789012345678901212345678901234567890123456789012123456
1234567890123456789012345678901212345678901234567890123456789012123456
aesthetic difficulties of painting darkness. excludes contemporary works from 7
1234567890123456789012345678901212345678901234567890123456789012123456
(15) (15)
7
1234567890123456789012345678901212345678901234567890123456789012123456 consideration. We do not know, they 7
1234567890123456789012345678901212345678901234567890123456789012123456 77
1 1. The passage suggests Remington’s major rightfully contend, if a novel published in
12 34567890123456789012345678901212345678901234567890123456789012123456 7
12 34567890123456789012345678901212345678901234567890123456789012123456the last few years will be read in a 7
12 34567890123456789012345678901212345678901234567890123456789012123456 7
artistic accomplishments were
12 34567890123456789012345678901212345678901234567890123456789012123456hundred years or not. And so they ask, 7
1234567890123456789012345678901212345678901234567890123456789012123456 7
1234567890123456789012345678901212345678901234567890123456789012123456 77
2 34567890123456789012345678901212345678901234567890123456789012123456
(A) magazine illustrations. (20) does this mean we cannot meaningfully
1234567890123456789012345678901212345678901234567890123456789012123456 discuss whether the work is important, or 7
1234567890123456789012345678901212345678901234567890123456789012123456
(B) sculptures.
7
1234567890123456789012345678901212345678901234567890123456789012123456 7
1 (C) paintings of nocturnal cityscapes. influential, or of great merit?
2 34567890123456789012345678901212345678901234567890123456789012123456
1234567890123456789012345678901212345678901234567890123456789012123456
(D) paintings of nocturnal landscapes. 7
1 7
12 34567890123456789012345678901212345678901234567890123456789012123456
(E) color studies. 7
1234567890123456789012345678901212345678901234567890123456789012123456 7
12 34567890123456789012345678901212345678901234567890123456789012123456 7
1234567890123456789012345678901212345678901234567890123456789012123456 7
1234567890123456789012345678901212345678901234567890123456789012123456 7
1234567890123456789012345678901212345678901234567890123456789012123456 7
1234567890123456789012345678901212345678901234567890123456789012123456 7
12345678901234567890123456789012123456789012345678901234567890121234567
36 Copyright © 2005 Thomson Peterson’s, a part of The Thomson Corporaton
SAT is a registered trademark of the College Entrance Examination Board, which
was not involved in the production of and does not endorse this product.
12345678901234567890123456789012123456789012345678901234567890121234567
1234567890123456789012345678901212345678901234567890123456789012123456 7
12234567890123456789012345678901212345678901234567890123456789012123456 7
1 34567890123456789012345678901212345678901234567890123456789012123456
2. The author uses the word “vindicates” to these were considerations that led the 7
12234567890123456789012345678901212345678901234567890123456789012123456 7
1234567890123456789012345678901212345678901234567890123456789012123456
emphasize that National Aeronautics and Space Admin- 7
1 34567890123456789012345678901212345678901234567890123456789012123456 7
12234567890123456789012345678901212345678901234567890123456789012123456
istration to Mars. Project Viking’s goal,
34567890123456789012345678901212345678901234567890123456789012123456
7
7
1 (A) all works of fiction claim to be
12234567890123456789012345678901212345678901234567890123456789012123456
(20) after making a soft landing on Mars, was 7
1234567890123456789012345678901212345678901234567890123456789012123456
literature.
to execute a set of scientific investigations 7
1234567890123456789012345678901212345678901234567890123456789012123456 7
1234567890123456789012345678901212345678901234567890123456789012123456
(B) all works of fiction are, in some
34567890123456789012345678901212345678901234567890123456789012123456
7
7
1 that would not only provide data on the
12234567890123456789012345678901212345678901234567890123456789012123456
sense, literature. 7
1234567890123456789012345678901212345678901234567890123456789012123456physical nature of the planet but also 7
1234567890123456789012345678901212345678901234567890123456789012123456
(C) literature is a much more prestigious 7
1234567890123456789012345678901212345678901234567890123456789012123456make
34567890123456789012345678901212345678901234567890123456789012123456
a first attempt at determining if 7
7
1 category than fiction.
12234567890123456789012345678901212345678901234567890123456789012123456
(25) detectable life forms were present. 7
1234567890123456789012345678901212345678901234567890123456789012123456
(D) the debate regarding what is litera- 7
1234567890123456789012345678901212345678901234567890123456789012123456 7
1234567890123456789012345678901212345678901234567890123456789012123456
ture is excessively erudite.
34567890123456789012345678901212345678901234567890123456789012123456
7
7
1 (E) for a work to establish itself as 4. Which of the following does the para-
12234567890123456789012345678901212345678901234567890123456789012123456 7
1234567890123456789012345678901212345678901234567890123456789012123456
literature is an incredible feat. graph most emphasize as the motivation 7
1234567890123456789012345678901212345678901234567890123456789012123456 7
1234567890123456789012345678901212345678901234567890123456789012123456
for the Viking trip to Mars? 7
34567890123456789012345678901212345678901234567890123456789012123456
1234567890123456789012345678901212345678901234567890123456789012123456 7
1234567890123456789012345678901212345678901234567890123456789012123456
3. The argument, given in the passage, 7
1234567890123456789012345678901212345678901234567890123456789012123456
(A) Fascination with chemistry on 7
1234567890123456789012345678901212345678901234567890123456789012123456
against the “test of time” approach is that 7
1234567890123456789012345678901212345678901234567890123456789012123456 77 another planet.
1234567890123456789012345678901212345678901234567890123456789012123456
(B) Four-century-old interest in the planet. 7
1 (A) it excludes by definition all writing
12234567890123456789012345678901212345678901234567890123456789012123456 7
1234567890123456789012345678901212345678901234567890123456789012123456
that is not fictional. (C) The advancement of space explora- 7
1234567890123456789012345678901212345678901234567890123456789012123456 7
1234567890123456789012345678901212345678901234567890123456789012123456 77
34567890123456789012345678901212345678901234567890123456789012123456
(B) it does not take trends in critical tion.
1234567890123456789012345678901212345678901234567890123456789012123456
(D) Possibility of a space station on Mars. 7
1234567890123456789012345678901212345678901234567890123456789012123456
interest into account.
1234567890123456789012345678901212345678901234567890123456789012123456
(C) it excludes contemporary fiction (E) Possibility of life on Mars. 7
1234567890123456789012345678901212345678901234567890123456789012123456 7
1234567890123456789012345678901212345678901234567890123456789012123456
from the discussion. 7
1234567890123456789012345678901212345678901234567890123456789012123456 7
1234567890123456789012345678901212345678901234567890123456789012123456
(D) it allows contemporary works to be 7
1234567890123456789012345678901212345678901234567890123456789012123456 77
Questions 5–16 are based on the following two
1 considered alongside the great works
12234567890123456789012345678901212345678901234567890123456789012123456
passages. 7
1234567890123456789012345678901212345678901234567890123456789012123456
of centuries past. 7
1 34567890123456789012345678901212345678901234567890123456789012123456
The following two passages discuss the English 7
1234567890123456789012345678901212345678901234567890123456789012123456
(E) it gives too much weight to popular 7
12234567890123456789012345678901212345678901234567890123456789012123456
and Metric systems of measurement. 7
1234567890123456789012345678901212345678901234567890123456789012123456 77
34567890123456789012345678901212345678901234567890123456789012123456
opinion.
1234567890123456789012345678901212345678901234567890123456789012123456 7
1 Line Since the sixteenth century, astronomers
12234567890123456789012345678901212345678901234567890123456789012123456
Passage 1 7
1234567890123456789012345678901212345678901234567890123456789012123456 7
34567890123456789012345678901212345678901234567890123456789012123456
1234567890123456789012345678901212345678901234567890123456789012123456
have recognized Mars for what it is—a Line It is an oft-repeated tale that the English 7
1234567890123456789012345678901212345678901234567890123456789012123456 7
1 relatively nearby planet not so unlike our measurement of the yard was standard- 7
1234567890123456789012345678901212345678901234567890123456789012123456 7
1234567890123456789012345678901212345678901234567890123456789012123456
own. The fourth planet from the sun and ized when an English royal stepped into 7
2 34567890123456789012345678901212345678901234567890123456789012123456
1234567890123456789012345678901212345678901234567890123456789012123456disputes about the measurement’s length 7
7
1234567890123456789012345678901212345678901234567890123456789012123456
(5) Earth’s closest neighbor, Mars has been
7
1234567890123456789012345678901212345678901234567890123456789012123456 7
1234567890123456789012345678901212345678901234567890123456789012123456
the subject of modern scientists’ careful (5) and declared the distance from his
7
1
1234567890123456789012345678901212345678901234567890123456789012123456 77
2 34567890123456789012345678901212345678901234567890123456789012123456
scrutiny with powerful telescopes, deep shoulder to the tip of his fingers as the
1234567890123456789012345678901212345678901234567890123456789012123456
space probes, and orbiting spacecraft. In 7
1234567890123456789012345678901212345678901234567890123456789012123456standard yard. Unlike many colorful
7
1234567890123456789012345678901212345678901234567890123456789012123456
1976, Earth-bound scientists were
7
1 anecdotes from history, this one is true.
2 34567890123456789012345678901212345678901234567890123456789012123456
1234567890123456789012345678901212345678901234567890123456789012123456
(10) brought significantly closer to their 7
1234567890123456789012345678901212345678901234567890123456789012123456Early in the twelfth century, the English 7
1234567890123456789012345678901212345678901234567890123456789012123456
subject of investigation when two Viking 7
1234567890123456789012345678901212345678901234567890123456789012123456 77
(10) king Henry I established the length of the
1 probes touched down on that red soil.
12234567890123456789012345678901212345678901234567890123456789012123456
yard as the distance from the tip of his 7
1 34567890123456789012345678901212345678901234567890123456789012123456
The possibility of life on Mars, clues to 7
12234567890123456789012345678901212345678901234567890123456789012123456
nose to the tip of his outstretched thumb. 7
1 34567890123456789012345678901212345678901234567890123456789012123456
the evolution of the solar system, 7
12234567890123456789012345678901212345678901234567890123456789012123456
34567890123456789012345678901212345678901234567890123456789012123456
In our scientific age such stories seem 7
7
1 (15) fascination with the chemistry, geology,
1234567890123456789012345678901212345678901234567890123456789012123456earthy at best and ridiculous at worst. 7
1234567890123456789012345678901212345678901234567890123456789012123456
and meteorology of another planet— 7
1234567890123456789012345678901212345678901234567890123456789012123456 7
12345678901234567890123456789012123456789012345678901234567890121234567
Copyright © 2005 Thomson Peterson’s, a part of The Thomson Corporaton 37
SAT is a registered trademark of the College Entrance Examination Board, which
was not involved in the production of and does not endorse this product.
12345678901234567890123456789012123456789012345678901234567890121234567
1234567890123456789012345678901212345678901234567890123456789012123456 7
12 34567890123456789012345678901212345678901234567890123456789012123456 7
1234567890123456789012345678901212345678901234567890123456789012123456
(15) But not all ancient units of measure have manageable size, it was defined as one 7
12 34567890123456789012345678901212345678901234567890123456789012123456 7
12 34567890123456789012345678901212345678901234567890123456789012123456
such arbitrary origins. The mile is a good one-forty-millionth of the Earth’s 7
1234567890123456789012345678901212345678901234567890123456789012123456 7
12
2
34567890123456789012345678901212345678901234567890123456789012123456 7
example of this. Though the mile is today circumference.
34567890123456789012345678901212345678901234567890123456789012123456
They employed the word
7
1
12 34567890123456789012345678901212345678901234567890123456789012123456
counted as part of the English system of meter to harken back to the ancient 7
12 34567890123456789012345678901212345678901234567890123456789012123456 7
12 34567890123456789012345678901212345678901234567890123456789012123456 7
measurement, the unit dates back to (60) Greek word metron, meaning measure.
12 34567890123456789012345678901212345678901234567890123456789012123456 7
1234567890123456789012345678901212345678901234567890123456789012123456
(20) ancient Rome. The English word mile The rest of the metric system is even 7
12 34567890123456789012345678901212345678901234567890123456789012123456 7
12 34567890123456789012345678901212345678901234567890123456789012123456 7
derives from the Latin term mille, which less arbitrary in origin. The other metric
12 34567890123456789012345678901212345678901234567890123456789012123456 7
12
2
34567890123456789012345678901212345678901234567890123456789012123456
means one thousand. For the Romans, units of length were generated by either
34567890123456789012345678901212345678901234567890123456789012123456
7
7
1
12 34567890123456789012345678901212345678901234567890123456789012123456
the mille was one thousand paces. A pace multiplying or dividing the meter by a 7
12 34567890123456789012345678901212345678901234567890123456789012123456 7
12 34567890123456789012345678901212345678901234567890123456789012123456 7
was two steps, or five feet roughly. This (65) factor of ten. Thus a kilometer is 1000
12 34567890123456789012345678901212345678901234567890123456789012123456 7
1234567890123456789012345678901212345678901234567890123456789012123456
(25) meant the mille was 5,000 feet. In meters, and a centimeter is one one- 7
12 34567890123456789012345678901212345678901234567890123456789012123456 7
12 34567890123456789012345678901212345678901234567890123456789012123456 7
medieval Europe, however, the 220-yard hundredths of a meter. It is the great asset
12 34567890123456789012345678901212345678901234567890123456789012123456
furlong became the dominant measure- 7
12 34567890123456789012345678901212345678901234567890123456789012123456
of the metric system, at least for scien- 7
2 34567890123456789012345678901212345678901234567890123456789012123456
tists, that units for measuring weight and 7
1234567890123456789012345678901212345678901234567890123456789012123456
ment used. To reconcile the two measure-
1234567890123456789012345678901212345678901234567890123456789012123456 7
1234567890123456789012345678901212345678901234567890123456789012123456
ments, the mile was lengthened to be (70) energy are also derived from the basic 7
1234567890123456789012345678901212345678901234567890123456789012123456 77
1234567890123456789012345678901212345678901234567890123456789012123456
(30) eight furlongs. This made the mile 5,280 unit of the meter. For instance, weight is
1234567890123456789012345678901212345678901234567890123456789012123456 7
1 feet. A sixteenth-century act of Parlia- measured in grams, which are determined 7
12 34567890123456789012345678901212345678901234567890123456789012123456 7
12 34567890123456789012345678901212345678901234567890123456789012123456
ment fixed this measurement for the mile. by the weight of one cubic centimeter 7
12 34567890123456789012345678901212345678901234567890123456789012123456 7
1234567890123456789012345678901212345678901234567890123456789012123456 77
2 34567890123456789012345678901212345678901234567890123456789012123456
It is true that the English system of of water.
1234567890123456789012345678901212345678901234567890123456789012123456 France made use of the metric system 7
1234567890123456789012345678901212345678901234567890123456789012123456
measurement, the system that includes (75)
1234567890123456789012345678901212345678901234567890123456789012123456 7
1234567890123456789012345678901212345678901234567890123456789012123456
(35) the mile, the yard, the foot, and the inch, compulsory in 1840. Other countries 7
1234567890123456789012345678901212345678901234567890123456789012123456 7
1234567890123456789012345678901212345678901234567890123456789012123456
has a certain quirkiness to it because it rapidly followed suit. The adoption of 7
1234567890123456789012345678901212345678901234567890123456789012123456 7
1234567890123456789012345678901212345678901234567890123456789012123456 77
has evolved through human history. This the metric system, also known as the
1
12 34567890123456789012345678901212345678901234567890123456789012123456
quirkiness might irritate scientists, but it international system, or S.I., coincided 7
12 34567890123456789012345678901212345678901234567890123456789012123456 7
1234567890123456789012345678901212345678901234567890123456789012123456 7
is part and parcel of the tradition that (80) with great advances in science. By 1900,
1234567890123456789012345678901212345678901234567890123456789012123456 7
12
2
34567890123456789012345678901212345678901234567890123456789012123456
(40) has been bequeathed, in its accumulated over 35 countries had officially adopted
34567890123456789012345678901212345678901234567890123456789012123456
7
7
1234567890123456789012345678901212345678901234567890123456789012123456
1234567890123456789012345678901212345678901234567890123456789012123456
form, to the English-speaking world. its use. In the United States, the system 7
1 7
12 34567890123456789012345678901212345678901234567890123456789012123456 7
has been dubbed “voluntary” and
12 34567890123456789012345678901212345678901234567890123456789012123456
Passage 2 7
12 34567890123456789012345678901212345678901234567890123456789012123456
“preferred,” but has never been made 7
12 34567890123456789012345678901212345678901234567890123456789012123456
compulsory. 7
1234567890123456789012345678901212345678901234567890123456789012123456
The metric system was conceived by (85)
7
1234567890123456789012345678901212345678901234567890123456789012123456 The measure of the meter has been 7
1234567890123456789012345678901212345678901234567890123456789012123456 7
twelve French scientists during the
2 34567890123456789012345678901212345678901234567890123456789012123456
refined three times since its conception in 7
1234567890123456789012345678901212345678901234567890123456789012123456
1234567890123456789012345678901212345678901234567890123456789012123456
French Revolution. Like many innova- 7
1234567890123456789012345678901212345678901234567890123456789012123456
1791. The latest was in 1983 when the 7
1234567890123456789012345678901212345678901234567890123456789012123456 77
(45) tions during the French Revolution, the
1 speed of light was employed to give the
12 34567890123456789012345678901212345678901234567890123456789012123456
metric system was formulated as a 7
12 34567890123456789012345678901212345678901234567890123456789012123456
(90) greatest precision for the measurement to 7
12 34567890123456789012345678901212345678901234567890123456789012123456
scientific system that would replace 7
12 34567890123456789012345678901212345678901234567890123456789012123456
date. The distance that light travels in a 7
1234567890123456789012345678901212345678901234567890123456789012123456
traditional ways of ordering society. The 7
2 34567890123456789012345678901212345678901234567890123456789012123456
1234567890123456789012345678901212345678901234567890123456789012123456
vacuum in 1/299,792,458 of a second is 7
1234567890123456789012345678901212345678901234567890123456789012123456
revolutionaries did not see it as a
the internationally accepted definition of 7
7
1234567890123456789012345678901212345678901234567890123456789012123456 7
1234567890123456789012345678901212345678901234567890123456789012123456
(50) coincidence that length was meted in
7
1 a meter.
measures based on the size of a medieval
1234567890123456789012345678901212345678901234567890123456789012123456 77
2 34567890123456789012345678901212345678901234567890123456789012123456
1 king. Instead of these arbitrary standards,
12 34567890123456789012345678901212345678901234567890123456789012123456 7
1234567890123456789012345678901212345678901234567890123456789012123456
the metric system’s basic unit of measure, 7
12
2
34567890123456789012345678901212345678901234567890123456789012123456 7
34567890123456789012345678901212345678901234567890123456789012123456 7
1 the meter, was based upon the circumfer-
1234567890123456789012345678901212345678901234567890123456789012123456 7
1234567890123456789012345678901212345678901234567890123456789012123456
(55) ence of the Earth. For the meter to be a 7
1234567890123456789012345678901212345678901234567890123456789012123456 7
12345678901234567890123456789012123456789012345678901234567890121234567
38 Copyright © 2005 Thomson Peterson’s, a part of The Thomson Corporaton
SAT is a registered trademark of the College Entrance Examination Board, which
was not involved in the production of and does not endorse this product.
12345678901234567890123456789012123456789012345678901234567890121234567
1234567890123456789012345678901212345678901234567890123456789012123456 7
12234567890123456789012345678901212345678901234567890123456789012123456 7
1 34567890123456789012345678901212345678901234567890123456789012123456
5. What is the author of Passage 1’s attitude 8. By “reconcile the two measurements,” 7
12234567890123456789012345678901212345678901234567890123456789012123456 7
1234567890123456789012345678901212345678901234567890123456789012123456
towards the English system of measure- (line 28–32) the author means 7
1 34567890123456789012345678901212345678901234567890123456789012123456 7
12234567890123456789012345678901212345678901234567890123456789012123456
ment?
34567890123456789012345678901212345678901234567890123456789012123456
7
7
1 (A) determine which one was accurate.
12234567890123456789012345678901212345678901234567890123456789012123456 7
1234567890123456789012345678901212345678901234567890123456789012123456
(A) Emphatic praise (B) develop a new system of measure- 7
1234567890123456789012345678901212345678901234567890123456789012123456 7
1234567890123456789012345678901212345678901234567890123456789012123456
(B) Qualified acceptance ment
34567890123456789012345678901212345678901234567890123456789012123456
without the inaccuracies of 7
7
1
12234567890123456789012345678901212345678901234567890123456789012123456
(C) Neutrality the old. 7
1234567890123456789012345678901212345678901234567890123456789012123456 7
1234567890123456789012345678901212345678901234567890123456789012123456
(D) Strong criticism (C) settle the public’s disagreement over 7
1234567890123456789012345678901212345678901234567890123456789012123456
34567890123456789012345678901212345678901234567890123456789012123456
7
7
1 (E) Antipathy which was better.
12234567890123456789012345678901212345678901234567890123456789012123456 7
1234567890123456789012345678901212345678901234567890123456789012123456
(D) find a metric equivalent. 7
1234567890123456789012345678901212345678901234567890123456789012123456
6. In Passage 1, the story of Henry I is 7
1234567890123456789012345678901212345678901234567890123456789012123456
(E) cease
34567890123456789012345678901212345678901234567890123456789012123456
using two different systems. 7
7
1 offered primarily to
12234567890123456789012345678901212345678901234567890123456789012123456 7
1234567890123456789012345678901212345678901234567890123456789012123456
9. The author refers to the English system’s 7
1234567890123456789012345678901212345678901234567890123456789012123456
(A) demonstrate that the reader’s 7
1234567890123456789012345678901212345678901234567890123456789012123456
“accumulated form” line 40–41 primarily 7
34567890123456789012345678901212345678901234567890123456789012123456
1234567890123456789012345678901212345678901234567890123456789012123456
preconceptions about the English 7
1234567890123456789012345678901212345678901234567890123456789012123456
to emphasize that the system 7
1234567890123456789012345678901212345678901234567890123456789012123456 77
system are wrong.
1234567890123456789012345678901212345678901234567890123456789012123456
1234567890123456789012345678901212345678901234567890123456789012123456
(B) illustrate the role of the English (A) ceased to change once officially 7
1234567890123456789012345678901212345678901234567890123456789012123456 7
1 monarchy in the development of the adopted. 7
12234567890123456789012345678901212345678901234567890123456789012123456 7
1234567890123456789012345678901212345678901234567890123456789012123456
English system. (B) derives from a variety of sources. 7
1234567890123456789012345678901212345678901234567890123456789012123456 7
1234567890123456789012345678901212345678901234567890123456789012123456 77
34567890123456789012345678901212345678901234567890123456789012123456
(C) reveal how far back in time the (C) stretches back further than reliable
1234567890123456789012345678901212345678901234567890123456789012123456
1234567890123456789012345678901212345678901234567890123456789012123456
English system goes. written history. 7
1234567890123456789012345678901212345678901234567890123456789012123456
(D) provide a concrete example of how (D) continues to evolve. 7
1234567890123456789012345678901212345678901234567890123456789012123456 77
1234567890123456789012345678901212345678901234567890123456789012123456
the arbitrariness of the English (E) was adopted wholesale.
1234567890123456789012345678901212345678901234567890123456789012123456 7
1234567890123456789012345678901212345678901234567890123456789012123456
system developed. 7
1234567890123456789012345678901212345678901234567890123456789012123456 7
1 (E) suggest the practicality of the English 10. The authors of the two passages would be 7
2 34567890123456789012345678901212345678901234567890123456789012123456
most likely to agree that the metric system 7
1234567890123456789012345678901212345678901234567890123456789012123456
1234567890123456789012345678901212345678901234567890123456789012123456
system. 7
1 7
1234567890123456789012345678901212345678901234567890123456789012123456 7
12234567890123456789012345678901212345678901234567890123456789012123456
(A) has a
34567890123456789012345678901212345678901234567890123456789012123456
shorter but equally interesting 7
7
1234567890123456789012345678901212345678901234567890123456789012123456
7. The word “earthy” in line 14 most closely
1234567890123456789012345678901212345678901234567890123456789012123456
history as the English system. 7
1 means 7
12234567890123456789012345678901212345678901234567890123456789012123456
(B) has a history that reaches back as far 7
1234567890123456789012345678901212345678901234567890123456789012123456 7
1234567890123456789012345678901212345678901234567890123456789012123456 77
34567890123456789012345678901212345678901234567890123456789012123456
(A) unrefined. as the English system’s.
1234567890123456789012345678901212345678901234567890123456789012123456
(B) musky. (C) has a longer history than the history 7
1
1234567890123456789012345678901212345678901234567890123456789012123456
(C) impractical. of the English system. 7
1234567890123456789012345678901212345678901234567890123456789012123456 7
2 34567890123456789012345678901212345678901234567890123456789012123456
(D) should not be thought of historically. 7
1234567890123456789012345678901212345678901234567890123456789012123456
(D) old-fashioned.
1234567890123456789012345678901212345678901234567890123456789012123456 7
(E) has a history that is equally long but 7
1234567890123456789012345678901212345678901234567890123456789012123456
(E) baffling.
1234567890123456789012345678901212345678901234567890123456789012123456 77
1 less colorful than the English
12234567890123456789012345678901212345678901234567890123456789012123456 7
1234567890123456789012345678901212345678901234567890123456789012123456
system’s. 7
1234567890123456789012345678901212345678901234567890123456789012123456 7
1234567890123456789012345678901212345678901234567890123456789012123456 7
1 34567890123456789012345678901212345678901234567890123456789012123456 7
1234567890123456789012345678901212345678901234567890123456789012123456 77
2 34567890123456789012345678901212345678901234567890123456789012123456
1234567890123456789012345678901212345678901234567890123456789012123456 7
1234567890123456789012345678901212345678901234567890123456789012123456 7
1234567890123456789012345678901212345678901234567890123456789012123456 7
1
12234567890123456789012345678901212345678901234567890123456789012123456 7
1 34567890123456789012345678901212345678901234567890123456789012123456 7
12234567890123456789012345678901212345678901234567890123456789012123456 7
1 34567890123456789012345678901212345678901234567890123456789012123456 7
12234567890123456789012345678901212345678901234567890123456789012123456 7
1 34567890123456789012345678901212345678901234567890123456789012123456 7
1234567890123456789012345678901212345678901234567890123456789012123456 7
1234567890123456789012345678901212345678901234567890123456789012123456 7
1234567890123456789012345678901212345678901234567890123456789012123456 7
12345678901234567890123456789012123456789012345678901234567890121234567
Copyright © 2005 Thomson Peterson’s, a part of The Thomson Corporaton 39
SAT is a registered trademark of the College Entrance Examination Board, which
was not involved in the production of and does not endorse this product.
12345678901234567890123456789012123456789012345678901234567890121234567
1234567890123456789012345678901212345678901234567890123456789012123456 7
12 34567890123456789012345678901212345678901234567890123456789012123456 7
1234567890123456789012345678901212345678901234567890123456789012123456
11. According to Passage 2, the invention of 14. In Passage 2, the reason for the 1983 defini- 7
12 34567890123456789012345678901212345678901234567890123456789012123456 7
12 34567890123456789012345678901212345678901234567890123456789012123456
the metric system was tion of the meter is probably that scientists 7
1234567890123456789012345678901212345678901234567890123456789012123456 7
12 34567890123456789012345678901212345678901234567890123456789012123456 7
1234567890123456789012345678901212345678901234567890123456789012123456
(A) one of the greatest accomplishments (A) have determined that the new meter 7
12 34567890123456789012345678901212345678901234567890123456789012123456 7
12 34567890123456789012345678901212345678901234567890123456789012123456 7
of the French Revolution. is a more manageable length.
12 34567890123456789012345678901212345678901234567890123456789012123456 7
12
2
34567890123456789012345678901212345678901234567890123456789012123456
(B) in contradiction to many of the other (B) have more sophisticated data on the
34567890123456789012345678901212345678901234567890123456789012123456
7
7
1
12 34567890123456789012345678901212345678901234567890123456789012123456
goals of the French Revolution. circumference of the earth. 7
12 34567890123456789012345678901212345678901234567890123456789012123456 7
12 34567890123456789012345678901212345678901234567890123456789012123456 7
(C) a side-effect of the French Revolu- (C) needed a way to bring the meter’s
12 34567890123456789012345678901212345678901234567890123456789012123456 7
1234567890123456789012345678901212345678901234567890123456789012123456
tion’s new calendar system. length closer to the yard’s. 7
12 34567890123456789012345678901212345678901234567890123456789012123456 7
12 34567890123456789012345678901212345678901234567890123456789012123456 7
(D) one of many anti-traditionalist (D) have developed more accurate ways
12 34567890123456789012345678901212345678901234567890123456789012123456 7
12
2
34567890123456789012345678901212345678901234567890123456789012123456
undertakings of the French Revolu- to calculate the original fraction.
34567890123456789012345678901212345678901234567890123456789012123456
7
7
1 tion. (E) wanted to disassociate the meter
12 34567890123456789012345678901212345678901234567890123456789012123456 7
12 34567890123456789012345678901212345678901234567890123456789012123456
(E) left incomplete at the end of the with the French Revolution. 7
12 34567890123456789012345678901212345678901234567890123456789012123456 7
12 34567890123456789012345678901212345678901234567890123456789012123456
French Revolution. 7
12 34567890123456789012345678901212345678901234567890123456789012123456 7
12 34567890123456789012345678901212345678901234567890123456789012123456
15. In at least one of the passages all of the 7
12 34567890123456789012345678901212345678901234567890123456789012123456 7
12 34567890123456789012345678901212345678901234567890123456789012123456 7
12. In the sentence beginning “In the United following are mentioned EXCEPT
12 34567890123456789012345678901212345678901234567890123456789012123456 7
12 34567890123456789012345678901212345678901234567890123456789012123456
States. . .” (line 82) the writer suggests 7
1234567890123456789012345678901212345678901234567890123456789012123456
(A) the kings who ruled during the 7
12 34567890123456789012345678901212345678901234567890123456789012123456
that the United States 7
12 34567890123456789012345678901212345678901234567890123456789012123456 7
standardization of measurements.
12 34567890123456789012345678901212345678901234567890123456789012123456 7
12 34567890123456789012345678901212345678901234567890123456789012123456
(A) has never seriously attempted to (B) the contemporary standing of the 7
12 34567890123456789012345678901212345678901234567890123456789012123456 7
12 34567890123456789012345678901212345678901234567890123456789012123456 7
implement the metric system. measuring system discussed.
12 34567890123456789012345678901212345678901234567890123456789012123456
(B) is likely to adopt the metric system (C) terms from ancient languages. 7
12 34567890123456789012345678901212345678901234567890123456789012123456 7
2 34567890123456789012345678901212345678901234567890123456789012123456
1234567890123456789012345678901212345678901234567890123456789012123456
fairly soon. (D) the refinement of measurement 7
1234567890123456789012345678901212345678901234567890123456789012123456 7
1234567890123456789012345678901212345678901234567890123456789012123456
(C) has created official policies regarding standards in recent years. 7
1234567890123456789012345678901212345678901234567890123456789012123456 77
1 use of the metric system. (E) the cultural heritage of each measur-
12 34567890123456789012345678901212345678901234567890123456789012123456 7
12 34567890123456789012345678901212345678901234567890123456789012123456
(D) has attempted to require use of the ing system. 7
1234567890123456789012345678901212345678901234567890123456789012123456 7
1234567890123456789012345678901212345678901234567890123456789012123456
metric system, but has been unable 7
12 34567890123456789012345678901212345678901234567890123456789012123456 7
1234567890123456789012345678901212345678901234567890123456789012123456 77
2 34567890123456789012345678901212345678901234567890123456789012123456
to enforce its policies. 16. The author of Passage 2 conveys an
1234567890123456789012345678901212345678901234567890123456789012123456 7
1 (E) reflects a clear bias for the superior- implicit belief that the
12 34567890123456789012345678901212345678901234567890123456789012123456 7
12 34567890123456789012345678901212345678901234567890123456789012123456
ity of the English system. 7
1234567890123456789012345678901212345678901234567890123456789012123456 77
2 34567890123456789012345678901212345678901234567890123456789012123456
(A) metric system facilitates scientific
1234567890123456789012345678901212345678901234567890123456789012123456endeavors. 7
1 13. The word “refined” in line 87 most
1234567890123456789012345678901212345678901234567890123456789012123456
(B) United States has damaged its reputa- 7
1234567890123456789012345678901212345678901234567890123456789012123456 7
12 34567890123456789012345678901212345678901234567890123456789012123456
closely means
tion in the international community by 7
12 34567890123456789012345678901212345678901234567890123456789012123456 7
12 34567890123456789012345678901212345678901234567890123456789012123456
(A) processed. refusing to adopt the metric system. 7
12 34567890123456789012345678901212345678901234567890123456789012123456 7
1234567890123456789012345678901212345678901234567890123456789012123456
(B) renegotiated. (C) metric system is best confined to 7
12 34567890123456789012345678901212345678901234567890123456789012123456 7
12 34567890123456789012345678901212345678901234567890123456789012123456
(C) made smaller. scientific use. 7
12 34567890123456789012345678901212345678901234567890123456789012123456 7
12 34567890123456789012345678901212345678901234567890123456789012123456
(D) challenged. (D) French Revolution was a high point 7
1234567890123456789012345678901212345678901234567890123456789012123456 7
1234567890123456789012345678901212345678901234567890123456789012123456 77
2 34567890123456789012345678901212345678901234567890123456789012123456
(E) modified. in the history of science.
1234567890123456789012345678901212345678901234567890123456789012123456
(E) metric system is a more fitting system 7
1234567890123456789012345678901212345678901234567890123456789012123456 7
1234567890123456789012345678901212345678901234567890123456789012123456 7
1 for a democratic society.
1234567890123456789012345678901212345678901234567890123456789012123456 77
2 34567890123456789012345678901212345678901234567890123456789012123456
1
12 34567890123456789012345678901212345678901234567890123456789012123456 7
1234567890123456789012345678901212345678901234567890123456789012123456 7
12 34567890123456789012345678901212345678901234567890123456789012123456 7
STOP
1234567890123456789012345678901212345678901234567890123456789012123456
Do not proceed to the next section until time is up.
1234567890123456789012345678901212345678901234567890123456789012123456 7
7
1234567890123456789012345678901212345678901234567890123456789012123456 7
1234567890123456789012345678901212345678901234567890123456789012123456 7
12345678901234567890123456789012123456789012345678901234567890121234567
40 Copyright © 2005 Thomson Peterson’s, a part of The Thomson Corporaton
SAT is a registered trademark of the College Entrance Examination Board, which
was not involved in the production of and does not endorse this product.
Section 7
13 Questions j Time—20 Minutes

12345678901234567890123456789012123456789012345678901234567890121234567
12234567890123456789012345678901212345678901234567890123456789012123456 7
1234567890123456789012345678901212345678901234567890123456789012123456
34567890123456789012345678901212345678901234567890123456789012123456
7
7
1
12234567890123456789012345678901212345678901234567890123456789012123456 7
1234567890123456789012345678901212345678901234567890123456789012123456
Notes: 7
1234567890123456789012345678901212345678901234567890123456789012123456 7
1234567890123456789012345678901212345678901234567890123456789012123456
1. All numbers used are real numbers. 7
1234567890123456789012345678901212345678901234567890123456789012123456 77
34567890123456789012345678901212345678901234567890123456789012123456
1234567890123456789012345678901212345678901234567890123456789012123456
1234567890123456789012345678901212345678901234567890123456789012123456
2. All angle measurements can be assumed to be positive unless otherwise noted. 7
1234567890123456789012345678901212345678901234567890123456789012123456 77
1234567890123456789012345678901212345678901234567890123456789012123456
1234567890123456789012345678901212345678901234567890123456789012123456
3. All figures lie in the same plane unless otherwise noted. 7
1 7
12234567890123456789012345678901212345678901234567890123456789012123456 7
1234567890123456789012345678901212345678901234567890123456789012123456
4. Drawings that accompany questions are intended to provide information useful in 7
1234567890123456789012345678901212345678901234567890123456789012123456 7
1234567890123456789012345678901212345678901234567890123456789012123456 77
34567890123456789012345678901212345678901234567890123456789012123456
answering the question. The figures are drawn closely to scale unless otherwise noted.
1234567890123456789012345678901212345678901234567890123456789012123456 7
1234567890123456789012345678901212345678901234567890123456789012123456 7
1234567890123456789012345678901212345678901234567890123456789012123456 7
1234567890123456789012345678901212345678901234567890123456789012123456
1234567890123456789012345678901212345678901234567890123456789012123456
Directions for Student Produced Responses 7
1234567890123456789012345678901212345678901234567890123456789012123456 77
1234567890123456789012345678901212345678901234567890123456789012123456
Enter your responses to questions 1–13 in the
1234567890123456789012345678901212345678901234567890123456789012123456 Answer: 1.4 7
1 special grids provided on your answer sheet. 7
1234567890123456789012345678901212345678901234567890123456789012123456 77
2 34567890123456789012345678901212345678901234567890123456789012123456 Either position is correct.
1234567890123456789012345678901212345678901234567890123456789012123456
1 Input your answers as indicated in the directions 7
1234567890123456789012345678901212345678901234567890123456789012123456
below. 1 . 4 1 . 4 7
12234567890123456789012345678901212345678901234567890123456789012123456
34567890123456789012345678901212345678901234567890123456789012123456
7
7
1234567890123456789012345678901212345678901234567890123456789012123456 /
O O/ /O O/

1234567890123456789012345678901212345678901234567890123456789012123456
4 Decimal → O Þ O O
. . . O O Þ O
. . .
7
1 Answer: or 4/9 7
12234567890123456789012345678901212345678901234567890123456789012123456
9 point 0
O O0 O0 0O 0
O O0
7
1234567890123456789012345678901212345678901234567890123456789012123456 Þ Þ 7
34567890123456789012345678901212345678901234567890123456789012123456
1234567890123456789012345678901212345678901234567890123456789012123456 7
1
O O1 O1 1
O 1
O O1

1234567890123456789012345678901212345678901234567890123456789012123456 7
2 2
O O O O 2 2 2 2
O O O O 2 2
Write answer → 4 / 9
1 in boxes. 3 3
O O O O 3 3 3 3
O O O O 3 3
7
1234567890123456789012345678901212345678901234567890123456789012123456
Þ O ← Fraction line 4
O 4
O Þ O4 4
O 4O 4
O Þ 7
1234567890123456789012345678901212345678901234567890123456789012123456 7
/

2 34567890123456789012345678901212345678901234567890123456789012123456
1234567890123456789012345678901212345678901234567890123456789012123456 7
. .
O O O O . .

Note: You may begin your answer in any col- 7


1234567890123456789012345678901212345678901234567890123456789012123456
⎧O
1234567890123456789012345678901212345678901234567890123456789012123456 7
0
O O O0 0

umn, space permitting. Leave blank any columns 7


1234567890123456789012345678901212345678901234567890123456789012123456
1 1
O O O1 1

1 ⎪Þ
2 2
O O O O 2 2
7
12234567890123456789012345678901212345678901234567890123456789012123456
3 3
O O O O 3 3 not needed. 7
1234567890123456789012345678901212345678901234567890123456789012123456 7
Grid in →

1234567890123456789012345678901212345678901234567890123456789012123456 7
4
O O O4 4

1234567890123456789012345678901212345678901234567890123456789012123456
• 7
5 5 5 5
result. O O O O
Writing your answer in the boxes at the top
1 34567890123456789012345678901212345678901234567890123456789012123456

6 6
O O O O 6 6
7
2 34567890123456789012345678901212345678901234567890123456789012123456
1234567890123456789012345678901212345678901234567890123456789012123456
7
O O7 O7 O7 of the columns will help you accurately grid 7
1234567890123456789012345678901212345678901234567890123456789012123456 7

1234567890123456789012345678901212345678901234567890123456789012123456 7
8 8 8 8
O O O O
your answer, but it is not required. You will
1234567890123456789012345678901212345678901234567890123456789012123456
O O Þ O
7
9 9 9

1 only receive credit for an answer if the ovals 7


2 34567890123456789012345678901212345678901234567890123456789012123456
1234567890123456789012345678901212345678901234567890123456789012123456 7
1 are filled in properly. 7
12234567890123456789012345678901212345678901234567890123456789012123456 7
1 34567890123456789012345678901212345678901234567890123456789012123456
• Only fill in one oval in each column. 7
12234567890123456789012345678901212345678901234567890123456789012123456
34567890123456789012345678901212345678901234567890123456789012123456
7
7
1
1234567890123456789012345678901212345678901234567890123456789012123456 7
1234567890123456789012345678901212345678901234567890123456789012123456 7
1234567890123456789012345678901212345678901234567890123456789012123456 7
12345678901234567890123456789012123456789012345678901234567890121234567
12345678901234567890123456789012123456789012345678901234567890121234567
1234567890123456789012345678901212345678901234567890123456789012123456 7
12 34567890123456789012345678901212345678901234567890123456789012123456 7
1234567890123456789012345678901212345678901234567890123456789012123456
• If a problem has several correct answers, Decimal Accuracy 7
12 34567890123456789012345678901212345678901234567890123456789012123456 7
12 34567890123456789012345678901212345678901234567890123456789012123456
just grid in one of them. 7
1234567890123456789012345678901212345678901234567890123456789012123456 7
Decimal answers must be gridded as accu-
12 34567890123456789012345678901212345678901234567890123456789012123456
rately as possible. The answer 0.3333 . . . 7
1234567890123456789012345678901212345678901234567890123456789012123456
• There are no negative answers. 7
12 34567890123456789012345678901212345678901234567890123456789012123456 7
12 34567890123456789012345678901212345678901234567890123456789012123456

must be gridded as .333. 7
12 34567890123456789012345678901212345678901234567890123456789012123456 7
Never grid in mixed numbers. The answer
12 34567890123456789012345678901212345678901234567890123456789012123456
Less accurate values, such as .33 are not 7
1234567890123456789012345678901212345678901234567890123456789012123456
1 7
12 34567890123456789012345678901212345678901234567890123456789012123456
3 must be gridded as 16/5 or 3.2. If 7
12 34567890123456789012345678901212345678901234567890123456789012123456
5 acceptable. 7
12 34567890123456789012345678901212345678901234567890123456789012123456
is gridded, it will be read 7
12
2
34567890123456789012345678901212345678901234567890123456789012123456
3 1 / 5
34567890123456789012345678901212345678901234567890123456789012123456 1 7
7
1 31 1 Acceptable ways to grid 5 .3333 . . .
12 34567890123456789012345678901212345678901234567890123456789012123456
/O Þ as , not 3 . 3 7
12 34567890123456789012345678901212345678901234567890123456789012123456
. . . .
5 5 7
12 34567890123456789012345678901212345678901234567890123456789012123456 7
O O O O

12
2
34567890123456789012345678901212345678901234567890123456789012123456
0O 0
O 0
O
34567890123456789012345678901212345678901234567890123456789012123456
7
7
1 Þ 1 / 3 . 3 3 3
12 34567890123456789012345678901212345678901234567890123456789012123456 7
1O 1
O 1
O

12 34567890123456789012345678901212345678901234567890123456789012123456
Þ O
7
2 2
O O O O2 2 / O O / /

12 34567890123456789012345678901212345678901234567890123456789012123456
Þ 3 3
O O O 3 O O O O . . .Þ O O O
. . . .
7
12 34567890123456789012345678901212345678901234567890123456789012123456
4O 4O 4
O 4
O 7
2 34567890123456789012345678901212345678901234567890123456789012123456
1234567890123456789012345678901212345678901234567890123456789012123456 7
O O O 0 0 0O O O 0 0 0
Þ
1234567890123456789012345678901212345678901234567890123456789012123456 7
5O 5O 5
O
Þ O O O 1 1 1
O O O O 1 1 1 1

1234567890123456789012345678901212345678901234567890123456789012123456 2 2 2O 2O O 2
O 2 2 2
7
7
O O O O
1234567890123456789012345678901212345678901234567890123456789012123456 O Þ Þ Þ
1234567890123456789012345678901212345678901234567890123456789012123456 7
O O Þ O 3 3 3 3

1234567890123456789012345678901212345678901234567890123456789012123456 7
O O O O 4 4 4 4
O O O O 4 4 4 4

1 O O O O 5 5 5 5
O O O O 5 5 5 5
7
12 34567890123456789012345678901212345678901234567890123456789012123456 7 6 6 6O 6O O 6
O 6 6 6

12 34567890123456789012345678901212345678901234567890123456789012123456 7
O O O O

12 34567890123456789012345678901212345678901234567890123456789012123456 7
1234567890123456789012345678901212345678901234567890123456789012123456
2 7
1234567890123456789012345678901212345678901234567890123456789012123456 7
1234567890123456789012345678901212345678901234567890123456789012123456 7
1234567890123456789012345678901212345678901234567890123456789012123456 7
1234567890123456789012345678901212345678901234567890123456789012123456
1. If 2=x 1 =x 5 y and 2y 5 12, then 5. If the area of a circle is 16p, what is the 7
1234567890123456789012345678901212345678901234567890123456789012123456 77
34567890123456789012345678901212345678901234567890123456789012123456
1234567890123456789012345678901212345678901234567890123456789012123456
what does x equal? diameter of the circle?
1234567890123456789012345678901212345678901234567890123456789012123456 77
1234567890123456789012345678901212345678901234567890123456789012123456 7
1 2. What is the product of the first five even
12 34567890123456789012345678901212345678901234567890123456789012123456
6. Lisa has three pizza toppings from which 7
12 34567890123456789012345678901212345678901234567890123456789012123456 7
1234567890123456789012345678901212345678901234567890123456789012123456 7
integers? to choose: pepperoni, anchovies, and red
1234567890123456789012345678901212345678901234567890123456789012123456 7
12
2
34567890123456789012345678901212345678901234567890123456789012123456
peppers. If she can choose as many
34567890123456789012345678901212345678901234567890123456789012123456
7
7
1234567890123456789012345678901212345678901234567890123456789012123456
3. Rider High has 400 students. A student
1234567890123456789012345678901212345678901234567890123456789012123456
toppings as three and as few toppings as 7
1 will be picked at random from the student 7
12 34567890123456789012345678901212345678901234567890123456789012123456 7
zero, how many different pizza orders are
12 34567890123456789012345678901212345678901234567890123456789012123456
body. If the probability that a senior 7
12 34567890123456789012345678901212345678901234567890123456789012123456
possible? (Lisa cannot order the same 7
12 34567890123456789012345678901212345678901234567890123456789012123456
would be picked is three-eighths, how 7
1234567890123456789012345678901212345678901234567890123456789012123456 7
topping twice.)
1234567890123456789012345678901212345678901234567890123456789012123456
many seniors are there? 7
1234567890123456789012345678901212345678901234567890123456789012123456 7
1234567890123456789012345678901212345678901234567890123456789012123456 77
2 34567890123456789012345678901212345678901234567890123456789012123456
7. The average (arithmetic mean) of five
1234567890123456789012345678901212345678901234567890123456789012123456 7
1234567890123456789012345678901212345678901234567890123456789012123456
4. numbers is 16. If one number is taken
7
1234567890123456789012345678901212345678901234567890123456789012123456 7
1 from the set, the new average is 14. What
2 34567890123456789012345678901212345678901234567890123456789012123456
1234567890123456789012345678901212345678901234567890123456789012123456 7
1234567890123456789012345678901212345678901234567890123456789012123456
number was taken from the set? 7
1234567890123456789012345678901212345678901234567890123456789012123456 77
1234567890123456789012345678901212345678901234567890123456789012123456 7
1
12 34567890123456789012345678901212345678901234567890123456789012123456 7
12 34567890123456789012345678901212345678901234567890123456789012123456 7
12 34567890123456789012345678901212345678901234567890123456789012123456 7
12 34567890123456789012345678901212345678901234567890123456789012123456 7
1234567890123456789012345678901212345678901234567890123456789012123456 7
1234567890123456789012345678901212345678901234567890123456789012123456
2 7
1 34567890123456789012345678901212345678901234567890123456789012123456 7
12 34567890123456789012345678901212345678901234567890123456789012123456
If the area of the striped side of the 7
1234567890123456789012345678901212345678901234567890123456789012123456 7
12
2
34567890123456789012345678901212345678901234567890123456789012123456 7
rectangular solid is 24, what is the volume
34567890123456789012345678901212345678901234567890123456789012123456 7
1
1234567890123456789012345678901212345678901234567890123456789012123456
of the box? 7
1234567890123456789012345678901212345678901234567890123456789012123456 77
1234567890123456789012345678901212345678901234567890123456789012123456
12345678901234567890123456789012123456789012345678901234567890121234567
42 Copyright © 2005 Thomson Peterson’s, a part of The Thomson Corporaton
SAT is a registered trademark of the College Entrance Examination Board, which
was not involved in the production of and does not endorse this product.
12345678901234567890123456789012123456789012345678901234567890121234567
1234567890123456789012345678901212345678901234567890123456789012123456 7
12234567890123456789012345678901212345678901234567890123456789012123456 7
1 34567890123456789012345678901212345678901234567890123456789012123456
8. 12. 7
12234567890123456789012345678901212345678901234567890123456789012123456 7
1234567890123456789012345678901212345678901234567890123456789012123456 7
1 34567890123456789012345678901212345678901234567890123456789012123456 7
12234567890123456789012345678901212345678901234567890123456789012123456 7
1 34567890123456789012345678901212345678901234567890123456789012123456 7
12234567890123456789012345678901212345678901234567890123456789012123456 7
1234567890123456789012345678901212345678901234567890123456789012123456 7
1234567890123456789012345678901212345678901234567890123456789012123456 7
1234567890123456789012345678901212345678901234567890123456789012123456
34567890123456789012345678901212345678901234567890123456789012123456
7
7
1
12234567890123456789012345678901212345678901234567890123456789012123456 7
1234567890123456789012345678901212345678901234567890123456789012123456 7
1234567890123456789012345678901212345678901234567890123456789012123456 7
1234567890123456789012345678901212345678901234567890123456789012123456
If the figure above is composed of a
34567890123456789012345678901212345678901234567890123456789012123456
7
7
1
12234567890123456789012345678901212345678901234567890123456789012123456
square and a triangle, what is the area of 7
1234567890123456789012345678901212345678901234567890123456789012123456 7
1234567890123456789012345678901212345678901234567890123456789012123456
the square? 7
1234567890123456789012345678901212345678901234567890123456789012123456
34567890123456789012345678901212345678901234567890123456789012123456
7
7
1 9. What is the slope a line that is defined by
12234567890123456789012345678901212345678901234567890123456789012123456 7
1234567890123456789012345678901212345678901234567890123456789012123456
What is the sum of the y-intercepts in the 7
1234567890123456789012345678901212345678901234567890123456789012123456
the following two points, (21, 22) and above graph? 7
1234567890123456789012345678901212345678901234567890123456789012123456 7
1234567890123456789012345678901212345678901234567890123456789012123456 77
34567890123456789012345678901212345678901234567890123456789012123456
(3,1)?
1234567890123456789012345678901212345678901234567890123456789012123456
1234567890123456789012345678901212345678901234567890123456789012123456
13. The sum of four positive distinct prime 7
1234567890123456789012345678901212345678901234567890123456789012123456 7
1234567890123456789012345678901212345678901234567890123456789012123456 77
10. Tom is twelve years older than Susan, and numbers is 21. What is the greatest
1234567890123456789012345678901212345678901234567890123456789012123456 7
1 Susan is three times the age of Gina, and possible value of one of those prime
12234567890123456789012345678901212345678901234567890123456789012123456
Gina is five years younger than Bo. If Bo is 7
1234567890123456789012345678901212345678901234567890123456789012123456
numbers? 7
1234567890123456789012345678901212345678901234567890123456789012123456
15, how old is Tom? 7
1234567890123456789012345678901212345678901234567890123456789012123456 77
34567890123456789012345678901212345678901234567890123456789012123456
1234567890123456789012345678901212345678901234567890123456789012123456 7
1234567890123456789012345678901212345678901234567890123456789012123456
1234567890123456789012345678901212345678901234567890123456789012123456
11. For all positive integers, let m* equal the 7
1234567890123456789012345678901212345678901234567890123456789012123456 7
1234567890123456789012345678901212345678901234567890123456789012123456 77
greatest prime divisor of m. What does
1234567890123456789012345678901212345678901234567890123456789012123456
1234567890123456789012345678901212345678901234567890123456789012123456
(15*)(12*) equal? 7
1234567890123456789012345678901212345678901234567890123456789012123456 77
1
12234567890123456789012345678901212345678901234567890123456789012123456 7
1234567890123456789012345678901212345678901234567890123456789012123456 7
1 34567890123456789012345678901212345678901234567890123456789012123456 7
1234567890123456789012345678901212345678901234567890123456789012123456 7
12234567890123456789012345678901212345678901234567890123456789012123456 7
1234567890123456789012345678901212345678901234567890123456789012123456 7
1234567890123456789012345678901212345678901234567890123456789012123456 7
1 34567890123456789012345678901212345678901234567890123456789012123456 7
12234567890123456789012345678901212345678901234567890123456789012123456 7
1234567890123456789012345678901212345678901234567890123456789012123456 7
1234567890123456789012345678901212345678901234567890123456789012123456 77
34567890123456789012345678901212345678901234567890123456789012123456
1234567890123456789012345678901212345678901234567890123456789012123456 7
1
1234567890123456789012345678901212345678901234567890123456789012123456 7
1234567890123456789012345678901212345678901234567890123456789012123456 7
12234567890123456789012345678901212345678901234567890123456789012123456 7
1234567890123456789012345678901212345678901234567890123456789012123456 7
1234567890123456789012345678901212345678901234567890123456789012123456 7
1234567890123456789012345678901212345678901234567890123456789012123456 7
1 34567890123456789012345678901212345678901234567890123456789012123456 7
1234567890123456789012345678901212345678901234567890123456789012123456 77
2 34567890123456789012345678901212345678901234567890123456789012123456
1234567890123456789012345678901212345678901234567890123456789012123456 7
1234567890123456789012345678901212345678901234567890123456789012123456 7
1234567890123456789012345678901212345678901234567890123456789012123456 7
1
12234567890123456789012345678901212345678901234567890123456789012123456 7
1234567890123456789012345678901212345678901234567890123456789012123456 7
1234567890123456789012345678901212345678901234567890123456789012123456 7
1234567890123456789012345678901212345678901234567890123456789012123456 7
1 34567890123456789012345678901212345678901234567890123456789012123456 7
1234567890123456789012345678901212345678901234567890123456789012123456 77
2 34567890123456789012345678901212345678901234567890123456789012123456
1
12234567890123456789012345678901212345678901234567890123456789012123456 7
1 34567890123456789012345678901212345678901234567890123456789012123456 7
12234567890123456789012345678901212345678901234567890123456789012123456 7
STOP
1 34567890123456789012345678901212345678901234567890123456789012123456
Do not proceed to the next section until time is up.
1234567890123456789012345678901212345678901234567890123456789012123456 7
7
1234567890123456789012345678901212345678901234567890123456789012123456 7
1234567890123456789012345678901212345678901234567890123456789012123456 7
12345678901234567890123456789012123456789012345678901234567890121234567
Copyright © 2005 Thomson Peterson’s, a part of The Thomson Corporaton 43
SAT is a registered trademark of the College Entrance Examination Board, which
was not involved in the production of and does not endorse this product.
Section 8
Time—25 Minutes

12345678901234567890123456789012123456789012345678901234567890121234567
12 34567890123456789012345678901212345678901234567890123456789012123456 7
12 34567890123456789012345678901212345678901234567890123456789012123456 7
1 34567890123456789012345678901212345678901234567890123456789012123456 7
2
12 34567890123456789012345678901212345678901234567890123456789012123456 7
12 34567890123456789012345678901212345678901234567890123456789012123456
Directions: Think carefully about the statement below and the assignment that follows it. 7
12 34567890123456789012345678901212345678901234567890123456789012123456 7
12 34567890123456789012345678901212345678901234567890123456789012123456 7
12 34567890123456789012345678901212345678901234567890123456789012123456 7
12 34567890123456789012345678901212345678901234567890123456789012123456
Consider the following statement. 7
12 34567890123456789012345678901212345678901234567890123456789012123456 7
12 34567890123456789012345678901212345678901234567890123456789012123456
Innovation is primarily accomplished by an individual, though groups often do work out the 7
12 34567890123456789012345678901212345678901234567890123456789012123456 7
2 34567890123456789012345678901212345678901234567890123456789012123456
1234567890123456789012345678901212345678901234567890123456789012123456 7
1 details of an individual’s innovations. 7
12 34567890123456789012345678901212345678901234567890123456789012123456 7
12 34567890123456789012345678901212345678901234567890123456789012123456
Assignment: Write an essay in which you argue for or against the preceding statement. Develop your 7
12 34567890123456789012345678901212345678901234567890123456789012123456 7
2 34567890123456789012345678901212345678901234567890123456789012123456
1234567890123456789012345678901212345678901234567890123456789012123456
point of view on this statement and be sure to support your stance with sufficient details. 7
1234567890123456789012345678901212345678901234567890123456789012123456 7
1234567890123456789012345678901212345678901234567890123456789012123456 77
1234567890123456789012345678901212345678901234567890123456789012123456 7
1234567890123456789012345678901212345678901234567890123456789012123456 7
1234567890123456789012345678901212345678901234567890123456789012123456 7
1234567890123456789012345678901212345678901234567890123456789012123456 7
1234567890123456789012345678901212345678901234567890123456789012123456 7
1234567890123456789012345678901212345678901234567890123456789012123456 7
1
12 34567890123456789012345678901212345678901234567890123456789012123456 7
12 34567890123456789012345678901212345678901234567890123456789012123456 7
1234567890123456789012345678901212345678901234567890123456789012123456 7
1234567890123456789012345678901212345678901234567890123456789012123456 7
12 34567890123456789012345678901212345678901234567890123456789012123456 7
12 34567890123456789012345678901212345678901234567890123456789012123456 7
12 34567890123456789012345678901212345678901234567890123456789012123456 7
1234567890123456789012345678901212345678901234567890123456789012123456 7
12 34567890123456789012345678901212345678901234567890123456789012123456 7
12 34567890123456789012345678901212345678901234567890123456789012123456 7
1234567890123456789012345678901212345678901234567890123456789012123456
2 7
1234567890123456789012345678901212345678901234567890123456789012123456 7
1 34567890123456789012345678901212345678901234567890123456789012123456 7
1234567890123456789012345678901212345678901234567890123456789012123456 7
1234567890123456789012345678901212345678901234567890123456789012123456 7
12 34567890123456789012345678901212345678901234567890123456789012123456 7
12 34567890123456789012345678901212345678901234567890123456789012123456 7
12 34567890123456789012345678901212345678901234567890123456789012123456 7
12 34567890123456789012345678901212345678901234567890123456789012123456 7
1234567890123456789012345678901212345678901234567890123456789012123456 7
1234567890123456789012345678901212345678901234567890123456789012123456
2 7
1234567890123456789012345678901212345678901234567890123456789012123456 7
1234567890123456789012345678901212345678901234567890123456789012123456 7
1234567890123456789012345678901212345678901234567890123456789012123456 7
1 34567890123456789012345678901212345678901234567890123456789012123456 7
1234567890123456789012345678901212345678901234567890123456789012123456 77
2 34567890123456789012345678901212345678901234567890123456789012123456
1234567890123456789012345678901212345678901234567890123456789012123456 7
1234567890123456789012345678901212345678901234567890123456789012123456 7
1234567890123456789012345678901212345678901234567890123456789012123456 7
1
12 34567890123456789012345678901212345678901234567890123456789012123456 7
1234567890123456789012345678901212345678901234567890123456789012123456 7
12 34567890123456789012345678901212345678901234567890123456789012123456 7
1234567890123456789012345678901212345678901234567890123456789012123456 7
12 34567890123456789012345678901212345678901234567890123456789012123456 7
1234567890123456789012345678901212345678901234567890123456789012123456 7
1234567890123456789012345678901212345678901234567890123456789012123456 7
1234567890123456789012345678901212345678901234567890123456789012123456 7
1234567890123456789012345678901212345678901234567890123456789012123456 7
12345678901234567890123456789012123456789012345678901234567890121234567
44 Copyright © 2005 Thomson Peterson’s, a part of The Thomson Corporaton
SAT is a registered trademark of the College Entrance Examination Board, which
was not involved in the production of and does not endorse this product.
12345678901234567890123456789012123456789012345678901234567890121234567
1234567890123456789012345678901212345678901234567890123456789012123456 7
12234567890123456789012345678901212345678901234567890123456789012123456 7
1 34567890123456789012345678901212345678901234567890123456789012123456 7
12234567890123456789012345678901212345678901234567890123456789012123456 7
1234567890123456789012345678901212345678901234567890123456789012123456 7
1 34567890123456789012345678901212345678901234567890123456789012123456 7
12234567890123456789012345678901212345678901234567890123456789012123456 7
1 34567890123456789012345678901212345678901234567890123456789012123456 7
12234567890123456789012345678901212345678901234567890123456789012123456 7
1234567890123456789012345678901212345678901234567890123456789012123456 7
1234567890123456789012345678901212345678901234567890123456789012123456 7
1234567890123456789012345678901212345678901234567890123456789012123456
34567890123456789012345678901212345678901234567890123456789012123456
7
7
1
12234567890123456789012345678901212345678901234567890123456789012123456 7
1234567890123456789012345678901212345678901234567890123456789012123456 7
1234567890123456789012345678901212345678901234567890123456789012123456 7
1234567890123456789012345678901212345678901234567890123456789012123456
34567890123456789012345678901212345678901234567890123456789012123456
7
7
1
12234567890123456789012345678901212345678901234567890123456789012123456 7
1234567890123456789012345678901212345678901234567890123456789012123456 7
1234567890123456789012345678901212345678901234567890123456789012123456 7
1234567890123456789012345678901212345678901234567890123456789012123456
34567890123456789012345678901212345678901234567890123456789012123456
7
7
1
12234567890123456789012345678901212345678901234567890123456789012123456 7
1234567890123456789012345678901212345678901234567890123456789012123456 7
1234567890123456789012345678901212345678901234567890123456789012123456 7
1234567890123456789012345678901212345678901234567890123456789012123456 7
1234567890123456789012345678901212345678901234567890123456789012123456 77
34567890123456789012345678901212345678901234567890123456789012123456
1234567890123456789012345678901212345678901234567890123456789012123456 7
1234567890123456789012345678901212345678901234567890123456789012123456 7
1234567890123456789012345678901212345678901234567890123456789012123456 7
1234567890123456789012345678901212345678901234567890123456789012123456 7
1234567890123456789012345678901212345678901234567890123456789012123456 7
1
12234567890123456789012345678901212345678901234567890123456789012123456 7
1234567890123456789012345678901212345678901234567890123456789012123456 7
1234567890123456789012345678901212345678901234567890123456789012123456 7
1234567890123456789012345678901212345678901234567890123456789012123456 77
34567890123456789012345678901212345678901234567890123456789012123456
1234567890123456789012345678901212345678901234567890123456789012123456 7
1234567890123456789012345678901212345678901234567890123456789012123456 7
1234567890123456789012345678901212345678901234567890123456789012123456 7
1234567890123456789012345678901212345678901234567890123456789012123456 7
1234567890123456789012345678901212345678901234567890123456789012123456 7
1234567890123456789012345678901212345678901234567890123456789012123456 7
1234567890123456789012345678901212345678901234567890123456789012123456 7
1234567890123456789012345678901212345678901234567890123456789012123456 7
1
12234567890123456789012345678901212345678901234567890123456789012123456 7
1234567890123456789012345678901212345678901234567890123456789012123456 7
1 34567890123456789012345678901212345678901234567890123456789012123456 7
1234567890123456789012345678901212345678901234567890123456789012123456 7
12234567890123456789012345678901212345678901234567890123456789012123456 7
1234567890123456789012345678901212345678901234567890123456789012123456 7
1234567890123456789012345678901212345678901234567890123456789012123456 7
1 34567890123456789012345678901212345678901234567890123456789012123456 7
12234567890123456789012345678901212345678901234567890123456789012123456 7
1234567890123456789012345678901212345678901234567890123456789012123456 7
1234567890123456789012345678901212345678901234567890123456789012123456 77
34567890123456789012345678901212345678901234567890123456789012123456
1234567890123456789012345678901212345678901234567890123456789012123456 7
1
1234567890123456789012345678901212345678901234567890123456789012123456 7
1234567890123456789012345678901212345678901234567890123456789012123456 7
12234567890123456789012345678901212345678901234567890123456789012123456 7
1234567890123456789012345678901212345678901234567890123456789012123456 7
1234567890123456789012345678901212345678901234567890123456789012123456 7
1234567890123456789012345678901212345678901234567890123456789012123456 7
1 34567890123456789012345678901212345678901234567890123456789012123456 7
1234567890123456789012345678901212345678901234567890123456789012123456 77
2 34567890123456789012345678901212345678901234567890123456789012123456
1234567890123456789012345678901212345678901234567890123456789012123456 7
1234567890123456789012345678901212345678901234567890123456789012123456 7
1234567890123456789012345678901212345678901234567890123456789012123456 7
1
12234567890123456789012345678901212345678901234567890123456789012123456 7
1234567890123456789012345678901212345678901234567890123456789012123456 7
1234567890123456789012345678901212345678901234567890123456789012123456 7
1234567890123456789012345678901212345678901234567890123456789012123456 7
1 34567890123456789012345678901212345678901234567890123456789012123456 7
1234567890123456789012345678901212345678901234567890123456789012123456 77
2 34567890123456789012345678901212345678901234567890123456789012123456
1
12234567890123456789012345678901212345678901234567890123456789012123456 7
1 34567890123456789012345678901212345678901234567890123456789012123456 7
12234567890123456789012345678901212345678901234567890123456789012123456 7
1 34567890123456789012345678901212345678901234567890123456789012123456 7
1234567890123456789012345678901212345678901234567890123456789012123456 7
1234567890123456789012345678901212345678901234567890123456789012123456 7
1234567890123456789012345678901212345678901234567890123456789012123456 7
12345678901234567890123456789012123456789012345678901234567890121234567
Copyright © 2005 Thomson Peterson’s, a part of The Thomson Corporaton 45
SAT is a registered trademark of the College Entrance Examination Board, which
was not involved in the production of and does not endorse this product.
12345678901234567890123456789012123456789012345678901234567890121234567
1234567890123456789012345678901212345678901234567890123456789012123456 7
12 34567890123456789012345678901212345678901234567890123456789012123456 7
1234567890123456789012345678901212345678901234567890123456789012123456 7
12 34567890123456789012345678901212345678901234567890123456789012123456 7
12 34567890123456789012345678901212345678901234567890123456789012123456 7
1234567890123456789012345678901212345678901234567890123456789012123456 7
12 34567890123456789012345678901212345678901234567890123456789012123456 7
1234567890123456789012345678901212345678901234567890123456789012123456 7
12 34567890123456789012345678901212345678901234567890123456789012123456 7
12 34567890123456789012345678901212345678901234567890123456789012123456 7
12 34567890123456789012345678901212345678901234567890123456789012123456 7
12 34567890123456789012345678901212345678901234567890123456789012123456 7
1 34567890123456789012345678901212345678901234567890123456789012123456 7
2
12 34567890123456789012345678901212345678901234567890123456789012123456 7
12 34567890123456789012345678901212345678901234567890123456789012123456 7
12 34567890123456789012345678901212345678901234567890123456789012123456 7
12 34567890123456789012345678901212345678901234567890123456789012123456 7
1 34567890123456789012345678901212345678901234567890123456789012123456 7
2
12 34567890123456789012345678901212345678901234567890123456789012123456 7
12 34567890123456789012345678901212345678901234567890123456789012123456 7
12 34567890123456789012345678901212345678901234567890123456789012123456 7
12 34567890123456789012345678901212345678901234567890123456789012123456 7
1 34567890123456789012345678901212345678901234567890123456789012123456 7
2
12 34567890123456789012345678901212345678901234567890123456789012123456 7
12 34567890123456789012345678901212345678901234567890123456789012123456 7
12 34567890123456789012345678901212345678901234567890123456789012123456 7
12 34567890123456789012345678901212345678901234567890123456789012123456 7
1234567890123456789012345678901212345678901234567890123456789012123456
2 7
1234567890123456789012345678901212345678901234567890123456789012123456 7
1234567890123456789012345678901212345678901234567890123456789012123456 7
1234567890123456789012345678901212345678901234567890123456789012123456 7
1234567890123456789012345678901212345678901234567890123456789012123456 7
1234567890123456789012345678901212345678901234567890123456789012123456 77
34567890123456789012345678901212345678901234567890123456789012123456
1
12 34567890123456789012345678901212345678901234567890123456789012123456 7
12 34567890123456789012345678901212345678901234567890123456789012123456 7
12 34567890123456789012345678901212345678901234567890123456789012123456 7
1234567890123456789012345678901212345678901234567890123456789012123456
2 7
1234567890123456789012345678901212345678901234567890123456789012123456 7
1234567890123456789012345678901212345678901234567890123456789012123456 7
1234567890123456789012345678901212345678901234567890123456789012123456 7
1234567890123456789012345678901212345678901234567890123456789012123456 7
1234567890123456789012345678901212345678901234567890123456789012123456 77
34567890123456789012345678901212345678901234567890123456789012123456
1234567890123456789012345678901212345678901234567890123456789012123456 7
1234567890123456789012345678901212345678901234567890123456789012123456 7
1234567890123456789012345678901212345678901234567890123456789012123456 7
1
12 34567890123456789012345678901212345678901234567890123456789012123456 7
12 34567890123456789012345678901212345678901234567890123456789012123456 7
1234567890123456789012345678901212345678901234567890123456789012123456 7
1234567890123456789012345678901212345678901234567890123456789012123456 7
12 34567890123456789012345678901212345678901234567890123456789012123456 7
12 34567890123456789012345678901212345678901234567890123456789012123456 7
12 34567890123456789012345678901212345678901234567890123456789012123456 7
1234567890123456789012345678901212345678901234567890123456789012123456 7
12 34567890123456789012345678901212345678901234567890123456789012123456 7
12 34567890123456789012345678901212345678901234567890123456789012123456 7
1234567890123456789012345678901212345678901234567890123456789012123456
2 7
1234567890123456789012345678901212345678901234567890123456789012123456 7
1 34567890123456789012345678901212345678901234567890123456789012123456 7
1234567890123456789012345678901212345678901234567890123456789012123456 7
1234567890123456789012345678901212345678901234567890123456789012123456 7
12 34567890123456789012345678901212345678901234567890123456789012123456 7
12 34567890123456789012345678901212345678901234567890123456789012123456 7
12 34567890123456789012345678901212345678901234567890123456789012123456 7
12 34567890123456789012345678901212345678901234567890123456789012123456 7
1234567890123456789012345678901212345678901234567890123456789012123456 7
1234567890123456789012345678901212345678901234567890123456789012123456
2 7
1234567890123456789012345678901212345678901234567890123456789012123456 7
1234567890123456789012345678901212345678901234567890123456789012123456 7
1234567890123456789012345678901212345678901234567890123456789012123456 7
1 34567890123456789012345678901212345678901234567890123456789012123456 7
1234567890123456789012345678901212345678901234567890123456789012123456 77
2 34567890123456789012345678901212345678901234567890123456789012123456
1234567890123456789012345678901212345678901234567890123456789012123456 7
1234567890123456789012345678901212345678901234567890123456789012123456 7
1234567890123456789012345678901212345678901234567890123456789012123456 7
1
12 34567890123456789012345678901212345678901234567890123456789012123456 7
1234567890123456789012345678901212345678901234567890123456789012123456 7
12 34567890123456789012345678901212345678901234567890123456789012123456 7
1234567890123456789012345678901212345678901234567890123456789012123456 7
12 34567890123456789012345678901212345678901234567890123456789012123456
When you are finished with your essay put your pencil down until the 7
1234567890123456789012345678901212345678901234567890123456789012123456
STOP 7
1234567890123456789012345678901212345678901234567890123456789012123456
time allotted is over. 7
1234567890123456789012345678901212345678901234567890123456789012123456 77
1234567890123456789012345678901212345678901234567890123456789012123456
12345678901234567890123456789012123456789012345678901234567890121234567
46 Copyright © 2005 Thomson Peterson’s, a part of The Thomson Corporaton
SAT is a registered trademark of the College Entrance Examination Board, which
was not involved in the production of and does not endorse this product.
Answers
and Explanations
1234567890123456789012345678901212345678901234567890123456789
123456789012345678901234567890121234567890123456789012345678 9
123456789012345678901234567890121234567890123456789012345678 9
123456789012345678901234567890121234567890123456789012345678
Section 1 9
123456789012345678901234567890121234567890123456789012345678 9
123456789012345678901234567890121234567890123456789012345678 9
23456789012345678901234567890121234567890123456789012345678
123456789012345678901234567890121234567890123456789012345678 9
1 1. B The passage says that H.R. means House of Representatives and that 9
123456789012345678901234567890121234567890123456789012345678
the number is given according to, “the order it was submitted in a 9
123456789012345678901234567890121234567890123456789012345678 9
123456789012345678901234567890121234567890123456789012345678
particular session.” (lines 12–13) So the 1 means that it was submitted 9
123456789012345678901234567890121234567890123456789012345678 99
23456789012345678901234567890121234567890123456789012345678
123456789012345678901234567890121234567890123456789012345678
first in a particular session. Choice (B) is the correct answer.
123456789012345678901234567890121234567890123456789012345678 9
123456789012345678901234567890121234567890123456789012345678 9
123456789012345678901234567890121234567890123456789012345678
2. C This is a tricky question. The major clue is that only three entities are 9
123456789012345678901234567890121234567890123456789012345678 9
123456789012345678901234567890121234567890123456789012345678
mentioned in the passage: the House of Representatives, the Senate, and 9
123456789012345678901234567890121234567890123456789012345678 9
123456789012345678901234567890121234567890123456789012345678
the President. You should suspect then that the answer will be the 9
1 9
123456789012345678901234567890121234567890123456789012345678 99
23456789012345678901234567890121234567890123456789012345678
Senate and the President since those are the only other two entities
123456789012345678901234567890121234567890123456789012345678 9
1 mentioned in the passage. You can get confirmation for this hunch in
123456789012345678901234567890121234567890123456789012345678 9
123456789012345678901234567890121234567890123456789012345678
the sentence that says “Bills are presented
23456789012345678901234567890121234567890123456789012345678
to the President for action 9
9
123456789012345678901234567890121234567890123456789012345678
123456789012345678901234567890121234567890123456789012345678
when approved in identical form by both the House of Representatives 9
1 9
123456789012345678901234567890121234567890123456789012345678
and the Senate.” Bills must pass both houses, so if the bills originate in 9
123456789012345678901234567890121234567890123456789012345678 9
123456789012345678901234567890121234567890123456789012345678 99
23456789012345678901234567890121234567890123456789012345678
the house, they have to go to the Senate before reaching the President.
123456789012345678901234567890121234567890123456789012345678 9
1 The answer is (C).
123456789012345678901234567890121234567890123456789012345678 9
123456789012345678901234567890121234567890123456789012345678
3. A The paragraph is about how Native Americans have “long recognized 9
123456789012345678901234567890121234567890123456789012345678 9
123456789012345678901234567890121234567890123456789012345678
and celebrated the connectedness among all natural things.” The 9
123456789012345678901234567890121234567890123456789012345678 9
123456789012345678901234567890121234567890123456789012345678
medicine wheel illustrates this point because it emphasizes the 9
123456789012345678901234567890121234567890123456789012345678 9
123456789012345678901234567890121234567890123456789012345678 99
23456789012345678901234567890121234567890123456789012345678
connectedness of the four basic elements. So (A) is the best choice.
123456789012345678901234567890121234567890123456789012345678 9
123456789012345678901234567890121234567890123456789012345678 9
123456789012345678901234567890121234567890123456789012345678
4. E Remember the Native American view stated in the passage is that all
9
1
123456789012345678901234567890121234567890123456789012345678 99
23456789012345678901234567890121234567890123456789012345678
natural things are connected; so we should expect that the scenario will
123456789012345678901234567890121234567890123456789012345678 9
123456789012345678901234567890121234567890123456789012345678
demonstrate this belief. (A) is specifically opposed to that. (C) is also, to
9
123456789012345678901234567890121234567890123456789012345678 9
1 a lesser degree. (B) is neutral toward the Native American idea—it
23456789012345678901234567890121234567890123456789012345678
123456789012345678901234567890121234567890123456789012345678 9
1 neither contradicts it nor realizes it. (D) and (E) are in accord with the 9
123456789012345678901234567890121234567890123456789012345678 9
123456789012345678901234567890121234567890123456789012345678 9
Native American view because they recognize the connectedness of an
123456789012345678901234567890121234567890123456789012345678 9
123456789012345678901234567890121234567890123456789012345678
organism and its habitat. However, (D) mention studying a specific 9
123456789012345678901234567890121234567890123456789012345678 9
123456789012345678901234567890121234567890123456789012345678 9
123456789012345678901234567890121234567890123456789012345678 9
1234567890123456789012345678901212345678901234567890123456789
Copyright © 2005 Thomson Peterson’s, a part of The Thomson Corporaton 47
SAT is a registered trademark of the College Entrance Examination Board, which
was not involved in the production of and does not endorse this product.
1234567890123456789012345678901212345678901234567890123456789
123456789012345678901234567890121234567890123456789012345678 9
123456789012345678901234567890121234567890123456789012345678 9
123456789012345678901234567890121234567890123456789012345678 9
123456789012345678901234567890121234567890123456789012345678 9
123456789012345678901234567890121234567890123456789012345678
organism in its habitat, while (E) mentions studying a habitat as a 9
123456789012345678901234567890121234567890123456789012345678 9
123456789012345678901234567890121234567890123456789012345678
whole. So, (E) is broader and, in this
23456789012345678901234567890121234567890123456789012345678
case, broader is better. The best 9
9
1
123456789012345678901234567890121234567890123456789012345678
answer is (E). 9
123456789012345678901234567890121234567890123456789012345678 9
123456789012345678901234567890121234567890123456789012345678 9
123456789012345678901234567890121234567890123456789012345678
5. C Does the author portray Copland’s
23456789012345678901234567890121234567890123456789012345678
music in a very negative or very 9
9
1 positive light? Neither, so (A) and (E) are out. The author actually just
123456789012345678901234567890121234567890123456789012345678 9
123456789012345678901234567890121234567890123456789012345678
describes the reactions of others to Copland’s music and interjects little 9
123456789012345678901234567890121234567890123456789012345678 9
123456789012345678901234567890121234567890123456789012345678
opinion of his or her own. That eliminates (D). Then again, the author
23456789012345678901234567890121234567890123456789012345678
9
9
1
123456789012345678901234567890121234567890123456789012345678
is not clinically objective in tone (just think, the passage does not sound 9
123456789012345678901234567890121234567890123456789012345678 9
123456789012345678901234567890121234567890123456789012345678
like a science passage). That makes (C) the best choice. 9
123456789012345678901234567890121234567890123456789012345678
23456789012345678901234567890121234567890123456789012345678
9
9
1
123456789012345678901234567890121234567890123456789012345678
6. C The virtuoso passages are passages only a virtuoso could play— 9
123456789012345678901234567890121234567890123456789012345678 9
123456789012345678901234567890121234567890123456789012345678
essentially, difficult. Eliminate (E). There is no negative judgment of 9
123456789012345678901234567890121234567890123456789012345678 9
123456789012345678901234567890121234567890123456789012345678 9
23456789012345678901234567890121234567890123456789012345678
that quality in the passage. That eliminates all but (C), which is the
123456789012345678901234567890121234567890123456789012345678 9
123456789012345678901234567890121234567890123456789012345678
answer. 9
123456789012345678901234567890121234567890123456789012345678 9
123456789012345678901234567890121234567890123456789012345678 9
123456789012345678901234567890121234567890123456789012345678
7. C The different fates were that the Piano Variation was known by many 9
1 9
123456789012345678901234567890121234567890123456789012345678
pianists, but played by few in concert, and the Symphonic Ode was 9
123456789012345678901234567890121234567890123456789012345678
largely unknown because it was too difficult. Choice (C) says as much, 9
123456789012345678901234567890121234567890123456789012345678 9
23456789012345678901234567890121234567890123456789012345678
123456789012345678901234567890121234567890123456789012345678
and so is the answer. 9
123456789012345678901234567890121234567890123456789012345678 9
123456789012345678901234567890121234567890123456789012345678 9
123456789012345678901234567890121234567890123456789012345678
8. A In the passage, Koussevitzky is first mentioned in connection to the 9
123456789012345678901234567890121234567890123456789012345678 9
123456789012345678901234567890121234567890123456789012345678 9
difficulty of playing the Symphonic Ode. The passage tells you that
123456789012345678901234567890121234567890123456789012345678 9
123456789012345678901234567890121234567890123456789012345678
even he, a “champion” of Copland’s music, did not conduct the piece 9
123456789012345678901234567890121234567890123456789012345678 9
1 until it had been simplified. Choice (A) correctly points this out. Don’t 9
23456789012345678901234567890121234567890123456789012345678
123456789012345678901234567890121234567890123456789012345678 9
123456789012345678901234567890121234567890123456789012345678
be tricked by the difficult name into answering that he was European. 9
1 9
123456789012345678901234567890121234567890123456789012345678 9
123456789012345678901234567890121234567890123456789012345678
9. C The second paragraph begins, “Perhaps
23456789012345678901234567890121234567890123456789012345678
as a reaction to the perfor- 9
9
123456789012345678901234567890121234567890123456789012345678
123456789012345678901234567890121234567890123456789012345678
mance problems of the Symphonic Ode, Copland’s next two orchestral 9
1 9
123456789012345678901234567890121234567890123456789012345678
works deal in shorter units of time.” So the author thinks that the pieces 9
123456789012345678901234567890121234567890123456789012345678 9
123456789012345678901234567890121234567890123456789012345678 9
23456789012345678901234567890121234567890123456789012345678
immediately subsequent to the Symphonic Ode were made shorter
123456789012345678901234567890121234567890123456789012345678
because the Ode was not being performed. Choice (C) says the same. 9
1 9
123456789012345678901234567890121234567890123456789012345678
(B) is tempting but the passage explicitly states that it was the length 9
123456789012345678901234567890121234567890123456789012345678 9
23456789012345678901234567890121234567890123456789012345678
123456789012345678901234567890121234567890123456789012345678
and not the complexity that Copland adjusted in the subsequent pieces. 9
123456789012345678901234567890121234567890123456789012345678 9
123456789012345678901234567890121234567890123456789012345678 9
123456789012345678901234567890121234567890123456789012345678
10. E This sentence is essentially saying that, although brief, these works were 9
1 9
123456789012345678901234567890121234567890123456789012345678 9
23456789012345678901234567890121234567890123456789012345678
also very difficult to perform. The answer is (E). This is essentially a
9
123456789012345678901234567890121234567890123456789012345678 9
123456789012345678901234567890121234567890123456789012345678
vocabulary question, asking you to define “agile.”
123456789012345678901234567890121234567890123456789012345678 9 9
1 11. C First off, what does the passage state about Copland’s views of the
123456789012345678901234567890121234567890123456789012345678 9
123456789012345678901234567890121234567890123456789012345678 9
123456789012345678901234567890121234567890123456789012345678
Americas and music? It says that Copland, “envisioned ‘American 9
123456789012345678901234567890121234567890123456789012345678 9
123456789012345678901234567890121234567890123456789012345678 9
music’ as being music of the Americas.” In other words, he thought
23456789012345678901234567890121234567890123456789012345678
123456789012345678901234567890121234567890123456789012345678 9
1 they were related, or should be. (A), (B), and (E) run contrary to this 9
123456789012345678901234567890121234567890123456789012345678 9
123456789012345678901234567890121234567890123456789012345678 9
meaning. (D) is irrelevant. The answer is (C).
123456789012345678901234567890121234567890123456789012345678 9
123456789012345678901234567890121234567890123456789012345678 9
123456789012345678901234567890121234567890123456789012345678 9
123456789012345678901234567890121234567890123456789012345678 9
123456789012345678901234567890121234567890123456789012345678 9
1234567890123456789012345678901212345678901234567890123456789
48 Copyright © 2005 Thomson Peterson’s, a part of The Thomson Corporaton
SAT is a registered trademark of the College Entrance Examination Board, which
was not involved in the production of and does not endorse this product.
1234567890123456789012345678901212345678901234567890123456789
123456789012345678901234567890121234567890123456789012345678 9
123456789012345678901234567890121234567890123456789012345678 9
123456789012345678901234567890121234567890123456789012345678 9
123456789012345678901234567890121234567890123456789012345678 9
123456789012345678901234567890121234567890123456789012345678
12. D The Mexican material is more accessible to audiences. The opposite of 9
123456789012345678901234567890121234567890123456789012345678 9
123456789012345678901234567890121234567890123456789012345678
accessible is inaccessible. (D) should
23456789012345678901234567890121234567890123456789012345678
look good to you right away. 9
9
1
123456789012345678901234567890121234567890123456789012345678
Don’t get confused by (C). The emphasis in the sentence is on 9
123456789012345678901234567890121234567890123456789012345678 9
123456789012345678901234567890121234567890123456789012345678
audiences, not musicians. 9
123456789012345678901234567890121234567890123456789012345678 9
1 13. C You can eliminate (B) because it is too specific. You can eliminate (D) 9
23456789012345678901234567890121234567890123456789012345678
123456789012345678901234567890121234567890123456789012345678 9
123456789012345678901234567890121234567890123456789012345678
because the author never talks about eating or brushing teeth or other 9
123456789012345678901234567890121234567890123456789012345678 9
123456789012345678901234567890121234567890123456789012345678
daily activities. (E) is also too specific. (A) is really off in emphasis; the
23456789012345678901234567890121234567890123456789012345678
9
9
1
123456789012345678901234567890121234567890123456789012345678
author focuses on describing the Earth’s appearance more than the 9
123456789012345678901234567890121234567890123456789012345678 9
123456789012345678901234567890121234567890123456789012345678
trajectory of the Space Station. The answer is (C). It’s also a good 9
123456789012345678901234567890121234567890123456789012345678
23456789012345678901234567890121234567890123456789012345678
9
9
1 answer because it captures a little bit of the tone of the piece; the author
123456789012345678901234567890121234567890123456789012345678 9
123456789012345678901234567890121234567890123456789012345678
seems to write the passage for an average person experiencing curiosity 9
123456789012345678901234567890121234567890123456789012345678 9
123456789012345678901234567890121234567890123456789012345678
about seeing the Earth from space. 9
123456789012345678901234567890121234567890123456789012345678 99
23456789012345678901234567890121234567890123456789012345678
123456789012345678901234567890121234567890123456789012345678
123456789012345678901234567890121234567890123456789012345678
14. E The second half of the paragraph contrasts seeing the whole Earth to 9
123456789012345678901234567890121234567890123456789012345678 9
123456789012345678901234567890121234567890123456789012345678 99
seeing just a limited part of it. But the emphasis is on seeing the whole
123456789012345678901234567890121234567890123456789012345678 9
1 Earth, because we’ve all seen limited parts of it. The answer is (E).
123456789012345678901234567890121234567890123456789012345678 9
123456789012345678901234567890121234567890123456789012345678
B Here you have to be careful because two different window views are 9
123456789012345678901234567890121234567890123456789012345678
15. 9
23456789012345678901234567890121234567890123456789012345678
123456789012345678901234567890121234567890123456789012345678
discussed in the passage. The downward-facing window is discussed in 9
123456789012345678901234567890121234567890123456789012345678 9
123456789012345678901234567890121234567890123456789012345678
the second half of the second paragraph. And there the author 9
123456789012345678901234567890121234567890123456789012345678 99
123456789012345678901234567890121234567890123456789012345678
compares the view to looking at a big blue beach ball up-close, choice
123456789012345678901234567890121234567890123456789012345678 9
123456789012345678901234567890121234567890123456789012345678
(B). 9
123456789012345678901234567890121234567890123456789012345678 99
123456789012345678901234567890121234567890123456789012345678
1 16. B This is where that second window view comes in. The author first 9
123456789012345678901234567890121234567890123456789012345678 9
123456789012345678901234567890121234567890123456789012345678
discusses the downward-facing window and then contrasts it with 9
123456789012345678901234567890121234567890123456789012345678 9
123456789012345678901234567890121234567890123456789012345678 9
looking through a sideward-facing window in the third paragraph. (B)
123456789012345678901234567890121234567890123456789012345678 9
123456789012345678901234567890121234567890123456789012345678
is the answer. 9
123456789012345678901234567890121234567890123456789012345678 9
123456789012345678901234567890121234567890123456789012345678 9
123456789012345678901234567890121234567890123456789012345678
17. B The passage discusses the “faint glow” in the last sentence of the third 9
123456789012345678901234567890121234567890123456789012345678 9
123456789012345678901234567890121234567890123456789012345678 99
23456789012345678901234567890121234567890123456789012345678
paragraph. There, he attributes it to the outer rim of the atmosphere.
123456789012345678901234567890121234567890123456789012345678
(B) says the same thing. (A) mentions the atmosphere, but it designates 9
1
123456789012345678901234567890121234567890123456789012345678
the wrong part of the atmosphere. 9
123456789012345678901234567890121234567890123456789012345678 9
23456789012345678901234567890121234567890123456789012345678
123456789012345678901234567890121234567890123456789012345678 9
123456789012345678901234567890121234567890123456789012345678
18. B Before the thought exercise is given the author states that, “a good way 9
123456789012345678901234567890121234567890123456789012345678 9
123456789012345678901234567890121234567890123456789012345678
to imagine our view is to stand up and look down at your feet.” In 9
1 9
123456789012345678901234567890121234567890123456789012345678 99
23456789012345678901234567890121234567890123456789012345678
“imagining the view” through the thought exercise the reader gets a
123456789012345678901234567890121234567890123456789012345678 9
123456789012345678901234567890121234567890123456789012345678
definite idea of the proportions of things within the space stations view
9
123456789012345678901234567890121234567890123456789012345678 9
1 (e.g. where San Francisco is in relation to Denver). Choice (B) says as
23456789012345678901234567890121234567890123456789012345678
123456789012345678901234567890121234567890123456789012345678 9
123456789012345678901234567890121234567890123456789012345678
much, and so is the best answer. Choice (C) is a tempting answer, 9
123456789012345678901234567890121234567890123456789012345678 9
123456789012345678901234567890121234567890123456789012345678 99
because it is the idea with which the paragraph begins, but it ends up
1
123456789012345678901234567890121234567890123456789012345678
being too narrow a description of the thought exercise. 9
123456789012345678901234567890121234567890123456789012345678 9
123456789012345678901234567890121234567890123456789012345678 9
123456789012345678901234567890121234567890123456789012345678 9
123456789012345678901234567890121234567890123456789012345678 9
123456789012345678901234567890121234567890123456789012345678 9
123456789012345678901234567890121234567890123456789012345678 9
123456789012345678901234567890121234567890123456789012345678 9
123456789012345678901234567890121234567890123456789012345678 9
1234567890123456789012345678901212345678901234567890123456789
Copyright © 2005 Thomson Peterson’s, a part of The Thomson Corporaton 49
SAT is a registered trademark of the College Entrance Examination Board, which
was not involved in the production of and does not endorse this product.
1234567890123456789012345678901212345678901234567890123456789
123456789012345678901234567890121234567890123456789012345678 9
123456789012345678901234567890121234567890123456789012345678 9
123456789012345678901234567890121234567890123456789012345678 9
123456789012345678901234567890121234567890123456789012345678 9
123456789012345678901234567890121234567890123456789012345678
19. D You may be tempted to answer (A), because the passage can be 9
123456789012345678901234567890121234567890123456789012345678 9
123456789012345678901234567890121234567890123456789012345678
confusing. But it’s not dry or scientific,
23456789012345678901234567890121234567890123456789012345678
so “technical” just isn’t a good 9
9
1
123456789012345678901234567890121234567890123456789012345678
word for it. Choice (B) doesn’t capture it’s informality. Choice (E) is too 9
123456789012345678901234567890121234567890123456789012345678 9
123456789012345678901234567890121234567890123456789012345678
extreme—as far as poetic descriptions of the planet go, this one doesn’t 9
123456789012345678901234567890121234567890123456789012345678
23456789012345678901234567890121234567890123456789012345678
9
9
1 even rate. That leaves (C) and (D). Choice (C) isn’t quite right, either,
123456789012345678901234567890121234567890123456789012345678 9
123456789012345678901234567890121234567890123456789012345678
because it’s not irreverent, it’s just casual. Choice (D) best captures that 9
123456789012345678901234567890121234567890123456789012345678 9
123456789012345678901234567890121234567890123456789012345678
sense.
23456789012345678901234567890121234567890123456789012345678
9
9
1
123456789012345678901234567890121234567890123456789012345678
20. D You don’t know a lot about what the astronauts do from the passage, 9
123456789012345678901234567890121234567890123456789012345678 9
123456789012345678901234567890121234567890123456789012345678
so you’ll have to dig this answer up. Choice (A) seems silly, and would 9
123456789012345678901234567890121234567890123456789012345678
23456789012345678901234567890121234567890123456789012345678
9
9
1 really be a disrespectful thing to say about the author. The SAT won’t
123456789012345678901234567890121234567890123456789012345678 9
123456789012345678901234567890121234567890123456789012345678
do that. So eliminate (A). There’s no reference to gravity in the passage, 9
123456789012345678901234567890121234567890123456789012345678 9
123456789012345678901234567890121234567890123456789012345678
so eliminate (B). There’s no real reference to physical activity, so (C) 9
23456789012345678901234567890121234567890123456789012345678
123456789012345678901234567890121234567890123456789012345678 9
123456789012345678901234567890121234567890123456789012345678
seems wrong. You are left with (D) and (E). The astronaut uses a lot of 9
123456789012345678901234567890121234567890123456789012345678 9
123456789012345678901234567890121234567890123456789012345678 9
numbers, and throws them around as though they were quite easy. (D)
123456789012345678901234567890121234567890123456789012345678 9
123456789012345678901234567890121234567890123456789012345678
seems like a reasonable answer. There’s no reference in the passage to 9
1 9
123456789012345678901234567890121234567890123456789012345678
communication procedures, which means you can eliminate (E) even 9
123456789012345678901234567890121234567890123456789012345678 9
123456789012345678901234567890121234567890123456789012345678
though it’s sort of a tempting answer. 9
123456789012345678901234567890121234567890123456789012345678 9
23456789012345678901234567890121234567890123456789012345678
123456789012345678901234567890121234567890123456789012345678 9
123456789012345678901234567890121234567890123456789012345678 9
123456789012345678901234567890121234567890123456789012345678
Section 2 9
123456789012345678901234567890121234567890123456789012345678 9
123456789012345678901234567890121234567890123456789012345678 9
123456789012345678901234567890121234567890123456789012345678
1. C This is the first question, so it should be one of the easiest, if not the 9
123456789012345678901234567890121234567890123456789012345678 9
123456789012345678901234567890121234567890123456789012345678
easiest, questions in the section. For this reason, you don’t have to 9
1 9
23456789012345678901234567890121234567890123456789012345678
123456789012345678901234567890121234567890123456789012345678
expect anything too tricky about this problem. Substitute x 5 2y into 9
123456789012345678901234567890121234567890123456789012345678 9
1 the first equation and solve for x: 9
123456789012345678901234567890121234567890123456789012345678 9
123456789012345678901234567890121234567890123456789012345678 9
123456789012345678901234567890121234567890123456789012345678
x 1 4y 5 3 9
123456789012345678901234567890121234567890123456789012345678 9
123456789012345678901234567890121234567890123456789012345678 9
123456789012345678901234567890121234567890123456789012345678
2y 1 4y 5 3 9
123456789012345678901234567890121234567890123456789012345678 9
123456789012345678901234567890121234567890123456789012345678 9
23456789012345678901234567890121234567890123456789012345678
3y 5 3
123456789012345678901234567890121234567890123456789012345678 9
1 3y 3 9
123456789012345678901234567890121234567890123456789012345678
5 9
123456789012345678901234567890121234567890123456789012345678
3 3 9
123456789012345678901234567890121234567890123456789012345678 9
23456789012345678901234567890121234567890123456789012345678
9
123456789012345678901234567890121234567890123456789012345678 9
123456789012345678901234567890121234567890123456789012345678
y51
123456789012345678901234567890121234567890123456789012345678 9 9
1 (C) is the answer.
123456789012345678901234567890121234567890123456789012345678 9
123456789012345678901234567890121234567890123456789012345678 9
123456789012345678901234567890121234567890123456789012345678
2. B There are two main parts to this problem: 9
123456789012345678901234567890121234567890123456789012345678 9
123456789012345678901234567890121234567890123456789012345678 9
123456789012345678901234567890121234567890123456789012345678 9
23456789012345678901234567890121234567890123456789012345678
1. the use of the variable d, and
123456789012345678901234567890121234567890123456789012345678 9 9
123456789012345678901234567890121234567890123456789012345678 9
123456789012345678901234567890121234567890123456789012345678
2. the change of units from hours to minutes.
9
1
23456789012345678901234567890121234567890123456789012345678
123456789012345678901234567890121234567890123456789012345678
Start with the variable since d is half as much as 2d, it should take him 9
1 9
123456789012345678901234567890121234567890123456789012345678
half the amount of time, or 1.5 hours. One hour is 60 minutes, and half 9
123456789012345678901234567890121234567890123456789012345678 9
123456789012345678901234567890121234567890123456789012345678
an hour is 30 minutes, so 1.5 hours is 90 minutes. That’s choice (B). 9
123456789012345678901234567890121234567890123456789012345678 9
123456789012345678901234567890121234567890123456789012345678 9
123456789012345678901234567890121234567890123456789012345678 9
123456789012345678901234567890121234567890123456789012345678 9
1234567890123456789012345678901212345678901234567890123456789
50 Copyright © 2005 Thomson Peterson’s, a part of The Thomson Corporaton
SAT is a registered trademark of the College Entrance Examination Board, which
was not involved in the production of and does not endorse this product.
1234567890123456789012345678901212345678901234567890123456789
123456789012345678901234567890121234567890123456789012345678 9
123456789012345678901234567890121234567890123456789012345678 9
123456789012345678901234567890121234567890123456789012345678 9
123456789012345678901234567890121234567890123456789012345678 9
123456789012345678901234567890121234567890123456789012345678
3. A There is more than one way to solve this problem, but all paths involve 9
123456789012345678901234567890121234567890123456789012345678 9
123456789012345678901234567890121234567890123456789012345678
manipulating the equation. You can solve
23456789012345678901234567890121234567890123456789012345678
for f in the first equation, and 9
9
1
123456789012345678901234567890121234567890123456789012345678
then plug that value into the second equation. You could also just leave 9
123456789012345678901234567890121234567890123456789012345678 9
123456789012345678901234567890121234567890123456789012345678
the 3f as is, and monkey with the formula this way: 9
123456789012345678901234567890121234567890123456789012345678
23456789012345678901234567890121234567890123456789012345678
9
9
1
123456789012345678901234567890121234567890123456789012345678
3f 1 15 5 27 9
123456789012345678901234567890121234567890123456789012345678 9
123456789012345678901234567890121234567890123456789012345678
3f 1 15 2 15 5 27 2 15 9
123456789012345678901234567890121234567890123456789012345678
23456789012345678901234567890121234567890123456789012345678
9
9
1 3f 5 12
123456789012345678901234567890121234567890123456789012345678 9
123456789012345678901234567890121234567890123456789012345678 9
123456789012345678901234567890121234567890123456789012345678
3f 2 6 5 12 2 6 9
123456789012345678901234567890121234567890123456789012345678
23456789012345678901234567890121234567890123456789012345678
9
9
1 3f 2 6 5 6
123456789012345678901234567890121234567890123456789012345678 9
123456789012345678901234567890121234567890123456789012345678 9
123456789012345678901234567890121234567890123456789012345678
Whichever path you take, if you manipulate the equation correctly 9
123456789012345678901234567890121234567890123456789012345678 9
123456789012345678901234567890121234567890123456789012345678 99
23456789012345678901234567890121234567890123456789012345678
you’ll get choice (A) as your answer.
123456789012345678901234567890121234567890123456789012345678 9
123456789012345678901234567890121234567890123456789012345678
4. A The first thing to notice is that this triangle is a 30-60-90 triangle, one
123456789012345678901234567890121234567890123456789012345678 9
123456789012345678901234567890121234567890123456789012345678
of the SAT’s favorite triangles. Since you can read off the relationships 9
123456789012345678901234567890121234567890123456789012345678 99
1 between different sides of a 30-60-90 triangle, it can be readily seen that
123456789012345678901234567890121234567890123456789012345678 9
123456789012345678901234567890121234567890123456789012345678
the x 1 y side is half the length of the hypotenuse (which is 14). Half of 9
123456789012345678901234567890121234567890123456789012345678 9
123456789012345678901234567890121234567890123456789012345678 99
23456789012345678901234567890121234567890123456789012345678
14 is 7, so (A) is the answer.
123456789012345678901234567890121234567890123456789012345678 9
123456789012345678901234567890121234567890123456789012345678
123456789012345678901234567890121234567890123456789012345678
5. D Translating the English into algebra is the key to all word problems. 9
123456789012345678901234567890121234567890123456789012345678 99
123456789012345678901234567890121234567890123456789012345678
123456789012345678901234567890121234567890123456789012345678
Since the snack costs twenty cents less than the drink, you can write 9
123456789012345678901234567890121234567890123456789012345678 9
123456789012345678901234567890121234567890123456789012345678 99
down d 2 20 5 s. Since a snack and drink together costs $1.30, you
1
123456789012345678901234567890121234567890123456789012345678
also know that s 1 d 5 130. You have two equations and two variables. 9
123456789012345678901234567890121234567890123456789012345678 9
123456789012345678901234567890121234567890123456789012345678 9
Substitute the first equation into the second and then solve:
123456789012345678901234567890121234567890123456789012345678 9
123456789012345678901234567890121234567890123456789012345678 9
123456789012345678901234567890121234567890123456789012345678
s 1 d 5 130 9
123456789012345678901234567890121234567890123456789012345678 9
123456789012345678901234567890121234567890123456789012345678
~d 2 20! 1 d 5 130 9
123456789012345678901234567890121234567890123456789012345678 9
123456789012345678901234567890121234567890123456789012345678
2d 2 20 5 130 9
123456789012345678901234567890121234567890123456789012345678 99
23456789012345678901234567890121234567890123456789012345678
123456789012345678901234567890121234567890123456789012345678 9
1 2d 2 20 1 20 5 130 1 20
123456789012345678901234567890121234567890123456789012345678 9
123456789012345678901234567890121234567890123456789012345678 9
2d 5 150
23456789012345678901234567890121234567890123456789012345678
123456789012345678901234567890121234567890123456789012345678 9
123456789012345678901234567890121234567890123456789012345678
2d 150 9
123456789012345678901234567890121234567890123456789012345678
5 9
123456789012345678901234567890121234567890123456789012345678
2 2 9
1 9
23456789012345678901234567890121234567890123456789012345678
123456789012345678901234567890121234567890123456789012345678 9
123456789012345678901234567890121234567890123456789012345678
d 5 75 9
123456789012345678901234567890121234567890123456789012345678 99
123456789012345678901234567890121234567890123456789012345678
A drink costs 75 cents ($0.75), so (D) is the answer.
9
1
123456789012345678901234567890121234567890123456789012345678 99
23456789012345678901234567890121234567890123456789012345678
123456789012345678901234567890121234567890123456789012345678 9
123456789012345678901234567890121234567890123456789012345678
For problems dealing with units of money, always decide whether you
9
123456789012345678901234567890121234567890123456789012345678 9
1
Tip

want to work in dollars or cents. If the amount of money is great, using


23456789012345678901234567890121234567890123456789012345678
123456789012345678901234567890121234567890123456789012345678 9
1 dollars is typically the best way to go. On this problem, converting to 9
123456789012345678901234567890121234567890123456789012345678 9
123456789012345678901234567890121234567890123456789012345678
cents might work better since you won’t have to deal with any decimals. 9
123456789012345678901234567890121234567890123456789012345678
23456789012345678901234567890121234567890123456789012345678
9
9
1
123456789012345678901234567890121234567890123456789012345678 9
123456789012345678901234567890121234567890123456789012345678 9
123456789012345678901234567890121234567890123456789012345678 9
1234567890123456789012345678901212345678901234567890123456789
Copyright © 2005 Thomson Peterson’s, a part of The Thomson Corporaton 51
SAT is a registered trademark of the College Entrance Examination Board, which
was not involved in the production of and does not endorse this product.
1234567890123456789012345678901212345678901234567890123456789
123456789012345678901234567890121234567890123456789012345678 9
123456789012345678901234567890121234567890123456789012345678 9
123456789012345678901234567890121234567890123456789012345678 9
123456789012345678901234567890121234567890123456789012345678 9
123456789012345678901234567890121234567890123456789012345678
6. D This problem looks really complicated, but don’t let that ruffle your test 9
123456789012345678901234567890121234567890123456789012345678 9
123456789012345678901234567890121234567890123456789012345678
feathers. With function problems, just
23456789012345678901234567890121234567890123456789012345678
plug in whatever the problem 9
9
1
123456789012345678901234567890121234567890123456789012345678
tells you to plug into the equation. Be a machine! Here, you are 9
123456789012345678901234567890121234567890123456789012345678 9
123456789012345678901234567890121234567890123456789012345678
considering when x 5 2, so f(2x) is the same thing as f(4). (Of course, 9
123456789012345678901234567890121234567890123456789012345678
23456789012345678901234567890121234567890123456789012345678
9
9
1 the question could have just said x 5 4, but that would have been too
123456789012345678901234567890121234567890123456789012345678 9
123456789012345678901234567890121234567890123456789012345678
easy). 9
123456789012345678901234567890121234567890123456789012345678 9
123456789012345678901234567890121234567890123456789012345678
Now just replace every x in the problem with a 4:
23456789012345678901234567890121234567890123456789012345678
9
9
1
123456789012345678901234567890121234567890123456789012345678 9
123456789012345678901234567890121234567890123456789012345678 9
123456789012345678901234567890121234567890123456789012345678 9
x
f(x) 5 x 1 x
123456789012345678901234567890121234567890123456789012345678
23456789012345678901234567890121234567890123456789012345678
9
9
1 f ~ 4! 5 4 1 4 4
123456789012345678901234567890121234567890123456789012345678 9
123456789012345678901234567890121234567890123456789012345678
f ~4! 5 4 1 256 9
123456789012345678901234567890121234567890123456789012345678 9
123456789012345678901234567890121234567890123456789012345678
~ ! 9
123456789012345678901234567890121234567890123456789012345678 9
23456789012345678901234567890121234567890123456789012345678
f 4 5 260
123456789012345678901234567890121234567890123456789012345678 9
123456789012345678901234567890121234567890123456789012345678
Your answer is (D). 9
123456789012345678901234567890121234567890123456789012345678 9
123456789012345678901234567890121234567890123456789012345678 9
123456789012345678901234567890121234567890123456789012345678 9
7. D This one looks strange, and it is as straightforward to solve as it
9
1
123456789012345678901234567890121234567890123456789012345678
= 2 9
123456789012345678901234567890121234567890123456789012345678
appears. If you only consider x to be x, then you only get the 9
123456789012345678901234567890121234567890123456789012345678
extraneous solution, and not the correct one. You start by squaring 9
123456789012345678901234567890121234567890123456789012345678 9
23456789012345678901234567890121234567890123456789012345678
123456789012345678901234567890121234567890123456789012345678
both sides and then factoring. 9
123456789012345678901234567890121234567890123456789012345678 9
123456789012345678901234567890121234567890123456789012345678 9
123456789012345678901234567890121234567890123456789012345678
2 2 9
123456789012345678901234567890121234567890123456789012345678
x 5 4x 1 12x 1 9 9
123456789012345678901234567890121234567890123456789012345678 9
123456789012345678901234567890121234567890123456789012345678 9
2
0 5 3x 1 12x 19
123456789012345678901234567890121234567890123456789012345678 9
1 0 5 3~x2 1 4x 1 3! 9
23456789012345678901234567890121234567890123456789012345678
123456789012345678901234567890121234567890123456789012345678 9
123456789012345678901234567890121234567890123456789012345678
2
0 5 x 1 4x 1 3 9
1 9
123456789012345678901234567890121234567890123456789012345678 9
123456789012345678901234567890121234567890123456789012345678
0 5 ~ x 1 3 !~ x 1 1 !
23456789012345678901234567890121234567890123456789012345678
9
9
123456789012345678901234567890121234567890123456789012345678
123456789012345678901234567890121234567890123456789012345678
x 5 23 or x 5 21 9
1 9
123456789012345678901234567890121234567890123456789012345678 9
123456789012345678901234567890121234567890123456789012345678
Checking both solutions in the original equation, you see that only x 5 9
23456789012345678901234567890121234567890123456789012345678
123456789012345678901234567890121234567890123456789012345678 9
123456789012345678901234567890121234567890123456789012345678
21 works. This is choice (D). 9
1 9
123456789012345678901234567890121234567890123456789012345678
E If |x 1| |y|, what do you know about x and y? Not much actually. 9
123456789012345678901234567890121234567890123456789012345678 9
8. 1 .
23456789012345678901234567890121234567890123456789012345678
123456789012345678901234567890121234567890123456789012345678
x could be 210 and y could be 5, and the inequality would be true. But 9
123456789012345678901234567890121234567890123456789012345678 9
123456789012345678901234567890121234567890123456789012345678
x could also be 10,000 and y could be 9,998, and the inequality would 9
123456789012345678901234567890121234567890123456789012345678 9 9
1 still be true. This means you cannot nail down the relationship between
123456789012345678901234567890121234567890123456789012345678 9
123456789012345678901234567890121234567890123456789012345678
x and y, so (E) is the answer. 9
123456789012345678901234567890121234567890123456789012345678 9
123456789012345678901234567890121234567890123456789012345678 9
123456789012345678901234567890121234567890123456789012345678 9
123456789012345678901234567890121234567890123456789012345678 9
23456789012345678901234567890121234567890123456789012345678
123456789012345678901234567890121234567890123456789012345678 9
123456789012345678901234567890121234567890123456789012345678 9
123456789012345678901234567890121234567890123456789012345678 9 9
1
123456789012345678901234567890121234567890123456789012345678 9
123456789012345678901234567890121234567890123456789012345678 9
123456789012345678901234567890121234567890123456789012345678 9
123456789012345678901234567890121234567890123456789012345678 9
123456789012345678901234567890121234567890123456789012345678 9
123456789012345678901234567890121234567890123456789012345678 9
123456789012345678901234567890121234567890123456789012345678 9
123456789012345678901234567890121234567890123456789012345678 9
123456789012345678901234567890121234567890123456789012345678 9
1234567890123456789012345678901212345678901234567890123456789
52 Copyright © 2005 Thomson Peterson’s, a part of The Thomson Corporaton
SAT is a registered trademark of the College Entrance Examination Board, which
was not involved in the production of and does not endorse this product.
1234567890123456789012345678901212345678901234567890123456789
123456789012345678901234567890121234567890123456789012345678 9
123456789012345678901234567890121234567890123456789012345678 9
123456789012345678901234567890121234567890123456789012345678 9
123456789012345678901234567890121234567890123456789012345678 9
123456789012345678901234567890121234567890123456789012345678
9. A This question is almost all Math Speak, but hopefully you are getting 9
123456789012345678901234567890121234567890123456789012345678 9
123456789012345678901234567890121234567890123456789012345678
better at this language. The whole thing
23456789012345678901234567890121234567890123456789012345678
sets up an equation. “Percent” 9
9
1
123456789012345678901234567890121234567890123456789012345678
means “divide by 100,” and “is the same value” works the same as an 9
123456789012345678901234567890121234567890123456789012345678 9
123456789012345678901234567890121234567890123456789012345678
equal sign. “What number” is Math Speak for “place a variable here.” 9
123456789012345678901234567890121234567890123456789012345678
23456789012345678901234567890121234567890123456789012345678
9
9
1 Here’s the translation:
123456789012345678901234567890121234567890123456789012345678 9
123456789012345678901234567890121234567890123456789012345678
“75 percent of 104 is the same value as 60 percent of what number” 9
123456789012345678901234567890121234567890123456789012345678 9
123456789012345678901234567890121234567890123456789012345678
23456789012345678901234567890121234567890123456789012345678
9
9
1 75
S D
~ !
60
S D
123456789012345678901234567890121234567890123456789012345678
123456789012345678901234567890121234567890123456789012345678
100
104 5 n
123456789012345678901234567890121234567890123456789012345678
100
9
9
9
123456789012345678901234567890121234567890123456789012345678
23456789012345678901234567890121234567890123456789012345678
9
9
1 ~ 0.75 !~ 104 ! 5 0.6n
123456789012345678901234567890121234567890123456789012345678 9
123456789012345678901234567890121234567890123456789012345678
78 5 0.6n 9
123456789012345678901234567890121234567890123456789012345678 9
123456789012345678901234567890121234567890123456789012345678 9
23456789012345678901234567890121234567890123456789012345678
123456789012345678901234567890121234567890123456789012345678
78 0.6n 9
123456789012345678901234567890121234567890123456789012345678
5 9
123456789012345678901234567890121234567890123456789012345678 99
0.6 0.6
123456789012345678901234567890121234567890123456789012345678
123456789012345678901234567890121234567890123456789012345678
130 5 n 9
123456789012345678901234567890121234567890123456789012345678 99
1
123456789012345678901234567890121234567890123456789012345678
Choice (A) is the answer. 9
123456789012345678901234567890121234567890123456789012345678 9
123456789012345678901234567890121234567890123456789012345678 9
123456789012345678901234567890121234567890123456789012345678 99
23456789012345678901234567890121234567890123456789012345678
123456789012345678901234567890121234567890123456789012345678
Note

The percentages were rewritten as decimals instead of fractions to


123456789012345678901234567890121234567890123456789012345678 9
123456789012345678901234567890121234567890123456789012345678
make it easier to use your calculator. Dividing 78 by 0.6 is not 9
123456789012345678901234567890121234567890123456789012345678 9
123456789012345678901234567890121234567890123456789012345678 99
something most people can do easily, but for a calculator, it’s a cinch.
123456789012345678901234567890121234567890123456789012345678 9
123456789012345678901234567890121234567890123456789012345678 9
123456789012345678901234567890121234567890123456789012345678 9
1 10. D Equations of lines can be put into the y 5 mx 1 b form, and then the
123456789012345678901234567890121234567890123456789012345678 9
123456789012345678901234567890121234567890123456789012345678 9
123456789012345678901234567890121234567890123456789012345678 9
y-intercept, b, can be read off.
123456789012345678901234567890121234567890123456789012345678 9
123456789012345678901234567890121234567890123456789012345678 9
123456789012345678901234567890121234567890123456789012345678
3y 2 x 5 12 9
123456789012345678901234567890121234567890123456789012345678 9
123456789012345678901234567890121234567890123456789012345678
3y 2 x 1 x 5 12 1 x 9
123456789012345678901234567890121234567890123456789012345678 9
123456789012345678901234567890121234567890123456789012345678
3y 5 x 1 12 9
123456789012345678901234567890121234567890123456789012345678 99
23456789012345678901234567890121234567890123456789012345678
123456789012345678901234567890121234567890123456789012345678 9
1 3y x 12
123456789012345678901234567890121234567890123456789012345678
5 1 9
123456789012345678901234567890121234567890123456789012345678 9
3 3 3
23456789012345678901234567890121234567890123456789012345678
123456789012345678901234567890121234567890123456789012345678 9
123456789012345678901234567890121234567890123456789012345678
x 9
123456789012345678901234567890121234567890123456789012345678
y5 14 9
123456789012345678901234567890121234567890123456789012345678
3 9
1 9
23456789012345678901234567890121234567890123456789012345678
123456789012345678901234567890121234567890123456789012345678 9
123456789012345678901234567890121234567890123456789012345678
If 4 is the y-intercept, and twice this number is 8, choice (D). 9
123456789012345678901234567890121234567890123456789012345678 99
123456789012345678901234567890121234567890123456789012345678 9
1
123456789012345678901234567890121234567890123456789012345678 9
123456789012345678901234567890121234567890123456789012345678 9
123456789012345678901234567890121234567890123456789012345678 9
123456789012345678901234567890121234567890123456789012345678 9
123456789012345678901234567890121234567890123456789012345678 9
123456789012345678901234567890121234567890123456789012345678 99
23456789012345678901234567890121234567890123456789012345678
1
123456789012345678901234567890121234567890123456789012345678 9
123456789012345678901234567890121234567890123456789012345678 9
123456789012345678901234567890121234567890123456789012345678 9
123456789012345678901234567890121234567890123456789012345678 9
123456789012345678901234567890121234567890123456789012345678 9
123456789012345678901234567890121234567890123456789012345678 9
123456789012345678901234567890121234567890123456789012345678 9
1234567890123456789012345678901212345678901234567890123456789
Copyright © 2005 Thomson Peterson’s, a part of The Thomson Corporaton 53
SAT is a registered trademark of the College Entrance Examination Board, which
was not involved in the production of and does not endorse this product.
1234567890123456789012345678901212345678901234567890123456789
123456789012345678901234567890121234567890123456789012345678 9
123456789012345678901234567890121234567890123456789012345678 9
123456789012345678901234567890121234567890123456789012345678 9
123456789012345678901234567890121234567890123456789012345678 9
123456789012345678901234567890121234567890123456789012345678
11. C 9
123456789012345678901234567890121234567890123456789012345678 9
123456789012345678901234567890121234567890123456789012345678 9
123456789012345678901234567890121234567890123456789012345678 9
123456789012345678901234567890121234567890123456789012345678 9
123456789012345678901234567890121234567890123456789012345678 9
123456789012345678901234567890121234567890123456789012345678 9
123456789012345678901234567890121234567890123456789012345678
23456789012345678901234567890121234567890123456789012345678
9
9
1
123456789012345678901234567890121234567890123456789012345678 9
123456789012345678901234567890121234567890123456789012345678 9
123456789012345678901234567890121234567890123456789012345678
As you can see from this figure a 5 a, and b 5 b because alternate 9
123456789012345678901234567890121234567890123456789012345678
23456789012345678901234567890121234567890123456789012345678
9
9
1 interior angles are congruent. If two angles of a triangle are congruent
123456789012345678901234567890121234567890123456789012345678 9
123456789012345678901234567890121234567890123456789012345678
to two angles of another triangle, what can you deduce about the 9
123456789012345678901234567890121234567890123456789012345678 9
123456789012345678901234567890121234567890123456789012345678
relationship between the third angles? The third angles must also be
23456789012345678901234567890121234567890123456789012345678
9
9
1
123456789012345678901234567890121234567890123456789012345678
congruent. If you don’t see this, pick values for a and b and remember 9
123456789012345678901234567890121234567890123456789012345678 9
123456789012345678901234567890121234567890123456789012345678
that the sum of the mesures of the interior angles of a triangle is 180. 9
123456789012345678901234567890121234567890123456789012345678 9
123456789012345678901234567890121234567890123456789012345678 9
23456789012345678901234567890121234567890123456789012345678
Choice (C) is the answer.
123456789012345678901234567890121234567890123456789012345678 9
123456789012345678901234567890121234567890123456789012345678 9
123456789012345678901234567890121234567890123456789012345678
12. D These next three problems test your ability to deal with unfamiliar 9
123456789012345678901234567890121234567890123456789012345678 9
123456789012345678901234567890121234567890123456789012345678 9
symbols. Recall that you will be told everything that you need to know
9
1
123456789012345678901234567890121234567890123456789012345678
about the new symbol. Carefully read the instructions and don’t get 9
123456789012345678901234567890121234567890123456789012345678 9
123456789012345678901234567890121234567890123456789012345678
rattled; everything you need to know about the new symbol will be 9
23456789012345678901234567890121234567890123456789012345678
123456789012345678901234567890121234567890123456789012345678 9
123456789012345678901234567890121234567890123456789012345678
handed to you on a platter. 9
123456789012345678901234567890121234567890123456789012345678 9
123456789012345678901234567890121234567890123456789012345678
In the subtraction problem, convert each term one at a time: 9
123456789012345678901234567890121234567890123456789012345678 9
123456789012345678901234567890121234567890123456789012345678 9
123456789012345678901234567890121234567890123456789012345678
{ 2323 5 3322 and { 2321 5 1322 9
123456789012345678901234567890121234567890123456789012345678 9
123456789012345678901234567890121234567890123456789012345678 9
1 (It’s helpful if you write out the middle unchanging middle digits first, 9
23456789012345678901234567890121234567890123456789012345678
123456789012345678901234567890121234567890123456789012345678 9
123456789012345678901234567890121234567890123456789012345678
and then write the first and last digits.) 9
1 9
123456789012345678901234567890121234567890123456789012345678 9
123456789012345678901234567890121234567890123456789012345678
So { 2323 2 { 2321 5 3322 2 1322 5 2000, choice (D). 9
123456789012345678901234567890121234567890123456789012345678 9
23456789012345678901234567890121234567890123456789012345678
123456789012345678901234567890121234567890123456789012345678 9
1 13. A You know that A is a two-digit number between 10 and 20, which 9
123456789012345678901234567890121234567890123456789012345678
narrows the field of possible answers to 11, 12, 13, 14, 15, 16, 17, 18, 9
123456789012345678901234567890121234567890123456789012345678 9
23456789012345678901234567890121234567890123456789012345678
123456789012345678901234567890121234567890123456789012345678
and 19. This may seem like a lot, but the second part of the problem 9
123456789012345678901234567890121234567890123456789012345678 9
1 2 2
will narrow things down. The equation ({ A) 5 {(A )looks confusing, 9
123456789012345678901234567890121234567890123456789012345678 9
123456789012345678901234567890121234567890123456789012345678 9
but the key word is “equation.” The two values are equal, even though
23456789012345678901234567890121234567890123456789012345678
123456789012345678901234567890121234567890123456789012345678 9
123456789012345678901234567890121234567890123456789012345678
you’ve flipped the first and last digits. Flipping 19 makes 91, which is 9
123456789012345678901234567890121234567890123456789012345678 9
123456789012345678901234567890121234567890123456789012345678 9
quite a difference, and it’s highly unlikely that 192
5 91 2
.
9
1
123456789012345678901234567890121234567890123456789012345678 9
123456789012345678901234567890121234567890123456789012345678
At this point, you might suspect that 11 was the answer because 9
123456789012345678901234567890121234567890123456789012345678 9
123456789012345678901234567890121234567890123456789012345678
reversing the digits does not change the value of the number. That is a 9
123456789012345678901234567890121234567890123456789012345678 9
123456789012345678901234567890121234567890123456789012345678 9
23456789012345678901234567890121234567890123456789012345678
good suspicion, and if you see it, you can easily read off that (A) is the
9
123456789012345678901234567890121234567890123456789012345678 9
123456789012345678901234567890121234567890123456789012345678
answer. If you did not have that suspicion, just dive into the problem
9
123456789012345678901234567890121234567890123456789012345678 9
1 trying different choices.
123456789012345678901234567890121234567890123456789012345678 9
23456789012345678901234567890121234567890123456789012345678
9
1
123456789012345678901234567890121234567890123456789012345678 9
123456789012345678901234567890121234567890123456789012345678 9
123456789012345678901234567890121234567890123456789012345678 9
123456789012345678901234567890121234567890123456789012345678 9
123456789012345678901234567890121234567890123456789012345678 9
123456789012345678901234567890121234567890123456789012345678 9
123456789012345678901234567890121234567890123456789012345678 9
1234567890123456789012345678901212345678901234567890123456789
54 Copyright © 2005 Thomson Peterson’s, a part of The Thomson Corporaton
SAT is a registered trademark of the College Entrance Examination Board, which
was not involved in the production of and does not endorse this product.
1234567890123456789012345678901212345678901234567890123456789
123456789012345678901234567890121234567890123456789012345678 9
123456789012345678901234567890121234567890123456789012345678 9
123456789012345678901234567890121234567890123456789012345678 9
123456789012345678901234567890121234567890123456789012345678 9
123456789012345678901234567890121234567890123456789012345678
14. E Since each variable is a digit and the inequality is true, plug in some 9
123456789012345678901234567890121234567890123456789012345678 9
123456789012345678901234567890121234567890123456789012345678
numbers to try to make the smallest difference
23456789012345678901234567890121234567890123456789012345678
possible: 9
9
1
123456789012345678901234567890121234567890123456789012345678 9
123456789012345678901234567890121234567890123456789012345678
A . B . C . D . E 9
123456789012345678901234567890121234567890123456789012345678
5 4 3 2 1 9
123456789012345678901234567890121234567890123456789012345678
. . . .
23456789012345678901234567890121234567890123456789012345678
9
9
1
123456789012345678901234567890121234567890123456789012345678
Now look at the subtraction problem and plug in the numbers above: 9
123456789012345678901234567890121234567890123456789012345678 9
123456789012345678901234567890121234567890123456789012345678 9
123456789012345678901234567890121234567890123456789012345678
ABCD 2 {(ABCD) 5 5432 2 2435
23456789012345678901234567890121234567890123456789012345678
5 2977 9
9
1
123456789012345678901234567890121234567890123456789012345678 9
123456789012345678901234567890121234567890123456789012345678
The difference is greater than one thousand, so choice (E) is the answer. 9
123456789012345678901234567890121234567890123456789012345678 9
123456789012345678901234567890121234567890123456789012345678
23456789012345678901234567890121234567890123456789012345678
9
9
1
123456789012345678901234567890121234567890123456789012345678
There is a theoretical path that leads to the same answer on this 9
123456789012345678901234567890121234567890123456789012345678 9
Note

123456789012345678901234567890121234567890123456789012345678
problem, but when dealing with weird symbols problems the 9
123456789012345678901234567890121234567890123456789012345678 9
123456789012345678901234567890121234567890123456789012345678 99
23456789012345678901234567890121234567890123456789012345678
theoretical path is usually not the best one to take. As you can see,
123456789012345678901234567890121234567890123456789012345678
generating some numbers and then placing them into the equation
123456789012345678901234567890121234567890123456789012345678 9
123456789012345678901234567890121234567890123456789012345678
works well and didn’t take too long. 9
123456789012345678901234567890121234567890123456789012345678 99
123456789012345678901234567890121234567890123456789012345678 9
1
123456789012345678901234567890121234567890123456789012345678
15. C There’s no answer choice that says, “It cannot be determined,” so you 9
123456789012345678901234567890121234567890123456789012345678 9
123456789012345678901234567890121234567890123456789012345678
have to realize that there is a way to determine the area of the circle. 9
23456789012345678901234567890121234567890123456789012345678
123456789012345678901234567890121234567890123456789012345678 9
123456789012345678901234567890121234567890123456789012345678
Since the area formula for a circle is A 5 pr2 this means there has to be 9
123456789012345678901234567890121234567890123456789012345678 9
123456789012345678901234567890121234567890123456789012345678
a way to find the radius of the circle, which in this case is line 9
123456789012345678901234567890121234567890123456789012345678 9
123456789012345678901234567890121234567890123456789012345678 99
segment RS.
123456789012345678901234567890121234567890123456789012345678 9
123456789012345678901234567890121234567890123456789012345678
123456789012345678901234567890121234567890123456789012345678
The two statements underneath the drawing give you the tools you 9
1 9
123456789012345678901234567890121234567890123456789012345678 99
23456789012345678901234567890121234567890123456789012345678
need. If “line q is tangent to circle R,” then angle RST is a right angle.
123456789012345678901234567890121234567890123456789012345678 9
1 RT
123456789012345678901234567890121234567890123456789012345678
And if RS 5 , then you can determine the value of RS since the 9
123456789012345678901234567890121234567890123456789012345678
3
23456789012345678901234567890121234567890123456789012345678
9
9
123456789012345678901234567890121234567890123456789012345678
problem gives you the length of RT as 18. For line segment RS, the
123456789012345678901234567890121234567890123456789012345678 9
1 18 9
123456789012345678901234567890121234567890123456789012345678
radius will be 6, since 5 6. If you place this value of the radius into 9
123456789012345678901234567890121234567890123456789012345678
3 9
23456789012345678901234567890121234567890123456789012345678
123456789012345678901234567890121234567890123456789012345678 9
123456789012345678901234567890121234567890123456789012345678
the area formula for a circle, you’ll find answer (C) at the end. 9
1 9
123456789012345678901234567890121234567890123456789012345678
2 9
123456789012345678901234567890121234567890123456789012345678
A 5 pr 9
23456789012345678901234567890121234567890123456789012345678
123456789012345678901234567890121234567890123456789012345678
2 9
123456789012345678901234567890121234567890123456789012345678
A 5 p6 9
123456789012345678901234567890121234567890123456789012345678 9
123456789012345678901234567890121234567890123456789012345678 99
A 5 36p
1
123456789012345678901234567890121234567890123456789012345678
16. E All of the answer choices mention August and September total sales, so 9
123456789012345678901234567890121234567890123456789012345678 9
123456789012345678901234567890121234567890123456789012345678
the first step in this problem is to figure out what these values are. In the 9
123456789012345678901234567890121234567890123456789012345678 9
123456789012345678901234567890121234567890123456789012345678
table, August has two squares (1,000 each) and three triangles (50 9
23456789012345678901234567890121234567890123456789012345678
123456789012345678901234567890121234567890123456789012345678 9
123456789012345678901234567890121234567890123456789012345678
returns each), so total August sales are: 9
123456789012345678901234567890121234567890123456789012345678 99
123456789012345678901234567890121234567890123456789012345678 9
1 2(1,000) 2 3(50)5 2000 2 150 5 1,850
123456789012345678901234567890121234567890123456789012345678 9
123456789012345678901234567890121234567890123456789012345678 9
123456789012345678901234567890121234567890123456789012345678 9
123456789012345678901234567890121234567890123456789012345678 9
123456789012345678901234567890121234567890123456789012345678 9
123456789012345678901234567890121234567890123456789012345678 9
123456789012345678901234567890121234567890123456789012345678 9
123456789012345678901234567890121234567890123456789012345678 9
123456789012345678901234567890121234567890123456789012345678 9
1234567890123456789012345678901212345678901234567890123456789
Copyright © 2005 Thomson Peterson’s, a part of The Thomson Corporaton 55
SAT is a registered trademark of the College Entrance Examination Board, which
was not involved in the production of and does not endorse this product.
1234567890123456789012345678901212345678901234567890123456789
123456789012345678901234567890121234567890123456789012345678 9
123456789012345678901234567890121234567890123456789012345678 9
123456789012345678901234567890121234567890123456789012345678 9
123456789012345678901234567890121234567890123456789012345678 9
123456789012345678901234567890121234567890123456789012345678
Doing the same box-and-triangle conversion, you should find that 9
123456789012345678901234567890121234567890123456789012345678 9
123456789012345678901234567890121234567890123456789012345678
September total sales were 1950. Therefore,
23456789012345678901234567890121234567890123456789012345678
there were 100 more sales 9
9
1
123456789012345678901234567890121234567890123456789012345678
in September than in August. This lets you cross out (B) and (D), 9
123456789012345678901234567890121234567890123456789012345678 9
123456789012345678901234567890121234567890123456789012345678
because they have September total sales as being less than August total 9
123456789012345678901234567890121234567890123456789012345678
23456789012345678901234567890121234567890123456789012345678
9
9
1 sales.
123456789012345678901234567890121234567890123456789012345678 9
123456789012345678901234567890121234567890123456789012345678
The final three answers all have different percentages. The difference in 9
123456789012345678901234567890121234567890123456789012345678 9
123456789012345678901234567890121234567890123456789012345678
monthly sales is 100 (1950 2 1850), so what percent of 1,850 is 100?
23456789012345678901234567890121234567890123456789012345678
9
9
1
123456789012345678901234567890121234567890123456789012345678 9
123456789012345678901234567890121234567890123456789012345678 9
1
S D
n
123456789012345678901234567890121234567890123456789012345678
123456789012345678901234567890121234567890123456789012345678
1850 5 100
23456789012345678901234567890121234567890123456789012345678
100
9
9
9
123456789012345678901234567890121234567890123456789012345678 9
~ ! 100
100 S D
123456789012345678901234567890121234567890123456789012345678
n
123456789012345678901234567890121234567890123456789012345678
1850 5 100 ~ !
100
123456789012345678901234567890121234567890123456789012345678
9
9
9
123456789012345678901234567890121234567890123456789012345678 9
23456789012345678901234567890121234567890123456789012345678
123456789012345678901234567890121234567890123456789012345678
1850n 5 10000 9
123456789012345678901234567890121234567890123456789012345678 9
123456789012345678901234567890121234567890123456789012345678 9
123456789012345678901234567890121234567890123456789012345678
1850n 10000 9
123456789012345678901234567890121234567890123456789012345678
5 9
1 1850 1850 9
123456789012345678901234567890121234567890123456789012345678 9
123456789012345678901234567890121234567890123456789012345678
n 5 5.4 9
123456789012345678901234567890121234567890123456789012345678 9
23456789012345678901234567890121234567890123456789012345678
123456789012345678901234567890121234567890123456789012345678 9
123456789012345678901234567890121234567890123456789012345678
100 is 5.4 percent of 1850, which means that September total sales 9
123456789012345678901234567890121234567890123456789012345678 9
123456789012345678901234567890121234567890123456789012345678 9
were 5.4 percent greater than August total sales. (E) says the same and
123456789012345678901234567890121234567890123456789012345678 9
123456789012345678901234567890121234567890123456789012345678
is the correct answer. 9
123456789012345678901234567890121234567890123456789012345678 9
123456789012345678901234567890121234567890123456789012345678
17. C You cannot solve this problem unless you do some creative line 9
123456789012345678901234567890121234567890123456789012345678 9
1 drawing. 9
123456789012345678901234567890121234567890123456789012345678 9
23456789012345678901234567890121234567890123456789012345678
123456789012345678901234567890121234567890123456789012345678 9 9
1
123456789012345678901234567890121234567890123456789012345678 9
123456789012345678901234567890121234567890123456789012345678 9
123456789012345678901234567890121234567890123456789012345678 9
123456789012345678901234567890121234567890123456789012345678 9
123456789012345678901234567890121234567890123456789012345678 9
123456789012345678901234567890121234567890123456789012345678 9
123456789012345678901234567890121234567890123456789012345678 9
123456789012345678901234567890121234567890123456789012345678 9
23456789012345678901234567890121234567890123456789012345678
123456789012345678901234567890121234567890123456789012345678 9 9
1
123456789012345678901234567890121234567890123456789012345678 9
123456789012345678901234567890121234567890123456789012345678 9
123456789012345678901234567890121234567890123456789012345678 9
123456789012345678901234567890121234567890123456789012345678
To find the unknown lengths, you have to subtract known values of 9
123456789012345678901234567890121234567890123456789012345678 9
123456789012345678901234567890121234567890123456789012345678
opposites. Take the top and bottom sides, for instance. The larger top 9
123456789012345678901234567890121234567890123456789012345678 9
23456789012345678901234567890121234567890123456789012345678
123456789012345678901234567890121234567890123456789012345678
side is 8, and the lower part is 5, so the difference must be 3. A dashed 9
123456789012345678901234567890121234567890123456789012345678 9
123456789012345678901234567890121234567890123456789012345678
line shows this value. Subtracting the right side (length 2) from the left 9
123456789012345678901234567890121234567890123456789012345678 9
1 side (length 6), you find another dashed line that has a value of 4. 9
123456789012345678901234567890121234567890123456789012345678 9
23456789012345678901234567890121234567890123456789012345678
9
123456789012345678901234567890121234567890123456789012345678 9
123456789012345678901234567890121234567890123456789012345678
From this sketch you can see that x is the hypotenuse of a 3-4-5 triangle,
9
123456789012345678901234567890121234567890123456789012345678 9
1 and so is equal to 5. This means the perimeter is 26, (C).
123456789012345678901234567890121234567890123456789012345678 9
23456789012345678901234567890121234567890123456789012345678
9
1
123456789012345678901234567890121234567890123456789012345678 9
123456789012345678901234567890121234567890123456789012345678 9
123456789012345678901234567890121234567890123456789012345678 9
123456789012345678901234567890121234567890123456789012345678 9
123456789012345678901234567890121234567890123456789012345678 9
123456789012345678901234567890121234567890123456789012345678 9
123456789012345678901234567890121234567890123456789012345678 9
1234567890123456789012345678901212345678901234567890123456789
56 Copyright © 2005 Thomson Peterson’s, a part of The Thomson Corporaton
SAT is a registered trademark of the College Entrance Examination Board, which
was not involved in the production of and does not endorse this product.
1234567890123456789012345678901212345678901234567890123456789
123456789012345678901234567890121234567890123456789012345678 9
123456789012345678901234567890121234567890123456789012345678 9
123456789012345678901234567890121234567890123456789012345678 9
123456789012345678901234567890121234567890123456789012345678 9
123456789012345678901234567890121234567890123456789012345678
18. A Unpack the tangle of terms, and you’ll find the answer. You are looking 9
123456789012345678901234567890121234567890123456789012345678 9
123456789012345678901234567890121234567890123456789012345678
for two prime numbers that:
23456789012345678901234567890121234567890123456789012345678
9
9
1
123456789012345678901234567890121234567890123456789012345678 9
123456789012345678901234567890121234567890123456789012345678
1. When you subtract them you get 21. 9
123456789012345678901234567890121234567890123456789012345678 9
123456789012345678901234567890121234567890123456789012345678
2. When you multiply them you get one of the answer choices.
23456789012345678901234567890121234567890123456789012345678
9
9
1
123456789012345678901234567890121234567890123456789012345678 9
123456789012345678901234567890121234567890123456789012345678
It might not sound like enough information to find the answer, but it is. 9
123456789012345678901234567890121234567890123456789012345678 9
123456789012345678901234567890121234567890123456789012345678
None of the answer choices are very great,
23456789012345678901234567890121234567890123456789012345678
so one or both of the prime 9
9
1
123456789012345678901234567890121234567890123456789012345678
numbers must be less than ten. If the two primes were greater, their 9
123456789012345678901234567890121234567890123456789012345678 9
123456789012345678901234567890121234567890123456789012345678
product would not be as small as the answer choices. The difference 9
123456789012345678901234567890121234567890123456789012345678
23456789012345678901234567890121234567890123456789012345678
9
9
1 between the two primes is 21, which means that one of the prime
123456789012345678901234567890121234567890123456789012345678 9
123456789012345678901234567890121234567890123456789012345678
numbers must be small. Good candidates for the lesser prime are 2, 3, 9
123456789012345678901234567890121234567890123456789012345678 9
123456789012345678901234567890121234567890123456789012345678
or 5. 9
123456789012345678901234567890121234567890123456789012345678 99
23456789012345678901234567890121234567890123456789012345678
123456789012345678901234567890121234567890123456789012345678
123456789012345678901234567890121234567890123456789012345678
Prime numbers starting in the 20s are 23, 29, 31, and 37. Use a little 9
123456789012345678901234567890121234567890123456789012345678
trial and error, and you’ll realize that the difference between 23 and 2 is 9
123456789012345678901234567890121234567890123456789012345678 99
123456789012345678901234567890121234567890123456789012345678
21. Those are the two primes. What is 2 times 23? The answer is 46,
9
1
123456789012345678901234567890121234567890123456789012345678
choice (A). 9
123456789012345678901234567890121234567890123456789012345678 9
123456789012345678901234567890121234567890123456789012345678 9
123456789012345678901234567890121234567890123456789012345678 99
23456789012345678901234567890121234567890123456789012345678
19. B First, it always helps to draw a quick sketch of the scenario to get a
123456789012345678901234567890121234567890123456789012345678
123456789012345678901234567890121234567890123456789012345678
better grasp on what is being asked. 9
123456789012345678901234567890121234567890123456789012345678 99
123456789012345678901234567890121234567890123456789012345678 9
123456789012345678901234567890121234567890123456789012345678 9
123456789012345678901234567890121234567890123456789012345678 9
123456789012345678901234567890121234567890123456789012345678 9
123456789012345678901234567890121234567890123456789012345678 9
1
123456789012345678901234567890121234567890123456789012345678 9
123456789012345678901234567890121234567890123456789012345678 9
123456789012345678901234567890121234567890123456789012345678 9
123456789012345678901234567890121234567890123456789012345678 9
123456789012345678901234567890121234567890123456789012345678 9
123456789012345678901234567890121234567890123456789012345678 9
123456789012345678901234567890121234567890123456789012345678 9
123456789012345678901234567890121234567890123456789012345678 9
123456789012345678901234567890121234567890123456789012345678 9
123456789012345678901234567890121234567890123456789012345678 9
123456789012345678901234567890121234567890123456789012345678 99
23456789012345678901234567890121234567890123456789012345678
123456789012345678901234567890121234567890123456789012345678 9
1
123456789012345678901234567890121234567890123456789012345678 9
123456789012345678901234567890121234567890123456789012345678 9
123456789012345678901234567890121234567890123456789012345678 9
123456789012345678901234567890121234567890123456789012345678 9
123456789012345678901234567890121234567890123456789012345678 9
123456789012345678901234567890121234567890123456789012345678 9
123456789012345678901234567890121234567890123456789012345678 9
23456789012345678901234567890121234567890123456789012345678
123456789012345678901234567890121234567890123456789012345678 9
The first thing to determine is the midpoint of DE, which you can do by
123456789012345678901234567890121234567890123456789012345678 9
123456789012345678901234567890121234567890123456789012345678
using the midpoint formula: 9
123456789012345678901234567890121234567890123456789012345678 99
1
123456789012345678901234567890121234567890123456789012345678 9
2
S ,
2
5 D S
123456789012345678901234567890121234567890123456789012345678
x1 1 x2 y1 1 y2 0 1 5 12 1 0
123456789012345678901234567890121234567890123456789012345678
2
,
123456789012345678901234567890121234567890123456789012345678
2
5
2
,D S D
5 12
2
5 ~ 2.5,6 ! 9
9
9
123456789012345678901234567890121234567890123456789012345678 9
23456789012345678901234567890121234567890123456789012345678
123456789012345678901234567890121234567890123456789012345678
You can keep the x-value as a fraction if you like, but converting it to 9
1 9
123456789012345678901234567890121234567890123456789012345678
2.5 will make it easier to punch into your calculator. 9
123456789012345678901234567890121234567890123456789012345678 9
123456789012345678901234567890121234567890123456789012345678 9
123456789012345678901234567890121234567890123456789012345678 9
123456789012345678901234567890121234567890123456789012345678 9
123456789012345678901234567890121234567890123456789012345678 9
123456789012345678901234567890121234567890123456789012345678 9
1234567890123456789012345678901212345678901234567890123456789
Copyright © 2005 Thomson Peterson’s, a part of The Thomson Corporaton 57
SAT is a registered trademark of the College Entrance Examination Board, which
was not involved in the production of and does not endorse this product.
1234567890123456789012345678901212345678901234567890123456789
123456789012345678901234567890121234567890123456789012345678 9
123456789012345678901234567890121234567890123456789012345678 9
123456789012345678901234567890121234567890123456789012345678 9
123456789012345678901234567890121234567890123456789012345678 9
123456789012345678901234567890121234567890123456789012345678
Looking at the dashed lines, you can see a right triangle with its 9
123456789012345678901234567890121234567890123456789012345678 9
123456789012345678901234567890121234567890123456789012345678
hypotenuse from the origin to the midpoint
23456789012345678901234567890121234567890123456789012345678
of DE. It has sides of 2.5 9
9
1
123456789012345678901234567890121234567890123456789012345678
and 6. Placing those values into the Pythagorean theorem yields: 9
123456789012345678901234567890121234567890123456789012345678 9
123456789012345678901234567890121234567890123456789012345678 9
123456789012345678901234567890121234567890123456789012345678 9
2 2 2
a 1 b 5 c
123456789012345678901234567890121234567890123456789012345678 9
123456789012345678901234567890121234567890123456789012345678
2.52 1 62 5 c2 9
123456789012345678901234567890121234567890123456789012345678 9
123456789012345678901234567890121234567890123456789012345678
6.25 36 c2 9
123456789012345678901234567890121234567890123456789012345678
1 5
23456789012345678901234567890121234567890123456789012345678
9
9
1 2
123456789012345678901234567890121234567890123456789012345678
42.25 5 c 9
123456789012345678901234567890121234567890123456789012345678 9
123456789012345678901234567890121234567890123456789012345678
= = 9
123456789012345678901234567890121234567890123456789012345678
42.25 5 c
23456789012345678901234567890121234567890123456789012345678
9
9
1
123456789012345678901234567890121234567890123456789012345678
6.5 5 c 9
123456789012345678901234567890121234567890123456789012345678 9
123456789012345678901234567890121234567890123456789012345678 9
123456789012345678901234567890121234567890123456789012345678
It’s choice (B). 9
123456789012345678901234567890121234567890123456789012345678 9
23456789012345678901234567890121234567890123456789012345678
123456789012345678901234567890121234567890123456789012345678
20. A It’s a big equation, but in the end it’s just that: an equation. You have to 9
123456789012345678901234567890121234567890123456789012345678 9
123456789012345678901234567890121234567890123456789012345678
pull out two different techniques—FOIL and PEMDAS—to solve it, 9
123456789012345678901234567890121234567890123456789012345678 9
123456789012345678901234567890121234567890123456789012345678 9
but so long as you write out your work carefully, you will reach the
9
1
123456789012345678901234567890121234567890123456789012345678
right answer. 9
123456789012345678901234567890121234567890123456789012345678 9
123456789012345678901234567890121234567890123456789012345678 9
23456789012345678901234567890121234567890123456789012345678
123456789012345678901234567890121234567890123456789012345678
2
2b 5 ~b 2 7!~b 1 3! 2 ~2b 1 2!~b 1 5! 9
123456789012345678901234567890121234567890123456789012345678 9
123456789012345678901234567890121234567890123456789012345678
2
~ 2
! ~ 2
! 9
123456789012345678901234567890121234567890123456789012345678
2b 5 b 1 3b 2 7b 2 21 2 2b 1 10b 1 2b 1 10 9
123456789012345678901234567890121234567890123456789012345678 9
123456789012345678901234567890121234567890123456789012345678 9
2 2 2
2b 5 ~ b 2 4b 2 21 ! 2 ~2b 1 12b 1 10 !
123456789012345678901234567890121234567890123456789012345678 9
123456789012345678901234567890121234567890123456789012345678
2b2 5 b2 2 4b 2 21 2 2b2 2 12b 2 10 9
123456789012345678901234567890121234567890123456789012345678 9
1 2 2
2b 5 2b 2 16b 2 31 9
23456789012345678901234567890121234567890123456789012345678
123456789012345678901234567890121234567890123456789012345678 9
123456789012345678901234567890121234567890123456789012345678
2 2 2 2 9
1 2b 1 b 5 2b 1 b 2 16b 2 31 9
123456789012345678901234567890121234567890123456789012345678 9
123456789012345678901234567890121234567890123456789012345678
0 5 216b 2 31 9
123456789012345678901234567890121234567890123456789012345678 9
23456789012345678901234567890121234567890123456789012345678
123456789012345678901234567890121234567890123456789012345678
0 1 16b 5 216b 1 16b 2 31 9
1 9
123456789012345678901234567890121234567890123456789012345678 9
123456789012345678901234567890121234567890123456789012345678
16b 5 231 9
23456789012345678901234567890121234567890123456789012345678
123456789012345678901234567890121234567890123456789012345678 9
123456789012345678901234567890121234567890123456789012345678
16b 231 9
1 5 9
123456789012345678901234567890121234567890123456789012345678
16 16 9
123456789012345678901234567890121234567890123456789012345678 9
23456789012345678901234567890121234567890123456789012345678
123456789012345678901234567890121234567890123456789012345678
31 15 9
123456789012345678901234567890121234567890123456789012345678
b 5 2 5 21 9
123456789012345678901234567890121234567890123456789012345678
16 16 9
123456789012345678901234567890121234567890123456789012345678 9 9
1
123456789012345678901234567890121234567890123456789012345678
Choice (A) is your answer. 9
123456789012345678901234567890121234567890123456789012345678 9
123456789012345678901234567890121234567890123456789012345678 9
123456789012345678901234567890121234567890123456789012345678
21. A There’s no table given for this question, so it’s up to you to create one. 9
123456789012345678901234567890121234567890123456789012345678 9
123456789012345678901234567890121234567890123456789012345678 9
23456789012345678901234567890121234567890123456789012345678
In this respect it’s like a diagram without a diagram given: You could
9
123456789012345678901234567890121234567890123456789012345678 9
123456789012345678901234567890121234567890123456789012345678
conceivably answer it without the visual aids, but it’s much easier
9
123456789012345678901234567890121234567890123456789012345678 9
1 with them.
123456789012345678901234567890121234567890123456789012345678 9
23456789012345678901234567890121234567890123456789012345678
9
1
123456789012345678901234567890121234567890123456789012345678
For the table, start with the years: 9
123456789012345678901234567890121234567890123456789012345678 9
123456789012345678901234567890121234567890123456789012345678
1950 1960 1970 1980 1990 2000 9
123456789012345678901234567890121234567890123456789012345678 9
123456789012345678901234567890121234567890123456789012345678 9
123456789012345678901234567890121234567890123456789012345678 9
123456789012345678901234567890121234567890123456789012345678 9
1234567890123456789012345678901212345678901234567890123456789
58 Copyright © 2005 Thomson Peterson’s, a part of The Thomson Corporaton
SAT is a registered trademark of the College Entrance Examination Board, which
was not involved in the production of and does not endorse this product.
1234567890123456789012345678901212345678901234567890123456789
123456789012345678901234567890121234567890123456789012345678 9
123456789012345678901234567890121234567890123456789012345678 9
123456789012345678901234567890121234567890123456789012345678 9
123456789012345678901234567890121234567890123456789012345678 9
123456789012345678901234567890121234567890123456789012345678
Now place the number of people in Cree County in 1950 in the 9
123456789012345678901234567890121234567890123456789012345678 9
123456789012345678901234567890121234567890123456789012345678
appropriate place, and then start multiplying
23456789012345678901234567890121234567890123456789012345678
by three. 9
9
1
123456789012345678901234567890121234567890123456789012345678 9
123456789012345678901234567890121234567890123456789012345678
1950 1960 1970 1980 1990 2000 9
123456789012345678901234567890121234567890123456789012345678
1000 3000 9000 27000 9
123456789012345678901234567890121234567890123456789012345678
23456789012345678901234567890121234567890123456789012345678
9
9
1
123456789012345678901234567890121234567890123456789012345678
In the 1980s, the population fell by one-half, so divide 27,000 by 2. 9
123456789012345678901234567890121234567890123456789012345678 9
123456789012345678901234567890121234567890123456789012345678
Then go back to multiplying by 3. 9
123456789012345678901234567890121234567890123456789012345678
23456789012345678901234567890121234567890123456789012345678
9
9
1
123456789012345678901234567890121234567890123456789012345678
1950 1960 1970 1980 1990 2000 9
123456789012345678901234567890121234567890123456789012345678 9
123456789012345678901234567890121234567890123456789012345678
1000 3000 9000 27000 13,500 40,500 9
123456789012345678901234567890121234567890123456789012345678
23456789012345678901234567890121234567890123456789012345678
9
9
1
123456789012345678901234567890121234567890123456789012345678
Your answer is choice (A). 9
123456789012345678901234567890121234567890123456789012345678 9
123456789012345678901234567890121234567890123456789012345678 9
123456789012345678901234567890121234567890123456789012345678 9
23456789012345678901234567890121234567890123456789012345678
123456789012345678901234567890121234567890123456789012345678
Section 3 9
123456789012345678901234567890121234567890123456789012345678 9
123456789012345678901234567890121234567890123456789012345678 9
123456789012345678901234567890121234567890123456789012345678
1. A Something that can be counted requires “fewer” rather than “less.” The 9
123456789012345678901234567890121234567890123456789012345678 9
123456789012345678901234567890121234567890123456789012345678 99
answer is (A).
1
123456789012345678901234567890121234567890123456789012345678 9
123456789012345678901234567890121234567890123456789012345678
2. B In for Ada and I, I is used here as the object of a preposition (for). That 9
123456789012345678901234567890121234567890123456789012345678 9
123456789012345678901234567890121234567890123456789012345678 99
23456789012345678901234567890121234567890123456789012345678
should never be. Me is the object of a preposition, or a direct or indirect
123456789012345678901234567890121234567890123456789012345678
123456789012345678901234567890121234567890123456789012345678
object. The answer is (B). 9
123456789012345678901234567890121234567890123456789012345678 99
123456789012345678901234567890121234567890123456789012345678
3. B Is the subject in this sentence singular or plural? Team is singular. Is the
123456789012345678901234567890121234567890123456789012345678 9
123456789012345678901234567890121234567890123456789012345678
verb singular or plural? Compete is plural. The clause which is 9
123456789012345678901234567890121234567890123456789012345678 99
123456789012345678901234567890121234567890123456789012345678
comprised of four cyclists obscures the subject-verb disagreement, but
9
1
23456789012345678901234567890121234567890123456789012345678
123456789012345678901234567890121234567890123456789012345678
it is still there. (B) is the answer. 9
123456789012345678901234567890121234567890123456789012345678 9
1 9
123456789012345678901234567890121234567890123456789012345678
4. E Did you hear any mistakes in this sentence? Hopefully you didn’t 9
123456789012345678901234567890121234567890123456789012345678 9
123456789012345678901234567890121234567890123456789012345678 99
23456789012345678901234567890121234567890123456789012345678
because there are not any. (E) is the answer.
123456789012345678901234567890121234567890123456789012345678 9
1
123456789012345678901234567890121234567890123456789012345678
5. A If you know that Louis and Clark were nineteenth-century explorers, 9
123456789012345678901234567890121234567890123456789012345678 9
123456789012345678901234567890121234567890123456789012345678 99
23456789012345678901234567890121234567890123456789012345678
the verb have surveyed might sound strange even before you read the
123456789012345678901234567890121234567890123456789012345678 9
1 rest of the sentence. If not, were well prepared suggests that the first
123456789012345678901234567890121234567890123456789012345678 9
123456789012345678901234567890121234567890123456789012345678 9
exploration happened prior to an event which is, itself, in the past. The
23456789012345678901234567890121234567890123456789012345678
123456789012345678901234567890121234567890123456789012345678
first verb should be had surveyed, and the answer is (A). 9
123456789012345678901234567890121234567890123456789012345678 9
123456789012345678901234567890121234567890123456789012345678 99
123456789012345678901234567890121234567890123456789012345678
6. D There is a parallel structure problem here. Focus, a verb, should be
9
1
23456789012345678901234567890121234567890123456789012345678
123456789012345678901234567890121234567890123456789012345678
paired with follow up, a verb, not follow-up, which is a noun. The 9
123456789012345678901234567890121234567890123456789012345678 9
123456789012345678901234567890121234567890123456789012345678
answer is (D). 9
123456789012345678901234567890121234567890123456789012345678 99
1
123456789012345678901234567890121234567890123456789012345678
7. C Nothing in this sentence sounds wrong, but (E) is not the answer. Here’s 9
123456789012345678901234567890121234567890123456789012345678 9
123456789012345678901234567890121234567890123456789012345678
why, to whom does the its refer? It is not clear if it refers to the Ottoman 9
123456789012345678901234567890121234567890123456789012345678 9
123456789012345678901234567890121234567890123456789012345678 9
or Austrian Empire. An ambiguous pronoun reference is an error, and
23456789012345678901234567890121234567890123456789012345678
123456789012345678901234567890121234567890123456789012345678 9
1 so (C) is the answer. 9
123456789012345678901234567890121234567890123456789012345678 9
123456789012345678901234567890121234567890123456789012345678 9
123456789012345678901234567890121234567890123456789012345678 9
123456789012345678901234567890121234567890123456789012345678 9
123456789012345678901234567890121234567890123456789012345678 9
123456789012345678901234567890121234567890123456789012345678 9
123456789012345678901234567890121234567890123456789012345678 9
1234567890123456789012345678901212345678901234567890123456789
Copyright © 2005 Thomson Peterson’s, a part of The Thomson Corporaton 59
SAT is a registered trademark of the College Entrance Examination Board, which
was not involved in the production of and does not endorse this product.
1234567890123456789012345678901212345678901234567890123456789
123456789012345678901234567890121234567890123456789012345678 9
123456789012345678901234567890121234567890123456789012345678 9
123456789012345678901234567890121234567890123456789012345678 9
123456789012345678901234567890121234567890123456789012345678 9
123456789012345678901234567890121234567890123456789012345678
8. A What is the subject, census (singular) or statistics (plural)? Statistics. 9
123456789012345678901234567890121234567890123456789012345678 9
123456789012345678901234567890121234567890123456789012345678
Census is used as an adjective. The
23456789012345678901234567890121234567890123456789012345678
plural subject means you would 9
9
1
123456789012345678901234567890121234567890123456789012345678
need a plural verb, were. The answer is (A). 9
123456789012345678901234567890121234567890123456789012345678 9
123456789012345678901234567890121234567890123456789012345678 9
123456789012345678901234567890121234567890123456789012345678
9. B Scarcely no one is a double negative. It
23456789012345678901234567890121234567890123456789012345678
should be scarcely anyone. (B) is 9
9
1 the answer.
123456789012345678901234567890121234567890123456789012345678 9
123456789012345678901234567890121234567890123456789012345678 9
123456789012345678901234567890121234567890123456789012345678
10. D This is a tricky question because the error arises with the implied part of 9
123456789012345678901234567890121234567890123456789012345678
23456789012345678901234567890121234567890123456789012345678
9
9
1 the sentence. The full sentence with the implied part written out
123456789012345678901234567890121234567890123456789012345678 9
123456789012345678901234567890121234567890123456789012345678
explicitly reads, “Even though he had the title of Vice-President of 9
123456789012345678901234567890121234567890123456789012345678 9
123456789012345678901234567890121234567890123456789012345678
Operations, his duties and responsibilities
23456789012345678901234567890121234567890123456789012345678
were not much greater than a 9
9
1
123456789012345678901234567890121234567890123456789012345678
midlevel manager’s duties and responsibilities.” The short version 9
123456789012345678901234567890121234567890123456789012345678 9
123456789012345678901234567890121234567890123456789012345678
should still say manager’s. The answer is (D). 9
123456789012345678901234567890121234567890123456789012345678 9
23456789012345678901234567890121234567890123456789012345678
123456789012345678901234567890121234567890123456789012345678 9
123456789012345678901234567890121234567890123456789012345678
11. D After both, you especially need parallel structure. It should read more 9
123456789012345678901234567890121234567890123456789012345678 9
123456789012345678901234567890121234567890123456789012345678 9
accessible and more understandable. The other answers only confuse
123456789012345678901234567890121234567890123456789012345678 9
123456789012345678901234567890121234567890123456789012345678
the original. The answer is (D). 9
1 9
123456789012345678901234567890121234567890123456789012345678
12. D The initial modifier should put you on the lookout for an incorrect 9
123456789012345678901234567890121234567890123456789012345678 9
123456789012345678901234567890121234567890123456789012345678
noun, or reference to the “thing” that is being modified. You should 9
123456789012345678901234567890121234567890123456789012345678 9
23456789012345678901234567890121234567890123456789012345678
123456789012345678901234567890121234567890123456789012345678
examine the noun that directly follows the modifier carefully. Does the 9
123456789012345678901234567890121234567890123456789012345678 9
123456789012345678901234567890121234567890123456789012345678
modifier modify the noun that follows it? Nope. The poet, Gerard 9
123456789012345678901234567890121234567890123456789012345678 9
123456789012345678901234567890121234567890123456789012345678 9
Manley Hopkins, not his poems, is what is being modified here. The
123456789012345678901234567890121234567890123456789012345678 9
123456789012345678901234567890121234567890123456789012345678
possessive form (Hopkins’s) and the noun poems messes things up in 9
123456789012345678901234567890121234567890123456789012345678 9
1 the original and answer choices (A), (B), and (C). To make it work, you 9
23456789012345678901234567890121234567890123456789012345678
123456789012345678901234567890121234567890123456789012345678 9
123456789012345678901234567890121234567890123456789012345678
need to get rid of the possessive on the author’s name. Only (D) and (E) 9
1 9
123456789012345678901234567890121234567890123456789012345678 9
do that. (E) makes an unnecessary change to the verb. The better
123456789012345678901234567890121234567890123456789012345678 9
123456789012345678901234567890121234567890123456789012345678
answer is (D). 9
123456789012345678901234567890121234567890123456789012345678 9
123456789012345678901234567890121234567890123456789012345678 9
123456789012345678901234567890121234567890123456789012345678
13. E The original underlined phrase is not glaringly obtuse or a grammatical 9
123456789012345678901234567890121234567890123456789012345678 9
123456789012345678901234567890121234567890123456789012345678 9
23456789012345678901234567890121234567890123456789012345678
disaster, but it is a little wordy. Also, this includes the Human Rights
123456789012345678901234567890121234567890123456789012345678
Commission is a complete sentence, so it would need to be attached 9
1 9
123456789012345678901234567890121234567890123456789012345678
with a semicolon. Only (E) addresses both problems without creating 9
123456789012345678901234567890121234567890123456789012345678 9
23456789012345678901234567890121234567890123456789012345678
123456789012345678901234567890121234567890123456789012345678
others. 9
123456789012345678901234567890121234567890123456789012345678 9
123456789012345678901234567890121234567890123456789012345678 9
123456789012345678901234567890121234567890123456789012345678
14. C The most blatant mistake here is that, again, the clause that follows a 9
1 9
123456789012345678901234567890121234567890123456789012345678 9
23456789012345678901234567890121234567890123456789012345678
comma is technically a complete sentence. The best way to approach
9
123456789012345678901234567890121234567890123456789012345678 9
123456789012345678901234567890121234567890123456789012345678
this question is to read all the way through each of the answers to see if
9
123456789012345678901234567890121234567890123456789012345678 9
1 they contain problems. (B) has tense problems. (D) also has two
23456789012345678901234567890121234567890123456789012345678
123456789012345678901234567890121234567890123456789012345678 9
123456789012345678901234567890121234567890123456789012345678
complete sentences attached by a comma (this is called a comma splice). 9
123456789012345678901234567890121234567890123456789012345678 9
123456789012345678901234567890121234567890123456789012345678 9
(E) just doesn’t work. The answer is (C).
9
1
123456789012345678901234567890121234567890123456789012345678 9
123456789012345678901234567890121234567890123456789012345678 9
123456789012345678901234567890121234567890123456789012345678 9
123456789012345678901234567890121234567890123456789012345678 9
123456789012345678901234567890121234567890123456789012345678 9
123456789012345678901234567890121234567890123456789012345678 9
123456789012345678901234567890121234567890123456789012345678 9
123456789012345678901234567890121234567890123456789012345678 9
123456789012345678901234567890121234567890123456789012345678 9
1234567890123456789012345678901212345678901234567890123456789
60 Copyright © 2005 Thomson Peterson’s, a part of The Thomson Corporaton
SAT is a registered trademark of the College Entrance Examination Board, which
was not involved in the production of and does not endorse this product.
1234567890123456789012345678901212345678901234567890123456789
123456789012345678901234567890121234567890123456789012345678 9
123456789012345678901234567890121234567890123456789012345678 9
123456789012345678901234567890121234567890123456789012345678 9
123456789012345678901234567890121234567890123456789012345678 9
123456789012345678901234567890121234567890123456789012345678
15. B The underlined portion of the sentence reads awkwardly for two 9
123456789012345678901234567890121234567890123456789012345678 9
123456789012345678901234567890121234567890123456789012345678
reasons. First, it includes the phrase
23456789012345678901234567890121234567890123456789012345678
her plans of, which is redundant 9
9
1
123456789012345678901234567890121234567890123456789012345678
because the list is a list of her plans. Second, the preceding plans in the 9
123456789012345678901234567890121234567890123456789012345678 9
123456789012345678901234567890121234567890123456789012345678
list are given in infinitive form (i.e. to refinance), but the last plan of 9
123456789012345678901234567890121234567890123456789012345678
23456789012345678901234567890121234567890123456789012345678
9
9
1 action is not in infinitive form. Any answer choice that deals with these
123456789012345678901234567890121234567890123456789012345678 9
123456789012345678901234567890121234567890123456789012345678
two issues will be the best choice. (B) is that answer choice. 9
123456789012345678901234567890121234567890123456789012345678 9
123456789012345678901234567890121234567890123456789012345678
16. B You know the correct answer won’t be a complete sentence, because it
23456789012345678901234567890121234567890123456789012345678
9
9
1
123456789012345678901234567890121234567890123456789012345678
is attached by a comma to the main clause. Eliminate (E). You will also 9
123456789012345678901234567890121234567890123456789012345678 9
123456789012345678901234567890121234567890123456789012345678
want to cut down on wordiness, rather than add to it. Eliminate (A) and 9
123456789012345678901234567890121234567890123456789012345678
23456789012345678901234567890121234567890123456789012345678
9
9
1 (C). Which suggests that the clause modifies a noun that comes just
123456789012345678901234567890121234567890123456789012345678 9
123456789012345678901234567890121234567890123456789012345678
before it. That isn’t the case. The answer is (B). 9
123456789012345678901234567890121234567890123456789012345678 9
123456789012345678901234567890121234567890123456789012345678 9
123456789012345678901234567890121234567890123456789012345678 99
23456789012345678901234567890121234567890123456789012345678
17. C Again, you want to move toward parallel structure as much as possible.
123456789012345678901234567890121234567890123456789012345678
123456789012345678901234567890121234567890123456789012345678
You don’t want nouns in one part of the list and verbs in another, as you 9
123456789012345678901234567890121234567890123456789012345678 9
123456789012345678901234567890121234567890123456789012345678 99
have now. Change the noun tolerance to a verb, tolerate. You can
123456789012345678901234567890121234567890123456789012345678 9
1 narrow the field to (C) and (E). (E) is wrong because you don’t use a
123456789012345678901234567890121234567890123456789012345678 9
123456789012345678901234567890121234567890123456789012345678
comma in a two-items list. 9
123456789012345678901234567890121234567890123456789012345678 9
23456789012345678901234567890121234567890123456789012345678
123456789012345678901234567890121234567890123456789012345678
18. D Guess what? Parallel structure again. To commit a crime needs to be 9
123456789012345678901234567890121234567890123456789012345678 9
123456789012345678901234567890121234567890123456789012345678
followed by (to) conceal it. That leaves (D), without you even having to 9
123456789012345678901234567890121234567890123456789012345678 99
123456789012345678901234567890121234567890123456789012345678
know that among suggests more than two people, whereas between
123456789012345678901234567890121234567890123456789012345678 9
123456789012345678901234567890121234567890123456789012345678
suggests two. 9
123456789012345678901234567890121234567890123456789012345678 99
123456789012345678901234567890121234567890123456789012345678
1 19. C Here’s the test writers’ other favorite trick: “not only” with “but also.” 9
123456789012345678901234567890121234567890123456789012345678 9
123456789012345678901234567890121234567890123456789012345678
Only (C) and (E) use this construction. (E) makes the tenses 9
123456789012345678901234567890121234567890123456789012345678 9
123456789012345678901234567890121234567890123456789012345678 9
unnecessarily complex. The answer is (C).
123456789012345678901234567890121234567890123456789012345678 9
123456789012345678901234567890121234567890123456789012345678 9
123456789012345678901234567890121234567890123456789012345678
20. D The sentence as it stands is muddy and unnecessarily wordy. These are 9
123456789012345678901234567890121234567890123456789012345678 9
123456789012345678901234567890121234567890123456789012345678
things to be avoided in good writing. “If” should be followed by a 9
123456789012345678901234567890121234567890123456789012345678 9
123456789012345678901234567890121234567890123456789012345678 99
23456789012345678901234567890121234567890123456789012345678
“then” clause. What follows here doesn’t fit that logical pattern. So if is
123456789012345678901234567890121234567890123456789012345678
out. That leaves (D) and (E). (D) gets rid of the repetitive and vague it in 9
1
123456789012345678901234567890121234567890123456789012345678
the second part of the sentence. Good. The answer is (D). 9
123456789012345678901234567890121234567890123456789012345678 9
23456789012345678901234567890121234567890123456789012345678
123456789012345678901234567890121234567890123456789012345678 9
123456789012345678901234567890121234567890123456789012345678
21. D The original version should wave a red flag at you that it is repetitive 9
123456789012345678901234567890121234567890123456789012345678 9
123456789012345678901234567890121234567890123456789012345678
and cumbersome and will need to be changed. (B) is even more vague 9
1 9
123456789012345678901234567890121234567890123456789012345678 99
23456789012345678901234567890121234567890123456789012345678
and wordy than the original, despite being a single sentence. (C) is weak
123456789012345678901234567890121234567890123456789012345678 9
123456789012345678901234567890121234567890123456789012345678
because it begins the essay with the word it, which is never a strong
9
123456789012345678901234567890121234567890123456789012345678 9
1 opener. (E) is suspect because the essay is about the word modern not
23456789012345678901234567890121234567890123456789012345678
123456789012345678901234567890121234567890123456789012345678 9
123456789012345678901234567890121234567890123456789012345678
modernity. (Yes, modernity is later introduced, but that does not 9
123456789012345678901234567890121234567890123456789012345678 9
123456789012345678901234567890121234567890123456789012345678 99
change the point that the essay is about the word modern.) (E) is also
1
123456789012345678901234567890121234567890123456789012345678
unnecessarily wordy. That leaves (D). 9
123456789012345678901234567890121234567890123456789012345678 9
123456789012345678901234567890121234567890123456789012345678 9
123456789012345678901234567890121234567890123456789012345678 9
123456789012345678901234567890121234567890123456789012345678 9
123456789012345678901234567890121234567890123456789012345678 9
123456789012345678901234567890121234567890123456789012345678 9
123456789012345678901234567890121234567890123456789012345678 9
123456789012345678901234567890121234567890123456789012345678 9
1234567890123456789012345678901212345678901234567890123456789
Copyright © 2005 Thomson Peterson’s, a part of The Thomson Corporaton 61
SAT is a registered trademark of the College Entrance Examination Board, which
was not involved in the production of and does not endorse this product.
1234567890123456789012345678901212345678901234567890123456789
123456789012345678901234567890121234567890123456789012345678 9
123456789012345678901234567890121234567890123456789012345678 9
123456789012345678901234567890121234567890123456789012345678 9
123456789012345678901234567890121234567890123456789012345678 9
123456789012345678901234567890121234567890123456789012345678
22. B Descartes is the thing that will link the two sentences. He is the subject 9
123456789012345678901234567890121234567890123456789012345678 9
123456789012345678901234567890121234567890123456789012345678
of the first and the object of the second.
23456789012345678901234567890121234567890123456789012345678
This is a prime example of 9
9
1
123456789012345678901234567890121234567890123456789012345678
when to use whom. (B) is better than (C) and the others are longer than 9
123456789012345678901234567890121234567890123456789012345678 9
123456789012345678901234567890121234567890123456789012345678
the original, not shorter. (B) is the answer. 9
123456789012345678901234567890121234567890123456789012345678
23456789012345678901234567890121234567890123456789012345678
9
9
1 23. E What is the logical connection between the opinions of academics other
123456789012345678901234567890121234567890123456789012345678 9
123456789012345678901234567890121234567890123456789012345678
than philosophers and the opinions of historians in particular? The 9
123456789012345678901234567890121234567890123456789012345678 9
123456789012345678901234567890121234567890123456789012345678
latter is an example of the former. The answer is (E).
23456789012345678901234567890121234567890123456789012345678
9
9
1
123456789012345678901234567890121234567890123456789012345678 9
123456789012345678901234567890121234567890123456789012345678
24. B The original sentence is clunky because it has both beginning of 9
123456789012345678901234567890121234567890123456789012345678 9
123456789012345678901234567890121234567890123456789012345678
modernity and beginning of modern times.
23456789012345678901234567890121234567890123456789012345678
You will want to replace the 9
9
1
123456789012345678901234567890121234567890123456789012345678
second with it. That leaves (B) and possibly (E). (E) adds unhelpful 9
123456789012345678901234567890121234567890123456789012345678 9
123456789012345678901234567890121234567890123456789012345678
extra verbiage in other places, so (B) is the best answer. 9
123456789012345678901234567890121234567890123456789012345678 9
23456789012345678901234567890121234567890123456789012345678
123456789012345678901234567890121234567890123456789012345678 9
123456789012345678901234567890121234567890123456789012345678
25. E Anything separated by commas should be detachable from the rest of 9
123456789012345678901234567890121234567890123456789012345678 9
123456789012345678901234567890121234567890123456789012345678 9
the sentence. Remove the phrase between commas and you get Most of
123456789012345678901234567890121234567890123456789012345678 9
123456789012345678901234567890121234567890123456789012345678
us. . . we are thinking. Bad. Most of us think. (E) is the only answer that 9
1 9
123456789012345678901234567890121234567890123456789012345678
addresses that problem correctly. (B) actually changes the sense by 9
123456789012345678901234567890121234567890123456789012345678
eliminating the subject of “think” so that it becomes a command. 9
123456789012345678901234567890121234567890123456789012345678 9
123456789012345678901234567890121234567890123456789012345678 9
23456789012345678901234567890121234567890123456789012345678
123456789012345678901234567890121234567890123456789012345678
26. B Choices (C) and (E) have an imprecise and unclear 2ing verb, so 9
123456789012345678901234567890121234567890123456789012345678 9
123456789012345678901234567890121234567890123456789012345678
eliminate them. (D) is just a wordier alternative to (B), so eliminate it. 9
123456789012345678901234567890121234567890123456789012345678 9
123456789012345678901234567890121234567890123456789012345678 9
(B) is so straightforward and clear that it is better than the original. The
123456789012345678901234567890121234567890123456789012345678 9
123456789012345678901234567890121234567890123456789012345678
answer is (B). 9
123456789012345678901234567890121234567890123456789012345678 9 9
1 27. E The passage is talking about the 1920s. Be as specific as possible,
123456789012345678901234567890121234567890123456789012345678 9
123456789012345678901234567890121234567890123456789012345678 9
123456789012345678901234567890121234567890123456789012345678 9
without being wordy as in (C) and (D). The answer is (E).
123456789012345678901234567890121234567890123456789012345678 9
123456789012345678901234567890121234567890123456789012345678 9
123456789012345678901234567890121234567890123456789012345678
28. C The only punctuation marks that are possible here are parentheses, a 9
123456789012345678901234567890121234567890123456789012345678
semicolon or a colon. The second sentence gives more specific 9
123456789012345678901234567890121234567890123456789012345678 9
123456789012345678901234567890121234567890123456789012345678
information to elaborate on the first sentence, so a colon is better than 9
123456789012345678901234567890121234567890123456789012345678 9
23456789012345678901234567890121234567890123456789012345678
123456789012345678901234567890121234567890123456789012345678
a semicolon or parentheses. The answer is (C). 9
123456789012345678901234567890121234567890123456789012345678 9
1 29. D What’s wrong with the transition in the original? Is it a logical problem 9
123456789012345678901234567890121234567890123456789012345678 9
123456789012345678901234567890121234567890123456789012345678 9
123456789012345678901234567890121234567890123456789012345678
or a grammatical problem? It sounds weird and wordy, so it’s a 9
123456789012345678901234567890121234567890123456789012345678 9
123456789012345678901234567890121234567890123456789012345678
grammatical problem. The sense is logical, so keep it. Eliminate (A); it 9
123456789012345678901234567890121234567890123456789012345678 9
123456789012345678901234567890121234567890123456789012345678 9
changes the sense. Which of the answers simplifies and clarifies the
23456789012345678901234567890121234567890123456789012345678
123456789012345678901234567890121234567890123456789012345678 9
123456789012345678901234567890121234567890123456789012345678
grammar? The answer is (D). 9
123456789012345678901234567890121234567890123456789012345678 9 9
123456789012345678901234567890121234567890123456789012345678 9
1 30. A Eliminate (D) because it’s grammatically incorrect. Eliminate (C)
23456789012345678901234567890121234567890123456789012345678
123456789012345678901234567890121234567890123456789012345678 9
123456789012345678901234567890121234567890123456789012345678
because it really doesn’t go with the ideas in the passage. If you look 9
123456789012345678901234567890121234567890123456789012345678 9
123456789012345678901234567890121234567890123456789012345678 9
back at the passage, you’ll see that the last sentence is not about Macy’s
9
1 but Bloomingdales. (E) would be out of place, so eliminate it. You’re
123456789012345678901234567890121234567890123456789012345678 9
123456789012345678901234567890121234567890123456789012345678
left with (A) and (B). While (B) would be an adequate conclusion to the 9
123456789012345678901234567890121234567890123456789012345678 9
123456789012345678901234567890121234567890123456789012345678
third paragraph, only (A) reflects back over the entire passage. The best 9
123456789012345678901234567890121234567890123456789012345678
23456789012345678901234567890121234567890123456789012345678
9
9
1 answer is (A).
123456789012345678901234567890121234567890123456789012345678 9
123456789012345678901234567890121234567890123456789012345678 9
123456789012345678901234567890121234567890123456789012345678 9
1234567890123456789012345678901212345678901234567890123456789
62 Copyright © 2005 Thomson Peterson’s, a part of The Thomson Corporaton
SAT is a registered trademark of the College Entrance Examination Board, which
was not involved in the production of and does not endorse this product.
1234567890123456789012345678901212345678901234567890123456789
123456789012345678901234567890121234567890123456789012345678 9
123456789012345678901234567890121234567890123456789012345678 9
123456789012345678901234567890121234567890123456789012345678 9
123456789012345678901234567890121234567890123456789012345678 9
123456789012345678901234567890121234567890123456789012345678
Section 4 9
123456789012345678901234567890121234567890123456789012345678 9
123456789012345678901234567890121234567890123456789012345678
1. D Jerome does not indulge much. He is an ascetic, (D). A teetotaler, (B), is 9
123456789012345678901234567890121234567890123456789012345678 9
123456789012345678901234567890121234567890123456789012345678
someone who does not consume alcohol. A gourmand, (C), is someone 9
123456789012345678901234567890121234567890123456789012345678 9
123456789012345678901234567890121234567890123456789012345678
who indulges a great deal in fine food. Neither (A) nor (E) have 9
123456789012345678901234567890121234567890123456789012345678
23456789012345678901234567890121234567890123456789012345678
9
9
1 anything to do with indulgence.
123456789012345678901234567890121234567890123456789012345678 9
123456789012345678901234567890121234567890123456789012345678 9
123456789012345678901234567890121234567890123456789012345678
2. A Begin with the first blank, since you know that it must be something 9
123456789012345678901234567890121234567890123456789012345678
negative since protests were planned. (B) and (D) are not negative, so
23456789012345678901234567890121234567890123456789012345678
9
9
1
123456789012345678901234567890121234567890123456789012345678
eliminate them. Going to the second blank, organized fits well, while 9
123456789012345678901234567890121234567890123456789012345678 9
123456789012345678901234567890121234567890123456789012345678
negotiated and theorized do not (you don’t theorize about a protest 9
123456789012345678901234567890121234567890123456789012345678
23456789012345678901234567890121234567890123456789012345678
9
9
1 event). This makes (A) the best answer.
123456789012345678901234567890121234567890123456789012345678 9
123456789012345678901234567890121234567890123456789012345678 9
123456789012345678901234567890121234567890123456789012345678
3. B We know the first word will be negative, since it engendered criticism. 9
123456789012345678901234567890121234567890123456789012345678 9
123456789012345678901234567890121234567890123456789012345678 99
23456789012345678901234567890121234567890123456789012345678
All of the words are negative, so you can’t eliminate anything yet. But
123456789012345678901234567890121234567890123456789012345678
what would a politician be if he “seized a ceremony” having to do with
123456789012345678901234567890121234567890123456789012345678 9
123456789012345678901234567890121234567890123456789012345678
a “girl’s tragic death to speak out against his opponent”? Not militaris- 9
123456789012345678901234567890121234567890123456789012345678 9
123456789012345678901234567890121234567890123456789012345678 99
tic, or unreceptive, certainly not passive or defeatist. The answer is (B).
1
123456789012345678901234567890121234567890123456789012345678 9
123456789012345678901234567890121234567890123456789012345678
4. E All of the options in this question have a first word that could be 9
123456789012345678901234567890121234567890123456789012345678 9
123456789012345678901234567890121234567890123456789012345678 99
23456789012345678901234567890121234567890123456789012345678
something one software did to another. For the second word, all the
123456789012345678901234567890121234567890123456789012345678
123456789012345678901234567890121234567890123456789012345678
choices, except (D), are things a user could likely do to software. 9
123456789012345678901234567890121234567890123456789012345678 9
123456789012345678901234567890121234567890123456789012345678 99
Preclude, (A), means make something impossible, but the “prior
123456789012345678901234567890121234567890123456789012345678
version” clearly isn’t impossible so eliminate (A). If the older version is
123456789012345678901234567890121234567890123456789012345678 9
123456789012345678901234567890121234567890123456789012345678
outdone, you don’t want to implement, or use, it. Eliminate (B). If 9
123456789012345678901234567890121234567890123456789012345678 9
1 something is infected, you don’t want to disregard it, so eliminate (C). 9
23456789012345678901234567890121234567890123456789012345678
123456789012345678901234567890121234567890123456789012345678 9
123456789012345678901234567890121234567890123456789012345678
The answer is (E). Supercede means replace by being better, which 9
1 9
123456789012345678901234567890121234567890123456789012345678 9
would mean you could throw out, or discard, the now useless thing.
123456789012345678901234567890121234567890123456789012345678 9
123456789012345678901234567890121234567890123456789012345678 9
123456789012345678901234567890121234567890123456789012345678
5. B Let’s start with the first blank since it has to be paired with long. Which 9
123456789012345678901234567890121234567890123456789012345678 9
123456789012345678901234567890121234567890123456789012345678
of the first answer choices pairs well with long? Only storied, 9
123456789012345678901234567890121234567890123456789012345678
exemplary, and reputable do (the others either are awkward or don’t fit 9
123456789012345678901234567890121234567890123456789012345678 99
23456789012345678901234567890121234567890123456789012345678
123456789012345678901234567890121234567890123456789012345678
with the meaning of the sentence). Plug in the second answer choices of
9
1
123456789012345678901234567890121234567890123456789012345678
(A), (B), and (E) into the sentence. Can poetry initiate the dignity of 9
123456789012345678901234567890121234567890123456789012345678 9
23456789012345678901234567890121234567890123456789012345678
123456789012345678901234567890121234567890123456789012345678
humanity? No. Eliminate (A). Can it articulate, or express? Definitely. 9
123456789012345678901234567890121234567890123456789012345678 9
123456789012345678901234567890121234567890123456789012345678
Choice (B) sounds too good to pass up. 9
123456789012345678901234567890121234567890123456789012345678 99
1
123456789012345678901234567890121234567890123456789012345678
6. C You should have a good sense of what kind of word goes in each blank 9
123456789012345678901234567890121234567890123456789012345678 9
123456789012345678901234567890121234567890123456789012345678
because each blank has a pair. The second blank has a more obvious 9
123456789012345678901234567890121234567890123456789012345678 9
123456789012345678901234567890121234567890123456789012345678 9
pair, so start with it. Which of the second answer choices goes with
23456789012345678901234567890121234567890123456789012345678
123456789012345678901234567890121234567890123456789012345678 9
123456789012345678901234567890121234567890123456789012345678
speedy? (A) and (C) are the best candidates. On the first blank, does 9
123456789012345678901234567890121234567890123456789012345678 9
123456789012345678901234567890121234567890123456789012345678 99
intricate and laborious or fragile and laborious sound better? Intricate
1
123456789012345678901234567890121234567890123456789012345678
does because it provides a better contrast with the second half of the 9
123456789012345678901234567890121234567890123456789012345678 9
123456789012345678901234567890121234567890123456789012345678
sentence. (Also, it is strange to speak of a surgery being fragile; someone 9
123456789012345678901234567890121234567890123456789012345678 9
123456789012345678901234567890121234567890123456789012345678
in surgery might be in fragile health, but
23456789012345678901234567890121234567890123456789012345678
the surgery itself would not be 9
9
1
123456789012345678901234567890121234567890123456789012345678
fragile.) (C) is the answer. 9
123456789012345678901234567890121234567890123456789012345678 99
123456789012345678901234567890121234567890123456789012345678
1234567890123456789012345678901212345678901234567890123456789
Copyright © 2005 Thomson Peterson’s, a part of The Thomson Corporaton 63
SAT is a registered trademark of the College Entrance Examination Board, which
was not involved in the production of and does not endorse this product.
1234567890123456789012345678901212345678901234567890123456789
123456789012345678901234567890121234567890123456789012345678 9
123456789012345678901234567890121234567890123456789012345678 9
123456789012345678901234567890121234567890123456789012345678 9
123456789012345678901234567890121234567890123456789012345678 9
123456789012345678901234567890121234567890123456789012345678
7. A Despite at the beginning of the sentence is the main clue to unlocking 9
123456789012345678901234567890121234567890123456789012345678 9
123456789012345678901234567890121234567890123456789012345678
the blank. You need one positive
23456789012345678901234567890121234567890123456789012345678
word and one negative word. 9
9
1
123456789012345678901234567890121234567890123456789012345678
Eliminate (B) and (C). Now what can clans do to each other? Assist, 9
123456789012345678901234567890121234567890123456789012345678 9
123456789012345678901234567890121234567890123456789012345678
possibly discount. A problem is ameliorated, not a person or group of 9
123456789012345678901234567890121234567890123456789012345678
23456789012345678901234567890121234567890123456789012345678
9
9
1 people. (A) is the better answer, especially since “feud” and “clan” are
123456789012345678901234567890121234567890123456789012345678 9
123456789012345678901234567890121234567890123456789012345678
both words that seem to refer to the past. 9
123456789012345678901234567890121234567890123456789012345678 9
123456789012345678901234567890121234567890123456789012345678
8. B From the structure of the sentence, we can see that the word in the
23456789012345678901234567890121234567890123456789012345678
9
9
1
123456789012345678901234567890121234567890123456789012345678
blank goes along with humor and merriment. Which of the answer 9
123456789012345678901234567890121234567890123456789012345678 9
123456789012345678901234567890121234567890123456789012345678
choices fits with these two positive words? Only (B), mirth, does. 9
123456789012345678901234567890121234567890123456789012345678
23456789012345678901234567890121234567890123456789012345678
9
9
1 (Irony, history, and mystery, are at best neutral terms in this context,
123456789012345678901234567890121234567890123456789012345678 9
123456789012345678901234567890121234567890123456789012345678
not positive.) 9
123456789012345678901234567890121234567890123456789012345678 9
123456789012345678901234567890121234567890123456789012345678 9
123456789012345678901234567890121234567890123456789012345678 9
23456789012345678901234567890121234567890123456789012345678
9. A The Latin makes this sentence complicated. You can combat this by
123456789012345678901234567890121234567890123456789012345678 9
123456789012345678901234567890121234567890123456789012345678
replacing the Latin words with the letters (A) and (B) in your head (i.e. 9
123456789012345678901234567890121234567890123456789012345678 9
123456789012345678901234567890121234567890123456789012345678 9
although (A) has more subspecies than (B) . . . ). The although is the key
123456789012345678901234567890121234567890123456789012345678 9
1 to the logic of the sentence structure. Although (A) has more subspecies 9
123456789012345678901234567890121234567890123456789012345678 9
123456789012345678901234567890121234567890123456789012345678
than (B), (B) has greater numbers. (A) and (E) match this pre-guess for 9
123456789012345678901234567890121234567890123456789012345678 9
123456789012345678901234567890121234567890123456789012345678 9
23456789012345678901234567890121234567890123456789012345678
the first blank. (E) isn’t really logical (how can a region be ominous?),
123456789012345678901234567890121234567890123456789012345678 9
123456789012345678901234567890121234567890123456789012345678
so the answer is (A). 9
123456789012345678901234567890121234567890123456789012345678 9
123456789012345678901234567890121234567890123456789012345678
10. D Looking at the first blank, (B) and (E) can be eliminated because both 9
123456789012345678901234567890121234567890123456789012345678 9
123456789012345678901234567890121234567890123456789012345678
are not conventional English. That leaves (A), (C), and (D). (A) is 9
123456789012345678901234567890121234567890123456789012345678 9
123456789012345678901234567890121234567890123456789012345678
suspect, though, because progression is an odd word choice for the 9
1 9
123456789012345678901234567890121234567890123456789012345678 9
23456789012345678901234567890121234567890123456789012345678
context. (C) is illogical. (D) is a best choice because it flows well in both
123456789012345678901234567890121234567890123456789012345678 9
1 blanks and because development is already a word you associate with 9
123456789012345678901234567890121234567890123456789012345678 9
123456789012345678901234567890121234567890123456789012345678
“pregnancy” and “fetus.” 9
123456789012345678901234567890121234567890123456789012345678 9
23456789012345678901234567890121234567890123456789012345678
123456789012345678901234567890121234567890123456789012345678 9
1 11. C The word, Though, tells you that the ideals and the practices of the 9
123456789012345678901234567890121234567890123456789012345678 9
123456789012345678901234567890121234567890123456789012345678
organization are at odds. So if the ideals are outward-focused then the 9
23456789012345678901234567890121234567890123456789012345678
123456789012345678901234567890121234567890123456789012345678 9
123456789012345678901234567890121234567890123456789012345678
practice must be inward-focused. Which of the answer choices matches 9
1 9
123456789012345678901234567890121234567890123456789012345678 9
this pre-guess? Only (C), insular (isolated, circumscribed), does.
123456789012345678901234567890121234567890123456789012345678 9
123456789012345678901234567890121234567890123456789012345678
Hermetic means tightly sealed. It’s not too far off in meaning, but it 9
123456789012345678901234567890121234567890123456789012345678
doesn’t refer to group behavior. 9
123456789012345678901234567890121234567890123456789012345678 9
123456789012345678901234567890121234567890123456789012345678 9
123456789012345678901234567890121234567890123456789012345678
12. B On the first blank we know that the word goes with base, and we know 9
23456789012345678901234567890121234567890123456789012345678
123456789012345678901234567890121234567890123456789012345678 9
123456789012345678901234567890121234567890123456789012345678
that the prosecuting attorney is probably not saying nice things about 9
123456789012345678901234567890121234567890123456789012345678 9
123456789012345678901234567890121234567890123456789012345678
the defendant. (A) and (B) fit this (motley doesn’t; even though it has a 9
1 9
123456789012345678901234567890121234567890123456789012345678 9
23456789012345678901234567890121234567890123456789012345678
negative connotation, it means a random or ungainly assortment). This
9
123456789012345678901234567890121234567890123456789012345678 9
123456789012345678901234567890121234567890123456789012345678
leaves misunderstanding the testimonials or misconstruing the testimo-
9
123456789012345678901234567890121234567890123456789012345678 9
1 nials, for the second blank. The latter is a better choice since the
23456789012345678901234567890121234567890123456789012345678
123456789012345678901234567890121234567890123456789012345678 9
1 prosecuting attorney is more likely to be making the defendant sound 9
123456789012345678901234567890121234567890123456789012345678 9
123456789012345678901234567890121234567890123456789012345678 9
bad on purpose than by mistake.
123456789012345678901234567890121234567890123456789012345678 9
123456789012345678901234567890121234567890123456789012345678 9
123456789012345678901234567890121234567890123456789012345678 9
123456789012345678901234567890121234567890123456789012345678 9
123456789012345678901234567890121234567890123456789012345678 9
1234567890123456789012345678901212345678901234567890123456789
64 Copyright © 2005 Thomson Peterson’s, a part of The Thomson Corporaton
SAT is a registered trademark of the College Entrance Examination Board, which
was not involved in the production of and does not endorse this product.
1234567890123456789012345678901212345678901234567890123456789
123456789012345678901234567890121234567890123456789012345678 9
123456789012345678901234567890121234567890123456789012345678 9
123456789012345678901234567890121234567890123456789012345678 9
123456789012345678901234567890121234567890123456789012345678 9
123456789012345678901234567890121234567890123456789012345678
13. B This sentence is either really positive or really negative about the new 9
123456789012345678901234567890121234567890123456789012345678 9
123456789012345678901234567890121234567890123456789012345678
drug regime. It is more likely positive
23456789012345678901234567890121234567890123456789012345678
since a new drug wouldn’t come 9
9
1
123456789012345678901234567890121234567890123456789012345678
out if it was known to be really negative. So your inclination should be 9
123456789012345678901234567890121234567890123456789012345678 9
123456789012345678901234567890121234567890123456789012345678
that the blank is very positive. Only (B) fits this, and it fits well. If you 9
123456789012345678901234567890121234567890123456789012345678
23456789012345678901234567890121234567890123456789012345678
9
9
1 have extra time, you can check to see if there is a really negative
123456789012345678901234567890121234567890123456789012345678 9
123456789012345678901234567890121234567890123456789012345678
response that might fit better. Bane could work in terms of meaning, 9
123456789012345678901234567890121234567890123456789012345678 9
123456789012345678901234567890121234567890123456789012345678
but it doesn’t work syntactically.
23456789012345678901234567890121234567890123456789012345678
9
9
1
123456789012345678901234567890121234567890123456789012345678
14. C The blanks in this sentence are a little more complicated because they 9
123456789012345678901234567890121234567890123456789012345678 9
123456789012345678901234567890121234567890123456789012345678
must be considered together. The first blank is negative and the second 9
123456789012345678901234567890121234567890123456789012345678
23456789012345678901234567890121234567890123456789012345678
9
9
1 blank is even more negative (in the same way, or to a greater degree).
123456789012345678901234567890121234567890123456789012345678 9
123456789012345678901234567890121234567890123456789012345678
The second blank isn’t negative in (A) and (D), so they are not it. That 9
123456789012345678901234567890121234567890123456789012345678 9
123456789012345678901234567890121234567890123456789012345678
leaves (B), (C), and (E). Of the three, (C) is the best since self-centered 9
23456789012345678901234567890121234567890123456789012345678
123456789012345678901234567890121234567890123456789012345678 9
123456789012345678901234567890121234567890123456789012345678
and egocentric are synonyms and solipsism is a more negative form of 9
123456789012345678901234567890121234567890123456789012345678 9
123456789012345678901234567890121234567890123456789012345678 99
both words. You might be tempted to go with (B) because you don’t
123456789012345678901234567890121234567890123456789012345678
123456789012345678901234567890121234567890123456789012345678
know what solipsistic means, but you can eliminate it because 9
1 9
123456789012345678901234567890121234567890123456789012345678
ill-tempered has little to do with self-centered. 9
123456789012345678901234567890121234567890123456789012345678 9
123456789012345678901234567890121234567890123456789012345678 9
123456789012345678901234567890121234567890123456789012345678 99
23456789012345678901234567890121234567890123456789012345678
15. B What is a good pre-guess for the blank? Really hectic (that is, opposite
123456789012345678901234567890121234567890123456789012345678
123456789012345678901234567890121234567890123456789012345678
of alleviating demands upon our time). Only (B) matches this pre-guess, 9
123456789012345678901234567890121234567890123456789012345678
and it fits in the flow of the sentence. 9
123456789012345678901234567890121234567890123456789012345678 99
123456789012345678901234567890121234567890123456789012345678 9
123456789012345678901234567890121234567890123456789012345678
16. A This is a global question since it cannot be answered by looking at one
123456789012345678901234567890121234567890123456789012345678 9
123456789012345678901234567890121234567890123456789012345678
specific place in the text but has to be weighed considering the passage 9
1 9
123456789012345678901234567890121234567890123456789012345678 99
23456789012345678901234567890121234567890123456789012345678
as a whole. Ask yourself, does the author present Dr. Rael positively or
123456789012345678901234567890121234567890123456789012345678 9
1 negatively, sympathetically or unsympathetically? The article describes
123456789012345678901234567890121234567890123456789012345678 9
123456789012345678901234567890121234567890123456789012345678
Dr. Rael’s career, generally in positive
23456789012345678901234567890121234567890123456789012345678
terms, and no criticisms of Dr. 9
9
123456789012345678901234567890121234567890123456789012345678
123456789012345678901234567890121234567890123456789012345678
Rael’s work is discussed. So the answer should be positive. That 9
1 9
123456789012345678901234567890121234567890123456789012345678
eliminates (C)—(E). (B) might seem appealing, but realize that an article 9
123456789012345678901234567890121234567890123456789012345678 9
123456789012345678901234567890121234567890123456789012345678 99
23456789012345678901234567890121234567890123456789012345678
can engage a person’s work without being positive about that work.
123456789012345678901234567890121234567890123456789012345678 9
1 Also the passage more tells about Dr. Rael’s work than engages it. So
123456789012345678901234567890121234567890123456789012345678 9
123456789012345678901234567890121234567890123456789012345678 9
(A) is the best choice.
123456789012345678901234567890121234567890123456789012345678 99
23456789012345678901234567890121234567890123456789012345678
123456789012345678901234567890121234567890123456789012345678
17. C Again this is a global question. To rephrase the question, what is the 9
123456789012345678901234567890121234567890123456789012345678 9
123456789012345678901234567890121234567890123456789012345678
passage about? It tells the story of Dr. Rael’s life with specific emphasis
9
1
23456789012345678901234567890121234567890123456789012345678
123456789012345678901234567890121234567890123456789012345678
on his professional career. Choice (C) captures this. You might have 9
123456789012345678901234567890121234567890123456789012345678 9
123456789012345678901234567890121234567890123456789012345678
been attracted by (E), but remember the passage only discusses the Rael 9
123456789012345678901234567890121234567890123456789012345678 99
1 family when it contributes to the story of Dr. Rael.
123456789012345678901234567890121234567890123456789012345678 9
123456789012345678901234567890121234567890123456789012345678 9
123456789012345678901234567890121234567890123456789012345678
18. B You can answer this either as a detail question or as a global question; 9
123456789012345678901234567890121234567890123456789012345678 9
123456789012345678901234567890121234567890123456789012345678 9
you will get the answer more quickly if you answer it as a detail
23456789012345678901234567890121234567890123456789012345678
123456789012345678901234567890121234567890123456789012345678 9
1 question. You can find in paragraph 3 that “José Ignacio had the 9
123456789012345678901234567890121234567890123456789012345678 9
123456789012345678901234567890121234567890123456789012345678 9
foresight to recognize the changes that were coming with the increasing
123456789012345678901234567890121234567890123456789012345678 9
123456789012345678901234567890121234567890123456789012345678
Americanization of New Mexico and realized that a fluent knowledge 9
123456789012345678901234567890121234567890123456789012345678 9
123456789012345678901234567890121234567890123456789012345678 9
123456789012345678901234567890121234567890123456789012345678 9
1234567890123456789012345678901212345678901234567890123456789
Copyright © 2005 Thomson Peterson’s, a part of The Thomson Corporaton 65
SAT is a registered trademark of the College Entrance Examination Board, which
was not involved in the production of and does not endorse this product.
1234567890123456789012345678901212345678901234567890123456789
123456789012345678901234567890121234567890123456789012345678 9
123456789012345678901234567890121234567890123456789012345678 9
123456789012345678901234567890121234567890123456789012345678 9
123456789012345678901234567890121234567890123456789012345678 9
123456789012345678901234567890121234567890123456789012345678
of English. . . would be necessary.” Choices (B) and (D) seem to echo 9
123456789012345678901234567890121234567890123456789012345678 9
123456789012345678901234567890121234567890123456789012345678
this sentence. But the passage does not
23456789012345678901234567890121234567890123456789012345678
go on to say that Dr. Rael did not 9
9
1
123456789012345678901234567890121234567890123456789012345678
learn English. The answer is (B). If you answer it as a global question, 9
123456789012345678901234567890121234567890123456789012345678 9
123456789012345678901234567890121234567890123456789012345678
use the process of elimination. The passage doesn’t mention job 9
123456789012345678901234567890121234567890123456789012345678
23456789012345678901234567890121234567890123456789012345678
9
9
1 discrimination. The passage refers to Rael studying far from his family,
123456789012345678901234567890121234567890123456789012345678 9
123456789012345678901234567890121234567890123456789012345678
but doesn’t mention that problem in regard to teaching. It strongly 9
123456789012345678901234567890121234567890123456789012345678 9
123456789012345678901234567890121234567890123456789012345678
suggests that Rael was successful in
23456789012345678901234567890121234567890123456789012345678
American schools with English 9
9
1 names, so eliminate (D). The passage doesn’t specify what Rael studied
123456789012345678901234567890121234567890123456789012345678 9
123456789012345678901234567890121234567890123456789012345678
before his Ph.D. Choices (C) and (E) are hard to eliminate, but an 9
123456789012345678901234567890121234567890123456789012345678 9
123456789012345678901234567890121234567890123456789012345678
overall view of the passage should convey that the folklore was precious
23456789012345678901234567890121234567890123456789012345678
9
9
1
123456789012345678901234567890121234567890123456789012345678
and disappearing. 9
123456789012345678901234567890121234567890123456789012345678 9
123456789012345678901234567890121234567890123456789012345678 9
123456789012345678901234567890121234567890123456789012345678
19. E Relinquished means he gave something up, but the passage does not state 9
23456789012345678901234567890121234567890123456789012345678
123456789012345678901234567890121234567890123456789012345678 9
123456789012345678901234567890121234567890123456789012345678
that Rael was relieved to give up his family duties. Eliminate (D). There is 9
123456789012345678901234567890121234567890123456789012345678 9
123456789012345678901234567890121234567890123456789012345678 9
no mention of sibling rivalry, so you can eliminate (A). The passage doesn’t
123456789012345678901234567890121234567890123456789012345678 9
123456789012345678901234567890121234567890123456789012345678
say if the family was wealthy or simply OK, so (B) is out. It certainly 9
1 9
123456789012345678901234567890121234567890123456789012345678
doesn’t say Rael had tried to be a rancher, choice (C). That leaves (E). 9
123456789012345678901234567890121234567890123456789012345678 9
123456789012345678901234567890121234567890123456789012345678 9
123456789012345678901234567890121234567890123456789012345678 9
23456789012345678901234567890121234567890123456789012345678
20. E This question is similar to question 18, but it is definitely a global
123456789012345678901234567890121234567890123456789012345678 9
123456789012345678901234567890121234567890123456789012345678
question. The basic gist of the passage is that Rael’s work was 9
123456789012345678901234567890121234567890123456789012345678
important. He collected Spanish-language folklore and studied the 9
123456789012345678901234567890121234567890123456789012345678 9
123456789012345678901234567890121234567890123456789012345678
particular Spanish used in the area. The answer is (E). 9
123456789012345678901234567890121234567890123456789012345678 9
123456789012345678901234567890121234567890123456789012345678 9
123456789012345678901234567890121234567890123456789012345678
21. C Someone you study under is your mentor. This is a vocabulary question. 9
1 9
123456789012345678901234567890121234567890123456789012345678 9
23456789012345678901234567890121234567890123456789012345678
The answer is (C).
123456789012345678901234567890121234567890123456789012345678 9 9
1 22. B Corpus refers to a body of work. The only answer choice that reflects
123456789012345678901234567890121234567890123456789012345678 9
123456789012345678901234567890121234567890123456789012345678 9
123456789012345678901234567890121234567890123456789012345678
the idea of more than one book is (B). 9
123456789012345678901234567890121234567890123456789012345678 9
123456789012345678901234567890121234567890123456789012345678 9
123456789012345678901234567890121234567890123456789012345678
23. B Looking over the choices, there is no reference to Dr. Rael enjoying 9
123456789012345678901234567890121234567890123456789012345678 9
123456789012345678901234567890121234567890123456789012345678 9
23456789012345678901234567890121234567890123456789012345678
working on the ranch (or using his scholarly pursuits to avoid it, which
123456789012345678901234567890121234567890123456789012345678
would already be using logic too strained for the SAT). The passage 9
1 9
123456789012345678901234567890121234567890123456789012345678
does mention his love of Pastores. Just after the mention of Pastores, 9
123456789012345678901234567890121234567890123456789012345678 9
23456789012345678901234567890121234567890123456789012345678
123456789012345678901234567890121234567890123456789012345678
the passage says this influenced his later work. Bingo. The answer is (B). 9
123456789012345678901234567890121234567890123456789012345678 9
123456789012345678901234567890121234567890123456789012345678 9
123456789012345678901234567890121234567890123456789012345678
24. A This is a tough one. But “diffusion of motifs” seems to be related to the 9
1 9
123456789012345678901234567890121234567890123456789012345678 9
23456789012345678901234567890121234567890123456789012345678
number of variants of a particular story. That eliminates (C) and (E).
9
123456789012345678901234567890121234567890123456789012345678 9
123456789012345678901234567890121234567890123456789012345678
The passage goes on to emphasize how many variants there were,
9
123456789012345678901234567890121234567890123456789012345678 9
1 suggesting that the answer will emphasize difference more than
23456789012345678901234567890121234567890123456789012345678
123456789012345678901234567890121234567890123456789012345678 9
123456789012345678901234567890121234567890123456789012345678
sameness. Eliminate (B) and (D). The answer is (A). 9
123456789012345678901234567890121234567890123456789012345678 9 9
123456789012345678901234567890121234567890123456789012345678 9
1 25. B If you do not know the definition of provenance you are not lost on this
23456789012345678901234567890121234567890123456789012345678
123456789012345678901234567890121234567890123456789012345678 9
1 one. Replace it with each of the answer choices and see which one 9
123456789012345678901234567890121234567890123456789012345678 9
123456789012345678901234567890121234567890123456789012345678 9
makes the most sense. If you do know the definition of provenance
123456789012345678901234567890121234567890123456789012345678
(origin), then (B) pops out as the answer. 9
123456789012345678901234567890121234567890123456789012345678 9
123456789012345678901234567890121234567890123456789012345678 9
123456789012345678901234567890121234567890123456789012345678 9
123456789012345678901234567890121234567890123456789012345678 9
1234567890123456789012345678901212345678901234567890123456789
66 Copyright © 2005 Thomson Peterson’s, a part of The Thomson Corporaton
SAT is a registered trademark of the College Entrance Examination Board, which
was not involved in the production of and does not endorse this product.
1234567890123456789012345678901212345678901234567890123456789
123456789012345678901234567890121234567890123456789012345678 9
123456789012345678901234567890121234567890123456789012345678 9
123456789012345678901234567890121234567890123456789012345678 9
123456789012345678901234567890121234567890123456789012345678 9
123456789012345678901234567890121234567890123456789012345678
26. D The end of the seventh paragraph states, “But inevitably the historic- 9
123456789012345678901234567890121234567890123456789012345678 9
123456789012345678901234567890121234567890123456789012345678
geographic approach led more to collection
23456789012345678901234567890121234567890123456789012345678
building than to analysis.” 9
9
1
123456789012345678901234567890121234567890123456789012345678
This is the most critical sentence in the passage. The answer (D) 9
123456789012345678901234567890121234567890123456789012345678 9
123456789012345678901234567890121234567890123456789012345678
includes the word “analysis” and is a fair paraphrase of this sentence. 9
123456789012345678901234567890121234567890123456789012345678
23456789012345678901234567890121234567890123456789012345678
9
9
1 The answer is (D).
123456789012345678901234567890121234567890123456789012345678 9
123456789012345678901234567890121234567890123456789012345678
27. B This one is also hard. First of all, do you know what formidable and 9
123456789012345678901234567890121234567890123456789012345678 9
123456789012345678901234567890121234567890123456789012345678
quixotic mean? Impressive, difficult, and errant—which is to say,
23456789012345678901234567890121234567890123456789012345678
9
9
1
123456789012345678901234567890121234567890123456789012345678
traveling a lot. This comes from Don Quixote, who traveled a lot. He 9
123456789012345678901234567890121234567890123456789012345678 9
123456789012345678901234567890121234567890123456789012345678
was also a little crazy, which is why the test writers try to trick you with 9
123456789012345678901234567890121234567890123456789012345678
23456789012345678901234567890121234567890123456789012345678
9
9
1 (A) and (C). But these don’t apply to Dr. Rael. Remember, the overall
123456789012345678901234567890121234567890123456789012345678 9
123456789012345678901234567890121234567890123456789012345678
tone is “laudatory.” (E) tries to trick you by getting you to admit that 9
123456789012345678901234567890121234567890123456789012345678 9
123456789012345678901234567890121234567890123456789012345678
you were confused at this point in the passage. Admitting you don’t 9
23456789012345678901234567890121234567890123456789012345678
123456789012345678901234567890121234567890123456789012345678 9
123456789012345678901234567890121234567890123456789012345678
know will never be the answer on the SAT. The answer is (B). 9
123456789012345678901234567890121234567890123456789012345678 99
123456789012345678901234567890121234567890123456789012345678 9
123456789012345678901234567890121234567890123456789012345678 9
123456789012345678901234567890121234567890123456789012345678
1 Section 5 9
123456789012345678901234567890121234567890123456789012345678 9
123456789012345678901234567890121234567890123456789012345678
1. A The wrinkle in this word problem is correctly translating what a 9
123456789012345678901234567890121234567890123456789012345678 9
123456789012345678901234567890121234567890123456789012345678 99
23456789012345678901234567890121234567890123456789012345678
“quarter” of the factory’s capacity means. A quarter is one-fourth of
123456789012345678901234567890121234567890123456789012345678
123456789012345678901234567890121234567890123456789012345678
something, so a quarter of full capacity—200 sheets of paper per 9
123456789012345678901234567890121234567890123456789012345678 9
123456789012345678901234567890121234567890123456789012345678 99
second—is one-fourth of 200, or 50. If a quarter-capacity is 50 sheets
123456789012345678901234567890121234567890123456789012345678
123456789012345678901234567890121234567890123456789012345678
per second, in 12 seconds the factory would produce 600 sheets since 9
123456789012345678901234567890121234567890123456789012345678
50 12 600. Choice (A) is what’s going on. 9
123456789012345678901234567890121234567890123456789012345678 99
3 5
1
123456789012345678901234567890121234567890123456789012345678
2. E This is a straightforward variable/equation substitution problem. If 9
123456789012345678901234567890121234567890123456789012345678 9
123456789012345678901234567890121234567890123456789012345678
2x 9
123456789012345678901234567890121234567890123456789012345678
Z 5 3 and Z 5 , then you substitute the value of 3 for Z in the 9
123456789012345678901234567890121234567890123456789012345678
5
23456789012345678901234567890121234567890123456789012345678
9
9
123456789012345678901234567890121234567890123456789012345678
123456789012345678901234567890121234567890123456789012345678
second equation to get: 9
1 9
123456789012345678901234567890121234567890123456789012345678 9
123456789012345678901234567890121234567890123456789012345678
2x 9
23456789012345678901234567890121234567890123456789012345678
123456789012345678901234567890121234567890123456789012345678
Z 5 9
123456789012345678901234567890121234567890123456789012345678
5 9
1 9
123456789012345678901234567890121234567890123456789012345678
2x 9
123456789012345678901234567890121234567890123456789012345678
35 9
23456789012345678901234567890121234567890123456789012345678
123456789012345678901234567890121234567890123456789012345678
5 9
123456789012345678901234567890121234567890123456789012345678 9
123456789012345678901234567890121234567890123456789012345678 9
123456789012345678901234567890121234567890123456789012345678
2x 9
1 ~ 5! 3 5 ~ 5 ! 9
123456789012345678901234567890121234567890123456789012345678 99
23456789012345678901234567890121234567890123456789012345678
5
123456789012345678901234567890121234567890123456789012345678 9
123456789012345678901234567890121234567890123456789012345678
15 5 2x
123456789012345678901234567890121234567890123456789012345678 99
1 15 2x
123456789012345678901234567890121234567890123456789012345678 9
123456789012345678901234567890121234567890123456789012345678
2
5
2 9
123456789012345678901234567890121234567890123456789012345678 9
123456789012345678901234567890121234567890123456789012345678 9
123456789012345678901234567890121234567890123456789012345678
15 9
23456789012345678901234567890121234567890123456789012345678
123456789012345678901234567890121234567890123456789012345678
5 x 9
1 2 9
123456789012345678901234567890121234567890123456789012345678 9
123456789012345678901234567890121234567890123456789012345678 9
123456789012345678901234567890121234567890123456789012345678
(E) is the answer.
23456789012345678901234567890121234567890123456789012345678
9
9
1
123456789012345678901234567890121234567890123456789012345678 9
123456789012345678901234567890121234567890123456789012345678 9
123456789012345678901234567890121234567890123456789012345678 9
1234567890123456789012345678901212345678901234567890123456789
Copyright © 2005 Thomson Peterson’s, a part of The Thomson Corporaton 67
SAT is a registered trademark of the College Entrance Examination Board, which
was not involved in the production of and does not endorse this product.
1234567890123456789012345678901212345678901234567890123456789
123456789012345678901234567890121234567890123456789012345678 9
123456789012345678901234567890121234567890123456789012345678 9
123456789012345678901234567890121234567890123456789012345678 9
123456789012345678901234567890121234567890123456789012345678 9
123456789012345678901234567890121234567890123456789012345678
3. A There are two prime ways to approach this problem. You can do a 9
123456789012345678901234567890121234567890123456789012345678 9
123456789012345678901234567890121234567890123456789012345678
factor tree for both numbers, find
23456789012345678901234567890121234567890123456789012345678
the common factors, and then look 9
9
1
123456789012345678901234567890121234567890123456789012345678
down the answer list for the greatest one. If you don’t know what a 9
123456789012345678901234567890121234567890123456789012345678 9
123456789012345678901234567890121234567890123456789012345678
“factor tree” is, you could go through the answer choices one by one to 9
123456789012345678901234567890121234567890123456789012345678
23456789012345678901234567890121234567890123456789012345678
9
9
1 see which is the greatest that divides 32 and 42. Start with choice (E),
123456789012345678901234567890121234567890123456789012345678 9
123456789012345678901234567890121234567890123456789012345678
since it’s the greatest number, and if it works, it’s the answer. 12 doesn’t 9
123456789012345678901234567890121234567890123456789012345678 9
123456789012345678901234567890121234567890123456789012345678
work, so try the next greatest one,
23456789012345678901234567890121234567890123456789012345678
8, choice (D). Eight doesn’t work, 9
9
1 and neither does 6 or 3. This leaves (A).
123456789012345678901234567890121234567890123456789012345678 9
123456789012345678901234567890121234567890123456789012345678 9
123456789012345678901234567890121234567890123456789012345678
4. B If you understand the idea of slope, the answer will fall into your lap. If 9
123456789012345678901234567890121234567890123456789012345678
23456789012345678901234567890121234567890123456789012345678
9
9
1 you have trouble with this problem, review pages 350–352 in the Math
123456789012345678901234567890121234567890123456789012345678 9
123456789012345678901234567890121234567890123456789012345678
section to make sure you are perfectly comfortable with slope and 9
123456789012345678901234567890121234567890123456789012345678 9
123456789012345678901234567890121234567890123456789012345678
linear equations in the form y 5 mx 1b. 9
123456789012345678901234567890121234567890123456789012345678 9
23456789012345678901234567890121234567890123456789012345678
123456789012345678901234567890121234567890123456789012345678 9
123456789012345678901234567890121234567890123456789012345678
Viewed from left to right, the line descends, which means the slope is 9
123456789012345678901234567890121234567890123456789012345678 9
123456789012345678901234567890121234567890123456789012345678 9
negative.
123456789012345678901234567890121234567890123456789012345678 9
1 That eliminates choices (C), (D), and (E). A slope is “rise over run,” 9
123456789012345678901234567890121234567890123456789012345678 9
123456789012345678901234567890121234567890123456789012345678
meaning you look at the change in y-values as the numerator of the 9
123456789012345678901234567890121234567890123456789012345678 9
123456789012345678901234567890121234567890123456789012345678 9
23456789012345678901234567890121234567890123456789012345678
fraction, while the change in x-values is the denominator. The line
123456789012345678901234567890121234567890123456789012345678 9
123456789012345678901234567890121234567890123456789012345678
drops two points along the y-axis for every three points it moves over 9
123456789012345678901234567890121234567890123456789012345678
2 9
123456789012345678901234567890121234567890123456789012345678 9
on the x-axis, so the slope is , choice
123456789012345678901234567890121234567890123456789012345678
2 (B). 9
123456789012345678901234567890121234567890123456789012345678
3 9
123456789012345678901234567890121234567890123456789012345678 9
123456789012345678901234567890121234567890123456789012345678
5. E You need to know the total number of students in each grade to answer 9
1 9
23456789012345678901234567890121234567890123456789012345678
123456789012345678901234567890121234567890123456789012345678 9
th
this question. The table tells you there are 32 in the 4 grade, so you
123456789012345678901234567890121234567890123456789012345678 rd 9
1 only need to determine the total in the 3 grade. There are 16 boys and 9
123456789012345678901234567890121234567890123456789012345678
rd 9
123456789012345678901234567890121234567890123456789012345678
14 girls in the 3 grade, which makes
23456789012345678901234567890121234567890123456789012345678
30 total. The overall total is 32 9
9
123456789012345678901234567890121234567890123456789012345678
123456789012345678901234567890121234567890123456789012345678
plus 30, which is 62, choice (E). 9
1 9
123456789012345678901234567890121234567890123456789012345678 9
123456789012345678901234567890121234567890123456789012345678 9
23456789012345678901234567890121234567890123456789012345678
123456789012345678901234567890121234567890123456789012345678 9
Note

The table is incomplete, but don’t let this worry you. The SAT will
123456789012345678901234567890121234567890123456789012345678 9
1 always provide the information needed to answer the question, one 9
123456789012345678901234567890121234567890123456789012345678 9
123456789012345678901234567890121234567890123456789012345678 9
way or another.
123456789012345678901234567890121234567890123456789012345678 9
23456789012345678901234567890121234567890123456789012345678
123456789012345678901234567890121234567890123456789012345678 9 9
123456789012345678901234567890121234567890123456789012345678 9
123456789012345678901234567890121234567890123456789012345678
th th
9
1 6. D If there are 18 girls in the 4 grade, then there are 14 boys in 4 grade
23456789012345678901234567890121234567890123456789012345678
123456789012345678901234567890121234567890123456789012345678 rd 9
123456789012345678901234567890121234567890123456789012345678
(32 2 18 5 14). The table says there are 16 boys in 3 grade, so there 9
123456789012345678901234567890121234567890123456789012345678 rd
are a total of 30 (16 1 14 5 30) in the 3 and 4 grade. Choice (D) is th 9
123456789012345678901234567890121234567890123456789012345678 9 9
1 the answer.
123456789012345678901234567890121234567890123456789012345678 9
123456789012345678901234567890121234567890123456789012345678 9
123456789012345678901234567890121234567890123456789012345678 9
123456789012345678901234567890121234567890123456789012345678 9
123456789012345678901234567890121234567890123456789012345678 9
123456789012345678901234567890121234567890123456789012345678 9
23456789012345678901234567890121234567890123456789012345678
9
1
123456789012345678901234567890121234567890123456789012345678 9
123456789012345678901234567890121234567890123456789012345678 9
123456789012345678901234567890121234567890123456789012345678 9
123456789012345678901234567890121234567890123456789012345678 9
123456789012345678901234567890121234567890123456789012345678 9
123456789012345678901234567890121234567890123456789012345678 9
123456789012345678901234567890121234567890123456789012345678 9
1234567890123456789012345678901212345678901234567890123456789
68 Copyright © 2005 Thomson Peterson’s, a part of The Thomson Corporaton
SAT is a registered trademark of the College Entrance Examination Board, which
was not involved in the production of and does not endorse this product.
1234567890123456789012345678901212345678901234567890123456789
123456789012345678901234567890121234567890123456789012345678 9
123456789012345678901234567890121234567890123456789012345678 9
123456789012345678901234567890121234567890123456789012345678 9
123456789012345678901234567890121234567890123456789012345678 9
123456789012345678901234567890121234567890123456789012345678
7. A Careful here. The problem asks for the value of the exponent, not the 9
123456789012345678901234567890121234567890123456789012345678 9
123456789012345678901234567890121234567890123456789012345678
simplified expression. Simplifying this expression
23456789012345678901234567890121234567890123456789012345678
looks a little wiggy, 9
9
1
123456789012345678901234567890121234567890123456789012345678
but you need to be prepared to work with wacky-looking exponents 9
123456789012345678901234567890121234567890123456789012345678 9
123456789012345678901234567890121234567890123456789012345678
problems like this: 9
123456789012345678901234567890121234567890123456789012345678
23456789012345678901234567890121234567890123456789012345678
9
9
1
123456789012345678901234567890121234567890123456789012345678 9
3 22
~ 4!
123456789012345678901234567890121234567890123456789012345678
m 1 1 1 1 24
11
9
123456789012345678901234567890121234567890123456789012345678
5 5 5 3 2 5 5 3 3 5 51313 5 11 5 m 9
123456789012345678901234567890121234567890123456789012345678
m m 4~m4! m 4m4m4 m
23456789012345678901234567890121234567890123456789012345678 m 9
9
1 4 4 4
123456789012345678901234567890121234567890123456789012345678 9
123456789012345678901234567890121234567890123456789012345678
11 9
123456789012345678901234567890121234567890123456789012345678
The exponent is 2 , which is choice (A). If this makes no sense to you, 9
123456789012345678901234567890121234567890123456789012345678
4
23456789012345678901234567890121234567890123456789012345678
9
9
1
123456789012345678901234567890121234567890123456789012345678
you’ll want to review the rules of multiplying and dividing exponents. 9
123456789012345678901234567890121234567890123456789012345678 9
123456789012345678901234567890121234567890123456789012345678 9
123456789012345678901234567890121234567890123456789012345678
8. C You can’t solve this problem without a little help from the algebraic 9
23456789012345678901234567890121234567890123456789012345678
123456789012345678901234567890121234567890123456789012345678 9
123456789012345678901234567890121234567890123456789012345678
expressions. You know that the two expressions sum to 90 (because the 9
123456789012345678901234567890121234567890123456789012345678
measures of the interior angles of a triangle sum to 180), but there are 9
123456789012345678901234567890121234567890123456789012345678 99
123456789012345678901234567890121234567890123456789012345678
two variables and only one equation: 2x 1 y 1 3x 2 y 5 90 (They
123456789012345678901234567890121234567890123456789012345678 9
1 equal 90 since the right angle takes up the other 90°.) 9
123456789012345678901234567890121234567890123456789012345678 9
123456789012345678901234567890121234567890123456789012345678 9
123456789012345678901234567890121234567890123456789012345678
If the y variables didn’t subtract out when you added the two 9
23456789012345678901234567890121234567890123456789012345678
123456789012345678901234567890121234567890123456789012345678 9
123456789012345678901234567890121234567890123456789012345678
expressions, you could not solve for x. The whole point to this problem 9
123456789012345678901234567890121234567890123456789012345678 9
123456789012345678901234567890121234567890123456789012345678 99
is to brain freeze students who look at it and say, “There are two
123456789012345678901234567890121234567890123456789012345678
123456789012345678901234567890121234567890123456789012345678
variables and only one equation. It can’t be solved!” If you just write 9
123456789012345678901234567890121234567890123456789012345678 9
123456789012345678901234567890121234567890123456789012345678 99
out the equation, you can avoid this type of paralysis. Writing down
123456789012345678901234567890121234567890123456789012345678 9
1 your work saves the day!
123456789012345678901234567890121234567890123456789012345678 9
123456789012345678901234567890121234567890123456789012345678 9
123456789012345678901234567890121234567890123456789012345678
2x 1 y 1 3x 2 y 5 90 9
123456789012345678901234567890121234567890123456789012345678 9
123456789012345678901234567890121234567890123456789012345678
5x 5 90 9
123456789012345678901234567890121234567890123456789012345678 9
123456789012345678901234567890121234567890123456789012345678 9
123456789012345678901234567890121234567890123456789012345678
5x 90 9
123456789012345678901234567890121234567890123456789012345678
5 9
123456789012345678901234567890121234567890123456789012345678
5 5 9
23456789012345678901234567890121234567890123456789012345678
123456789012345678901234567890121234567890123456789012345678 9
123456789012345678901234567890121234567890123456789012345678
x 5 18 9
1 9
123456789012345678901234567890121234567890123456789012345678 9
123456789012345678901234567890121234567890123456789012345678 9
(C) is the answer.
123456789012345678901234567890121234567890123456789012345678 99
23456789012345678901234567890121234567890123456789012345678
123456789012345678901234567890121234567890123456789012345678 9
123456789012345678901234567890121234567890123456789012345678 9
123456789012345678901234567890121234567890123456789012345678 9
1
123456789012345678901234567890121234567890123456789012345678 9
123456789012345678901234567890121234567890123456789012345678 9
123456789012345678901234567890121234567890123456789012345678 9
123456789012345678901234567890121234567890123456789012345678 9
123456789012345678901234567890121234567890123456789012345678 9
123456789012345678901234567890121234567890123456789012345678 99
23456789012345678901234567890121234567890123456789012345678
123456789012345678901234567890121234567890123456789012345678 9
123456789012345678901234567890121234567890123456789012345678 9
123456789012345678901234567890121234567890123456789012345678 9
1
123456789012345678901234567890121234567890123456789012345678 9
123456789012345678901234567890121234567890123456789012345678 9
123456789012345678901234567890121234567890123456789012345678 9
123456789012345678901234567890121234567890123456789012345678 9
123456789012345678901234567890121234567890123456789012345678 9
123456789012345678901234567890121234567890123456789012345678 9
123456789012345678901234567890121234567890123456789012345678 9
123456789012345678901234567890121234567890123456789012345678 9
123456789012345678901234567890121234567890123456789012345678 9
1234567890123456789012345678901212345678901234567890123456789
Copyright © 2005 Thomson Peterson’s, a part of The Thomson Corporaton 69
SAT is a registered trademark of the College Entrance Examination Board, which
was not involved in the production of and does not endorse this product.
1234567890123456789012345678901212345678901234567890123456789
123456789012345678901234567890121234567890123456789012345678 9
123456789012345678901234567890121234567890123456789012345678 9
123456789012345678901234567890121234567890123456789012345678 9
123456789012345678901234567890121234567890123456789012345678 9
123456789012345678901234567890121234567890123456789012345678
9. E For word problems, always translate the English into algebra. Call the 9
123456789012345678901234567890121234567890123456789012345678 9
123456789012345678901234567890121234567890123456789012345678
distance from Easton to Bethsaida via
23456789012345678901234567890121234567890123456789012345678
highway x. Traveling this route is 9
9
1
123456789012345678901234567890121234567890123456789012345678
7 miles longer than going on surface streets, so going via surface streets 9
123456789012345678901234567890121234567890123456789012345678 9
123456789012345678901234567890121234567890123456789012345678
is x 2 7. Traveling both routes is 31 miles, which translates to: 9
123456789012345678901234567890121234567890123456789012345678
23456789012345678901234567890121234567890123456789012345678
9
9
1 x 1 (x 2 7) 5 31. Once you have an equation, you can solve for x:
123456789012345678901234567890121234567890123456789012345678 9
123456789012345678901234567890121234567890123456789012345678 9
123456789012345678901234567890121234567890123456789012345678
x 1 x 2 7 5 31 9
123456789012345678901234567890121234567890123456789012345678
23456789012345678901234567890121234567890123456789012345678
9
9
1 2x 2 7 5 31
123456789012345678901234567890121234567890123456789012345678 9
123456789012345678901234567890121234567890123456789012345678
2x 2 7 1 7 5 31 1 7 9
123456789012345678901234567890121234567890123456789012345678 9
123456789012345678901234567890121234567890123456789012345678
23456789012345678901234567890121234567890123456789012345678
9
9
1 2x 5 38
123456789012345678901234567890121234567890123456789012345678 9
123456789012345678901234567890121234567890123456789012345678
2x 38 9
123456789012345678901234567890121234567890123456789012345678
5 9
123456789012345678901234567890121234567890123456789012345678
2 2 9
123456789012345678901234567890121234567890123456789012345678 9
23456789012345678901234567890121234567890123456789012345678
123456789012345678901234567890121234567890123456789012345678
x59 9
123456789012345678901234567890121234567890123456789012345678 9
123456789012345678901234567890121234567890123456789012345678 9
123456789012345678901234567890121234567890123456789012345678
Here you might to double check that x is the highway route distance 9
123456789012345678901234567890121234567890123456789012345678 9
1 and not the surface street distance. Choice (E) is the answer. 9
123456789012345678901234567890121234567890123456789012345678 9
123456789012345678901234567890121234567890123456789012345678 9
123456789012345678901234567890121234567890123456789012345678
10. E A function is undefined at a certain value, if at that value the function 9
23456789012345678901234567890121234567890123456789012345678
123456789012345678901234567890121234567890123456789012345678 9
123456789012345678901234567890121234567890123456789012345678
does not make any sense. For this function, the most probable way that 9
123456789012345678901234567890121234567890123456789012345678 9
123456789012345678901234567890121234567890123456789012345678 9
it could be undefined is if the denominator is zero.
123456789012345678901234567890121234567890123456789012345678 9
123456789012345678901234567890121234567890123456789012345678
When is the denominator zero? Solve it like a quadratic to see: 9
123456789012345678901234567890121234567890123456789012345678 9
123456789012345678901234567890121234567890123456789012345678 9
123456789012345678901234567890121234567890123456789012345678
2 9
1 x 1 3x 2 18 5 0 9
23456789012345678901234567890121234567890123456789012345678
123456789012345678901234567890121234567890123456789012345678 9
123456789012345678901234567890121234567890123456789012345678
~x 1 6!~x 2 3! 5 0 9
1 9
123456789012345678901234567890121234567890123456789012345678 9
123456789012345678901234567890121234567890123456789012345678
The denominator is zero if x equals 26
23456789012345678901234567890121234567890123456789012345678
or 3, which is choice (E). 9
9
123456789012345678901234567890121234567890123456789012345678
123456789012345678901234567890121234567890123456789012345678 2
11. D The graph is a parabola, so it will have an x term in it. Sometimes that 9
1 9
123456789012345678901234567890121234567890123456789012345678
will help you cross out an answer choice or two, but on this question all 9
123456789012345678901234567890121234567890123456789012345678 9
23456789012345678901234567890121234567890123456789012345678
123456789012345678901234567890121234567890123456789012345678
2
answer choices have an x in them. Even so, it was a good technique; it 9
123456789012345678901234567890121234567890123456789012345678 9
1 didn’t work on this problem, but it will work on others. 9
123456789012345678901234567890121234567890123456789012345678 9
123456789012345678901234567890121234567890123456789012345678 9
123456789012345678901234567890121234567890123456789012345678
The parabola is facing downwards, so the x2 term must have a negative 9
123456789012345678901234567890121234567890123456789012345678 9
123456789012345678901234567890121234567890123456789012345678
in front of it. If you don’t see why, plug some numbers into 2x . That 2
9
123456789012345678901234567890121234567890123456789012345678 9
123456789012345678901234567890121234567890123456789012345678 9
eliminates (A) and (B).
123456789012345678901234567890121234567890123456789012345678 9
23456789012345678901234567890121234567890123456789012345678
9
123456789012345678901234567890121234567890123456789012345678 9
123456789012345678901234567890121234567890123456789012345678
Our last clue will come from where the figure crosses the y-axis. At this
9
123456789012345678901234567890121234567890123456789012345678 2
9
1 point, x must equal zero, which means that 2x will also be zero. When
23456789012345678901234567890121234567890123456789012345678
123456789012345678901234567890121234567890123456789012345678 9
123456789012345678901234567890121234567890123456789012345678
x equals zero on the graph, the value of f(x is 22. So you are looking 9
123456789012345678901234567890121234567890123456789012345678
for an answer choice that has both 2
and Although it’s hiding a 9
123456789012345678901234567890121234567890123456789012345678
2x 22. 9
1 2
bit inside some parentheses, (D) is the answer since 2(x 1 2) 5 2x 2 2. 2 9
123456789012345678901234567890121234567890123456789012345678 9
23456789012345678901234567890121234567890123456789012345678
9
1
123456789012345678901234567890121234567890123456789012345678 9
123456789012345678901234567890121234567890123456789012345678 9
123456789012345678901234567890121234567890123456789012345678 9
123456789012345678901234567890121234567890123456789012345678 9
123456789012345678901234567890121234567890123456789012345678 9
123456789012345678901234567890121234567890123456789012345678 9
123456789012345678901234567890121234567890123456789012345678 9
1234567890123456789012345678901212345678901234567890123456789
70 Copyright © 2005 Thomson Peterson’s, a part of The Thomson Corporaton
SAT is a registered trademark of the College Entrance Examination Board, which
was not involved in the production of and does not endorse this product.
1234567890123456789012345678901212345678901234567890123456789
123456789012345678901234567890121234567890123456789012345678 9
123456789012345678901234567890121234567890123456789012345678 9
123456789012345678901234567890121234567890123456789012345678 9
123456789012345678901234567890121234567890123456789012345678 9
123456789012345678901234567890121234567890123456789012345678
12. D With congruent triangles, corresponding angles and sides are congru- 9
123456789012345678901234567890121234567890123456789012345678 9
123456789012345678901234567890121234567890123456789012345678
ent. Both are right triangles, so you
23456789012345678901234567890121234567890123456789012345678
should smell the Pythagorean 9
9
1
123456789012345678901234567890121234567890123456789012345678
theorem wafting about this problem as a method of solving for y. 9
123456789012345678901234567890121234567890123456789012345678 9
123456789012345678901234567890121234567890123456789012345678 9
123456789012345678901234567890121234567890123456789012345678 9
2 2 2
a 1 b 5 c
123456789012345678901234567890121234567890123456789012345678 9
123456789012345678901234567890121234567890123456789012345678
y2 1 52 5 102 9
123456789012345678901234567890121234567890123456789012345678 9
123456789012345678901234567890121234567890123456789012345678
y2
25 100 9
123456789012345678901234567890121234567890123456789012345678
1 5
23456789012345678901234567890121234567890123456789012345678
9
9
1 2
123456789012345678901234567890121234567890123456789012345678
y 1 25 2 25 5 100 2 25 9
123456789012345678901234567890121234567890123456789012345678 9
123456789012345678901234567890121234567890123456789012345678
2
y 5 75 9
123456789012345678901234567890121234567890123456789012345678
23456789012345678901234567890121234567890123456789012345678
9
9
1 y 5 =75
123456789012345678901234567890121234567890123456789012345678 9
123456789012345678901234567890121234567890123456789012345678 9
123456789012345678901234567890121234567890123456789012345678 9
123456789012345678901234567890121234567890123456789012345678
You might also have noticed that both figures are 30-60-90 triangles 9
23456789012345678901234567890121234567890123456789012345678
123456789012345678901234567890121234567890123456789012345678
(you can infer this from the relationship between the hypotenuse and 9
123456789012345678901234567890121234567890123456789012345678 9
123456789012345678901234567890121234567890123456789012345678 = 9
123456789012345678901234567890121234567890123456789012345678 99
the smallest side of the triangle on the left). This means y 5 5 3
123456789012345678901234567890121234567890123456789012345678
123456789012345678901234567890121234567890123456789012345678
which brings you once again to =75. Either method, choice (D) is 9
1 9
123456789012345678901234567890121234567890123456789012345678
correct. 9
123456789012345678901234567890121234567890123456789012345678 9
123456789012345678901234567890121234567890123456789012345678 9
123456789012345678901234567890121234567890123456789012345678 99
23456789012345678901234567890121234567890123456789012345678
13. A Plug in the answer choices and see which ones make the equation true.
123456789012345678901234567890121234567890123456789012345678
123456789012345678901234567890121234567890123456789012345678
Many of the numbers repeat and the equation isn’t that involved, so 9
123456789012345678901234567890121234567890123456789012345678 9
123456789012345678901234567890121234567890123456789012345678 99
this doesn’t take as long as you might think. 1 works, but 21 doesn’t. If
123456789012345678901234567890121234567890123456789012345678
123456789012345678901234567890121234567890123456789012345678
you try all the answer choices, you will see that only 1 works, which 9
123456789012345678901234567890121234567890123456789012345678 9
123456789012345678901234567890121234567890123456789012345678 99
makes (A) the answer.
1
123456789012345678901234567890121234567890123456789012345678
14. B There are multiple ways to approach this problem. Possibly the 9
123456789012345678901234567890121234567890123456789012345678 9
123456789012345678901234567890121234567890123456789012345678
quickest is to count up the multiples of three that are even and 50 or less 9
123456789012345678901234567890121234567890123456789012345678 9
123456789012345678901234567890121234567890123456789012345678
(if they are even, they will be divisible by 2). This list starts 6, 12, 18, 24 9
123456789012345678901234567890121234567890123456789012345678 9
123456789012345678901234567890121234567890123456789012345678
... 9
123456789012345678901234567890121234567890123456789012345678 9
123456789012345678901234567890121234567890123456789012345678 9
123456789012345678901234567890121234567890123456789012345678
You might stop here and realize that the members of set Z are all 9
23456789012345678901234567890121234567890123456789012345678
123456789012345678901234567890121234567890123456789012345678 9
123456789012345678901234567890121234567890123456789012345678
multiples of 6. Even if you don’t, you’ll continue with: 30, 36, 42, and 9
1 9
123456789012345678901234567890121234567890123456789012345678 9
48. 54 is the next item, but it’s too great, so there are eight members of
123456789012345678901234567890121234567890123456789012345678 9
123456789012345678901234567890121234567890123456789012345678
set Z. (B) is the answer. 9
123456789012345678901234567890121234567890123456789012345678 9
123456789012345678901234567890121234567890123456789012345678 9
123456789012345678901234567890121234567890123456789012345678
15. D With similar figures, the ratios of each pair of corresponding sides must 9
123456789012345678901234567890121234567890123456789012345678 9
123456789012345678901234567890121234567890123456789012345678 99
23456789012345678901234567890121234567890123456789012345678
be equal. The tricky part to this problem is the bent-leg formation of the
123456789012345678901234567890121234567890123456789012345678
given figure. The L-shape could roughly be described as, “three squares 9
123456789012345678901234567890121234567890123456789012345678 9
123456789012345678901234567890121234567890123456789012345678
up, then one to the right.” Choice (A) is too thin, i.e. there was no
9
1
23456789012345678901234567890121234567890123456789012345678
123456789012345678901234567890121234567890123456789012345678
corresponding increase in width even though there is an increase in 9
123456789012345678901234567890121234567890123456789012345678 9
123456789012345678901234567890121234567890123456789012345678
length. With (B), the small part of L seems to be too big, and with (C) 9
123456789012345678901234567890121234567890123456789012345678 99
1 the big part of the L is too large in proportion to the skinny part.
123456789012345678901234567890121234567890123456789012345678 9
123456789012345678901234567890121234567890123456789012345678
Now look at choice (D). The fact that the figure has been rotated makes 9
123456789012345678901234567890121234567890123456789012345678 9
123456789012345678901234567890121234567890123456789012345678
no difference with similarity. If you draw the following dashed lines on 9
123456789012345678901234567890121234567890123456789012345678
23456789012345678901234567890121234567890123456789012345678
9
9
1 choice (D), you’ll see why it’s the right answer.
123456789012345678901234567890121234567890123456789012345678 9
123456789012345678901234567890121234567890123456789012345678 9
123456789012345678901234567890121234567890123456789012345678 9
1234567890123456789012345678901212345678901234567890123456789
Copyright © 2005 Thomson Peterson’s, a part of The Thomson Corporaton 71
SAT is a registered trademark of the College Entrance Examination Board, which
was not involved in the production of and does not endorse this product.
1234567890123456789012345678901212345678901234567890123456789
123456789012345678901234567890121234567890123456789012345678 9
123456789012345678901234567890121234567890123456789012345678 9
123456789012345678901234567890121234567890123456789012345678 9
123456789012345678901234567890121234567890123456789012345678 9
123456789012345678901234567890121234567890123456789012345678 9
123456789012345678901234567890121234567890123456789012345678 9
123456789012345678901234567890121234567890123456789012345678 9
123456789012345678901234567890121234567890123456789012345678 9
123456789012345678901234567890121234567890123456789012345678 9
123456789012345678901234567890121234567890123456789012345678 9
123456789012345678901234567890121234567890123456789012345678 9
123456789012345678901234567890121234567890123456789012345678
23456789012345678901234567890121234567890123456789012345678
9
9
1
123456789012345678901234567890121234567890123456789012345678 9
123456789012345678901234567890121234567890123456789012345678 9
123456789012345678901234567890121234567890123456789012345678 9
123456789012345678901234567890121234567890123456789012345678
23456789012345678901234567890121234567890123456789012345678
9
9
1
123456789012345678901234567890121234567890123456789012345678 9
123456789012345678901234567890121234567890123456789012345678 9
123456789012345678901234567890121234567890123456789012345678 9
123456789012345678901234567890121234567890123456789012345678
23456789012345678901234567890121234567890123456789012345678
9
9
1
123456789012345678901234567890121234567890123456789012345678 9
123456789012345678901234567890121234567890123456789012345678
Here you have the same figure, “three squares, then one to the right.” 9
123456789012345678901234567890121234567890123456789012345678 9
123456789012345678901234567890121234567890123456789012345678
The squares are much larger, but they are in the same corresponding 9
23456789012345678901234567890121234567890123456789012345678
123456789012345678901234567890121234567890123456789012345678 9
123456789012345678901234567890121234567890123456789012345678
proportion. Choice (D) is correct. 9
123456789012345678901234567890121234567890123456789012345678 9
123456789012345678901234567890121234567890123456789012345678 9
123456789012345678901234567890121234567890123456789012345678
16. C The figure is a square since the side lengths are all equal and all the 9
123456789012345678901234567890121234567890123456789012345678 9
1 angles are ninety degrees (if you don’t see this, rotate the figure ninety 9
123456789012345678901234567890121234567890123456789012345678 9
123456789012345678901234567890121234567890123456789012345678
degrees). To find the side length, it’s Pythagoras time, but you get a 9
123456789012345678901234567890121234567890123456789012345678
shortcut since you have a special triangle. Each side length is the 9
123456789012345678901234567890121234567890123456789012345678 9
23456789012345678901234567890121234567890123456789012345678
123456789012345678901234567890121234567890123456789012345678
hypotenuse of a 45-45-90 triangle whose equal sides are of length one. 9
123456789012345678901234567890121234567890123456789012345678 9
123456789012345678901234567890121234567890123456789012345678 = 9
123456789012345678901234567890121234567890123456789012345678 9
So the hypotenuse/side length of the square is 2. Put this into the
123456789012345678901234567890121234567890123456789012345678 9
123456789012345678901234567890121234567890123456789012345678
area of a square formula and you’ll find answer (C): 9
123456789012345678901234567890121234567890123456789012345678 9
123456789012345678901234567890121234567890123456789012345678
A5s 2
9
1 9
23456789012345678901234567890121234567890123456789012345678
123456789012345678901234567890121234567890123456789012345678
A 5 ~=2!
2
9
123456789012345678901234567890121234567890123456789012345678 9
1 9
123456789012345678901234567890121234567890123456789012345678
A52 9
123456789012345678901234567890121234567890123456789012345678
23456789012345678901234567890121234567890123456789012345678
9
9
123456789012345678901234567890121234567890123456789012345678
123456789012345678901234567890121234567890123456789012345678
17. B First, make sure you understand what all those words mean. You have 9
1 9
123456789012345678901234567890121234567890123456789012345678
eight numbers that are integers, and are also even, and when you add 9
123456789012345678901234567890121234567890123456789012345678 9
123456789012345678901234567890121234567890123456789012345678 9
23456789012345678901234567890121234567890123456789012345678
them up, they sum to 50. Furthermore, only two of the eight integers
123456789012345678901234567890121234567890123456789012345678 9
1 can be the same value. In other words, there are never three integers 9
123456789012345678901234567890121234567890123456789012345678 9
123456789012345678901234567890121234567890123456789012345678 9
that are the same.
123456789012345678901234567890121234567890123456789012345678 9
23456789012345678901234567890121234567890123456789012345678
9
123456789012345678901234567890121234567890123456789012345678
To find the greatest possible integer in the set, first make all the other
9
123456789012345678901234567890121234567890123456789012345678 9
123456789012345678901234567890121234567890123456789012345678
integers as least as possible. The set could be 2, 2, 4, 4, 6, 6, 8, x. This
9
1
23456789012345678901234567890121234567890123456789012345678
123456789012345678901234567890121234567890123456789012345678
would maximize x. Now solve: 9
123456789012345678901234567890121234567890123456789012345678 9 9
123456789012345678901234567890121234567890123456789012345678 9
123456789012345678901234567890121234567890123456789012345678
2 1 2 1 4 1 4 1 6 1 6 1 8 1 x 5 50
9
1
123456789012345678901234567890121234567890123456789012345678 9
23456789012345678901234567890121234567890123456789012345678
32 1 x 5 50
123456789012345678901234567890121234567890123456789012345678 9 9
123456789012345678901234567890121234567890123456789012345678
32 2 32 1 x 5 50 2 32
9
123456789012345678901234567890121234567890123456789012345678 9
1 x 5 18
23456789012345678901234567890121234567890123456789012345678
123456789012345678901234567890121234567890123456789012345678 9
1 This is choice (B). 9
123456789012345678901234567890121234567890123456789012345678 9
123456789012345678901234567890121234567890123456789012345678
You can also start with the greatet numerical answer choice and then 9
123456789012345678901234567890121234567890123456789012345678
23456789012345678901234567890121234567890123456789012345678
9
9
1 see if it works, but this will take more time than setting up a formula.
123456789012345678901234567890121234567890123456789012345678 9
123456789012345678901234567890121234567890123456789012345678 9
123456789012345678901234567890121234567890123456789012345678 9
1234567890123456789012345678901212345678901234567890123456789
72 Copyright © 2005 Thomson Peterson’s, a part of The Thomson Corporaton
SAT is a registered trademark of the College Entrance Examination Board, which
was not involved in the production of and does not endorse this product.
1234567890123456789012345678901212345678901234567890123456789
123456789012345678901234567890121234567890123456789012345678 9
123456789012345678901234567890121234567890123456789012345678 9
123456789012345678901234567890121234567890123456789012345678 9
123456789012345678901234567890121234567890123456789012345678 9
123456789012345678901234567890121234567890123456789012345678
18. E Time to sketch! Here’s one where two of the triangle’s vertices coincide 9
123456789012345678901234567890121234567890123456789012345678 9
123456789012345678901234567890121234567890123456789012345678
with two of the square’s corners.
23456789012345678901234567890121234567890123456789012345678
9
9
1
123456789012345678901234567890121234567890123456789012345678 9
123456789012345678901234567890121234567890123456789012345678 9
123456789012345678901234567890121234567890123456789012345678 9
123456789012345678901234567890121234567890123456789012345678
23456789012345678901234567890121234567890123456789012345678
9
9
1
123456789012345678901234567890121234567890123456789012345678 9
123456789012345678901234567890121234567890123456789012345678 9
123456789012345678901234567890121234567890123456789012345678 9
123456789012345678901234567890121234567890123456789012345678
23456789012345678901234567890121234567890123456789012345678
9
9
1
123456789012345678901234567890121234567890123456789012345678 9
123456789012345678901234567890121234567890123456789012345678 9
123456789012345678901234567890121234567890123456789012345678 9
123456789012345678901234567890121234567890123456789012345678
23456789012345678901234567890121234567890123456789012345678
9
9
1
123456789012345678901234567890121234567890123456789012345678 9
123456789012345678901234567890121234567890123456789012345678 9
123456789012345678901234567890121234567890123456789012345678 9
123456789012345678901234567890121234567890123456789012345678 9
123456789012345678901234567890121234567890123456789012345678 99
23456789012345678901234567890121234567890123456789012345678
123456789012345678901234567890121234567890123456789012345678 9
123456789012345678901234567890121234567890123456789012345678 9
123456789012345678901234567890121234567890123456789012345678
123456789012345678901234567890121234567890123456789012345678
You could also have AB run along DE. In fact, using AB you could put 9
123456789012345678901234567890121234567890123456789012345678 9
1 the triangle in the square four different ways (along DF, DE, EG, GF). 9
123456789012345678901234567890121234567890123456789012345678 9
123456789012345678901234567890121234567890123456789012345678
Using BC, you could put the triangle in the square four different ways 9
123456789012345678901234567890121234567890123456789012345678 9
23456789012345678901234567890121234567890123456789012345678
123456789012345678901234567890121234567890123456789012345678
also. The same is true for AC. That is a total of 12 different ways that 9
123456789012345678901234567890121234567890123456789012345678 9
123456789012345678901234567890121234567890123456789012345678
the triangle could be placed in the square such that two angles of the 9
123456789012345678901234567890121234567890123456789012345678 99
123456789012345678901234567890121234567890123456789012345678
triangle coincided with two corners of the square. It means that (E) is
123456789012345678901234567890121234567890123456789012345678 9
123456789012345678901234567890121234567890123456789012345678
the answer. 9
123456789012345678901234567890121234567890123456789012345678 99
123456789012345678901234567890121234567890123456789012345678
1 19. D You only know the distances between these points. You don’t know 9
123456789012345678901234567890121234567890123456789012345678 9
123456789012345678901234567890121234567890123456789012345678
their orientation in relation to each other. If T is on a line between 9
123456789012345678901234567890121234567890123456789012345678 9
123456789012345678901234567890121234567890123456789012345678 9
G and S, then G and T are 4 away from each other (9 2 5 5 4). That is
123456789012345678901234567890121234567890123456789012345678 9
123456789012345678901234567890121234567890123456789012345678
the closest that they could be to each other. But if T is on a line with 9
123456789012345678901234567890121234567890123456789012345678 9
123456789012345678901234567890121234567890123456789012345678 9
S and G, and S is between G and T, then they are 14 away from each
123456789012345678901234567890121234567890123456789012345678 9
123456789012345678901234567890121234567890123456789012345678
other (9 1 5 5 14). This is the farthest that they could be from each 9
23456789012345678901234567890121234567890123456789012345678
123456789012345678901234567890121234567890123456789012345678 9
123456789012345678901234567890121234567890123456789012345678
other. Therefore I is not possible, but II and III are. Choice (D) is 9
1 9
123456789012345678901234567890121234567890123456789012345678 9
the answer.
123456789012345678901234567890121234567890123456789012345678 9
123456789012345678901234567890121234567890123456789012345678
20. C If you connect A, B, and C, you have an equilateral triangle with a side 9
123456789012345678901234567890121234567890123456789012345678 9
123456789012345678901234567890121234567890123456789012345678
length of six. BD cuts the triangle into two 30-60-90 triangles. This 9
123456789012345678901234567890121234567890123456789012345678 9
123456789012345678901234567890121234567890123456789012345678 9
123456789012345678901234567890121234567890123456789012345678 99
23456789012345678901234567890121234567890123456789012345678
means that AD 5 3—because of the relationship between the sides of a
123456789012345678901234567890121234567890123456789012345678
30-60-90 triangle—and so BD 5 3=3, choice (C). 9
123456789012345678901234567890121234567890123456789012345678
123456789012345678901234567890121234567890123456789012345678 99
1 21. E This problem has many steps, which is why it’s number 21. First, ask
123456789012345678901234567890121234567890123456789012345678 9
123456789012345678901234567890121234567890123456789012345678 9
123456789012345678901234567890121234567890123456789012345678
yourself, “What point on the fan travels the greatest distance through 9
123456789012345678901234567890121234567890123456789012345678 9
123456789012345678901234567890121234567890123456789012345678 9
one revolution?” The answer is the point on the corner tip of the fan
23456789012345678901234567890121234567890123456789012345678
123456789012345678901234567890121234567890123456789012345678 9
1 blade, as it is the farthest from the axis of rotation. In going through 9
123456789012345678901234567890121234567890123456789012345678
one revolution, this point describes a circle because it rotates about a 9
123456789012345678901234567890121234567890123456789012345678 9
123456789012345678901234567890121234567890123456789012345678
fixed point with a constant radius. 9
123456789012345678901234567890121234567890123456789012345678 9
123456789012345678901234567890121234567890123456789012345678 9
123456789012345678901234567890121234567890123456789012345678 9
123456789012345678901234567890121234567890123456789012345678 9
1234567890123456789012345678901212345678901234567890123456789
Copyright © 2005 Thomson Peterson’s, a part of The Thomson Corporaton 73
SAT is a registered trademark of the College Entrance Examination Board, which
was not involved in the production of and does not endorse this product.
1234567890123456789012345678901212345678901234567890123456789
123456789012345678901234567890121234567890123456789012345678 9
123456789012345678901234567890121234567890123456789012345678 9
123456789012345678901234567890121234567890123456789012345678 9
123456789012345678901234567890121234567890123456789012345678 9
123456789012345678901234567890121234567890123456789012345678
To find out how far this point travels, you need to determine the radius 9
123456789012345678901234567890121234567890123456789012345678 9
123456789012345678901234567890121234567890123456789012345678
of this circle. This can be done with
23456789012345678901234567890121234567890123456789012345678
some help from the triangle shape 9
9
1
123456789012345678901234567890121234567890123456789012345678
at the top of the fan. The dashed lines through the two sides show that 9
123456789012345678901234567890121234567890123456789012345678 9
123456789012345678901234567890121234567890123456789012345678
these sides are congruent. Combine this with the right angle, and you 9
123456789012345678901234567890121234567890123456789012345678
23456789012345678901234567890121234567890123456789012345678
9
9
1 have a 45-45-90 right triangle with two sides of length 4. The
123456789012345678901234567890121234567890123456789012345678
hypotenuse of this triangle will be 4=2, and this length will also be
9
123456789012345678901234567890121234567890123456789012345678 9
123456789012345678901234567890121234567890123456789012345678 9
123456789012345678901234567890121234567890123456789012345678
the diameter of the circle.
23456789012345678901234567890121234567890123456789012345678
9
9
1
123456789012345678901234567890121234567890123456789012345678 9
123456789012345678901234567890121234567890123456789012345678
Therefore, in one rotation that point travels the circumference of the 9
123456789012345678901234567890121234567890123456789012345678 9
123456789012345678901234567890121234567890123456789012345678
circle it describes:
23456789012345678901234567890121234567890123456789012345678
9
9
1
123456789012345678901234567890121234567890123456789012345678 9
123456789012345678901234567890121234567890123456789012345678
C 5 pd 5 4=2p 9
123456789012345678901234567890121234567890123456789012345678 9
123456789012345678901234567890121234567890123456789012345678 9
123456789012345678901234567890121234567890123456789012345678 9
23456789012345678901234567890121234567890123456789012345678
But there’s more! In 30 seconds, the fan can do 300 revolutions. You
123456789012345678901234567890121234567890123456789012345678 9
123456789012345678901234567890121234567890123456789012345678
know this because the problem states that maximum blade speed is 100 9
123456789012345678901234567890121234567890123456789012345678 9
123456789012345678901234567890121234567890123456789012345678 9
revolutions in 10 seconds, and you can multiply both of these numbers
123456789012345678901234567890121234567890123456789012345678 9
1 by 3 to get 300 revolutions in 30 seconds. This means that the farthest 9
123456789012345678901234567890121234567890123456789012345678 9
123456789012345678901234567890121234567890123456789012345678
the point could travel would be: 9
123456789012345678901234567890121234567890123456789012345678 9
123456789012345678901234567890121234567890123456789012345678 9
23456789012345678901234567890121234567890123456789012345678
123456789012345678901234567890121234567890123456789012345678
300 3 4=2p 5 1200=2 9
123456789012345678901234567890121234567890123456789012345678 9
123456789012345678901234567890121234567890123456789012345678 9
123456789012345678901234567890121234567890123456789012345678 9
Choice (E).
123456789012345678901234567890121234567890123456789012345678 9
123456789012345678901234567890121234567890123456789012345678 9
123456789012345678901234567890121234567890123456789012345678 9
123456789012345678901234567890121234567890123456789012345678 9
1 Section 6 9
23456789012345678901234567890121234567890123456789012345678
123456789012345678901234567890121234567890123456789012345678 9
123456789012345678901234567890121234567890123456789012345678
1. D The passage reads, “As he matured, however, Remington turned his 9
1 9
123456789012345678901234567890121234567890123456789012345678 9
attention away from illustration, concentrating instead on painting and
123456789012345678901234567890121234567890123456789012345678 9
123456789012345678901234567890121234567890123456789012345678
sculpture.” (lines 5–9) So the passage links Remington’s concentration 9
123456789012345678901234567890121234567890123456789012345678 9
123456789012345678901234567890121234567890123456789012345678 9
upon painting and sculpture as key for his maturation. Eliminate (A).
123456789012345678901234567890121234567890123456789012345678 9
123456789012345678901234567890121234567890123456789012345678
However, the rest of the passage focuses on his paintings, so the answer 9
23456789012345678901234567890121234567890123456789012345678
123456789012345678901234567890121234567890123456789012345678 9
123456789012345678901234567890121234567890123456789012345678
can’t just be sculpture. Eliminate (B). We know he paints nocturnal 9
1 9
123456789012345678901234567890121234567890123456789012345678 9
scenes, so eliminate (E) (this answer isn’t specific enough). He is a great
123456789012345678901234567890121234567890123456789012345678
artist of the American West, painting before there were any big cities in 9
123456789012345678901234567890121234567890123456789012345678 9
123456789012345678901234567890121234567890123456789012345678
the West. The passage also implies that he paints natural scenes. The 9
123456789012345678901234567890121234567890123456789012345678 9
123456789012345678901234567890121234567890123456789012345678
best answer is (D). 9
123456789012345678901234567890121234567890123456789012345678 9
23456789012345678901234567890121234567890123456789012345678
123456789012345678901234567890121234567890123456789012345678 9
123456789012345678901234567890121234567890123456789012345678
2. C Go back to the passage. In the same sentence with vindicates is claim to 9
123456789012345678901234567890121234567890123456789012345678 9
123456789012345678901234567890121234567890123456789012345678
the status of literature. This phrase indicates that literature has more 9
1 9
123456789012345678901234567890121234567890123456789012345678 9
23456789012345678901234567890121234567890123456789012345678
status than fiction. You can confirm this hunch in the next sentence,
9
123456789012345678901234567890121234567890123456789012345678 9
123456789012345678901234567890121234567890123456789012345678
too. It says a work clearly rises to the auspicious status of literature.
9
123456789012345678901234567890121234567890123456789012345678 9
1 Again, the emphasis is on the loftiness of literature. Choice (C) best
23456789012345678901234567890121234567890123456789012345678
123456789012345678901234567890121234567890123456789012345678 9
1 captures this sense. 9
123456789012345678901234567890121234567890123456789012345678 9
123456789012345678901234567890121234567890123456789012345678 9
123456789012345678901234567890121234567890123456789012345678 9
123456789012345678901234567890121234567890123456789012345678 9
123456789012345678901234567890121234567890123456789012345678 9
123456789012345678901234567890121234567890123456789012345678 9
123456789012345678901234567890121234567890123456789012345678 9
1234567890123456789012345678901212345678901234567890123456789
74 Copyright © 2005 Thomson Peterson’s, a part of The Thomson Corporaton
SAT is a registered trademark of the College Entrance Examination Board, which
was not involved in the production of and does not endorse this product.
1234567890123456789012345678901212345678901234567890123456789
123456789012345678901234567890121234567890123456789012345678 9
123456789012345678901234567890121234567890123456789012345678 9
123456789012345678901234567890121234567890123456789012345678 9
123456789012345678901234567890121234567890123456789012345678 9
123456789012345678901234567890121234567890123456789012345678
3. C The passage states that the test of time method, by definition excludes 9
123456789012345678901234567890121234567890123456789012345678 9
123456789012345678901234567890121234567890123456789012345678
contemporary works from consideration.
23456789012345678901234567890121234567890123456789012345678
Choice (C) is a reasonable 9
9
1
123456789012345678901234567890121234567890123456789012345678
paraphrase. 9
123456789012345678901234567890121234567890123456789012345678 9
123456789012345678901234567890121234567890123456789012345678 9
123456789012345678901234567890121234567890123456789012345678
4. E There is only one topic that the paragraph
23456789012345678901234567890121234567890123456789012345678
mentions twice in relation to 9
9
1 the exploration of Mars: detection of life. So this topic is the best
123456789012345678901234567890121234567890123456789012345678 9
123456789012345678901234567890121234567890123456789012345678
candidate for what the paragraph most emphasizes as the motivation 9
123456789012345678901234567890121234567890123456789012345678 9
123456789012345678901234567890121234567890123456789012345678
for the Mars exploration. The paragraph also concludes with this point.
23456789012345678901234567890121234567890123456789012345678
9
9
1
123456789012345678901234567890121234567890123456789012345678
Choice (E) is the answer. 9
123456789012345678901234567890121234567890123456789012345678 9
123456789012345678901234567890121234567890123456789012345678 9
123456789012345678901234567890121234567890123456789012345678
5. B Does the author of Passage 1 criticize
23456789012345678901234567890121234567890123456789012345678
the English system? He says it is 9
9
1
123456789012345678901234567890121234567890123456789012345678
quirky, but he does not outright criticize it. That eliminates (D) and (E). 9
123456789012345678901234567890121234567890123456789012345678 9
123456789012345678901234567890121234567890123456789012345678
The author does not strongly praise the system either. That eliminates 9
123456789012345678901234567890121234567890123456789012345678 9
123456789012345678901234567890121234567890123456789012345678 99
23456789012345678901234567890121234567890123456789012345678
(A). The author is also not neutral toward it (he argues that it is part of
123456789012345678901234567890121234567890123456789012345678
123456789012345678901234567890121234567890123456789012345678
the inheritance of the English-speaking world). That eliminates (C). 9
123456789012345678901234567890121234567890123456789012345678 9
123456789012345678901234567890121234567890123456789012345678 99
Choice (B), qualified acceptance, sounds about right since the author
123456789012345678901234567890121234567890123456789012345678 9
1 does accept the system but still points out its quirkiness.
123456789012345678901234567890121234567890123456789012345678 9
123456789012345678901234567890121234567890123456789012345678
D What is Henry I’s role in the passage? Not simply to demonstrate that 9
123456789012345678901234567890121234567890123456789012345678
6. 9
23456789012345678901234567890121234567890123456789012345678
123456789012345678901234567890121234567890123456789012345678
the monarchy was involved in the development of the English system. 9
123456789012345678901234567890121234567890123456789012345678 9
123456789012345678901234567890121234567890123456789012345678
Eliminate (B). The story of Henry I is definitely not included to suggest 9
123456789012345678901234567890121234567890123456789012345678 99
123456789012345678901234567890121234567890123456789012345678
the practicality of the English system, since that is contrary to the basic
123456789012345678901234567890121234567890123456789012345678 9
123456789012345678901234567890121234567890123456789012345678
thesis of the passage. Eliminate (E). The basic point of the passage is to 9
123456789012345678901234567890121234567890123456789012345678 9
123456789012345678901234567890121234567890123456789012345678
argue that the English system developed arbitrarily over time. Choice 9
1 9
123456789012345678901234567890121234567890123456789012345678 99
23456789012345678901234567890121234567890123456789012345678
(D) sounds like a good answer. (A) isn’t right, since the author says the
123456789012345678901234567890121234567890123456789012345678 9
1 anecdote is surprisingly true. Answer (C) is off base, because the
123456789012345678901234567890121234567890123456789012345678 9
123456789012345678901234567890121234567890123456789012345678
passage isn’t simply interested in the
23456789012345678901234567890121234567890123456789012345678
length, but also the quality, of the 9
9
123456789012345678901234567890121234567890123456789012345678
123456789012345678901234567890121234567890123456789012345678
English system’s history. Choice (D) is correct. 9
1 9
123456789012345678901234567890121234567890123456789012345678 9
123456789012345678901234567890121234567890123456789012345678
7. C Earthy in context must mean something slightly better than ridiculous, 9
23456789012345678901234567890121234567890123456789012345678
123456789012345678901234567890121234567890123456789012345678 9
123456789012345678901234567890121234567890123456789012345678
because it is used in the discussion of the arbitrary nature of the English 9
1 9
123456789012345678901234567890121234567890123456789012345678 9
system. Choice (C) is the best answer.
123456789012345678901234567890121234567890123456789012345678 9
123456789012345678901234567890121234567890123456789012345678
8. E Go back to the phrase in the passage. It talks about one system 9
123456789012345678901234567890121234567890123456789012345678 9
123456789012345678901234567890121234567890123456789012345678
developing in Rome and another in medieval Europe. They reconciled 9
123456789012345678901234567890121234567890123456789012345678 9
123456789012345678901234567890121234567890123456789012345678
the two by making an estimate of one (the mile) that is an even number 9
23456789012345678901234567890121234567890123456789012345678
123456789012345678901234567890121234567890123456789012345678 9
123456789012345678901234567890121234567890123456789012345678
of the other (furlongs). Is that synonymous with determining which one 9
123456789012345678901234567890121234567890123456789012345678 9
123456789012345678901234567890121234567890123456789012345678
is accurate? No. Eliminate choice (A). With developing a more accurate 9
1 9
123456789012345678901234567890121234567890123456789012345678 99
23456789012345678901234567890121234567890123456789012345678
system? No. (B) is out. Settling the public’s disagreement? Possibly.
123456789012345678901234567890121234567890123456789012345678 9
123456789012345678901234567890121234567890123456789012345678
Hang on to (C). Finding a metric equivalent? Definitely not, so
9
123456789012345678901234567890121234567890123456789012345678 9
1 eliminate (D). Ceasing to use two different systems? Definitely. Between
23456789012345678901234567890121234567890123456789012345678
123456789012345678901234567890121234567890123456789012345678 9
1 definitely (E)and possibly (C), pick definitely. The answer is (E). 9
123456789012345678901234567890121234567890123456789012345678 9
123456789012345678901234567890121234567890123456789012345678 9
123456789012345678901234567890121234567890123456789012345678 9
123456789012345678901234567890121234567890123456789012345678 9
123456789012345678901234567890121234567890123456789012345678 9
123456789012345678901234567890121234567890123456789012345678 9
123456789012345678901234567890121234567890123456789012345678 9
1234567890123456789012345678901212345678901234567890123456789
Copyright © 2005 Thomson Peterson’s, a part of The Thomson Corporaton 75
SAT is a registered trademark of the College Entrance Examination Board, which
was not involved in the production of and does not endorse this product.
1234567890123456789012345678901212345678901234567890123456789
123456789012345678901234567890121234567890123456789012345678 9
123456789012345678901234567890121234567890123456789012345678 9
123456789012345678901234567890121234567890123456789012345678 9
123456789012345678901234567890121234567890123456789012345678 9
123456789012345678901234567890121234567890123456789012345678
9. B Again, use your knowledge of the main idea to help guide you. You can 9
123456789012345678901234567890121234567890123456789012345678 9
123456789012345678901234567890121234567890123456789012345678
therefore eliminate (A) and (E). Does
23456789012345678901234567890121234567890123456789012345678
the English system continue to 9
9
1
123456789012345678901234567890121234567890123456789012345678
evolve? Use common sense—no. You are left with (B) and (C). The 9
123456789012345678901234567890121234567890123456789012345678 9
123456789012345678901234567890121234567890123456789012345678
passage specifically mentions a variety of sources. (B) is the best answer. 9
123456789012345678901234567890121234567890123456789012345678
23456789012345678901234567890121234567890123456789012345678
9
9
1 (C) is less good because it emphasizes the length of history instead of
123456789012345678901234567890121234567890123456789012345678 9
123456789012345678901234567890121234567890123456789012345678
the quality of that history. 9
123456789012345678901234567890121234567890123456789012345678 9
123456789012345678901234567890121234567890123456789012345678
10. A You’ve established that the history of the English system is long and
23456789012345678901234567890121234567890123456789012345678
9
9
1
123456789012345678901234567890121234567890123456789012345678
colorful. In the second passage, the author is also interested in history. 9
123456789012345678901234567890121234567890123456789012345678 9
123456789012345678901234567890121234567890123456789012345678
So she/he would probably not happily agree that that history is less 9
123456789012345678901234567890121234567890123456789012345678
23456789012345678901234567890121234567890123456789012345678
9
9
1 interesting. But there are dates in both passages. The English system
123456789012345678901234567890121234567890123456789012345678 9
123456789012345678901234567890121234567890123456789012345678
goes back to the sixteenth century. Even if you forget that’s actually the 9
123456789012345678901234567890121234567890123456789012345678 9
123456789012345678901234567890121234567890123456789012345678
1500s, you still know it’s before 1840. So the history is factually 9
23456789012345678901234567890121234567890123456789012345678
123456789012345678901234567890121234567890123456789012345678 9
123456789012345678901234567890121234567890123456789012345678
shorter. That makes the answer (A). 9
123456789012345678901234567890121234567890123456789012345678 9
123456789012345678901234567890121234567890123456789012345678 9
123456789012345678901234567890121234567890123456789012345678
11. D Go back to the reference to the French Revolution in the passage. The 9
123456789012345678901234567890121234567890123456789012345678 9
1 author specifically mentions its anti-traditionalist bent. So (D) looks 9
123456789012345678901234567890121234567890123456789012345678 9
123456789012345678901234567890121234567890123456789012345678
good. She/he doesn’t go so far as to indicate that the greatest 9
123456789012345678901234567890121234567890123456789012345678 9
123456789012345678901234567890121234567890123456789012345678 9
23456789012345678901234567890121234567890123456789012345678
accomplishment of the French Revolution was the metric system.
123456789012345678901234567890121234567890123456789012345678 9
123456789012345678901234567890121234567890123456789012345678
Eliminate (A). The author indicates that the metric system was in 9
123456789012345678901234567890121234567890123456789012345678
keeping with the rest of the revolutionary thinking. So eliminate (B). 9
123456789012345678901234567890121234567890123456789012345678 9
123456789012345678901234567890121234567890123456789012345678
There’s no mention of the revolutionary calendar in the passage. 9
123456789012345678901234567890121234567890123456789012345678 9
123456789012345678901234567890121234567890123456789012345678
Eliminate (C). (E) is the only tricky wrong answer. The passage says the 9
123456789012345678901234567890121234567890123456789012345678 9 9
1 metric system wasn’t adopted in France until 1840, but it doesn’t say it
123456789012345678901234567890121234567890123456789012345678 9
123456789012345678901234567890121234567890123456789012345678
wasn’t fully invented before then. The answer is (D). 9
123456789012345678901234567890121234567890123456789012345678 9
123456789012345678901234567890121234567890123456789012345678 9
123456789012345678901234567890121234567890123456789012345678
12. C In the sentence in question, you find that it has been called, or dubbed,
23456789012345678901234567890121234567890123456789012345678
9
9
123456789012345678901234567890121234567890123456789012345678
123456789012345678901234567890121234567890123456789012345678
“voluntary” and “preferred.” The implication is these are some sort of 9
1 9
123456789012345678901234567890121234567890123456789012345678
official designation, but they specifically are not required. Choice (C) 9
123456789012345678901234567890121234567890123456789012345678 9
123456789012345678901234567890121234567890123456789012345678 9
23456789012345678901234567890121234567890123456789012345678
sounds like a reasonable, noncommittal paraphrase. Eliminate (D).
123456789012345678901234567890121234567890123456789012345678 9
1 These words also don’t suggest any pending official adoption. 9
123456789012345678901234567890121234567890123456789012345678 9
123456789012345678901234567890121234567890123456789012345678 9
Eliminate (B). (A) overstates the case in the other direction: the view of
23456789012345678901234567890121234567890123456789012345678
123456789012345678901234567890121234567890123456789012345678 9
123456789012345678901234567890121234567890123456789012345678
the metric system reflected in the sentence in the passage is favorable. 9
123456789012345678901234567890121234567890123456789012345678 9
123456789012345678901234567890121234567890123456789012345678 9
(E) overstates how favorable—the words in the passage are warm but
9
1
123456789012345678901234567890121234567890123456789012345678
don’t suggest “superiority.” The answer is (C). 9
123456789012345678901234567890121234567890123456789012345678 9
123456789012345678901234567890121234567890123456789012345678
13. E Refined in the passage is synonymous with recalculated. The passage 9
123456789012345678901234567890121234567890123456789012345678 9
123456789012345678901234567890121234567890123456789012345678
doesn’t say whether it’s to make the meter smaller or bigger. That only 9
23456789012345678901234567890121234567890123456789012345678
123456789012345678901234567890121234567890123456789012345678 9
123456789012345678901234567890121234567890123456789012345678
leaves (E). 9
123456789012345678901234567890121234567890123456789012345678 9
123456789012345678901234567890121234567890123456789012345678 9 9
1
123456789012345678901234567890121234567890123456789012345678 9
123456789012345678901234567890121234567890123456789012345678 9
123456789012345678901234567890121234567890123456789012345678 9
123456789012345678901234567890121234567890123456789012345678 9
123456789012345678901234567890121234567890123456789012345678 9
123456789012345678901234567890121234567890123456789012345678 9
123456789012345678901234567890121234567890123456789012345678 9
123456789012345678901234567890121234567890123456789012345678 9
123456789012345678901234567890121234567890123456789012345678 9
1234567890123456789012345678901212345678901234567890123456789
76 Copyright © 2005 Thomson Peterson’s, a part of The Thomson Corporaton
SAT is a registered trademark of the College Entrance Examination Board, which
was not involved in the production of and does not endorse this product.
1234567890123456789012345678901212345678901234567890123456789
123456789012345678901234567890121234567890123456789012345678 9
123456789012345678901234567890121234567890123456789012345678 9
123456789012345678901234567890121234567890123456789012345678 9
123456789012345678901234567890121234567890123456789012345678 9
123456789012345678901234567890121234567890123456789012345678
14. B There is no indication of any ideological shift motivating the 1983 9
123456789012345678901234567890121234567890123456789012345678 9
123456789012345678901234567890121234567890123456789012345678
move. That eliminates (A), (C), and
23456789012345678901234567890121234567890123456789012345678
(E). Which is more logical: new 9
9
1
123456789012345678901234567890121234567890123456789012345678
calculations of the Earth’s circumference or new ways to calculate 9
123456789012345678901234567890121234567890123456789012345678 9
123456789012345678901234567890121234567890123456789012345678
fractions? The former, especially where the speed of light is involved. 9
123456789012345678901234567890121234567890123456789012345678
23456789012345678901234567890121234567890123456789012345678
9
9
1 The answer is (B).
123456789012345678901234567890121234567890123456789012345678 9
123456789012345678901234567890121234567890123456789012345678
15. A As soon as you find an answer that isn’t addressed in one passage, 9
123456789012345678901234567890121234567890123456789012345678 9
123456789012345678901234567890121234567890123456789012345678
you’re done. Do both passages mention kings who were ruling at the
23456789012345678901234567890121234567890123456789012345678
9
9
1
123456789012345678901234567890121234567890123456789012345678
time of their invention? No. Passage 2 talks about the French 9
123456789012345678901234567890121234567890123456789012345678 9
123456789012345678901234567890121234567890123456789012345678
Revolution, which overthrew the king; it only mentions a king in 9
123456789012345678901234567890121234567890123456789012345678
23456789012345678901234567890121234567890123456789012345678
9
9
1 regards to the English system. Choice (A) is the answer.
123456789012345678901234567890121234567890123456789012345678 9
123456789012345678901234567890121234567890123456789012345678 9
123456789012345678901234567890121234567890123456789012345678
16. A Which of the answers is something that the author of Passage 2 would 9
123456789012345678901234567890121234567890123456789012345678 9
123456789012345678901234567890121234567890123456789012345678 99
23456789012345678901234567890121234567890123456789012345678
agree with but does not explicitly state? Which is most plausible?
123456789012345678901234567890121234567890123456789012345678
123456789012345678901234567890121234567890123456789012345678
Choice (A) is the most reasonable answer on its face. You can find 9
123456789012345678901234567890121234567890123456789012345678 9
123456789012345678901234567890121234567890123456789012345678 99
evidence for it when the author says It is the great asset of the metric
123456789012345678901234567890121234567890123456789012345678 9
1 system, at least for scientists, that units for measuring weight and
123456789012345678901234567890121234567890123456789012345678 9
123456789012345678901234567890121234567890123456789012345678
energy are also derived from the basic unit of the meter and The 9
123456789012345678901234567890121234567890123456789012345678 9
123456789012345678901234567890121234567890123456789012345678 99
23456789012345678901234567890121234567890123456789012345678
adoption of the metric system, also known as the international system,
123456789012345678901234567890121234567890123456789012345678
123456789012345678901234567890121234567890123456789012345678
or S.I., coincided with great advances in science. 9
123456789012345678901234567890121234567890123456789012345678 99
123456789012345678901234567890121234567890123456789012345678 9
123456789012345678901234567890121234567890123456789012345678 9
123456789012345678901234567890121234567890123456789012345678
Section 7
123456789012345678901234567890121234567890123456789012345678 9
123456789012345678901234567890121234567890123456789012345678
1. There is nothing fancy about this problem. Substitute and solve for x. 9
1 9
123456789012345678901234567890121234567890123456789012345678 99
23456789012345678901234567890121234567890123456789012345678
123456789012345678901234567890121234567890123456789012345678
2y 5 12
9
1
123456789012345678901234567890121234567890123456789012345678 9
123456789012345678901234567890121234567890123456789012345678
y 5 6 9
123456789012345678901234567890121234567890123456789012345678 99
23456789012345678901234567890121234567890123456789012345678
123456789012345678901234567890121234567890123456789012345678 9
1 2=x 1 =x 5 y
123456789012345678901234567890121234567890123456789012345678 9
123456789012345678901234567890121234567890123456789012345678 9
123456789012345678901234567890121234567890123456789012345678 99
23456789012345678901234567890121234567890123456789012345678
2 = x 1 = x 5 6
123456789012345678901234567890121234567890123456789012345678 9
1 3=x 5 6
123456789012345678901234567890121234567890123456789012345678 9
123456789012345678901234567890121234567890123456789012345678 9
23456789012345678901234567890121234567890123456789012345678
123456789012345678901234567890121234567890123456789012345678
=x 5 2 9
123456789012345678901234567890121234567890123456789012345678 9
123456789012345678901234567890121234567890123456789012345678 9
123456789012345678901234567890121234567890123456789012345678 99
x 5 4
1
123456789012345678901234567890121234567890123456789012345678 9
123456789012345678901234567890121234567890123456789012345678
2. The first five even integers are 2, 4, 6, 8, and 10. Rev up that calculator start 9
123456789012345678901234567890121234567890123456789012345678 9
123456789012345678901234567890121234567890123456789012345678
multiplying. The answer is 3,840. 9
123456789012345678901234567890121234567890123456789012345678 9
23456789012345678901234567890121234567890123456789012345678
123456789012345678901234567890121234567890123456789012345678
3. If the probability that a senior would be picked is three-eighths, then seniors 9
123456789012345678901234567890121234567890123456789012345678 9
123456789012345678901234567890121234567890123456789012345678
are three-eighths of the entire student body. Since Rider High has 400 9
123456789012345678901234567890121234567890123456789012345678 9
1 students, the equation would be: 9
123456789012345678901234567890121234567890123456789012345678 99
23456789012345678901234567890121234567890123456789012345678
1
123456789012345678901234567890121234567890123456789012345678
3 1200 9
123456789012345678901234567890121234567890123456789012345678
400 3 5 5 150 9
123456789012345678901234567890121234567890123456789012345678
8 8
23456789012345678901234567890121234567890123456789012345678
9
9
1
123456789012345678901234567890121234567890123456789012345678 9
123456789012345678901234567890121234567890123456789012345678 9
123456789012345678901234567890121234567890123456789012345678 9
1234567890123456789012345678901212345678901234567890123456789
Copyright © 2005 Thomson Peterson’s, a part of The Thomson Corporaton 77
SAT is a registered trademark of the College Entrance Examination Board, which
was not involved in the production of and does not endorse this product.
1234567890123456789012345678901212345678901234567890123456789
123456789012345678901234567890121234567890123456789012345678 9
123456789012345678901234567890121234567890123456789012345678 9
123456789012345678901234567890121234567890123456789012345678 9
123456789012345678901234567890121234567890123456789012345678 9
123456789012345678901234567890121234567890123456789012345678
4. The volume of any rectangular solid is the length times the width times the 9
123456789012345678901234567890121234567890123456789012345678 9
123456789012345678901234567890121234567890123456789012345678
height. The figure tells us the width and the
23456789012345678901234567890121234567890123456789012345678
height, and you can determine 9
9
1
123456789012345678901234567890121234567890123456789012345678
the length from what you know about the area of the shaded side. Start with 9
123456789012345678901234567890121234567890123456789012345678 9
123456789012345678901234567890121234567890123456789012345678
the area of a rectangle formula and you can find the length: 9
123456789012345678901234567890121234567890123456789012345678
23456789012345678901234567890121234567890123456789012345678
9
9
1
123456789012345678901234567890121234567890123456789012345678
A5l3h 9
123456789012345678901234567890121234567890123456789012345678 9
123456789012345678901234567890121234567890123456789012345678
24 5 4 3 l 9
123456789012345678901234567890121234567890123456789012345678
23456789012345678901234567890121234567890123456789012345678
9
9
1 65l
123456789012345678901234567890121234567890123456789012345678 9
123456789012345678901234567890121234567890123456789012345678 9
123456789012345678901234567890121234567890123456789012345678
Place this length of 6 into the volume formula for the box: 9
123456789012345678901234567890121234567890123456789012345678
23456789012345678901234567890121234567890123456789012345678
9
9
1
123456789012345678901234567890121234567890123456789012345678
V 5 l 3 h 3 w 5 3 3 4 3 6 5 72 9
123456789012345678901234567890121234567890123456789012345678 9
123456789012345678901234567890121234567890123456789012345678 9
123456789012345678901234567890121234567890123456789012345678
5. First use the area of a circle formula to determine the radius. 9
23456789012345678901234567890121234567890123456789012345678
123456789012345678901234567890121234567890123456789012345678 9
123456789012345678901234567890121234567890123456789012345678 9
123456789012345678901234567890121234567890123456789012345678 9
2
A 5 pr
123456789012345678901234567890121234567890123456789012345678 9
123456789012345678901234567890121234567890123456789012345678
16p 5 pr2 9
123456789012345678901234567890121234567890123456789012345678 9
1 16p pr 2 9
123456789012345678901234567890121234567890123456789012345678 9
123456789012345678901234567890121234567890123456789012345678
5 9
123456789012345678901234567890121234567890123456789012345678
p p 9
23456789012345678901234567890121234567890123456789012345678
123456789012345678901234567890121234567890123456789012345678 9
123456789012345678901234567890121234567890123456789012345678 9
2
16 5 r
123456789012345678901234567890121234567890123456789012345678 9
123456789012345678901234567890121234567890123456789012345678
45r 9
123456789012345678901234567890121234567890123456789012345678 9
123456789012345678901234567890121234567890123456789012345678 9
123456789012345678901234567890121234567890123456789012345678
The diameter of a circle is twice the radius, so the diameter is 8. 9
123456789012345678901234567890121234567890123456789012345678 9
123456789012345678901234567890121234567890123456789012345678 9
1 6. There are some fancier ways to solve this problem, but the surest way is to 9
23456789012345678901234567890121234567890123456789012345678
123456789012345678901234567890121234567890123456789012345678 9
123456789012345678901234567890121234567890123456789012345678
count up the options. She could have: 9
1 9
123456789012345678901234567890121234567890123456789012345678
1. No toppings. 9
123456789012345678901234567890121234567890123456789012345678
23456789012345678901234567890121234567890123456789012345678
9
9
123456789012345678901234567890121234567890123456789012345678
2. Just a
123456789012345678901234567890121234567890123456789012345678 9
1 3. Just p 9
123456789012345678901234567890121234567890123456789012345678 9
123456789012345678901234567890121234567890123456789012345678
4. Just r 9
23456789012345678901234567890121234567890123456789012345678
123456789012345678901234567890121234567890123456789012345678 9
123456789012345678901234567890121234567890123456789012345678
5. a and p 9
1 9
123456789012345678901234567890121234567890123456789012345678
6. a and r 9
123456789012345678901234567890121234567890123456789012345678 9
123456789012345678901234567890121234567890123456789012345678 9
23456789012345678901234567890121234567890123456789012345678
7. p and r
9
123456789012345678901234567890121234567890123456789012345678 9
123456789012345678901234567890121234567890123456789012345678
8. All the toppings.
123456789012345678901234567890121234567890123456789012345678 9 9
1
123456789012345678901234567890121234567890123456789012345678
That is a total of 8. Sure, there’s a fancier math way of handling this 9
123456789012345678901234567890121234567890123456789012345678 9
123456789012345678901234567890121234567890123456789012345678
problem, but since you have the right answer, what does it matter? Is your 9
123456789012345678901234567890121234567890123456789012345678 9
123456789012345678901234567890121234567890123456789012345678 9
SAT score in any way determined by whether you used the “fancy method”
23456789012345678901234567890121234567890123456789012345678
123456789012345678901234567890121234567890123456789012345678 9
123456789012345678901234567890121234567890123456789012345678
or not? 9
123456789012345678901234567890121234567890123456789012345678 9
123456789012345678901234567890121234567890123456789012345678 9 9
1
123456789012345678901234567890121234567890123456789012345678 9
123456789012345678901234567890121234567890123456789012345678 9
123456789012345678901234567890121234567890123456789012345678 9
123456789012345678901234567890121234567890123456789012345678 9
123456789012345678901234567890121234567890123456789012345678 9
123456789012345678901234567890121234567890123456789012345678 9
123456789012345678901234567890121234567890123456789012345678 9
123456789012345678901234567890121234567890123456789012345678 9
123456789012345678901234567890121234567890123456789012345678 9
1234567890123456789012345678901212345678901234567890123456789
78 Copyright © 2005 Thomson Peterson’s, a part of The Thomson Corporaton
SAT is a registered trademark of the College Entrance Examination Board, which
was not involved in the production of and does not endorse this product.
1234567890123456789012345678901212345678901234567890123456789
123456789012345678901234567890121234567890123456789012345678 9
123456789012345678901234567890121234567890123456789012345678 9
123456789012345678901234567890121234567890123456789012345678 9
123456789012345678901234567890121234567890123456789012345678 9
123456789012345678901234567890121234567890123456789012345678
7. If x is the sum of the five numbers, you know: 9
123456789012345678901234567890121234567890123456789012345678 9
123456789012345678901234567890121234567890123456789012345678 9
123456789012345678901234567890121234567890123456789012345678
sum 9
123456789012345678901234567890121234567890123456789012345678
5 Average 9
123456789012345678901234567890121234567890123456789012345678
number of items 9
123456789012345678901234567890121234567890123456789012345678 9
123456789012345678901234567890121234567890123456789012345678
x
23456789012345678901234567890121234567890123456789012345678
9
9
1
123456789012345678901234567890121234567890123456789012345678
5 16 9
123456789012345678901234567890121234567890123456789012345678
5 9
123456789012345678901234567890121234567890123456789012345678 9
123456789012345678901234567890121234567890123456789012345678
x
23456789012345678901234567890121234567890123456789012345678
9
9
1 ~ 5 ! 5 16 ~ 5 !
123456789012345678901234567890121234567890123456789012345678
5 9
123456789012345678901234567890121234567890123456789012345678 9
123456789012345678901234567890121234567890123456789012345678
x 5 80 9
123456789012345678901234567890121234567890123456789012345678
23456789012345678901234567890121234567890123456789012345678
9
9
1
123456789012345678901234567890121234567890123456789012345678
Make y the number taken away from the set y. You know that 80 minus y is 9
123456789012345678901234567890121234567890123456789012345678 9
123456789012345678901234567890121234567890123456789012345678
the new sum, and that the new average is 14. With this information you can 9
123456789012345678901234567890121234567890123456789012345678 9
123456789012345678901234567890121234567890123456789012345678 99
23456789012345678901234567890121234567890123456789012345678
solve for y using the equation:
123456789012345678901234567890121234567890123456789012345678 9
123456789012345678901234567890121234567890123456789012345678
123456789012345678901234567890121234567890123456789012345678
80 2 y 9
123456789012345678901234567890121234567890123456789012345678
5 14 9
123456789012345678901234567890121234567890123456789012345678 99
4
1
123456789012345678901234567890121234567890123456789012345678
80 2 y 9
123456789012345678901234567890121234567890123456789012345678
~ 4! 14 ~ 4! 9
123456789012345678901234567890121234567890123456789012345678
4
5 9
123456789012345678901234567890121234567890123456789012345678 99
23456789012345678901234567890121234567890123456789012345678
123456789012345678901234567890121234567890123456789012345678
123456789012345678901234567890121234567890123456789012345678
80 2 y 5 56 9
123456789012345678901234567890121234567890123456789012345678 9
123456789012345678901234567890121234567890123456789012345678 99
80 2 56 2 y 5 56 2 56
123456789012345678901234567890121234567890123456789012345678 9
123456789012345678901234567890121234567890123456789012345678
24 2 y 5 0
123456789012345678901234567890121234567890123456789012345678 9
123456789012345678901234567890121234567890123456789012345678 9
1 24 5 y 9
123456789012345678901234567890121234567890123456789012345678 99
23456789012345678901234567890121234567890123456789012345678
123456789012345678901234567890121234567890123456789012345678
8. Since the triangle is a right triangle, you can use the Pythagorean theorem to
9
1
123456789012345678901234567890121234567890123456789012345678
determine the third side of the triangle: 9
123456789012345678901234567890121234567890123456789012345678
23456789012345678901234567890121234567890123456789012345678
9
9
123456789012345678901234567890121234567890123456789012345678
123456789012345678901234567890121234567890123456789012345678
c2
a2
b 2 9
1 5 1 9
123456789012345678901234567890121234567890123456789012345678
2 9
123456789012345678901234567890121234567890123456789012345678
100 5 64 1 b 9
23456789012345678901234567890121234567890123456789012345678
123456789012345678901234567890121234567890123456789012345678 9
123456789012345678901234567890121234567890123456789012345678
36 5 b2 9
1 9
123456789012345678901234567890121234567890123456789012345678
b56 9
123456789012345678901234567890121234567890123456789012345678 9
23456789012345678901234567890121234567890123456789012345678
123456789012345678901234567890121234567890123456789012345678 9
123456789012345678901234567890121234567890123456789012345678
If 6 is the length of one side of the square, the area of the square is the square 9
123456789012345678901234567890121234567890123456789012345678 9
123456789012345678901234567890121234567890123456789012345678
of that, 36. 9
1 9
23456789012345678901234567890121234567890123456789012345678
123456789012345678901234567890121234567890123456789012345678 9
123456789012345678901234567890121234567890123456789012345678
9. You need to remember the right formula for this one. At least you know that 9
123456789012345678901234567890121234567890123456789012345678 9
123456789012345678901234567890121234567890123456789012345678 99
since you can’t grid in negative numbers, the slope must be positive. Here’s
1
123456789012345678901234567890121234567890123456789012345678
the rise over run calculation: 9
123456789012345678901234567890121234567890123456789012345678 9
123456789012345678901234567890121234567890123456789012345678 9
123456789012345678901234567890121234567890123456789012345678
rise y 2 y 22 2 1 23 3 9
123456789012345678901234567890121234567890123456789012345678 9
1 2
slope 5 5 5 5 5
23456789012345678901234567890121234567890123456789012345678
123456789012345678901234567890121234567890123456789012345678
run x 2 x 21 2 3 24 4 9
1 1 2
9
123456789012345678901234567890121234567890123456789012345678 9
123456789012345678901234567890121234567890123456789012345678 9
123456789012345678901234567890121234567890123456789012345678 9
123456789012345678901234567890121234567890123456789012345678 9
123456789012345678901234567890121234567890123456789012345678 9
123456789012345678901234567890121234567890123456789012345678 9
123456789012345678901234567890121234567890123456789012345678 9
1234567890123456789012345678901212345678901234567890123456789
Copyright © 2005 Thomson Peterson’s, a part of The Thomson Corporaton 79
SAT is a registered trademark of the College Entrance Examination Board, which
was not involved in the production of and does not endorse this product.
1234567890123456789012345678901212345678901234567890123456789
123456789012345678901234567890121234567890123456789012345678 9
123456789012345678901234567890121234567890123456789012345678 9
123456789012345678901234567890121234567890123456789012345678 9
123456789012345678901234567890121234567890123456789012345678 9
123456789012345678901234567890121234567890123456789012345678
10. If you work backward from what you know, this problem contains no 9
123456789012345678901234567890121234567890123456789012345678 9
123456789012345678901234567890121234567890123456789012345678
difficult steps. If Bo is 15 and Gina is five
23456789012345678901234567890121234567890123456789012345678
years younger than he, then Gina 9
9
1
123456789012345678901234567890121234567890123456789012345678
is 10. And if Gina is 10 and Susan is three times the age of Gina, then Susan 9
123456789012345678901234567890121234567890123456789012345678 9
123456789012345678901234567890121234567890123456789012345678
is 30. And if Susan is 30 and Tom is twelve years older than Susan, then Tom 9
123456789012345678901234567890121234567890123456789012345678
23456789012345678901234567890121234567890123456789012345678
9
9
1 is 42. There’s your answer, 42.
123456789012345678901234567890121234567890123456789012345678 9
123456789012345678901234567890121234567890123456789012345678
11. This one tries to intimidate you with a new symbol and a complicated 9
123456789012345678901234567890121234567890123456789012345678 9
123456789012345678901234567890121234567890123456789012345678
definition. By now, this sort of attempted distraction should not even faze
23456789012345678901234567890121234567890123456789012345678
9
9
1
123456789012345678901234567890121234567890123456789012345678
you, as you are well aware that everything you need to know about the new 9
123456789012345678901234567890121234567890123456789012345678 9
123456789012345678901234567890121234567890123456789012345678
symbol is right in front of you. 9
123456789012345678901234567890121234567890123456789012345678
23456789012345678901234567890121234567890123456789012345678
9
9
1
123456789012345678901234567890121234567890123456789012345678
The phrase greatest prime divisor means the greatest number that is prime and 9
123456789012345678901234567890121234567890123456789012345678 9
123456789012345678901234567890121234567890123456789012345678
also divides the original number. So the greatest prime divisor of 15 is 5 since 9
123456789012345678901234567890121234567890123456789012345678 9
123456789012345678901234567890121234567890123456789012345678 9
23456789012345678901234567890121234567890123456789012345678
no prime numbers greater than 5 evenly divide into 15. As for 12, the greatest
123456789012345678901234567890121234567890123456789012345678 9
123456789012345678901234567890121234567890123456789012345678
prime divisor is 3. This means (15*)(12*) 5 (5)(3) 5 15. 9
123456789012345678901234567890121234567890123456789012345678 9
123456789012345678901234567890121234567890123456789012345678 9
123456789012345678901234567890121234567890123456789012345678
12. The graph might look messy, but you only need to pick out the five 9
1 9
123456789012345678901234567890121234567890123456789012345678
y-intercepts and add them up. 9
123456789012345678901234567890121234567890123456789012345678 9
123456789012345678901234567890121234567890123456789012345678
Line 1: at (0, 2) 9
123456789012345678901234567890121234567890123456789012345678 9
23456789012345678901234567890121234567890123456789012345678
123456789012345678901234567890121234567890123456789012345678
Line 2: at (0, 1) 9
123456789012345678901234567890121234567890123456789012345678 9
123456789012345678901234567890121234567890123456789012345678
Lines 3 and 4: at (0, 0) 9
123456789012345678901234567890121234567890123456789012345678 9
123456789012345678901234567890121234567890123456789012345678 9
Line 5: at (0, 21)
123456789012345678901234567890121234567890123456789012345678 9
123456789012345678901234567890121234567890123456789012345678 9
123456789012345678901234567890121234567890123456789012345678
Adding up these five y-values gives you: 2 1 1 1 0 1 0 2 1 5 2 9
1 9
23456789012345678901234567890121234567890123456789012345678
123456789012345678901234567890121234567890123456789012345678 9
123456789012345678901234567890121234567890123456789012345678
13. The problem says that the numbers are distinct, so none of the four numbers 9
1 9
123456789012345678901234567890121234567890123456789012345678 9
are the same. That’s your first clue. Since the sum of the four numbers is 26, the
123456789012345678901234567890121234567890123456789012345678 9
123456789012345678901234567890121234567890123456789012345678
numbers in the sum must be less than 21. Here’s a Rogue’s Gallery List of all 9
123456789012345678901234567890121234567890123456789012345678
the less prime numbers: 2, 3, 5, 7, 11, 13, 17, and 19. To find the greatest 9
123456789012345678901234567890121234567890123456789012345678 9
123456789012345678901234567890121234567890123456789012345678
possible integer in the set, first make all the other integers as least as possible. 9
123456789012345678901234567890121234567890123456789012345678 9
23456789012345678901234567890121234567890123456789012345678
123456789012345678901234567890121234567890123456789012345678
The set could be 2, 3, 5, x. This would maximize x. Now solve: 9
123456789012345678901234567890121234567890123456789012345678 9
1 9
123456789012345678901234567890121234567890123456789012345678 9
123456789012345678901234567890121234567890123456789012345678 9
2 1 31 51 x 5 21
23456789012345678901234567890121234567890123456789012345678
123456789012345678901234567890121234567890123456789012345678 9
123456789012345678901234567890121234567890123456789012345678
10 1 x 5 21 9
123456789012345678901234567890121234567890123456789012345678 9
123456789012345678901234567890121234567890123456789012345678 9
10 2 10 1 x 5 21 2 10
9
1
123456789012345678901234567890121234567890123456789012345678
x 5 11, also prime. 9
123456789012345678901234567890121234567890123456789012345678 9
123456789012345678901234567890121234567890123456789012345678 9
123456789012345678901234567890121234567890123456789012345678
11 must be the answer. 9
123456789012345678901234567890121234567890123456789012345678 9
123456789012345678901234567890121234567890123456789012345678 9
23456789012345678901234567890121234567890123456789012345678
123456789012345678901234567890121234567890123456789012345678 9 9
123456789012345678901234567890121234567890123456789012345678
Section 8 9
123456789012345678901234567890121234567890123456789012345678 9
1 As you might expect, answers will vary.
123456789012345678901234567890121234567890123456789012345678 9
23456789012345678901234567890121234567890123456789012345678 If possible, politely ask a teacher,
9
1 fellow student, or some other person knowledgeable about formal essay
123456789012345678901234567890121234567890123456789012345678 9
123456789012345678901234567890121234567890123456789012345678
writing to review your essay and provide feedback on ways in which the 9
123456789012345678901234567890121234567890123456789012345678
23456789012345678901234567890121234567890123456789012345678
9
9
1 essay is commendable and on areas where it could be improved.
123456789012345678901234567890121234567890123456789012345678 9
123456789012345678901234567890121234567890123456789012345678 9
123456789012345678901234567890121234567890123456789012345678 9
1234567890123456789012345678901212345678901234567890123456789
80 Copyright © 2005 Thomson Peterson’s, a part of The Thomson Corporaton
SAT is a registered trademark of the College Entrance Examination Board, which
was not involved in the production of and does not endorse this product.
12345678901234567890123456789012123456789012345678901234567890121234567
1234567890123456789012345678901212345678901234567890123456789012123456 7
12234567890123456789012345678901212345678901234567890123456789012123456 7
1 34567890123456789012345678901212345678901234567890123456789012123456 7
12234567890123456789012345678901212345678901234567890123456789012123456 7
1234567890123456789012345678901212345678901234567890123456789012123456 7
Scoring Worksheet
1 34567890123456789012345678901212345678901234567890123456789012123456 7
12234567890123456789012345678901212345678901234567890123456789012123456
1 34567890123456789012345678901212345678901234567890123456789012123456 7
7
12234567890123456789012345678901212345678901234567890123456789012123456 7
1234567890123456789012345678901212345678901234567890123456789012123456 7
1234567890123456789012345678901212345678901234567890123456789012123456
MATH 7
1234567890123456789012345678901212345678901234567890123456789012123456
34567890123456789012345678901212345678901234567890123456789012123456
7
7
1
12234567890123456789012345678901212345678901234567890123456789012123456
Number Number Raw 7
1234567890123456789012345678901212345678901234567890123456789012123456 7
1234567890123456789012345678901212345678901234567890123456789012123456
Correct Incorrect 5 Score 7
1234567890123456789012345678901212345678901234567890123456789012123456
34567890123456789012345678901212345678901234567890123456789012123456
7
7
1
12234567890123456789012345678901212345678901234567890123456789012123456 7
1234567890123456789012345678901212345678901234567890123456789012123456 7
1234567890123456789012345678901212345678901234567890123456789012123456
Section 2 2 (.25 3 ) 5 7
1234567890123456789012345678901212345678901234567890123456789012123456
34567890123456789012345678901212345678901234567890123456789012123456
7
7
1
12234567890123456789012345678901212345678901234567890123456789012123456 7
1234567890123456789012345678901212345678901234567890123456789012123456 7
1234567890123456789012345678901212345678901234567890123456789012123456
Section 5 2 (.25 3 ) 5 7
1234567890123456789012345678901212345678901234567890123456789012123456 7
1234567890123456789012345678901212345678901234567890123456789012123456 77
34567890123456789012345678901212345678901234567890123456789012123456
1234567890123456789012345678901212345678901234567890123456789012123456 7
1234567890123456789012345678901212345678901234567890123456789012123456
Section 7 2 (.25 3 ) 5
1234567890123456789012345678901212345678901234567890123456789012123456 7
1234567890123456789012345678901212345678901234567890123456789012123456 77
1234567890123456789012345678901212345678901234567890123456789012123456 7
1
12234567890123456789012345678901212345678901234567890123456789012123456
CRITICAL READING 7
1234567890123456789012345678901212345678901234567890123456789012123456 7
1234567890123456789012345678901212345678901234567890123456789012123456 7
34567890123456789012345678901212345678901234567890123456789012123456
1234567890123456789012345678901212345678901234567890123456789012123456
Sections 7
1234567890123456789012345678901212345678901234567890123456789012123456 7
1234567890123456789012345678901212345678901234567890123456789012123456
1, 4, and 6 7
1234567890123456789012345678901212345678901234567890123456789012123456 77
2 (.25 3 ) 5
1234567890123456789012345678901212345678901234567890123456789012123456 7
1234567890123456789012345678901212345678901234567890123456789012123456 7
1234567890123456789012345678901212345678901234567890123456789012123456 7
1234567890123456789012345678901212345678901234567890123456789012123456
WRITING
1234567890123456789012345678901212345678901234567890123456789012123456 7
1 7
2 34567890123456789012345678901212345678901234567890123456789012123456
1234567890123456789012345678901212345678901234567890123456789012123456 7
1234567890123456789012345678901212345678901234567890123456789012123456
Section 3 2 (.25 3 ) 5 7
1 7
1234567890123456789012345678901212345678901234567890123456789012123456 7
12234567890123456789012345678901212345678901234567890123456789012123456 7
1234567890123456789012345678901212345678901234567890123456789012123456
Section 8 Go to www.petersons.com/satessayedge/ 7
1234567890123456789012345678901212345678901234567890123456789012123456 7
1 34567890123456789012345678901212345678901234567890123456789012123456 7
12234567890123456789012345678901212345678901234567890123456789012123456
for instant online scoring and feedback. 7
1234567890123456789012345678901212345678901234567890123456789012123456 7
1234567890123456789012345678901212345678901234567890123456789012123456 77
34567890123456789012345678901212345678901234567890123456789012123456
1234567890123456789012345678901212345678901234567890123456789012123456 7
1
1234567890123456789012345678901212345678901234567890123456789012123456 7
1234567890123456789012345678901212345678901234567890123456789012123456 7
12234567890123456789012345678901212345678901234567890123456789012123456 7
1234567890123456789012345678901212345678901234567890123456789012123456 7
1234567890123456789012345678901212345678901234567890123456789012123456 7
1234567890123456789012345678901212345678901234567890123456789012123456 7
1 34567890123456789012345678901212345678901234567890123456789012123456 7
1234567890123456789012345678901212345678901234567890123456789012123456 77
2 34567890123456789012345678901212345678901234567890123456789012123456
1234567890123456789012345678901212345678901234567890123456789012123456 7
1234567890123456789012345678901212345678901234567890123456789012123456 7
1234567890123456789012345678901212345678901234567890123456789012123456 7
1
12234567890123456789012345678901212345678901234567890123456789012123456 7
1234567890123456789012345678901212345678901234567890123456789012123456 7
1234567890123456789012345678901212345678901234567890123456789012123456 7
1234567890123456789012345678901212345678901234567890123456789012123456 7
1 34567890123456789012345678901212345678901234567890123456789012123456 7
1234567890123456789012345678901212345678901234567890123456789012123456 77
2 34567890123456789012345678901212345678901234567890123456789012123456
1
12234567890123456789012345678901212345678901234567890123456789012123456 7
1 34567890123456789012345678901212345678901234567890123456789012123456 7
12234567890123456789012345678901212345678901234567890123456789012123456 7
1 34567890123456789012345678901212345678901234567890123456789012123456 7
1234567890123456789012345678901212345678901234567890123456789012123456 7
1234567890123456789012345678901212345678901234567890123456789012123456 7
1234567890123456789012345678901212345678901234567890123456789012123456 7
12345678901234567890123456789012123456789012345678901234567890121234567
Copyright © 2005 Thomson Peterson’s, a part of The Thomson Corporaton 81
SAT is a registered trademark of the College Entrance Examination Board, which
was not involved in the production of and does not endorse this product.
12345678901234567890123456789012123456789012345678901234567890121234567
1234567890123456789012345678901212345678901234567890123456789012123456 7
12 34567890123456789012345678901212345678901234567890123456789012123456 7
12 Score Charts
1234567890123456789012345678901212345678901234567890123456789012123456 7
12 34567890123456789012345678901212345678901234567890123456789012123456 7
34567890123456789012345678901212345678901234567890123456789012123456 7
1234567890123456789012345678901212345678901234567890123456789012123456 7
12 34567890123456789012345678901212345678901234567890123456789012123456 7
1234567890123456789012345678901212345678901234567890123456789012123456 7
12 34567890123456789012345678901212345678901234567890123456789012123456
MATH CRITICAL READING 7
12 34567890123456789012345678901212345678901234567890123456789012123456 7
12 34567890123456789012345678901212345678901234567890123456789012123456 7
12
2
34567890123456789012345678901212345678901234567890123456789012123456
Raw Math Scaled Raw Math Scaled Raw Verbal Scaled Raw Verbal Scaled
34567890123456789012345678901212345678901234567890123456789012123456
7
7
1 Score Score Score Score Score Score Score Score
12 34567890123456789012345678901212345678901234567890123456789012123456 7
12 34567890123456789012345678901212345678901234567890123456789012123456 7
60 800 28 500 78 800 37 510
12 34567890123456789012345678901212345678901234567890123456789012123456
77 800 36 510 7
12
2
34567890123456789012345678901212345678901234567890123456789012123456
59 800 27 490
34567890123456789012345678901212345678901234567890123456789012123456
7
7
1 58 790 26 490 76 800 35 500
12 34567890123456789012345678901212345678901234567890123456789012123456 7
12 34567890123456789012345678901212345678901234567890123456789012123456
57 770 25 480 75 790 34 500 7
12 34567890123456789012345678901212345678901234567890123456789012123456 7
12
2
34567890123456789012345678901212345678901234567890123456789012123456
56 760 24 470 74 780 33 490
34567890123456789012345678901212345678901234567890123456789012123456
7
7
1 55 73 770 32 490
12 34567890123456789012345678901212345678901234567890123456789012123456
740 23 460 7
12 34567890123456789012345678901212345678901234567890123456789012123456
72 760 31 480 7
12 34567890123456789012345678901212345678901234567890123456789012123456
54 720 22 460
7
12 34567890123456789012345678901212345678901234567890123456789012123456
53 710 21 450 71 750 30 480 7
2 34567890123456789012345678901212345678901234567890123456789012123456
1234567890123456789012345678901212345678901234567890123456789012123456
70 740 29 470 7
1234567890123456789012345678901212345678901234567890123456789012123456
52 700 20 440 7
1234567890123456789012345678901212345678901234567890123456789012123456
69 730 28 460 7
1234567890123456789012345678901212345678901234567890123456789012123456
51 690 19 430 7
1234567890123456789012345678901212345678901234567890123456789012123456
68 720 27 460 7
1234567890123456789012345678901212345678901234567890123456789012123456
50 680 18 420
67 710 26 450 7
1 49 670 17 420 7
12 34567890123456789012345678901212345678901234567890123456789012123456
66 700 25 450 7
12 34567890123456789012345678901212345678901234567890123456789012123456
48 660 16 410 7
12 34567890123456789012345678901212345678901234567890123456789012123456
65 700 24 440 7
2 34567890123456789012345678901212345678901234567890123456789012123456
1234567890123456789012345678901212345678901234567890123456789012123456
47 650 15 410 7
1234567890123456789012345678901212345678901234567890123456789012123456
64 690 23 440 7
1234567890123456789012345678901212345678901234567890123456789012123456
46 640 14 400 63 680 22 430 7
1234567890123456789012345678901212345678901234567890123456789012123456 7
1234567890123456789012345678901212345678901234567890123456789012123456 77
45 630 13 390 62 670 21 420
1234567890123456789012345678901212345678901234567890123456789012123456
44 620 12 380
1234567890123456789012345678901212345678901234567890123456789012123456
61 670 20 410 7
1234567890123456789012345678901212345678901234567890123456789012123456
43 610 11 370 60 660 19 410 7
1234567890123456789012345678901212345678901234567890123456789012123456 7
1 42 600 10 360 59 650 18 400 7
2 34567890123456789012345678901212345678901234567890123456789012123456
1234567890123456789012345678901212345678901234567890123456789012123456 7
1234567890123456789012345678901212345678901234567890123456789012123456
41 600 9 350 58 640 17 390 7
1 40 7
1234567890123456789012345678901212345678901234567890123456789012123456
590 8 340 57 640 16 380
7
12
2
34567890123456789012345678901212345678901234567890123456789012123456
39 580 7 330 56 630 15 380
34567890123456789012345678901212345678901234567890123456789012123456
7
7
1234567890123456789012345678901212345678901234567890123456789012123456
1234567890123456789012345678901212345678901234567890123456789012123456
38 570 6 320 55 620 14 370 7
1 37 7
12 34567890123456789012345678901212345678901234567890123456789012123456 7
560 5 310 54 610 13 360
12 34567890123456789012345678901212345678901234567890123456789012123456
53 610 12 360 7
12 34567890123456789012345678901212345678901234567890123456789012123456
36 560 4 300 7
12 34567890123456789012345678901212345678901234567890123456789012123456
35 550 3 280 52 600 11 350 7
1234567890123456789012345678901212345678901234567890123456789012123456 7
1234567890123456789012345678901212345678901234567890123456789012123456
34 540 2 270 51 600 10 340 7
1234567890123456789012345678901212345678901234567890123456789012123456
50 590 9 330 7
2 34567890123456789012345678901212345678901234567890123456789012123456
1234567890123456789012345678901212345678901234567890123456789012123456
33 540 1 250 7
1234567890123456789012345678901212345678901234567890123456789012123456
49 590 8 320 7
1234567890123456789012345678901212345678901234567890123456789012123456
32 530 0 240 7
1234567890123456789012345678901212345678901234567890123456789012123456
48 580 7 310 7
1 31 520 21 220 7
2 34567890123456789012345678901212345678901234567890123456789012123456
1234567890123456789012345678901212345678901234567890123456789012123456
30 510 22 210
47 570 6 300 7
1234567890123456789012345678901212345678901234567890123456789012123456
46 570 5 290 7
1234567890123456789012345678901212345678901234567890123456789012123456
29 510 23 and 200 7
1234567890123456789012345678901212345678901234567890123456789012123456
45 560 4 270 7
1 below 7
1234567890123456789012345678901212345678901234567890123456789012123456 77
2 34567890123456789012345678901212345678901234567890123456789012123456
44 550 3 260
1234567890123456789012345678901212345678901234567890123456789012123456
43 550 2 250
7
1234567890123456789012345678901212345678901234567890123456789012123456 7
1234567890123456789012345678901212345678901234567890123456789012123456
42 540 1 240
7
1
1234567890123456789012345678901212345678901234567890123456789012123456 77
2 34567890123456789012345678901212345678901234567890123456789012123456
41 540 0 230
1 40 530 220
12 34567890123456789012345678901212345678901234567890123456789012123456
21
7
1234567890123456789012345678901212345678901234567890123456789012123456
39 520 22 210 7
12
2
34567890123456789012345678901212345678901234567890123456789012123456
34567890123456789012345678901212345678901234567890123456789012123456
7
7
1 38 520 23 and 200
1234567890123456789012345678901212345678901234567890123456789012123456
below 7
1234567890123456789012345678901212345678901234567890123456789012123456 77
1234567890123456789012345678901212345678901234567890123456789012123456
12345678901234567890123456789012123456789012345678901234567890121234567
82 Copyright © 2005 Thomson Peterson’s, a part of The Thomson Corporaton
SAT is a registered trademark of the College Entrance Examination Board, which
was not involved in the production of and does not endorse this product.

Você também pode gostar